You are on page 1of 165

1200+

Krishna's MCQs

B.Sc. Objective
GEOMETRY and VECTOR CALCULUS
(For I year B.Sc. Students of all Colleges affiliated to Universities in U.P.)

As Per U.P. Unified Syllabus

Solved Question Papers


By
2018 & 2019

Mukesh Singh

M.Sc. (Gold Medalist),


NET (JRF), Ph.D.

Head, Department of Basic Science


Uttar Pradesh Textile Technology Institute
(Formely known as G.C.T.I)
Kanpur (U.P.)

Manoj Garg Mradul Dixit


M.Sc., (Gold Medalist), Ph.D.,
C.S.I.R. NET (JRF), GATE
M.Sc., M.Phil, Ph.D.
Assistant Professor & Head, Dept. of Mathematics
C.S.I.R. NET (JRF)
Saraswati (P.G.) College, Hathras (U.P.)
Associate Professor & Head, Dep’t of Mathematics
Nehru P.G. College
Chhibramau, Kannauj (U.P.)

KRISHNA Prakashan Media (P) Ltd.


KRISHNA HOUSE, 11, Shivaji Road, Meerut-250 001 (U.P.), India
Syllabus
GEOMETRY and VECTOR CALCULUS
B.Sc. I Year; III Paper

U.P. UNIFIED Syllabus (w.e.f. 2011-12)

GEOMETRY
Unit 1
General equation of second degree, Tracing of conics, System of conics, Confocal conics, Polar equation of a conic
and its properties.
Unit 2
Three dimensional system of co-ordinates, Projection and direction cosines, Plane, Straight line.

Unit 3
Sphere, cone and cylinder.

Unit 4
Central conicoids, Reduction of general equation of second degree, Tangent plane and normal to a conicoid, Pole
and polar, Conjugate diameters, Generating lines, Plane sections.

VECTOR CALCULUS
Unit 5
Vector differentiation and integration, Gradient, divergence and curl and their properties, Line integrals, Theorems
of Gauss, Green and Stokes and problems based on these.

(iii)
Brief Contents
Syllabus.... ..............................................................................................III
Brief Contents..........................................................................................IV

GEOMETRY AND VECTOR CALCULUS

Unit-I
Chapter 1: The General Equation of the Second Degree (Tracing of the Conics)...........................(01-10)
Chapter 2: General Conics, Contacts and Confocals......................................................................(11-16)
Chapter 3: Polar Equation of Conic...............................................................................................(17-24)
Unit-II
Chapter 4: Systems of Co-ordinates...............................................................................................(25-30)
Chapter 5: Direction Cosines and Projections................................................................................(31-38)
Chapter 6: The Plane.....................................................................................................................(39-46)
Chapter 7: The Straight Lines........................................................................................................(47-56)
Unit-III
Chapter 8: The Sphere..................................................................................................................(57-66)
Chapter 9: The Cone....................................................................................................................(67-76)
Chapter 10:The Cylinder...............................................................................................................(77-82)
Unit-IV
Chapter 11: The Central Conicoids ...............................................................................................(83-90)
Chapter 12: The Paraboloids.........................................................................................................(91-94)
Chapter 13: Generating Lines........................................................................................................(95-98)
Chapter 14: The Plane sections of Conicoids...............................................................................(99-104)
Chapter 15: Reduction of General Equation of Second Degree.................................................(105-108)
Unit-V
Chapter 16: Vector Calculus......................................................................................................(109-128)
Examination Paper 2019 (BRA Univ., Agra)..................................................................................(01-08)
Examination Paper 2019 (Allahabad State Univ.)...........................................................................(09-16)
Examination Paper 2019 (CSJM Univ., Kanpur)............................................................................(17-25)
Examination Paper 2019 (RML Avadh Univ., Faizabad).................................................................(26-36)

(iv)
B.Sc. Objective Mathematics

Book-3
GEOMETRY AND VECTOR CALCULUS
Unit-I
Chapter -1 : The General Equation of the Second Degree (Tracing of the Conics)
Chapter -2 : General Conics, Contacts and Confocals
Chapter -3 : Polar Equation of a Conic
Chapter -4 : Systems of Co-ordinates

Unit-II
Chapter -5 : Directions Cosines and Projections
Chapter -6 : The Plane
Chapter -7 : The Straight Line

Unit-III
Chapter -8 : The Sphere
Chapter -9 : The Cone
Chapter-10: The Cylinder

Unit-IV
Chapter -11: The Central Conicoids
Chapter -12: The Parabooids
Chapter -13: Generating Lines
Chapter -14: The Plane Sections of Conicoids
Chapter -15: Reduction of General Equations of Second Degree
Chapter -16: Vector Calculus
C
The General Equation of ... (Tracing of the Conics)
HAPTER Unit-I3

1 The General Equation of the Second Degree


(Tracing of the Conics)

CONIC SECTIONS ∆ ≡ abc + 2 fgh − af 2 − bg 2 − ch 2


1. A conic section or conic is the locus of a point
Then equation of conic with respect to centre of
which moves so that its distance from a fixed
origin is
point (focus) is in a constant ratio to its
2 2
perpendicular distance from a fixed straight line Case I: If ab − h = 0 i.e. h = ab i.e. if the second
(directrix). The constant ratio is called the degree terms in the general equation of the conic are
eccentricity and is denoted by e. not in a perfect square then conic is either a pair of
2. If the focus does not lie upon the directrix, the intersecting straight lines, or a circle, or an ellipse or a
conic is a parabola, an ellipse or a hyperbola hyperbola.
according as e =, < or > 1. The circle is a Case II: If ab − h 2 ≠ 0 and ∆ = 0 then the conic always
special case of an ellipse when e = 0 and the represents a pair of straight lines.
focus is the centre of the circle and directrix is at
a h g
an infinite distance from the focus. If the focus Case III: If ab − h 2 = 0 and = = then the conic
h b f
lies on the directrix, the conic section is a pair of
always represents a pair of parallel straight lines.
straight line.
Case IV: If ab − h 2 = 0 and hf − bg or gh − af is not
3. The general equation of the second degree
zero, the conic represents a parabola which has no
ax 2 + 2hxy + by 2 + 2gx + 2 fy + c = 0,
centre.
in every case represents a conic section.
TO FIND THE COORDINATES OF THE CENTRE
CENTRE
The general equation of the conic is
1. The centre of a conic section is a point which
2 2
bisects all those chords of the conic that pass ax + 2hxy + by + 2gx + 2 fy + c = 0.
through it.  hf − bg gh − af 
2 Its centre is  ,  . Now consider
2. If the equation of the conic is ax + 2hxy  ab − h 2 ab − h 2 
+ by 2 + c = 0 then the origin is the centre of the
F (x , y) ≡ ax 2 + 2hxy + by 2 + 2gx + 2 fy + c = 0 then
conic.
∂F ∂F
3. The general equation of the second degree = 2(ax + hy + g ) and = 2(hx + by + f ). The
2 2 ∂ x ∂y
ax + 2hxy + by + 2gx + 2 fy + c = 0 will
centre of the conic F (x , y) = 0 is obtained by solving
represents a conic with centre at the origin only ∂F ∂F
if f = g = 0 i.e. only if the first degree terms are the equation ∂x = 0 and ∂y = 0 i.e. ax + hy + g = 0
absent from the equation of the conic.
and hx + by + f = 0.
CENTRE OF A CONIC
ASYMPTOTES
1. The general equation of the conic
is The general equation of the conic is
2 2
ax + 2hxy + by + 2gx + 2 fy + c = 0 and its
ax 2 + 2hxy + by 2 + 2gx + 2 fy + c = 0
discriminant is
4

and its discriminant is LENGTH OF SEMI-AXES


1. The length of semi-axes of the conic
∆ = abc + 2 fgh − af 2 − bg 2 − ch 2 then
Ax 2 + 2Hxy + By 2 = 1
Case I: The equation of the asymptotes of the general
 1  1
conic is is  A −   B −  = H 2 , it gives two roots of
 r2  r2

ax 2 + 2hxy + by 2 + 2gx + 2 fy + c − =0 r 2 say r12 and r22 .
ab − h 2
Case I: If both r12 and r22 are positive i.e. AB > H 2 then
Case II: The equation of the hyperbolic conjugate of
the conic is an ellipse. If r1 > r2 then
the general conic is
2∆ Semi-major axes = r1
ax 2 + 2hxy + by 2 + 2gx + 2 fy + c − =0
2 Semi-minor axes = r2
ab − h
2 2 2
Case III: If (x , y) be the coordinates of the centre of the Case II: If r1 is positive and r2 is negative i.e. AB < H
hyperbola then the conic is hyperbola.
f (x , y) = ax 2 + 2 hxy + by 2 + 2gx + 2 fy + c = 0 then Semi-transverse axes = r1

(A) The equation of the asymptotes is Semi-conjugate axes = |r22 |


f (x , y) = f (x 1, y 1).
2. The eccentricity of the conic is e then
(B) The equation of the conjugate hyperbola is
r22
f (x , y) = 2 f (x 1, y 1). e= 1− .
r12
NATURE OF A CONIC
|r22 |
The equation ax 2 + 2hxy + by 2 + 2gx + 2 fy + c = 0 3. The semi-latus rectum is .
r1
represents
4. If (x 1, y 1) be the centre of the conic
1. A pair of straight lines if ∆ = 0.
ax 2 + 2hxy + by 2 + 2gx + 2 fy + c = 0 then
2. A pair of parallel straight lines if ∆ = 0 and
the co-ordinate of foci are
h 2 = ab.
(x 1 ± er1 cos θ, y 1 ± er1 sin θ) where
3. A circle if a = b and h = 0.
er1 = r12 − r22 and θ be the angle which the
4. A parabola if h 2 = ab and ∆ ≠ 0.
major axes (ellipse) or transverse axes
5. An ellipse if h 2 < ab and ∆ ≠ 0. (hyperbola) makes with x-axis.
6. A hyperbola if h 2 > ab and ∆ ≠ 0. 5. The equation of the directrix of the conic is
r
7. A rectangular hyperbola if ∆ ≠ 0, h 2 > ab and (x − x 1) cos θ + (y − y 1) sin θ = ± 1 .
e
a + b = 0.
6. The distance of the directrix from the centre C
r
is 1 .
e
5

EXERCISE
MULTIPLE CHOICE QUESTIONS 9. 2x 2 − 3xy − 2y 2 + 2x + y + 1 = 0
1. The co-ordinate of the centre of the conic always represents
ax 2 + 2hxy + by 2 + c = 0 are (a) an ellipse
(a) (a, b, c) (b) (− a, − b, − c) (b) a hyperbola
(c) (a, h, c) (d) (0, 0, 0) (c) a parabola

2. The conic (d) a rectangular hyperbola


10. If the equation of hyperbola is
2x 2 + xy − y 2 − x + 5y + 6 = 0 represents
2x 2 + xy − y 2 − x + 5y + 6 = 0 and its asymptote
(a) a parabola
(b) a circle is y 2 − xy − 2x 2 − 5y + x + 6 = 0 then equation of
(c) a hyperbola conjugate hyperbola is
(d) an ellipse (a) y 2 − xy − 2x 2 − 5y + x + 12 = 0
3. The general equation of the second degree (b) y 2 − xy − 2x 2 + 5y + x + 18 = 0
ax 2 + by 2 + 2hxy + 2gx + 2 fy + c = 0 represents
(c) y 2 − xy − 2x 2 − 5y + x = 0
a conic with centre at origin only when
(d) y 2 − xy − 2x 2 − 5y + x + 6 = 0
(a) f = g = h = 0 (b) f = g = 0
(c) f = 0 (d) a = b = c = 0 11. If the roots r12 and r22 of the quadratic

4. The centre of the conic  1  1 2 2


 A − 2   B − 2  = H in r are of the signs then
2 2  r  r 
14x − 4xy + 11y − 44x − 58y + 71 = 0 is
the conic Ax 2 + 2Hxy + By 2 = 1 is
(a) (2, 3) (b) (3, 2)
(c) (14, 11) (d) (14, − 4) (a) parabola (b) hyperbola
(c) ellipse (d) circle
5. The asymptotes of the hyperbola xy = hx + ky are
(a) x = k, y = h (b) x = h, y = k 12. In two dimensional geometry, the general equation
(c) x = h + k = y (d) x = 0, y = 0 ax 2 + 2hxy + by 2 + 2gx + 2 fy + c = 0

6. The eccentricity e of the ellipse is always represents a


b 2
b (a) coincide
(a) 1+ (b) 1+
a 2 a (b) two planes
(c) conic
b2 b
(c) 1− (d) 1− (d) central coincide
a2 a
13. The co-ordinate of the centre of the conic
7. If the focus does not lie upon the directrix, the conic
is circle when x 2 − 4xy − 2y 2 + 10x + 4y = 0 are
(a) e > 1 (b) e < 1 (a) (1, 2) (b) (−1, 2)
(c) e = 1 (d) e = 0 (c) (1, − 2) (d) (−1, − 2)
8. If the equation of hyperbola and asymptotes are 14. The conic ax + 2hxy + by 2 = 1 is an ellipse of
2
H = 0 and A = 0 respectively then the equation of
(a) h2 < ab (b) h2 > ab
conjugate hyperbola is
(a) 2 A − H = 0 (b) A + H = 0 (c) h2 = ab (d) h = 0
(c) A − 2H = 0 (d) A − H = 0
6

15. The conic ax 2 + 2hxy + by 2 + 2gx + 2 fy + c = 0 23. The minor axis of the ellipse

represents a pair of straight line only when x 2 + xy + y 2 + x + y − 1 = 0 is


(a) ∆ = 0, h2 = ab (b) ∆ ≠ 0, h2 = ab (a) 3x + 3y + 2 = 0 (b) x − y = 2
2
(c) ∆ = 0, h ≠ ab 2
(d) ∆ ≠ 0, h ≠ ab (c) 3x + y − 2 = 0 (d) x − y = 0
2
24. If h = ab for the conic
16. If the conic x 2 + 2xy + (1 + λ)y 2 + 2λy − 1 = 0
represents an ellipse then the value of λ is ax 2 + 2hxy + by 2 + 2gx + 2 fy + c = 0 then
(a) λ = 0 (b) λ < 0 it represents
(c) λ > 0 (d) any real number (a) only parabola
17. The length of the transverse axis of the hyperbola (b) only straight lines
2 2
x − 3xy + y + 10x − 10y + 21 = 0 is (c) a parabola or a pair of straight lines
(a) 2 2 (b) 2 (d) none of these
(c) 2 (d) 2 3 25. The conic ax + by = 1 represents a
18. The equation (a) hyperbola (b) ellipse
(c) circle (d) parabola
ax 2 + 2hxy + by 2 + 2gx + 2 fy + c = 0
26. The length of the major axis of the ellipse
represents a circle if
x 2 + xy + y 2 + x + y − 1 = 0 is
(a) a ≠ b, h = 0 (b) a = b, h ≠ 0
(c) a = b, h = 0 (d) a ≠ b, h ≠ 0 2 6 3 6
(a) (b)
3 2
19. The centre of the conic
4 6 3 6
(c) (d)
9x 2 − 24xy + 16y 2 − 18x − 101y + 19 = 0 is 3 4
(a) (3, − 5) (b) (5, − 3) 27. The conic 5x 2 − 2xy + 5y 2 − 10y − 7 = 0
(c) (−3, − 5) (d) no centre represents [Meerut 2009, 10 B, Avadh 2014]
20. If in a conic, the focus lies on the directrix, the conic (a) parabola (b) ellipse
rectum formed (c) hyperbola (d) straight lines
(a) an ellipse 28. The equation of the transverse axis of the hyperbola
(b) a parabola x 2 + 4xy + y 2 = 4 is

(c) a pair of straight lines (a) x − y = 0 (b) x − y = 1


(d) a hyperbola (c) x + y = 2 (d) not exist
21. The centre of the conic ax 2 + 2hxy + by 2 = 1 is 29. If the roots r12 and r22 of the quadratic
 1  1 2 2
(a) (0, 0) (b) (a, b)  A − 2   B − 2  = H in r are of opposite signs,
 r  r 
(c) (h, 0) (d) (0, h)
then the conic Ax 2 + 2Hxy + By 2 = 1 is a
22. The eccentricity of the conic
(a) parabola (b) ellipse
x 2 − 3xy + y 2 − 10x − 10y + 21 = 0 is
(c) hyperbola (d) circle
5 6
(a) (b) 30. 2 2
The conic x + xy + y + x + y = 4 is
6 5
10 21 (a) a parabola (b) an ellipse
(c) (d)
3 10 (c) a hyperbola (d) none of these
7

31. The length of the conjugate axis of the conic 39. The intersect point of the conic xy − x 2 = 3y with
7 x 2 + 52xy − 32y 2 + 125 = 0 is co-ordinate axes are
(a) 5 (b) 3 (a) (1, 3) only (b) (0, 0) (1, 3)
(c) 1 (d) 7 (c) (0, 3) only (d) (0, 0) only
32. The intersection point of the conic 40. The latus rectum of the conic whose length of
2 2
x + y + 2x + 3y + 2 = 0 with y-axis are semi-axes are given by r12 = 25 and r22 = 4 is
8 5
(a) (0, 2) (0, − 1) (b) (0, − 1) (0, − 2) (a) (b)
5 8
(c) (0, 0) (−1, − 2) (d) none of these 2 4
(c) (d)
2 2 5 25
33. If the equation ax − y = 0 represents a pair of
41. The eccentricity of the conic x 2 − 3xy + y 2 + 1 = 0
perpendicular straight lines then a is
is
(a) −1 (b) 2
1 6
(a) (b)
(c) 3 (d) 1 5 5
34. If the length of the semi-axes of an ellipse are 3 and 5
(c) 6 (d)
5 then its eccentricity is 6
1 2 42. If for a conic r12 = − 16 and r22 = 25 then its semi latus
(a) (b)
5 5
rectum is
3 4 32 16
(c) (d) (a) (b)
5 5 5 5
35. The eccentricity of the conic 48 64
(c) (d)
5 5
5x 2 − 6xy + 5y 2 − 8 = 0 is
43. If 4x 2 + 12xy + 9y 2 + 6x + 9y + 2 = 0 represent
3 1
(a) (b) two straight lines then angle between them is
2 2
π π
(a) (b)
2 2 2 4
(c) (d)
3 3 π
(c) (d) 0°
36. If the equation ax 2 − 17 xy + 6y 2 + 23x − 2y = 20 3

represents a pair of straight lines then a is 44. Conjugate hyperbola + hyperbola is equal to
(a) asymptote (b) 2 asymptotes
(a) 17 (b) 6
(c) 4 asymptotes (d) 0
(c) 7 (d) 2
45. Latus rectum of the conic is
37. The equation x 2 + y 2 − 6x − 8y = 25 represents
2|r22| 2|r1|
(a) (b)
(a) circle (b) parabola r1 r2

(c) hyperbola (d) ellipse 2|r2|


(c) 2 r1|r2| (d)
r1
38. If the curve px 2 − 6xy + 5y 2 + 22x − 26y + 29 = 0
46. The eccentricity of the conic is
represents an ellipse then p is
3 3 r2 r22
(a) ≥ (b) > (a) (b) 1+
5 5 r1 r12
3 3 r22 r12
(c) < (d) =
5 5 (c) 1− (d) 1+
r12 r22
8

47. The equation x 2 + 2xy + y 2 − 2x = 1 always 54. Conic 3x 2 − 4xy + 9y = 0 represents


represents a [Agra 2017]
(a) parabola (b) hyperbola (a) Parabola (b) Ellipse
(c) ellipse (d) circle (c) Hyperbola (d) None of these
48. The centre of the conic 55. The centre of the conic
2 2
8x + 4xy + 5y − 24x − 24y = 0 is x 2 − xy + y 2 + 5x − 5y + 11 = 0 is

(a) (2, 1) (b) (−1, 2) [Avadh 2018]

(c) (1, 2) (d) (−1, − 2) (a) (1, 0) (b) (2, − 2)

49. If r1 and r2 be the lengths of semi-axes of the conic (c) (−1, 1) (d) (−5 / 3, 5 / 3)
such that r12 = 9 and r22 = 4 then conic is 56. The conic x 2 + xy + y 2 + x + y = 1 is

(a) parabola (b) ellipse [Avadh 2018]

(c) hyperbola (d) none of these (a) Ellipse (b) Parabola


4 (c) Hyperbola (d) None of these
50. If for a conic r12 = and r22 = − 4 then its eccentricity
3 57. The eccentricity of the conic
is
3/ r = 1 + 3 cos θ + sin θ is [Avadh 2018]
(a) 4 (b) 1
(a) 1 (b) 2
2
(c) 2 (d)
3 (c) 1/2 (d) 3/4

3 58. The centre of the circle


51. If for a conic r12 is 4 and eccentricity is then the
2 ax 2 + (2a − 3) y 2 − 4x − 1 = 0 is [Avadh 2018]
distance of directrix from its centre is
2 2
2 1 (a)  , 0 (b)  − , 0
(a) (b) 3   3 
3 3
2 2
4 8 (c)  0,  (d)  0, − 
(c) (d)  3  3
3 3

52. The coordinates of the centre of the conic 59. Conic 6x 2 − 5xy − 6y 2 + 14x + 5y + 4 = 0

14x 2 − 4xy + 11y 2 − 44x − 58y + 71 = 0 are represents

[Kanpur 2018] (a) Ellipse

(a) (−2, 3) (b) (3, − 3) (b) A straight line

(c) (2, 3) (d) (−2, − 3) (c) Pair of straight line

53. The coordinates of focus of conic (d) Hyperbola

16x 2 − 24xy + 9y 2 − 104x − 172y + 44 = 0 are 60. x 2 − 3xy + y 2 + 10x − 10y + c = 0 represents

[Kanpur 2018] a hyperbola, when c is equal to

(a) (2, 1) (b) (2, 2) (a) c = 20 (b) c ≠ 20

(c) (1, 1) (d) (1, 2) (c) c = 16 (d) c ≠ 16


9

ANSWERS
MULTIPLE CHOICE QUESTIONS

1. (d) 2. (c) 3. (b) 4. (a) 5. (a) 6. (c) 7. (d) 8. (c) 9. (d) 10. (b)

11. (c) 12. (c) 13. (b) 14. (a) 15. (c) 16. (c) 17. (a) 18. (c) 19. (d) 20. (c)

21. (a) 22. (b) 23. (d) 24. (c) 25. (d) 26. (c) 27. (b) 28. (a) 29. (c) 30. (b)

31. (a) 32. (b) 33. (d) 34. (d) 35. (a) 36. (c) 37. (d) 38. (b) 39. (d) 40. (a)

41. (b) 42. (d) 43. (d) 44. (b) 45. (a) 46. (c) 47. (a) 48. (c) 49. (b) 50. (c)

51. (c) 52. (c) 53. (d) 54. (c) 55. (d) 56. (a) 57. (b) 58. (a) 59. (c) 60. (b)

HINTS AND SOLUTIONS


1 Here a = 1, b = 1 + λ, h = 1 so for ellipse h2 < ab i.e.
2. Here a = 2, b = − 1, h = so h2 > ab i.e. hyperbola. 16.
2
1 < 1 + λ i.e. λ > 0.
4. Let F (x, y) = 14x 2 − 4xy + 11y 2 − 44x − 58y + 71
17. Let F (x, y) = x 2 − 3xy + y 2 + 10x − 10y + 21 = 0
∂F
= 28x − 4y − 44 = 0 i.e. 7 x − y = 11 then its centre is (−2, 2) also c1 = yx1 + fy1 + c = 1
∂x
Thus equation of the conic referred to the centre c as
∂F
= − 4x + 22y − 58 = 0 i.e. 2x − 11y = − 29 origin is − x 2 + 3xy − y 2 = 1 reducing to the
∂y
standard form Ax 2 + 2Hxy + By 2 = 1. The square
After solving these equations we get x = 2, y = 3
which is centre. of the semi-axes are given by
 1  1 2
10. Let F (x, y) = 2x 2 + xy − y 2 − x + 5y + 6  A − 2 B − 2 = H
 r   r 
∂F ∂F
= 4x + y − 1 = 0, = x − 2y + 5 = 0 2
∂x ∂y  1  1   3
i.e.  −1 − 2   −1 − 2  =  
 r   r   2
After solving we get
−2
−1 7 After solving we get r12 = 2 and r22 = , so the curve
x= , y = so F (x1, y1) = 12 3
3 3
is hyperbola and length of transverse axes
So, asymptote is F (x, y) = F (x1, y1) i.e.
= 2r1 = 2 2
2 2
2x + xy − y − x + 5y − 6 = 0.
22. See the solution of (17) and find r12, r22 then
Thus, equation of conjugate hyperbola is
r22
eccentricity is defined by e = 1 −
F (x, y) = 2F (x1, y1) i.e. r12
2x 2 + xy − y 2 − x − 5y = 18
10

28. F = x 2 + 4xy + y 2 − 4, centre is (0, 0) and So equation of transverse axes is


c1 = gx1 + fy1 + c = − y. So equation of the conic  
 A − 1  x + Hy = 0 i.e. x − y = 0.
referred to the centre C as origin is  2
r1 

2 2 1 2 1
x + 4xy + y = 4 i.e. x + xy + y 2 = 1
4 4 33. Lines are y= ax and y = − ax for
2 2
Compare Ax + 2Hxy + By = 1. The square of the perpendicularity m1m2 = − 1 i.e. − a = − 1 i.e. a = 1.
❍❍❍
 1  1
semi-axes are  A −   B −  = H 2. So we get
 r2   r2 
4
r22 = − 4 and r12 = .
3
C HAPTER Unit-I
11

2 General Conics, Contacts and Confocals

EQUATION OF A CONIC SECTION 3. The co-ordinates (x 1, y 1) of the point of contact


1. The equation of a conic section is given by the are obtained from the relations
general equation of the second degree in x and ax 1 + hy 1 + g hx 1 + by 1 + f
=
y i.e. l m
gx 1 + fy 1 + c
F (x , y) = ax 2 + 2hxy + by 2 + 2gx + 2 fy + c = 0 =
n
2. The general equation of a conic is
POLAR OF A POINT OR CHORD OF CONTACT
ax 2 + 2hxy + by 2 + 2gx + 2 fy + c = 0, which
1. The equation of the chord of contact of the
may be point P (x 1, y 1) with respect to the conic
a 2 2h b 2g 2f
x + xy + y 2 + x+ y+1= 0 ax 2 + 2hxy + by 2 + 2gx + 2 fy + c = 0 is
c c c c c
Thus five conditions are required to determine axx 1 + h (xy 1 + x 1y) + byy 1 + g (x + x 1)
a h b g f + f (y + y 1) + c = 0
five contacts namely , , , and .
c c c c c 2. Let P be a given point in the plane of a conic.
3. Every straight line through a given point cuts Draw a straight line through the point P to meet
the conic in two points. the conic in the point Q and R. Draw the
tangents at the points Q and R on the conic and
TANGENT TO THE GENERAL CONIC
let these tangents intersect at the point T . The
1. The equation of the tangent at any point (x 1, y 1)
locus of point T is called the polar of a point P
on the conic
with respect to the given conic and point P is
F (x , y) = ax 2 + 2hxy + by 2 + 2gx + 2 fy + c = 0 called the pole of the polar.
is axx 1 + h (x 1y + xy 1) + hyy 1 + g (x + x 1) P
Q
+ f (y + y 1) + c = 0
2. The condition that the line lx + my + n = 0 may
touch the conic P T

F (x , y) = ax 2 + 2hxy + by 2 + 2gx
R
+ 2 fy + c = 0
3. The equation of the polar of the point P(x 1, y 1)
is Al 2 + Bm 2 + Cn 2 + 2Fmn + 2Gnl
with respect to the conic
+ 2Hlm = 0 ax 2 + 2hxy + by 2 + 2gx + 2 fy + c = 0

where A, B, C, F, G, H are the cofactors of a, b, is axx 1 + h(xy 1 + x 1y) + byy 1 + g (x + x 1)


a h g + f (y + y 1) + c = 0.
c, f , g, h in the determinant h b f . 4. If the polar of a point P with respect to a given
conic passes through a point Q, then the polar
g f c
12

of Q will also pass through P. Then points P and If ab = a 2 , the conic F (x , y) = 0 represents a
Q are called conjugate points. parabola then equation of director circle is
5. Two lines are called conjugate lines if the pole 2Gx + 2Fy − ( A + B) = 0.
of one lies on the other. 3. Every central conic has four and only four foci,
6. The equation of the chord with (x 1, y 1) as the two of which are real and two imaginary.
middle point of the conic
4. The tangents from a focus to a conic satisfy the
F (x , y) ≡ ax 2 + 2hxy + by 2 + 2gx + 2 fy + c = 0
conditions for a circle, conversely if the tangents
is
from a point to a conic satisfy the conditions of
 1 ∂F   1 ∂F  a circle, the point must be a focus of the conic.
x  + y  + c1
 2 ∂x  ( x  2 ∂y  ( x
1, y1)
1, y1) CONTACTS OF CONIC
= F (x 1, y 1) Two conics intersects in four points which are all real, or
where c1 = gx 1 + fy 1 + c. two real and two imaginary, or all the four imaginary.
DIAMETER AND CONJUGATE DIAMETERS We will discuss the contacts of different orders :

1. The locus of the middle points of a system of 1. The contact is said to be of zeroth order if all the
parallel chords is called a diameter. four points of intersection namely P, Q, R and S
2. The equation of a diameter of the conic are distinct.

F (x , y) ≡ ax 2 + 2hxy + by 2 + 2gx + 2 fy + c = 0 P
Q
is (a + hm) x + (h + bm) y + g + mf = 0
3. If two diameters are such that each diameter
bisects the chords parallel to the other, they are
called conjugate diameters.
S
R
4. The condition that y = mx and y = m 1x be
parallel to conjugate diameters of the conic
F (x , y) = 0 is a + h (m + m 1) + bmm 1 = 0 2. If two of the four points say P and Q, coincide at
P and the other two points (R and S) are
5. The condition that the two straight lines
distinct, the contact at P is said to be of the first
Ax 2 + 2Hxy + By 2 = 0 may be conjugate
order.
diameters of the conic ax 2 + 2hxy + by 2 = 1 is
aB − 2hH + bA = 0
DIRECTOR CIRCLE OF A CONIC AND FOCI
1. The locus of the points of intersection of
perpendicular tangents, is called a director
circle.
2. The equation of the director circle of the conic
3. If the points P and Q coincide, as also R and S,
F (x , y) ≡ ax 2 + 2hxy + by 2 + 2gx + 2 fy + c = 0
but P and R do not coincide. The conics then
is (ab − h 2 )(x 2 + y 2 ) + 2x (bg − hf ) touch at two points P and R and have a double
+ 2y (af − gh) + c (a + b) − g 2 − f 2 = 0. contacts.
13

P, Q P
Q, R, S

R, S
CONFOCAL CONICS
4. If three points P, Q, R coincide at P and the
fourth point S is distinct from them, the contact 1. All those conics which have the same two
is said to be of the second order. points or their foci are called confocal conics.
P, Q, R 2. The equation of confocal to an ellipse
x 2 y2 x2 y2
+ = 1 is + = 1.
a 2 b2 a 2 + λ b2 + λ
3. Through any given point in the plane of a conic,
S two confocals can be drawn, one of which is a
hyperbola and the other an ellipse.
5. If all the four points P, Q, R, S coincide at P, the
contact is said to be of the third order. 4. Confocals cut at right angles.
5. One and only one members of confocals
touches a given line.

EXERCISE
MULTIPLE CHOICE QUESTIONS 5. In a conic if all the four points of intersection are
1. The independent contacts in the general conic distinct then the contact is of
equation of the second degree F (x, y) = 0 are (a) zeroth order (b) first order
(a) 3 (b) 4 (c) second order (d) third order
(c) 5 (d) 6 6. If one confocal of the conic is ellipse then other one
2. The equation of the tangent at (1, 0) on the conic is
x 2 + 2xy + y 2 − 1 = 0 is (a) parabola (b) circle
(c) hyperbola (d) ellipse
(a) x + 2y = 2 (b) x + y = 1
(c) x − 2y = 2 (d) 2x + y = 1 7. The angle between two confocals is
[Kanpur 2018, Avadh 2018]
3. The locus of the middle points of a system of parallel
π π
chords of a conic is called (a) (b)
2 4
(a) chord (b) radius π
(c) (d) 0
(c) diameter (d) conic 3
4. Confocal conics have 8. If all the four points of two conics coincide at one
(a) same axes only point then contact is said to be of
(b) same centre only (a) first order (b) second order
(c) same axes and centre both (c) third order (d) fourth order
(d) none of these
14

9. The equation of the tangent at (1, 1) of the conic 17. The condition that two straight lines
2x + 2y + 1 = 0 is 2 2
x + y + 2xy = 0 be the conjugate diameters of
(a) x + y + 1 = 0 (b) x + y + 2 = 0
the conic ax 2 + 2hxy + by 2 = 1 is
(c) x + y + 3 = 0 (d) x + y = 0
(a) a + h + b = 0 (b) a − h + b = 0
10. If the equation of the system of parallel chords of the
2 2 (c) a + b = 0 (d) a − 2h + b = 0
conic x + 2xy + y + 1 = 0 is y = x + 1 then
equation of diameter is 18. The condition that y = mx and y = m1x be parallel to
(a) x + y + 1 = 0 (b) x + y = 0 conjugate diameters of the conic
(c) x + y + 3 = 0 (d) x + y + 2 = 0
ax 2 + 2hxy + by 2 + 2gx + 2 fy + c = 0 is
11. If a central conic has four and only four foci and
these are [Kanpur 2018] (a) a + hmm1 + b = 0
(a) all reals (b) a + h(m + m1) + b = 0
(b) all imaginary
(c) two real and two imaginary (c) a + h(m + m1) + bmm1 = 0

(d) none of these (d) a + hmm1 + b(m + m1) = 0


12. If two diameters are such that each diameter bisects
19. If x + λ = 0 be the tangent of the conic
the chords parallel to the other then these diameters 2 2
x + 2xy − y + 1 = 0 then the value of λ is
are called
(a) parallel diameters (a) 2 (b) −2
(b) bisected diameters (c) 2 (d) not exist
(c) conjugate diameters 20. The system of conic confocal to the ellipse
(d) perpendicular diameters x2 y2
+ = 1 is
2
13. Through any given point in the plane of conic, two a b2
confocals can be drawn which are
(a) a parabola (b) an ellipse
(a) circle and ellipse
(c) a hyperbola (d) a plane
(b) circle and hyperbola
(c) ellipse and hyperbola 21. The locus of the pole of a given straight line w.r.t. a
series of confocal conics is a
(d) plane and ellipse
14. In the plane of the conics, confocal cuts at (a) circle (b) parabola

(a) acute angle (b) obtuse angle (c) ellipse (d) straight line

(c) right angle (d) none of these 22. The locus of the points of contact of the tangents
drawn from a given point to a system of confocal is
15. The locus of a point from which two perpendicular
a:
tangents can be drawn one to each of two given
confocals is (a) linear curve (b) quadratic curve

(a) an ellipse (b) a circle (c) cubic curve (d) none of these

(c) a hyperbola (d) a plane 23. The two parabolas which have common focus and
16. If two conic have their axes parallel then the conic their axes in opposite directions always intersect at
that pass through their point of intersection is π
(a) 0 (b)
6
(a) parabola (b) hyperbola
π π
(c) ellipse (d) circle (c) (d)
4 2
15

24. The equation of the circle which has contact of the 31. The equation of a diameter of the conic
second order with the conic
F (x, y) = ax 2 + 2h xy + by 2 + 2gx + 2 fy + c = 0 is
ax 2 + 2hxy + by 2 + 2cx = 0 at the origin is
[Kanpur 2018]
(a) bx 2 + by 2 + 2cx = 0 (b) ax 2 + ay 2 + 2bx = 0 (a) (a + hm) x + (h + bm) y + g + mf = 0
(c) ax 2 + ay 2 + 2x = 0 (d) by 2 + ax 2 + 2cx = 0 (b) (a + hm) x − (h + bm) y + g − mf = 0
(c) (a + hm) x + (h + bm) y − g − mf = 0
25. The confocal conic of the parabola y 2 = 4ax is
(d) None of these
(a) y 2 = λ (x + λ) (b) y 2 = 2λ (x + λ)
32. The polar equation of the chord of contact of the
(c) y 2 = 4λ (x + λ) (d) y 2 = 4λ x l
point (r1, θ1) with respect to the conic = 1 + e cosθ
26. The number of confocal conics through a given r
point of an ellipse are is [Kanpur 2018]
(a) 3 (b) 4 (c) 2 (d) infinite l l 
(a)  − e cos θ  − e cos θ1  = cos (θ − θ1)
27. The confocal conic of an ellipse is r   r1 
(a) ellipse only (b) hyperbola only
l l 
(c) ellipse and hyperbola both (b)  + e cos θ  + e cos θ1  = cos (θ − θ1)
r   r1 
(d) not exist
l l 
28. The confocal conic of x 2 + 2y 2 = 4 is (c)  + e cos θ  − e cos θ1  = cos (θ + θ1)
r   r1 
x2 x 2 y2
(a) + y2 = 4 (b) + =1
2 4 48 l l 
(d)  − e cos θ  + e cos θ1  = cos (θ + θ1)
2 2 2 2 r   r1 
x y x y
(c) + =1 (d) + =1
45 48 48 45 33. Let P (a cos α, b sin α) be a part on the ellipse
29. Two conics are confocal if they have common x2 y2
+ = 1. If P and Q are extremities of conjugate
(a) foci (b) centre a 2
b2
(c) axes (d) latus rectum diameter, then the coordinate of Q is
30. The confocal conic of the conic y 2 = 8(x + 2) is [Avadh 2018]
2 2
(a) y = 8(x + 2) (b) y = 12(x + 3) (a) (a sin α, b cos α) (b) (a cos α, b sin α)
2 2
(c) y = 8(x − 2) (d) y = 12(x − 3) (c) (− a sin α, b cos α) (d) (a sin α, − b cos α)

ANSWERS
MULTIPLE CHOICE QUESTIONS

1. (c) 2. (b) 3. (c) 4. (b) 5. (a) 6. (c) 7. (a) 8. (c) 9. (c) 10. (b)

11. (c) 12. (c) 13. (c) 14. (c) 15. (b) 16. (d) 17. (d) 18. (c) 19. (d) 20. (b)

21. (d) 22. (c) 23. (d) 24. (a) 25. (c) 26. (c) 27. (c) 28. (d) 29. (a) 30. (b)

31. (a) 32. (a) 33. (c)


16

HINTS AND SOLUTIONS


2. Tangent is axx1 + h(x1y + xy1) + hyy1 + g (x + x1) a h g l

+ f (y + y1) + c = 0 h b f m
=0
i.e. x + 1(y + 0) + 0 + 0 + 0 − 1 = 0 i.e. x + y = 1. g f c n

10. The equation of diameter is l m n o


(a + hm) x + (h + bm) y + g + mf = 0 where a = 1, 1 1 0 1
h = 1, b = 1, m = 1, g = 0, f = 0 so we get x + y = 0
1 −1 0 0
19. The condition of tangents is i.e. =0
0 0 1 λ

1 0 λ 0

It gives an imaginary value of λ.


❍❍❍
C HAPTER Unit-I17

3 Polar Equation of a Conic

CONIC SECTION
1. A conic section is the locus of a point which
x
moves so that its distance from a fixed point is
in a constant ratio to its perpendicular distance
from a fixed straight line.
2. In polar system, the position of a point in a 5. If the conic is a parabola i.e. e = 1 then equation
plane is determined by its distance r from a l θ
of parabola is r = sec 2
fixed point O, called the pole or origin and the 2 2
angle θ that the line OP makes with OX is called 6. The equation of the directrix of the conic
vectorial angle. Fixed line OX is called initial l l
= 1 + e cosθ is = e cosθ
line. The distance r is called the radius vector r r
and is taken to be positive if measured along 7. The polar equation of a conic with its focus as
the line bonding the vectorial angle, and the pole and its axis inclined at angle α to the
negative if measured in opposite direction. In l
initial line is = 1 + e cos(θ − α)
polar system the co-ordinates of a point are not r
unique.
8. The equation of directrix in case 7 is
P(r, θ)
l
= e cos(θ − α)
r r

O θ CHORD AND TANGENT OF THE CONIC


X
1. The equation of the chord of the conic
l
Q (–r, θ) = 1 + e cos θ, whose extremities are P (r1, θ 1)
r
and Q (r2 , θ 2 ) is
3. The polar equation of a conic with its latus
rectum of length 2l, eccentricity e and the focus l  θ − θ1   θ1 + θ2  
= e cos θ + sec   cos θ −  
l r  2    2 
being the pole is = 1 + e cosθ
r 2. If the points P and Q are such that their vectorial
angles are α − β and α + β, so that the sum of
the angles is 2α and their difference is 2β then
x
the chord PQ in case (1) is
l
= e cos θ + sec β cos(θ − α)
r
4. If we take the positive direction of the initial line 3. The equation of the tangent at the point (r1, θ 1)
opposite to the direction directed from the focus l
of the conic = 1 + e cosθ is
towards the directrix, the equation of the conic r
l l
is = 1 − e cosθ = e cos θ + cos(θ − θ 1)
r r
18

l
4. If the conic is = 1 + e cos(θ − α) then the 7. The polar equation of the chord of contact of
r l
the point (r ′, θ ′) w.r.t the conic = 1 + e cosθ is
tangent at θ 1 is r
l l l 
= e cos(θ − α) + cos(θ − θ 1)  − e cos θ  − e cos θ ′ = cos(θ − θ ′)
r r  r′ 
l
5. If the conic is = 1 − e cosθ then the tangent at POLAR AND NORMAL
r
θ 1 is 1. The equation of the polar of a point (r 1, θ 1)
l l
= − e cos θ + cos(θ − θ 1) w.r.t. the conic = 1 + e cosθ is
r r
l l 
ASYMPTOTES  − e cos θ  − e cos θ ′ = cos(θ − θ ′).
r  r′ 
1. The equation of the asymptotes of the conic
l 2. The pole of a line is the point of intersection of
= 1 + e cosθ are the tangents at its extremities.
r
le 3. The equation of any line perpendicular to
= (e 2 − 1) cos θ ± (e 2 − 1) ⋅ sin θ l
r = A cos θ + B sin θ is obtained by writing
r
2. These two asymptotes are real only when e > 1
π
and then conic is a hyperbola. θ + for θ and changing l to a new constant L.
2
AUXILIARY CIRCLE AND DIRECTOR CIRCLE 4. The equation of the normal at point α on the
1. The locus of the foot of the perpendicular from l
conic = 1 + e cosθ is
the focus on any tangent to a conic is a circle r
called the auxiliary circle of the conic. le sin α 1
⋅ = sin (θ − α) + e sin θ
2. The polar equation of the auxiliary circle of the (1 + e cos α) r
l
conic = 1 + e cosθ is 5. In case of parabola i.e. e = 1 by point 4
r
equation of the normal at a point α is
(e 2 − 1) r 2 − 2 ler cos θ + l 2 = 0 l sin α 1
⋅ = sin (θ − α) + sin θ
1 + cos α r
3. If the conic is parabola i.e. e = 1 then by point 2
l 6. The equation of the normal at the point (r1, α) to
auxiliary circle is = 2 cosθ
r l
the conic = 1 − e cosθ is
4. The locus of the point of intersection of two r
perpendicular tangents to a conic is called the le sin α 1
− ⋅ = sin (θ − α) − e sin θ
director circle of the conic. 1 − e cos α r
5. The equation of the director circle of the conic TRANSFORMATION OF CO-ORDINATES
l
= 1 + e cosθ is 1. If the cartesian co-ordinate of a point P is (x , y)
r
and the corresponding polar co-ordinate is (r , θ)
(1 − e 2 ) r 2 + 2 ler cos θ − 2l 2 = 0 then
6. If the conic is parabola i.e. e = 1 then by 5 the x = r cos θ, y = r sin θ so r = x 2 + y 2
l
director circle is = cosθ
r  y
and θ = tan −1  
x
19

2. The distance between two points A (r1, θ 1) and sin (θ 2 − θ 1) sin (θ − θ 1) sin (θ 2 − θ)
= +
B (r2 , θ 2 ) is r r2 r1

AB = r12 + r22 − 2 r1r2 cos (θ 2 − θ 1) 3. Area of a triangle whose vertices are


(r1, θ 1), (r2 , θ 2 ) and (r3 , θ 3 ) is
GENERAL EQUATION OF LINE
1
1. The general equation of the straight line in polar D= {r1r2 sin (θ 2 − θ 1) + r2r3 sin (θ 3 − θ 2 )
co-ordinate system is 2

r cos (θ − θ 1) = p + r1r3 sin (θ 1 − θ 3 )}


2. The equation of straight line passing through
(r1, θ 1) and (r2 , θ 2 ) is

M (p, θ1)

θ1
X
O

EXERCISE
MULTIPLE CHOICE QUESTIONS 5.
π
The equation of the tangent at the point  r1,  of
l  2
1. The conic = 1 + e cosθ represents a parabola if l
r the conic = 1 + e cosθ is
r
(a) e = 1 (b) e > 1 (c) e < 1 (d) e = 0
l l
2. The equation of the directrix of the conic (a) = cos θ + sin θ (b) = e cos θ − sin θ
r r
l
= 1 + e cosθ is l l
r (c) = e cos θ + sin θ (d) = cos θ − sin θ
r r
l r
(a) = − e cosθ (b) = e cosθ l
r l 6. The slope of the tangent to the conic = 1 + e cosθ
r
r l
(c) = − e cosθ (d) = e cosθ at the point α on it is given by
l r
e + cos α e − cos α
3. The semi latus rectum between the segments of a (a) (b)
sin α sin α
focal chord is always a
e + cos α e 2 − cos α
(a) arithmetic mean (b) geometric mean (c) − (d)
sin α sin α
(c) harmonic mean (d) none of these
7. The number of normals that can be drawn from a
l
4. The equation of the conic = 1 + e cosθ is point to a parabola are
r
(a) 1 (b) 2
equivalent to the following equation[Avadh 2018]
(c) 3 (d) 6
l l
(a) = 1 − e cosθ (b) = 1 + e cosθ
r r 8. In a polar system the representation of the
−l l co-ordinates of a point are
(c) = 1 + e sinθ (d) = − 1 + e cosθ
r r (a) unique (b) finite
(c) infinite (d) none of these
20

9. If l be the semi latus rectum of a conic then the sum π


16. The equation of the normal at the point  r,  of the
of the reciprocals of the segments of any focal chord  2
is l
parabola = 1 + cosθ is
e 2 r
(a) (b)
l l 1 1
(a) = sin θ − cos θ (b) = 1 − cosθ
l l r r
(c) (d)
e 2 1 1
(c) = sin θ + cos θ (d) = 1 − sinθ
l r r
10. The condition that the line = A cos θ + B sin θ to be
r 17. The equation of the auxiliary circle of the conic
l l
a tangent to the conic = 1 + e cosθ is = 1 + cosθ is given by
r [Avadh 2018] r
(a) ( A + e)2 + B 2 = 1 (b) (B − e)2 + A2 = 1 (a) (e 2 − 1) r 2 + 2lr cos θ + l 2 = 0
(c) (B + e)2 + A2 = 1 (d) ( A − e)2 + B 2 = 1 (b)
l
= cosθ
r
11. The locus of the point of intersection of two tangents
l
to the parabola, which cut one another at a constant (c) = 2cosθ
r
angle is a l
(d) = 1 + cosθ
(a) circle (b) parabola r
(c) hyperbola (d) ellipse l θ
18. The conic r = sec2 always represents a
15 2 2
12. The nature of the conic = 3 − 4 cosθ is
r (a) parabola (b) hyperbola
(a) parabola (c) ellipse (d) circle
(b) ellipse 1
19. The asymptotes of the conic = 1 + 2 cosθ are
(c) hyperbola r
(d) rectangular hyperbola (a) r( 3 cos θ ± sin θ) = 2
l (b) 3 ( 3 cos θ ± sin θ) = 2r
13. Two real asymptotes of the conic = 1 + e cosθ
r (c) r( 3 cos θ ± sin θ) = 3
exist only when
(d) r 3 ( 3 cos θ ± sin θ) = 2
(a) e < 1 (b) e > 1
20. The coincident point of (r, θ) is
(c) 0 < e < 1 (d) e is real
(a) (r, θ + 2 π) (b) (− r, θ + π)
14. The locus of the point of intersection of two
perpendicular tangents to a conic is called (c) (−r, θ) (d) (− r, − θ)
(a) circle (b) director circle 21. The equation of the directrix of the conic
(c) auxiliary circle (d) parabola 2 1
= 1 + cosθ is
15. The equation of the director circle of the conic r 2
l 4 2
= 1 + cosθ is (a) = cosθ (b) = cosθ
r r r
(a) 2r 2 + 2 rl cosθ − 2l 2 = 0 1
(c) = 2 cosθ (d) none of these
(b) r 2 + lr cosθ + l 2 = 0 r

l l
(c) = 1 + cosθ 22. The conic = 2 + 3cosθ represents a
r r
l (a) parabola (b) circle
(d) = cosθ
r
(c) hyperbola (d) ellipse
21

14
23. The point on the curve = 3 − 8 cosθ whose radius 31. The equation of the director circle of the conic
r 1
= 1 + e cosθ is
vector is 2, is represented by r
(a)  3,
2π 

π
(b)  2,  (a) (1 + e 2) r 2 + 2er cos θ − 2l 2 = 0
 3  3
(b) (1 − e 2) r 2 + 2er cos θ − 2l 2 = 0
π 2π 
(c)  3,  (d)  2, 
 3  3 (c) (1 + e 2) r 2 + 2er cos θ + 2l 2 = 0
4 2 (d) none of these
24. The directrix of the conic = + cosθ is
r 3 32. The slope of the tangent at the point (r1, θ1) of the
4 6 l
(a) = cosθ (b) = cosθ conic = 1 + cosθ is
r r r
1 2 cos θ1 1 + cos θ1
(c) = + cosθ (d) none of these (a) (b)
r 3 sin θ1 sin θ1
1 5 1 + cos θ1 1 + sin θ1
25. The semi latus rectum of the conic = + 2 cosθ (c) − (d) −
2r 3 sin θ1 cos θ1
is
5 3 3 10 33. The locus of the foot of the perpendicular from the
(a) (b) (c) (d)
3 5 10 3 focus of any tangent to a conic is called
l θ (a) auxiliary circle (b) director circle
26. r= cosec2 represents a
2 2 [Agra 2017] (c) ellipse (d) parabola
(a) parabola (b) hyperbola l
34. The condition that the line = 2cos θ + sin θ be a
(c) ellipse (d) circle r
l
27. The polar equation of the auxiliary circle of the tangent to the conic = 1 + e cosθ is
r
1
conic = 1 + e cosθ is (a) e = 1 (b) e = 2
r
3
(a) (e 2 − 1) r 2 − 2er cos θ + 1 = 0 (c) e = (d) e = 3
2
(b) (e 2 + 1) r 2 − 2er cos θ + 1 = 0 35. The equation of the director circle of the conic
2 2
(c) (e + 1) r + 2er cos θ + 1 = 0 3
= 1 + 4 cosθ
r
(d) (e 2 − 1) r 2 + 2er cos θ − 1 = 0
(a) 5r 2 − 8r cosθ + 6 = 0
28. If two real asymptotes exist for a conic then it is
(b) 5r 2 + 8r cosθ + 6 = 0
(a) parabola (b) hyperbola
(c) 5r 2 + 8r cosθ − 6 = 0
(c) ellipse (d) circle
(d) 5r 2 − 8r cosθ − 6 = 0
29. r = a represents a
36. If e12 + e22 = 1 for two different conics of same nature
(a) circle (b) parabola
then conic is
(c) ellipse (d) hyperbola (a) ellipse (b) parabola
l
30. The condition that the line = A cos θ + B sin θ be a (c) circle (d) hyperbola
r
1 1 1
l 37. The eccentricity of the conic = + cosθ is
tangent to the conic = 1 + cosθ is r 6 18
r
1 1
(a) (b)
(a) A2 + B 2 + 2 A = 0 (b) A + B = 0 18 3
1 1
(c) A2 + B 2 − 2 A = 0 (d) A2 + B 2 = 0 (c) (d)
6 2
22

38. For the rectangular hyperbola, eccentricity is 48. The equation of director circle of the conic
(a) 3 (b) 2 3
= 1 + cosθ is
r
(c) 3 (d) 2
(a) r secθ = 3 (b) r cosθ = 3
39. The perpendicular focal chords of a rectangular
(c) r cosecθ = 3 (d) none of these
hyperbola are
2 π
(a) unequal 49. Equation of normal of = 1 + cosθ at  r,  is
r  2
(b) equal
2 2
(c) may be equal or unequal (a) = sin θ + cos θ (b) = sinθ
r r
(d) none of these 2 2
(c) = sin θ cos θ (d) = cosθ
π r r
40. If e = , then the conic is
2
50. The maximum asymptotes of the hyperbola are
(a) circle (b) parabola
(a) 1 (b) 2
(c) ellipse (d) hyperbola
(c) 3 (d) 4
π
41. If e = cosθ for 0 < θ < then the conic is 2
2 51. The asymptotes of the conic = 1 + 3 cosθ are
r
(a) parabola (b) ellipse 1
(a) = 4 cos θ ± sin θ
(c) hyperbola (d) circle r
42. The semi latus rectum of the conic 1
(b) = 4 cos θ ± 2 sin θ
r(cos θ + cos α) = 2 is r
3
(a) 2sinα (b) sinα (c) = cos θ ± sin θ
r
(c) 4sinα (d) cosecα
3
(d) = 4 cos θ ± 2 sin θ
43. If e and e ′ be the eccentricities of ellipse and r
1
hyperbola respectively then e 2 + is 52. The equation of the director circle of the conic
e ′2 1 1
= + 3 cosθ is
(a) > 2 (b) < 2 r 2
(c) = 2 (d) none of these (a) 35r 2 − 24r cosθ + 8 = 0
2
44. If for a conic e is given by 2e − 3e − 1 = 0 then the (b) 35r 2 − 16r cosθ + 8 = 0
conic is
(c) 5r 2 − 24r cosθ − 8 = 0
(a) circle (b) parabola
(d) 3r 2 − 16r cosθ − 8 = 0
(c) ellipse (d) hyperbola
θ θ 1 53. Tangents from the focus to a conic satisfy the
45. The conic cos + sin = represents a
2 3 2 conditions for an [Kanpur 2018]
(a) ellipse (b) parabola (a) ellipse (b) hyperbola
(c) hyperbola (d) circle (c) circle (d) None of these
46. Which of the following curve has an asymptotes? 54. The equation of the director circle of the conic
(a) circle (b) ellipse l
= 1 + e cos θ is
r
(c) rectangle (d) hyperbola
2 (a) (1 − e 2) r 2 + 2 ler cos θ + 2 l 2 = 0
47. The auxiliary circle of the conic = 1 + cosθ is
r (b) (1 + e 2) r 2 + 2 ler cos θ − 2 l 2 = 0
(a) r = secθ (b) r = cosecθ (c) (1 − e 2) r 2 + 2 ler cos θ − 2 l 2 = 0
(c) r = sinθ (d) r = cosθ
(d) None of these
23

π π
55. The equation of the auxiliary circle of the comic 60. The distance between  1,  and  2,  is
l  6  3
= 1 + e cos θ is
r [Kanpur 2018] [Avadh 2018]

(a) (e 2 − 1) r 2 + 2 ler cos θ + l 2 = 0 (a) 5− 2 3 (b) 5+ 2 3


(c) 2 + 2 (d) 2 − 2
(b) (e 2 + 1) r 2 + 2 ler cos θ − l 2 = 0
61. The polar equation of a circle which is passing
(c) (e 2 + 1) r 2 − 2 ler cos θ + l 2 = 0
through pole and whose centre lies on the initial line
(d) (e 2 + 1) r 2 + 2 ler cos θ + l 2 = 0 is [Avadh 2018]
(a) r = 2 a sin θ
56. The equation of tangent at the point (r1, θ1) of the
(b) r = a sin 2 θ
conic l / r = 1 + e cos θ is
θ
[Kanpur 2018, Avadh 2018] (c) r = a cos
2
(a) l / r = e cos (θ − θ1) + cos θ (d) r = a cos θ
(b) l / r = e cos θ + cos (θ − θ1) l
62. Let PSQ be a chord of the conic = 1 + cos θ
r
(c) l / r = e cos (θ + θ1) + cos θ
passing through the focus and inclined at an angle α
(d) l / r = e cos θ + cos (θ + θ1)
with initial line then SP ⋅ SQ is [Avadh 2018]
57. The equation of normal at a point α on the conic
1 l2
l / r = 1 + cos θ is [Kanpur 2018] (a) (b)
1 − cos 2 α sin 2 α
l sin α
(a) = sin (θ + α) + sin α l2 1
r (1 + cos α) (c) (d)
cos 2 α 1 + cos 2 α
l sin α
(b) = sin (θ − α) + sin θ
r (1 + cos α) 63. If e is the eccentricity of a parabola and e ′ is the
eccentricity of conic such that e + e ′2 = 5 then
l sin α
(c) = sin (θ − α) − sin θ
r (1 + cos α) conic represents a
(a) parabola (b) ellipse
l sin α
(d) = sin (θ + α) − sin θ (c) circle (d) hyperbola
r (1 − cos α)
l 64. The equation of the normal of the conic
58. The conic = 1 + 3 cos θ is
5 π
r [Avadh 2018] = 4 + 2 cos θ at  r,  is
r  2
(a) ellipse (b) parabola
5
(a) = 4 sin θ + 6 cos θ
(c) hyperbola (d) None of these r
l 5
59. The conic = 1 + e cos θ is a hyperbola if (b) = 4 sin θ − 8 cos θ
r r
[Avadh 2018] 5
(c) = 4 cos θ − 8 sin θ
r
(a) e < 1 (b) e = 1 5
(d) = 4 sin θ + 8 cos θ
(c) e > 1 (d) e = 0 r
24

ANSWERS
MULTIPLE CHOICE QUESTIONS

1. (a) 2. (d) 3. (c) 4. (d) 5. (c) 6. (c) 7. (c) 8. (c) 9. (b) 10. (d)

11. (c) 12. (c) 13. (b) 14. (b) 15. (d) 16. (a) 17. (c) 18. (a) 19. (d) 20. (b)

21. (a) 22. (c) 23. (d) 24. (a) 25. (c) 26. (a) 27. (a) 28. (b) 29. (a) 30. (c)

31. (b) 32. (c) 33. (a) 34. (b) 35. (a) 36. (a) 37. (b) 38. (d) 39. (b) 40. (d)

41. (b) 42. (a) 43. (b) 44. (d) 45. (b) 46. (d) 47. (a) 48. (b) 49. (c) 50. (b)

51. (d) 52. (a) 53. (c) 54. (c) 55. (d) 56. (b) 57. (b) 58. (c) 59. (c) 60. (a)

HINTS AND SOLUTIONS


l 1 5 3 6 3
5. The tangent of = 1 + e cosθ at (r1, θ1) is 25. = + 2 cosθ ⇒ = 1 + cosθ here l = .
r 2r 3 10r 5 10
l l
= e cos θ + cos(θ − θ1) so required tangent at 35. The director circle of = 1 + e cosθ is
r r
 r, π  l π
  is = e cos θ + cos  θ −  i.e. (1 − e 2)r 2 + 2 ler cos θ − 2 l 2 = 0 so director circle of
 2 r  2
3
l = 1 + 4 cosθ is l = 3, e=4
= e cos θ + sin θ. r
r
−15r 2 + 24r cosθ − 18 = 0 ⇒ 5r 2 − 8r cosθ + 6 = 0.
15 5 4
12. = 3 − 4 cosθ may be written as = 1 − cosθ so
r r 3 36. If e12 + e22 = 1 then e1 and e2 both are less than or
−4
e= < 1 so it is hyperbola. equal to 1. So conics are ellipse.
3
1 1 1 6 1 1
l 37. Here = + sinθ i.e. = 1 + sinθ so e = .
19. The asymptotes of = 1 + e cosθ are r 6 18 r 3 3
r
le π
= (e − 1) cos θ ± (e + 1) ⋅ sin θ so asymptotes 41.
2 2 Since cosθ lies between 0 to 1 when r < θ <
2
so it
r
1 represents an ellipse.
of = 1 + 2 cosθ are e = 1, e = 2,
r 3± 9+ 8 3± 17
2 44. 2e 2 − 3e − 1 = 0 i.e. e = = i.e.
= 3 cos θ ± 3 sin θ ⇒ 2 = r 3 ( 3 cos θ ± sin θ). 4 4
r
e > 1 so it is a hyperbola.
❍❍❍
C HAPTER Unit-II
25

4 Systems of Co-ordinates

z
CARTESIAN CO-ORDINATES SYSTEM x¢

1. In three dimension system we have three C M


non-coplanar lines X ′ OX , Y ′ OY and Z ′ OZ
L
which are called the x-axis, y-axis and z-axis
zx-plane
respectively. The planes XOY, YOZ and ZOX yz-plane
are called coordinate planes. These planes y¢
O
y
B
divide the space into eight parts which are
xy-plane
called the octants. The point O is called the
A
origin. x

2. The following table determine the signs in eight z¢


octants :

octant OXYZ OXY ′ Z OXY ′ Z ′ OXYZ ′ OX ′YZ OX ′Y ′ Z OX ′YZ ′ OX ′Y ′ Z ′


X + + + + – – – –
Y + – – + + – + –
Z + + – – + + – –

3. Let OX, OY and OZ be a rectangular set of SPHERICAL POLAR CO-ORDINATES


axes. Referred to these axes let the co-ordinates 1. Let X ′ OX , Y ′ OY and Z ′ OZ be the set of
of two points P and Q be (x 1, y 1, z 1) and rectangular axes. Let P be a point in space.
(x 2 , y 2 , z 2 ) respectively then the co-ordinates Draw PN perpendicular from P to the xy plane.
of Q referred to P as origin are Suppose OP = r , ∠ZOP = θ and ∠XON = φ
measured positively in the direction shown by
(x 2 − x 1, y 2 − y 1, z 2 − z 1).
arrows in the figure. The quantities r, θ, φ are
z z1
called the spherical polar co-ordinates of P and
Q denoted by (r , θ, φ).
z
P x1
P
y1 y'
r
θ v
O x O
x' x
φ
y
y u

Q
z'
26

2. The relations between spherical polar mx 2 + nx 1 my 2 + ny 1


x= , y= ,
co-ordinates and cartesian co-ordinates are m+n m+n
x = r cos φ sin θ, y = r sin φ sin θ, z = r cosθ,
mz 2 + nz 1
(x 2 + y 2 ) z= .
2 2
x + y + z =r , 2 2
tan θ = , m+n
2
y 2. The co-ordinates of the mid point of the join of
tan φ =
x P (x 1, y 1, z 1) and Q (x 2 , y 2 , z 2 ) are
x1 + x2 y1 + y2 z1 + z2
CYLINDRICAL CO-ORDINATES x= ,y= ,z = .
2 2 2
1. Let P be a point in space. Suppose OQ = u 3. If m : n = λ : 1 in (point 1) then the co-ordinates
∠XOQ = φ. QP = z then the quantities u, φ, z x + λx 2 y + λy 2
of the points are x = 1 ,y = 1 ,
are called cylindrical co-ordinates of P and is λ +1 λ +1
denoted by (u, φ, z ). z 1 + λz 2
z= .
z λ +1
P

m
4. If the ratio is positive, then the point R divides
n
v
x¢ O x PQ internally and if it is negative then divides
f
PQ externally.
y u
5. The co-ordinates of point R (x , y, z ) which
Q
divides the join of the line joining the points P
z¢ and Q externally in the ratio m : n are
mx 2 − nx 1 my 2 − ny 1
2. The relation between the cylindrical and x= , y= ,
m−n m−n
cartesian co-ordinates are x = u cos φ,
y mz 2 − nz 1
y = u sin φ, z = z , u 2 = x 2 + y 2 , tan φ = . z= .
x m−n

DISTANCE BETWEEN TWO POINTS CENTROID OF A TRIANGLE AND TETRAHEDRON


1. The distance between the points P (x 1, y 1, z 1) 1. The centroid of the triangle with vertices
and Q (x 2 , y 2 , z 2 ) is given by (x 1, y 1, z 1), (x 2 , y 2 , z 2 ) and (x 3 , y 3 , z 3 ) is
PQ = (x 2 − x 1)2 + (y 2 − y 1)2 + (z 2 − z 1)2  x 1 + x 2 + x 3 y1 + y2 + y3 z 1 + z 2 + z 3 
 , , .
 3 3 3 
2. The distance between the points P (x , y, z ) and
the origin O(0, 0, 0) is OP = x 2 + y 2 + z 2 . 2. The centroid of the tetrahedron with vertices
(x 1, y 1, z 1), (x 2 , y 2 , z 2 ), (x 3 , y 3 , z 3 ) and
SECTION-FORMULA (x 4 , y 4 , z 4 ) is
1. The co-ordinates of a point say R (x , y, z )  x 1 + x 2 + x 3 + x 4 y1 + y2 + y3 + y4
 , ,
dividing internally the join of two given points  4 4
P (x 1, y 1, z 1) and Q (x 2 , y 2 , z 2 ) in the ratio m : n z1 + z2 + z3 + z4
is .
4 
27

EXERCISE
MULTIPLE CHOICE QUESTIONS 10. The distance of the point (+1, − 2, + 3) from
the yz-plane is
1. The octant position of the point (1, − 2, 3) is
(a) 1 (b) 2
(a) OXYZ (b) OX ′ YZ
(c) 3 (d) –2
(c) OXY ′ Z (d) OXYZ ′
11. If (r, θ, φ) be the spherical polar co-ordinates of a
2. The distance between the points (1, 2, 3) and
point P (x, y, z) then θ is equal to
(2, 3, − 1) is
y  x 2 + y2 
(a) 3 2 (b) 2 3 (a) tan −1   (b) tan −1  
x  z 
(c) 6 (d) 0  
3. The points A(0, 1, 2), B(2, − 1, 3) and C(1, − 3, 1)  y2 + z 2 
z
forms an (c) tan −1   (d) tan −1  
x  x 
 
(a) isosceles triangle
(b) equilateral triangle 12. If point P (a, b, 7) lies on the line whose end points
(c) isosceles right angle triangle are A(1, 2, 3) and B(2, 10, 1) then b is

(d) none of these (a) –1 (b) –14


(c) –7 (d) 14
4. If A, B and C are the points A(−1, − 1, − 1), B(1, 3, 2)
13. The cylindrical co-ordinates of the point (3, 4, 5) is
and C(5, 11, 8) then the ratio in which B divides AC
4 4
is (a)  5, tan −1 , 5 (b)  3, tan −1 , 5
 3   3 
(a) 2 : 1 (b) 3 : 1
3 4
(c) 1 : 3 (d) 1 : 2 (c)  5, tan −1 , 5 (d)  5, tan −1 , 3
 4   3 
5. The equation f (x, y, z) = 0 represents a
14. The ratio in which the yz-plane divides the join of
(a) curve (b) surface the points (−2, 4, 7) and (3, − 5, 8) is
(c) line (d) circle (a) 4 : 3 (b) 3 : 2
6. In three dimension geometry x = a represents a (c) 2 : 3 (d) 3 : 4
(a) point (b) line 15. The octant position of the point (− a, − b, + c) is
(c) plane (d) cone (a) OX ′ Y ′ Z ′ (b) OXYZ
7. The locus of the point whose x-co-ordinate is a will (c) OX ′ Y ′ Z (d) OXYZ ′
be 16. The distance between the points (2, 3, 1) and
(a) yz = a (b) x = a (−1, 3, − 3) is
(a) 5 (b) 4
(c) y = a (d) z = a
(c) 5 (d) 2
8. The three mutually perpendicular plane divide the
17. The locus of the point whose z-co-ordinate is a will
space into
be
(a) three parts (b) five parts
(a) z = a (b) xy = a
(c) six parts (d) eight parts
(c) x = a (d) y = a
9. The distance of the point (a, b, c) from x-axis is
18. The distance of the point (4, 5, 6) from y-axis is
(a) b2 + c 2 (b) a (a) 39 (b) 61
2 2 2 2 (c) 52 (d) 5
(c) a + b (d) b + c
28

19. The distance of the point (2, 4, 6) from the xz-plane 28. If (r, θ, φ) be the spherical co-ordinate of the point
is : (1, 3, 6) then tanθ is
(a) 2 (b) 4 (a) 2 (b) 3 5
(c) 6 (d) 40 37 10
(c) (d)
20. The perpendicular distance of the point (a, b, c) from 3 6
the z-axis is 29. The y-co-ordinate of the point which divides the
(a) 2
a + b + c 2 2
(b) 2
a + b 2 ratio of the points A(1, 3, 2) and B(2, ,3 − 2)
externally in the ratio 2 : 3 is
(c) c (d) b2 + c 2 (a) −1 (b) 8
21. If A(2, 3, 4), B(3, − 2, 2) and C(6, − 17, − 4) are three (c) −3 (d) 3
points then the ratio in which C divides AB is 30. The distance of the point (a, b, c) from z-axis is
(a) 4 : 3 (b) 2 : 3 (a) a2 + c 2 (b) a 2 + b2
(c) 3 : 4 (d) −4 : 3
(c) b2 + c 2 (d) a 2 + b2 + c 2
22. The y-co-ordinate of the point which divides the join
of (2, 3, 4) and (3, − 4, 7) in the ratio 2 : − 4 is 31. If (u, φ, z) be the cylindrical co-ordinate of a point
P (x, y, z) in 3D-geometry then u is
(a) 21 (b) 14
(c) 10 (d) 12 (a) x 2 + y 2 + z 2 (b) x 2 + z 2

23. The centroid of the triangle whose vertices (c) x 2 + y 2 (d) y 2 + z 2

co-ordinates are (1, 0, 2), (2, 3, − 1), (3, 4, 2) is 32. If (x1, y1, z1) and (x 2, y2, z 2) be the co-ordinates of P
−7  −7 and Q with respect to origin O then the co-ordinates
(a)  −2, , 1 (b)  2, , − 1
 3   3  of Q with respect to P are
7 7 (a) (x 2, y2, z 2)
(c)  2, , 1 (d)  2, , − 1
 3   3  (b) (x1 + x 2, y1 + y2, z1 + z 2)
24. The plane x + y + z = 1 divide the points A(1, 2, 4) (c) (x1 − x 2, y1 − y2, z1 − z 2)
and B(2, 4, 5) in the ratio (d) (x 2 − x1, y2 − y1, z 2 − z1)
(a) −3 : 4 (b) 3 : 4 33. If (r, θ, φ) be the spherical polar co-ordinates of
(c) 4 : 3 (d) −4 : 3 (1, 2, 3) then θ is
 10 
25. The distance of the point (a, b, c) from the xy-plane (a) tan −1( 13) (b) tan −1  
is :  2 

(a) a (b) b  5 π
(c) tan −1   (d)
2 2  3  2
(c) c (d) a + b
34. The co-ordinates of the point which divides the join
26. If (4, φ, z) be the cylindrical co-ordinates of the point
of the points A(3, 5, 7) and (1, − 1, − 3) in equal ratio
(2, 4, 6) then tan φ is
are
(a) 3 (b) 2
(a) (1, 3, 5) (b) (2, 2, 2)
3 1
(c) (d) (c) (2, 3, 5) (d) (1, 3, 2)
2 2
35. The centroid of the triangle with vertices (1, − 1, 2),
27. The x-co-ordinate of the point which divides the
(2, 3, 5) and (3, 2, 1) is
joint of (1, 2, 3) and (3, 5, 7) in the ratio 1 : 2 is
8 8  2, −4 , 8 
5 1 (a)  2, ,  (b)  
(a) (b)  3 3  3 3
3 3
4 8 8 4
13 10 (c)  2, ,  (d)  2, , 
(c) (d)  3 3  3 3
3 3
29

36. If (u, φ, z) be the cylindrical co-ordinate of a point 45. The locus of the points which are equidistance from
P (3, 4, 5) in 3D-geometry then u is equal to the points A(1, 2, 3) and B(3, 2, 1) is
(a) 34 (b) 41 (a) x + y + z = 0 (b) x − z = 0
(c) 3 (d) 5 (c) y − z = 0 (d) x + y − z = 0
37. If (1, 2, 3) and (3, 4, 6) be the co-ordinates of P and Q 46. The perpendicular distance of the point (2, 3, 4) from
with respect to origin (0, 0, 0) then the co-ordinates OX-axis is
of Q with respect to P are (a) 5 (b) 13
(a) (−2, − 2, − 3) (b) (2, 2, 3) (c) 20 (d) none of these
(c) (1, 2, 3) (d) (3, 4, 6)
47. The plane 2x + 2y − 2z = 1 divide the points
38. The locus of the point whose y-coordinate is Q will A(2, 1, 5) and B(3, 4, 3) in the ratio
be (a) 7 : 5 (b) 7 : 3
(a) x = Q (b) y = b (c) 5 : 7 (d) 3 : 7
(c) z = b (d) xz = a
48. If the vertices of a triangle are the points (1, − 1, 2),
39. If (0, 1, 2) and (2, 3, 5) be the co-ordinates of P and Q (2, 3, 4) and (3, 2, 1) then the co-ordinates of its
with respect to origin O then y-coordinate of the centroid are
point Q with respect to P is (a) (2, 0, 1) (b) (2, 3, 4)
(a) 2 (b) −2 4 7
(c) (2, 1, 4) (d)  2, , 
(c) 1 (d) 3  3 3

40. If (r, θ, φ) be the spherical polar co-ordinates of 49. The x-co-ordinate of the point which divides the
(3, 4, 2) then r is equal to join of the points A(1, 2, 3) and B(2, 4, − 1) internally
(a) 5 (b) 2 5 in the ratio 1 : 2 is
(c) 29 (d) 13 4 3
(a) (b)
3 4
41. If (u, φ, z) be the cylindrical co-ordinates of a point
8
(3, 4, 6) then z is (c) 2 (d)
3
(a) 5 (b) 3
50. If (r, θ, φ) be the spherical co-ordinate of the point
(c) 4 (d) 6
(2, 3, 4) then φ is
42. The distance between the points (0, 0, 1) and 3
(a) tan −1(2) (b) tan −1  
(1, 2, − 1) is  2
(a) 1 (b) 2 4 π
(c) tan −1   (d)
(c) 3 (d) 2  3 2

43. The points (1, 2, 3), (2, 3, 1) and (3, 1, 2) forms a 51. If (u, φ, z) be the cylindrical co-ordinates of the point
triangle which is (1, 4, 6) then u is
(a) isosceles right triangle (a) 17 (b) 37
(b) equilateral (c) 52 (d) none of these
(c) isosceles 52. If the vertices of a triangle ABC are the points
(d) none of these A(2, − 1, 0), B (3, 3, − 3), C(0, 1, 4), then coordinates of
44. The cylindrical equation of the sphere whose centre its centroid are [Kanpur 2018]
is the origin and radius a is 5 1 −5 −1
(a)  , 1,  (b)  , 2, 
(a) u 2 + a 2 = z 2 (b) u 2 + z 2 = a 2 3 3  3 3
1 2
(c) z 2 + a 2 = u 2 (d) x 2 + y 2 + z 2 = a 2 (c) (5, 2, 1) (d)  4, , 
 3 3
30

ANSWERS
MULTIPLE CHOICE QUESTIONS

1. (c) 2. (a) 3. (c) 4. (d) 5. (b) 6. (c) 7. (b) 8. (d) 9. (a) 10. (a)

11. (b) 12. (b) 13. (a) 14. (c) 15. (c) 16. (a) 17. (a) 18. (c) 19. (b) 20. (b)

21. (d) 22. (c) 23. (c) 24. (a) 25. (c) 26. (b) 27. (a) 28. (d) 29. (d) 30. (b)

31. (c) 32. (d) 33. (c) 34. (b) 35. (c) 36. (d) 37. (b) 38. (b) 39. (a) 40. (c)

41. (d) 42. (c) 43. (b) 44. (b) 45. (b) 46. (a) 47. (c) 48. (d) 49. (a) 50. (b)

HINTS AND SOLUTIONS


4. l B 1 x 2 + y2 1+ 9 10
A C 28. tanθ = = =
(–1, –1, –1) (1, 3, 2) (5, 11, 8) z 6 6
m1x 2 + m2x1 40. r= x 2 + y2 + z 2 = 9 + 16 + 4 = 29
∵ x=
m1 + m2 45. P
A B
5λ − 1 1
So 1= ⇒λ = (1, 2, 3) (x, y, z) (3, 2, 1)
λ+1 2
Here AP = PB ⇒ (x − 1)2 + (y − 2)2 + (z − 3)2
12. l P 1
A B
= (x − 3)2 + (y − 2)2 + (z − 1)2
(1, 2, 3) (9, b, 7) (2, 10, 1)
⇒ x−z=0
m1z 2 + m2z1 3+ λ
∵ z= ⇒7 = ⇒ 6λ = − 4 47. Let the point be P (x, y, z) divide in ratio λ : 1 then
m1 + m2 λ+1 3λ + 2 4λ + 1 3λ + 5
−2 m y + m2y1 x= ,y = , z=
λ= now y= 1 2 λ+1 λ+1 λ+1
3 m1 + m2
−2
⋅ 10 + 2 l P 1
λ ⋅ 10 + 2 −14
⇒ b= = 3 = = − 14 A B
λ+1 −2 1 (2, 1, 5) (x, y, z) (3, 4, 3)
+1
3
13. Here x = 3, y = 4, z = 5 so u = x 2 + y 2 = 5,
y 4 These lies on the plane 2x + 2y − 2z = 1.
tan φ = = ,
x 3 4λ − 2 1
So =
4 λ+1 2
φ = tan −1   and x = z = 5
 3
5
⇒ 8λ − 4 = λ + 1 ⇒ 7 λ = 5 ⇒ λ =
16. d = (2 + 1)2 + (3 − 3)2 + (1 + 3)2 = 5 7
23. The centroid of triangle is y 3 3
50. tan φ = = ⇒ φ = tan −1  
 x1 + x 2 + x 3 , y1 + y2 + y3 , z1 + z 2 + z 3  x 2  2
 
 3 3 3 
1 + 2 + 3 0 + 3 + 4 2 − 1 + 2 51. u= x 2 + y 2 = 1 + 16 = 17
i.e.   7 
, ,  i.e.  2, , 1
 3 3 3   3  ❍❍❍
C HAPTER Unit-II
31

5 Direction Cosines and Projections

DIRECTION COSINES OF A LINE OR VECTOR a b


, ,
2 2 2
1. If a directed line or a vector makes angles α, β a +b +c a + b2 + c 2
2

and γ with positive directions of x-axis, y-axis c


.
and z-axis respectively then cosα, cosβ, cos γ a 2 + b2 + c 2
are called the direction cosines (d.c.) of that
directed line. These are usually denoted by l, m, 10. The direction ratios of a line PQ joining two
n. points P (x 1, y 1, z 1) and Q (x 2 , y 2 , z 2 ) are
x 2 − x 1, y 2 − y 1, z 2 − z 1 and its direction
2. If the d.c.’s of a directed line AB are l, m, n then
cosines are
the d.c.’s of the directed line BA are −l, −m, −n.
x 2 − x 1 y2 − y1 z 2 − z 1
Z , , .
B
PQ PQ PQ
A P
PROJECTIONS
γ
O
β
Y
1. Let P be a given point and AB the given straight
line. Draw PN perpendicular from P to AB,
α meeting AB in M. Then the foot M of the
X
perpendicular PM is called the projection of the
3. The direction cosines of the x-axis are 1, 0, 0. given point P on the given line AB.
Similarly the d.c.’s of the y-axis are 0, 1, 0 and P
d.c.’s of the z-axis are 0, 0, 1.
4. Suppose l, m, n are the d.c.’s of a line then any
three numbers a, b, c which are proportional to 90°
l m n A B
l, m, n i.e. = = are called direction ratios N
a b c
of that line. 2. The projection of the line joining two points
5. Direction cosines of a line are unique but P (x 1, y 1, z 1) and Q (x 2 , y 2 , z 2 ) on another line
direction ratio are not unique. whose direction cosines are l, m, n is
l (x 2 − x 1) + m (y 2 − y 1) + n (z 2 − z 1).
6. If l, m, n are the direction cosines of a line
OP (OP = r ), then the co-ordinates of P are Q
(lr , mr , nr ).
7. If l, m, n are the direction cosines of a line then
P R
l 2 + m2 + n 2 = 1
8. If α, β, γ are the angles which a line makes with
the axes, then cos2 α + cos2 β + cos2 γ = 1 90°
A B
M N
9. The direction cosines of a line whose direction
ratios are a, b, c are
32

3. If the projection of PQ on AB is zero, then PQ is 3. Two lines in point 1 are parallel if and only if
perpendicular to AB. l 1 m1 n 1
= =
4. If O and P are two points (0, 0, 0) and l 2 m2 n 2
(x 1, y 1, z 1), then the projection of OP on a line 4. If θ be the angle between two lines whose
whose direction cosines are l, m, n is direction ratios are a 1, a 2 , a 3 and b1, b2 , b3 then
lx 1 + my 1 + nz 1. a 1b1 + a 2 b2 + a 3 b3
cosθ =
5. The projection of the line joining the points a 1 + a 22 + a 32 b12 + b22 + b32
2

P (x 1, y 1, z 1) and Q (x 2 , y 2 , z 2 ) on a line whose


5. Two lines in point 4 are perpendicular iff
direction ratios are a, b, c is
(x 2 − x 1) a + (y 2 − y 1) b + (z 2 − z 1) c a 1b1 + a 2 b2 + a 3 b3 = 0
6. Two lines in point 4 are parallel iff
a 2 + b2 + c 2
a1 a2 a3
= =
ANGLE BETWEEN TWO LINES b1 b2 b3

1. If θ is the angle between two lines or vectors 7. Three points P, Q, R are collinear if direction
whose direction cosines are l 1, m1, n 1 and l 2 , ratios of PQ and RS are proportional and PQ
m2 , n 2 then cosθ = l 1l 2 + m1m2 + n 1n 2 and PR pass through P.

2. Two lines in point 1 are perpendicular if and


only if l 1l 2 + m1m2 + n 1n 2 = 0

EXERCISE
MULTIPLE CHOICE QUESTIONS 5. The angle between two diagonals of a cube is
[Avadh 2018]
1. The direction cosines of the y-axis are
−1  1  −1  1 
(a) sin   (b) cos  
(a) 1, 0, 0 (b) 0, 0, 1  3  3
(c) 0, 1, 0 (d) 0, 1, 1 1 1
(c) tan −1   (d) sec−1  
 3  3
2. If l, m, n are the direction cosines of a straight line
then l 2 + m 2 + n2 is 6. The projection of the line joining the points (1, 2, 4)
and (3, 5, 7) on a line whose direction cosines are
(a) 0 (b) 1 1, 2, 3 is
(c) 2 (d) 3 (a) 17 (b) −17 (c) 15 (d) 16
3. The direction ratios of the line joining the points 7. If ∆ DEF is the projection of ∆ ABC on a given plane
(1, 2, 3) and (2, 3, 4) are and θ be the angle between the triangle ABC plane
and then ∆ DEF is equal to
(a) 1, 1, 1 (b) 1, 2, 3
(a) ∆ ABC (b) ∆ ABCsinθ
2 3 4 1 2 3
(c) , , (d) , , (c) ∆ ABCtanθ (d) ∆ ABCcosθ
5 5 5 5 5 5
8. If α, β, γ be the angles which a line makes with
4. The length of a line whose projection on axes are 5,
positive direction of the co-ordinate axes then
10, 10 is
cos 2 α + cos 2 β + cos 2 γ is
(a) 25 (b) 5
(a) 0 (b) 1
(c) 10 (d) 15 (c) –1 (d) 2
33

9. The angle between two lines whose direction 17. The direction ratios of the x-axis are
cosines are given by l + m + n = 0, [Avadh 2018]
2 mn + 3 ln − 5 lm = 0 is (a) 1, 0, 0 (b) 0, 1, 0
π π π
(a) (b) (c) (d) π (c) 0, 0, 1 (d) 1, 1, 0
2 3 6
18. If α, β, γ are the angles which a line makes with the
10. If l1, m1, n1 and l 2, m2, n2 be the direction cosines of
axes then sin 2 α + sin 2 β + sin 2 γ is
two lines then these lines will be perpendicular if
l1l 2 + m1m2 + n1n2 is [Avadh 2018]
(a) 0 (b) 1 (a) 1 (b) 2
(c) –1 (d) 2 (c) 3 (d) 0

11. The co-ordinates of the point in which 19. If the projection of a line on co-ordinate axes are 2,
perpendicular from (0, 0, 0) to join of (−9, 4, 5) and 3, 6 then its length is
(11, 0, − 1) meets is (a) 2 (b) 3
(a) (1, 2, 1) (b) (1, 2, 2) (c) 6 (d) 7
(c) (2, 1, 1) (d) (2, 2, 1) 1 −1
20. If a, , are the direction cosines of a given line
2 2
12. If 3, 3, 3 are the direction ratios of a line then its
then the value of a is
direction cosines are
1 1
1 1 1 (a) ± (b) ±
(a) , , (b) 1, 1, 1 2 3
3 3 3
1 1
1 1 1 (c) ± (d) ±
(c) 3, 3, 3 (d) , , 2 3
3 3 3 3 3 3
21. The projection of the line AB, where the
13. If the projection of a line on the co-ordinate axes are
co-ordinates of A and B are (0, 0, 0) and (a, b, c) on a
5, 10, 10 then direction ratios of the line are
given line x = y = z is
(a) 1, 2, 3 (b) 1, 3, 2 a+ b+ c a+ b+ c
(a) (b)
(c) 1, 2, 2 (d) 1, 3, 3 3 3
l m n a+ b− c a+ b− c
14. If , , are the direction cosines of a line then (c) (d)
2 2 2 3 3
(a) l 2 + m 2 + n2 = 1 (b) l 2 + m 2 + n2 = 0 22. If the points are A(1, 1, 0) and B(0, 1, 1) then the angle
2 2 2 2 2 2 between OA and OB, where O is origin is
(c) l + m + n = 4 (d) l + m + n = 2
π π π π
15. The projection of line AB on the line CD where A, B, (a) (b) (c) (d)
2 3 4 6
C, D are (1, 2, 4), (2, 0, 1), (−1, 0, 2), (1, 2, − 1)
23. If θ be the angle between two diagonals of a cube
respectively is
then sinθ is equal to
7 7
(a) (b) 2 2
17 17 (a) (b)
3 3
7 7 2 2 2
(c) (d) (c) (d)
17 17 3 3
16. The co-ordinates of the foot of perpendicular drawn 24. The projection of a line on axes are 5, 10, 10 then its
from the point (0, 0, 0) to the line x − 1 = y = z are direction cosines are
2 1 1 2 1 1 1 2 3 2 1 3
(a)  , ,  (b)  , , −  (a) , , (b) , ,
 3 3 3  3 3 3 15 15 5 15 13 5
1 2 3 1 2 2
2 1 1 2 1 1 (c) , , (d) , ,
(c)  , − , −  (d)  − , ,  15 15 5 15 15 15
 3 3 3  3 3 3
34

25. If 1, –1, 0 and –1, 1, 2 are direction cosines of the 33. The sum of the direction cosines of a line whose
two lines then the direction ratios of the line direction ratios are 2, 3, –6 is
perpendicular to both are 1
(a) (b) 1
(a) 2, 2, 1 (b) –2, –2, –1 7
(c) –2, –2, 0 (d) –2, +2, 1 1
(c) − (d) 0
26. The projection of a line on the plane is 7
(a) a point (b) a line 34. If the length of a line OP is r and l, m, n are the
(c) a plane (d) none of these direction cosines of the line OP then the
27. The straight lines whose direction cosines are given co-ordinates of P are
by the relations al + bm + cn = 0 and l m n
fmn + gln + hlm = 0 are perpendicular if (a) (lr 2, mr 2, nr 2) (b)  , , 
r r r
a b c
(a) + + = 0 (b) af + bg + ch = 0 r r r
f g h (c)  , ,  (d) (lr, mr, nr)
l m n
f g h a b c
(c) + + = 0 (d) = =
a b c f g h 35. If O and P are the points (0, 0, 0) and (x, y, z) then the
projection of OP on a line whose direction cosines
28. The direction ratios of the line which is equally
are l, m, n is
inclined to the axes are
(a) 1, 1, 1 only (b) –1, –1, –1 only (a) x l + y m + z n (b) l 2x + m 2y + n2z

(c) ±1, ±1, ±1 (d) 1, 2, 3 x y z


(c) lx + my + nz (d) + +
l m n
29. The condition that the two lines whose direction
36. The condition that the two lines whose direction
ratios are a, 0, 1 and 2, b, c are perpendicular is
ratios are a, −1, 1 and 1, 3, c are perpendicular is
(a) 2a + b + c = 0 (b) 2a + c = 0
(a) 2a + c = 1 (b) a + 3c = 2
(c) a + b + c = 0 (d) 2a − c = 6
(c) a − c = 3 (d) a + c = 3
30. If the vertices A, B, C of a triangle have co-ordinates
37. If the projection of a line PQ on another line is zero
(2, 3, 5), (−1, 3, 2) and (3, 5, − 2) respectively then
then PQ is always
angle B is
(a) parallel to AB
 2 1 
(a) cos −1   (b) cos −1  
 3  3 (b) perpendicular to AB

2  π (c) having some angle to AB


(c) cos −1   (d)
 3 2 (d) none of these
31. The projection of a line on axes are 2, 4, 6 then the 38. The projection of a line joining the points (1, 0, 2)
length of the line is and (2, 3, 4) on a line whose direction ratios are 1, 1,
(a) 2 (b) 4 1 is
(c) 14 (d) 2 14 (a) 2 (b) 3
32. The perpendicular distance of a point (1, 0, 2) from a (c) 2 3 (d) 3 2
line through origin and whose d.c.’s are 1, 1, 0 is
39. If the d.c.’s of a line are 2, 3, 1 then which one is not
3 1 its direction ratios
(a) (b)
2 2
(a) 4, 6, 2 (b) 12,18, 6
3
(c) 3 (d) (c) 6, 9, 2 (d) 14, 21, 7
2
35

40. The sum of the direction cosines of a line whose 47. If O, A, B are the points (0, 0, 0), (3, 3, 6), (3, 5, − 1)
direction ratios are 1, 2, 3 is respectively then the angle between OA and OB is
1 2 π
(a) (b) (a) 0 (b)
14 14 3
3 6 π π
(c) (d) (c) (d)
14 14 4 2

41. If the length of a line OP through the origin O is 4 48. If the vertices A, B and C of a triangle ABC have
1 1 1 co-ordinates (2, 3, 5), (−1, 3, 2) and (3, 5, − 2)
and , , are the direction cosines of the line
3 3 3 respectively then angle B is
OP then the co-ordinates of P are π
(a) 0 (b)
4 4 4  4
(a) (4 3, 4 3, 4 3) (b)  , ,  π π
 3 3 3 (c) (d)
2 3
1 1 1 
(c)  , ,  (d) (12, 12, 12)
 4 3 4 3 4 3 49. The projections of a line on co-ordinate axes are 1,
4, 5 then its length is
42. The angle between the lines whose direction cosines
(a) 1 (b) 4
are given by the equations l + m + n = 0 and
(c) 5 (d) 42
l 2 + m 2 − n2 = 0 is [Avadh 2018]
50. If two lines whose direction ratios are a, 2, 4 and −1,
π π
(a) (b) 2, 2 are perpendicular then a is
2 3
(a) 4 (b) 8
π π
(c) (d) (c) 9 (d) 12
6 4
51. If a line makes equal angle with axes then its
43. If the points are A(1, 2, 3) and B(2, 1, 2) then the
direction ratios are
angle between OA and OB, where O is origin is
(a) 1, 1, 1 (b) 1, 0, 0
10  10
(a) cos −1   (b) cos −1   (c) 1, 1, 0 (d) 0, 1, 1
 14   3
1 1 1
(c) cos −1 
10 
 (d) cos −1 
10 
 52. If the direction cosines of a line are  , ,  then a
 6 14   3 14   a a a
is
44. The co-ordinates of the foot of perpendicular drawn
(a) ± 3 (b) 3 only
from the point (1, 2, 1) to the line joining (1, 4, 6) and
(c) − 3 only (d) 1
(5, 4, 4) are
53. If a, 0, 0 are the d.c.’s of a line then a is
(a) (3, 5, 4) (b) (3, 4, 5)
(c) (4, 3, 5) (d) (5, 4, 3) (a) +1 (b) –1

45. If A(6, 3, 2), B(5, 1, 4), C(3, − 4, 7), D(0, 2, 5) are four (c) 0 (d) 1 or –1
points then the projection of AB on CD is 54. The condition that the two lines whose d.c.’s are 0, a,
[Kanpur 2018] 1 1 1
and , b, are perpendicular is
13 12 15 11 2 3 2
(a) (b) (c) (d)
7 7 7 7 (a) 4ab + 1 = 0 (b) ab − 1 = 0
46. The direction cosines of a segment of a line whose (c) 2ab + 1 = 0 (d) 4ab − 1 = 0
projections on the axes are 2, 3, 6 are given by
55. If the mid points of a triangle are (3, 0, 1), (1, 2, − 3)
2 3 4 3 5 6
(a) , , (b) , , and (−1, 1, − 4) then its centre is
7 7 7 7 7 7
2 4 5 2 3 6 (a) (1, 1, 2) (b) (1, − 1, 2)
(c) , , (d) , ,
7 7 7 7 7 7 (c) (1, 1, − 2) (d) (−1, 1, 2)
36

56. If a variable line in two adjacent positions has d.c.’s 61. The direction cosines of a line equally inclined with
l, m, n and l + δl, m + δm, n + δn the small angle coordinate axes are
between them is δθ then (δθ)2 is [Avadh 2018]

(a) −(δl)2 − (δm)2 − (δn)2 (a) 1, 1, 1 (b) 1, 0, 0


1 1 1
(b) (δl)2 + (δm)2 + (δn)2 (c) , , (d) 0, 1, 0
3 3 3
(c) (δl)2 + (δm)2 − (δn)2 62. If the direction ratio of two lines intersect at
π
are
4
(d) −(δl)2 − (δm)2 + δn2
2, − 1, 2 and p, 3, 5 then the value of p is
57. The direction cosines of the line segment joining the (a) 1 (b) 2 (c) 4 (d) 8
points P(−2, 1, − 8) and Q(4, 3, − 5) are
63. If a line makes angle α, β, γ, δ with four diagonal of a
[Kanpur 2018]
cube, then sin 2 α + sin 2 β + sin 2 γ + sin 2 δ is equal
5 2 3 4 3 2
(a) , , (b) , , to
7 7 7 7 7 7
6 2 3 5 6 3 4 7 10 8
(c) , , (d) , , (a) (b) (c) (d)
7 7 7 7 7 7 3 3 3 3

58. The projection of join of points (0, 0, 0) and (1, 1, 1) on 64. The triangle formed by the vertices (a, b, c), (b, c, a)
1 1 1 and (c, a, b) is
a straight line with direction cosines , , is
3 3 3 (a) right angle triangle only
[Avadh 2018]
(b) isoscale triangle only
1
(a) 1 (b) (c) equilateral triangle only
3
(d) right angle isoscale triangle
(c) 3 (d) 3 3
65. The direction cosines of the line whose equations
59. The direction cosines of the line joining two points
are x + y = 3 and x + y + z = 0 are
(0, 1, 0) and (2, 1, 1) are
[Kanpur 2018]
[Avadh 2018]
1 1 1 1
1 1 1 (a) 0, ,− (b) 0, ,−
(a) , , (b) 1, 0, 1 2 2 3 3
3 3 3
1 1 1 1
2 1 1 2 (c) 1, ,− (d) 1, − ,
(c) , 0, (d) , 0, 2 2 3 3
5 5 5 5
66. The direction cosines of the line joining the points
60. If α, β, γ be the angles which a line makes with the
(1, 0, 1) and (0, 1, 0) are [Agra 2017]
positive directions of the axes, then the value of
1 1 1 1 1 1
cos 2 α + cos 2 β + cos 2 γ is [Avadh 2018] (a) , , (b) ,− ,
3 3 3 3 3 3
(a) −1 (b) +1
1 1 1
(c) , ,− (d) None of these
(c) −2 (d) +2 3 3 3
37

ANSWERS

MULTIPLE CHOICE QUESTIONS


1. (c) 2. (b) 3. (a) 4. (d) 5. (b) 6. (a) 7. (d) 8. (c) 9. (a) 10. (a)

11. (b) 12. (a) 13. (c) 14. (c) 15. (d) 16. (c) 17. (a) 18. (b) 19. (d) 20. (c)

21. (a) 22. (b) 23. (d) 24. (d) 25. (c) 26. (b) 27. (c) 28. (c) 29. (b) 30. (d)

31. (d) 32. (d) 33. (c) 34. (d) 35. (c) 36. (d) 37. (b) 38. (c) 39. (c) 40. (d)

41. (b) 42. (b) 43. (d) 44. (b) 45. (a) 46. (d) 47. (d) 48. (c) 49. (d) 50. (d)

HINTS AND SOLUTIONS


4. Given that lr = 5, mr = 10, nr = 10 ⇒ sin 2 α + sin 2 β + sin 2 γ = 2
So l 2r 2 + m 2r 2 + n2r 2 = 25 + 100 + 100 11. d.r’s of AB is 20, −4, −6
⇒ r 2 = 225 ⇒ r = 15 x+ 9 y−4 z−5
Equation of line AB is = = =r
20 −4 −6
5. The direction ratios of the diagonals are 1, −1, −1
and 1, 1, −1 so angle between them is Then point P is (20r − 9, − 4r + 4, − 6r + 5)
1−1+ 1 1 1
cosθ = = i.e. θ = cos −1   O(0, 0, 0)
3 3 3  3
6. Required projection is
l (x 2 − x1) + m (y2 − y1) + n(z 2 − z1)
1(3 − 1) + 2(5 − 2) + 3(7 − 4)
A B
= 2 + 6 + 9 = 17 P (11, 0, –1)
(–9, 4, 5)

(3, 5, 7) = (x2, y2, z2)


, z 1) So direction ratio of OP is 20r − 9, −4r + 4, −6r + 5
= (x 1, y 1
, 4)
(1 , 2 Now OP and AB are perpendicular so
20(20r − 9) − 4(−4r + 4) − 6(−6r + 5) = 0
113
After solving we get r =
46
d.c.’s 1, 2, 3 = (l, m, n) Put in P we get P (1, 2, 2)

8. 2 2
cos α + cos β + cos γ = 1 − 2 sin α + 2 2 16. Let the point P is given by x − 1 = y = z = r

1 − 2 sin 2 β + 1 − 2 sin 2 γ O (0, 0, 0)

= 3 − 2(sin 2 α + sin 2 β + sin 2 γ) x–1 y z


= =
= 3 − 2(2) = − 1 1 1 1
∵ cos 2 α + cos 2 β + cos 2 γ = 1 P

⇒ 1 − sin 2 α + 1 − sin 2 β + 1 − sin 2 γ = 1


38

So P (r + 1, r, r). Thus direction of P are r + 1, r, r and 1 1


direction ratio of given line are 1, 1, 1 but both are = + + 4
4 4
perpendicular so r + 1 + r + r = 0 ⇒ 3r = − 1 ⇒
−1 3
r= i.e. AP =
3 2
2 −1 −1 
So required co-ordinate of P is  , , .
3 3 3  36. (a) ⋅ (1) + (−1)(3) + 1 ⋅ c = 0 ⇒ a + c = 3
25. If a, b, c be the required direction ratios of 40. 1 2 3
Since d.r’s are 1, 2, 3 so its d.c.’s are , ,
perpendicular line then a − b + c = 0 and 14 14 14
a b c 6
− a + b + 2c = 0 then = = and their sums is .
−2 −2 0 14
⇒ a : b: c = − 2: − 2: 0
27. The given relations are al + bm + cn = 0 and 46. Let l, m, n are the required d.c.’s then given that
fmn + gln + hlm = 0 lr = 2, mr = 3, nr = 6 so
−(al + bm)
n= put in other equation
c (lr)2 + (mr)2 + (nr)2 = 22 + 32 + 62 = 49 i.e. r = 7
(al + bm)
− hlm + ( fm + gl) =0 2 3 6
c So required d.c.’s are , , .
7 7 7
agl 2 + bfm 2 + (af + bg − ch)lm = 0
2
48. d.r.’s of BA are 3, 0, 3 and d.r.’s of BC are 4, 2, –4. If
l l
i.e. ag   + (af + bg − ch)   + bf = 0 θ be the angle between BC and BA then
m m
12 + 0 − 12 π
If l1,m1, n1 and l 2,m2, n2 are the d.c’s of two lines then cosθ = = 0 ⇒θ =
l1l 2 cbf ll mm nn a + 0 + 9 16 + 4 + 16 2
= i.e. 1 2 = 1 2 = 1 2
m1m2 cag cbf cag abh 1 1 1 1 1 1
52. Since d.c.’s are , , so + + =1
2 2
The lines will be perpendicular if a a a a a a2
l1l 2 + m1m2 + n1n2 = 0
f g h i.e. a 2 = 3 i.e. a = ± 3
i.e. cbf + cag + abh = 0 i.e., + + = 0
a b c 56. cos δθ = l (l + δl) + m (m + δm) + n(n + δn)
29. a1a2 + b1b2 + c1c 2 = 0
θ
i.e. 2a + 0b + c = 0 ⇒ 2a + c = 0 1 − 2 sin 2 δ = 1 + (l δl + m δm + nδn)
x y z 2
32. Equation of line OB is = = = r
1 1 0 θ
i.e. −2 sin 2 δ = l δl + m δm + nδn …(1)
2
A(1, 0, 2)
Also l 2 + m 2 + n2 = 1

and (l + δl)2 + (m + δm)2 + (n + δn)2 = 1

O B Subtracting these two equations we get


P d.c.’s 1, 1, 0
(0, 0, 0)
δl 2 + δm 2 + δn2 = − (2l δl + 2m δm + 2nδn) …(2)

So co-ordinate of P is (r, r, 0) d.r’s of AP is r − 1, By (1) and (2)


r, − 2 which is perpendicular to d.r’s of OB i.e. θ 1
−2 sin 2 δ = − (δl 2 + δm 2 + δn2)
1(r − 1) + 1(r) + 0(−2) = 0 2 2
1 θ θ
⇒ r= Since δθ is small so sinδ =δ
2 2 2
1 1 2
So P is  , , 0 . Thus δθ 1
We get −2   = − (δl 2 + δm 2 + δn2)
2 2   2 2
2 2
AP =  1 − 1 +  1 − 0 + (−2)2
    i.e. δθ2 = δl 2 + δm 2 + δn2
2  2 
❍❍❍
C HAPTER Unit-II
39

6 The Plane

PLANE x y z 1
1. A plane is a surface such that every straight line x1 y1 z1 1
joining any two points on it lies wholly on it. =0
x2 y2 z2 1
2. A straight line which is perpendicular to every
line lying in a plane is called a normal to that x3 y3 z3 1
plane. All the normals to a plane are parallel 11. The condition that four points (x 1, y 1, z 1),
lines. (x 2 , y 2 , z 2 ), (x 3 , y 3 , z 3 ) and (x 4 , y 4 , z 4 ) are
3. The equation of a plane whose perpendicular coplanar is
distance from the origin is p and cosα, cosβ, x1 y1 z1 1
cos γ are direction-cosines of this
perpendicular, is x cos α + y cos β + z cos γ = p x2 y2 z2 1
=0
i.e. lx + my + nz = p. This is known as the x3 y3 z3 1
equation of a plane in normal form.
x4 y4 z4 1
4. The general equation of a plane is
ax + by + cz + d = 0. Here the coefficients a, b, 12. The equation of the plane parallel to the yz-
c of x, y and z are direction ratios of normal to plane (respectively xz-plane, xy-plane) and at a
this plane. distance a from it is x = a (respectively y = a,
5. The number of arbitrary constants in the z = a).
general equation of the plane are three. 13. The equation of the plane perpendicular to the
6. The equation of any plane passing through the x-axis (respectively y-axis and z-axis) is x = a
origin is ax + by + cz = 0. (respectively y = a and z = a).

7. If the plane makes intercepts a, b and c on the 14. The equation of plane parallel to x-axis
axes of x, y and z respectively, then the (respectively y-axis and z-axis) is
x y z by + cz + d = 0 (respectively ax + cz + d = 0
equation of the plane is + + = 1 and ax + by + d = 0).
a b c
8. The equation of xy-plane is z = 0, the equation 15. The angle between two planes is defined as the
of yz-plane is x = 0 and the equation of angle between their normal drawn from any
zx-plane is y = 0. point to the planes.

9. The equation of a plane through a given point 16. If the equation of the plane is
A (x 1, y 1, z 1) and perpendicular to a line whose ax + by + cz + d = 0 and A (x 1, y 1, z 1) and
direction ratios are a, b, c is B (x 2 , y 2 , z 2 ) be any two points.
a (x − x 1) + b (y − y 1) + c (z − z 1) = 0. (i) If the points A and B lie on the same side of
10. The equation of a plane which passes through the plane then ax 1 + by 1 + cz 1 + d and
three points whose co-ordinates are (x , y , z ), ax 2 + by 2 + cz 2 + d have the same signs.
1 1 1
(x 2 , y 2 , z 2 ) and (x 3 , y 3 , z 3 ) is (ii) If the points A and B lie on opposite sides
of the plane then ax 1 + by 1 + cz 1 + d and
ax 2 + by 2 + cz 2 + d have opposite signs.
40

17. The length of the perpendicular from the point a 1x + b1y + c1z + d 1 a 2 x + b2 y + c2 z + d 2
=−
(x 1, y 1, z 1) to a given plane a 12 + b12 + c12 a 22 + b22 + c22
ax + by + cz + d = 0 is
5. If in (point 2), the constant terms d 1 and d 2 are
ax 1 + by 1 + cz 1 + d
± of the same sign then :
a 2 + b2 + c 2
(i) If a 1a 2 + b1b2 + c1c2 < 0, then the angle
18. The distance between two planes first take a between the planes in which the origin lies
point on one plane and then find the length of is acute.
perpendicular from this point to the other (ii) If a 1a 2 + b1b2 + c1c2 > 0, then the angle
plane. between the planes in which the origin lies
19. The equation of any plane passing through the is obtuse.
line of intersection of two planes P = 0 and 6. The equation
Q = 0 is P + λQ = 0 where λ is a parameter.
ax 2 + by 2 + cz 2 + 2 fyz + 2gzx + 2hxy = 0
20. The condition that the line whose d.c.’s are l, m,
represents a pair of lines if
n is parallel to the plane ax + by + cz + d = 0 is
al + bm + cn = 0. abc + 2 fgh − af 2 − bg 2 − ch 2 = 0.
21. The condition that the line whose d.c.’s are l, m, 7. The angle between two planes defined in
n is perpendicular to the plane (point 6) is
a b c  2 f 2 + g 2 + h 2 − ab − bc − ac 
ax + by + cz + d = 0 is = = .
l m n tan −1  .
 a+ b+ c 
 
THE ANGLE BETWEEN A LINE AND A PLANE
8. Two planes defined in (point 6) are
1. The angle between a line and a plane is defined
perpendicular if a + b + c = 0.
to be the complement of the angle between the
line and the normal to the plane. PROJECTION ON A PLANE AND AREA OF A
2. The equation of the planes bisecting the angles TRIANGLE
between the planes a 1x + b1y + c1z + d 1 = 0 1. If the projection of an area A on the co-ordinate
and a 2 x + b2 y + c2 z + d 2 = 0 are planes yz, zx and xy by Ax, Ay and Az
a 1x + b1y + c1z + d 1 respectively, then
a 12 + b12 + c12 A 2 = Ax 2 + Ay 2 + Az 2
a 2 x + b2 y + c2 z + d 2 2. The area of a triangle ABC, the co-ordinates of

a 22 + b22 + c22 whose vertices are A (x 1, y 1, z 1), B (x 2 , y 2 , z 2 )
and C (x 3 , y 3 , z 3 ) is ∆ where
3. The equation of the plane (see point 2)
∆ = ∆x 2 + ∆y 2 + ∆z 2 and
bisecting the angle in which the origin lies is
a 1x + b1y + c1z + d 1 a 2 x + b2 y + c2 z + d 2 y1 z1 1 x1 z1 1
= .
1 1
a 12 + b12 + c12 a 22 + b22 + c22 ∆x = y2 z2 1 , ∆y = x2 z2 1 ,
2 2
4. The equation of the plane (see point 2) y3 z3 1 x3 z3 1
bisecting the angle in which the origin does not
lies is
41

x1 y1 1 x1 y1 z1 1
1
∆z = x2 y2 1 . 1 x2 y2 z2 1
2 V= .
x3 y3 1 6 x3 y3 z3 1

x4 y4 z4 1
VOLUME OF A TETRAHEDRON
1. The volume of the tetrahedron ABCD, whose 2. If V is the volume of the tetrahedron OABC
vertices are the points A (x 1, y 1, z 1), whose vertices are the points O(0, 0, 0),
B (x 2 , y 2 , z 2 ), C (x 3 , y 3 , z 3 ) and D (x 4 , y 4 , z 4 ) A (x 1, y 1, z 1), B (x 2 , y 2 , z 2 ) and C (x 3 , y 3 , z 3 )
x 1 y1 z 1
is
1
then V = x 2 y2 z 2 .
6
x 3 y3 z 3

EXERCISE
MULTIPLE CHOICE QUESTIONS 7. The angle between the planes 2x − y + z = 7 and
x + y + 2z = 9 is
1. The equation of the co-ordinate plane xy is
π π π
(a) x = 0 (b) y = 0 (a) (b) (c) (d) 0
2 3 4
(c) xy = 0 (d) z = 0
8. The equation of the plane which makes intercepts 1,
2. The number of arbitrary constant in the general 2, 3 on the co-ordinate axes is
equation of the plane ax + by + cz + d = 0 are
(a) x + 2y + 3z = 6 (b) 2x + y + 3z = 6
(a) 1 (b) 2 (c) 3 (d) 4
(c) 6x + 3y + 2z = 6 (d) none of these
3. The equation of the plane through (2, 0, 1) and
9. The equation of the plane through A(2, 3, − 1) at
perpendicular to a plane whose d.r. are 1, 1, 1 is
right angle to OA, where O is origin is
(a) x + y + z = 3 (b) x + y + z = 2
(a) 2x + 3y − z = 14 (b) 2x + 3y − z = 12
(c) x + y + z = 0 (d) 2x + z = 1
(c) 2x + 3y + z = 14 (d) 2x + 3y + z = 12
4. The equation of the plane perpendicular to z-axis
10. The equation of the plane which makes equal
is :
intercepts on the axes and pass through the point
(a) x = a (b) y = a (2, 3, 4) is
(c) z = a (d) xy = a (a) x + y + z = 1 (b) x + y + z = 5
5. ax + cz + d = 0 always represents a plane parallel (c) x + y + z = 7 (d) x + y + z = 9
to
11. The distance between the planes x + 2y + z = 1 and
(a) x-axis (b) y-axis 2x + 4y + 2z = 2 is
(c) z-axis (d) any axis (a) 1 (b) 2 (c) 0 (d) 3
6. The length of the perpendicular from the point 12. The foot of the perpendicular from the origin to a
(1, 2, 3) to a plane x − 2y + 2z = 0 is plane is (2, 1, 3), then the equation of the plane is
(a) 0 (b) 2 (a) 2x + y + 3z = 14 (b) x + 2y + 3z = 14
(c) 3 (d) 1 (c) 3x + 2y + z = 14 (d) x + 3y + 2z = 14
42

13. The position of the points (1, − 1, 3) and (3, 3, 3) from 21. a (x − α) + b(y − β) + c (z − γ) = 0 represents a
the plane 5x + 2y − 7 z + 9 = 0 is plane passing through the point
(a) equidistance and same side (a) (a, b, c) (b) (α, β, r)
(b) only equidistance  a b c
(c) (aα, bβ, cγ) (d)  , , 
(c) only opposite side α β γ 
(d) equidistance and opposite side 22. The equation of the plane passing through the
14. The equation of the plane passing through the line origin and parallel to the plane x − 3y + 4z + 2 = 0
of intersection of the planes x + y + z = 1, is
2x − y + 2z = 2 and the point (0, 0, 2) is (a) x − 3y + 4z = 0 (b) x − 3y + 4z = 2
(a) x + y + z = 3 (b) x = 0 (c) x − 3y + 4z = 4 (d) x − 3y + 4z = − 2
(c) z = 0 (d) y = 0 23. The area of a triangle included between the plane
15. If d1 and d2 are both positive and the origin lies in 3x − 4y + z = 12 and the coordinate planes is
the obtuse angle between the two planes (a) 6 26 (b) 2 26 (c) 26 (d) 3 26
a1x + b1y + c1z + d1 = 0 and 24. The equation of the plane perpendicular to x-axis
a2x + b2y + c 2z + d2 = 0 then the value of is :
a1a2 + b1b2 + c1c 2 is (a) x = a (b) y = a
(a) positive (c) z = a (d) yz = 1
(b) negative 25. The equation of the plane through (0, 1, 0) and
(c) 0 perpendicular to a line whose d.r. are 0, 1, 0 is
(d) may be positive or negative (a) z = 1 (b) y = 1

16. The equation of the plane through P (a, b, c) and (c) x = 1 (d) x + z = 1
perpendicular to OP is 26. The volume of the tetrahedron formed by the planes
2 2 2 x + y + z = 1, x + y = 0, y + z = 0 and z + x = 0 is
(a) ax + by + cz = a + b + c
2 3
(b) ax + by + cz = 1 (a) (b) (c) 1 (d) 0
3 2
(c) ax + by + cz = a 2 + b2 + c 2 27. The equation of the plane through the line of
(d) ax + by + cz = 0 intersection of the planes x + y + z + 1 = 0 and
17. The intercepts form of the plane is − x + y + z − 1 = 0 and parallel to x-axis is

(a) lx + my + nz = p (b) ax + by + cz + d = 0 (a) 2y + 2z = 1 (b) y + z = 1


x y z x y z (c) 2y + z = 1 (d) y + z = 0
(c) = = (d) + + =1
a b c a b c 28. The equation of the plane passes through the z-axis
18. The length of the perpendicular drawn from the is
origin to the plane x + 4y − 8z + 18 = 0 is (a) x + λy = 0 (b) x − λy = 1
(a) 2 (b) 3 (c) 4 (d) 6 (c) z = 0 (d) z = 1
19. The equation of the plane parallel to the xy-plane 29. The condition that a line whose d.r.s are l, m, n is
and at a distance a from it is perpendicular to the plane ax + by + cz + d = 0 is
(a) x = a (b) y = a a b c
(a) al + bm + cn = 0 (b) = =
(c) xy = a (d) z = a l m n
(c) al + bm + cn = 1 (d) none of these
20. The intercept made by the plane ax + by + cz = d
on x-axis is 30. The angle between the planes x = a and y = b is
d d a d π π π
(a) (b) (c) (d) (a) 0 (b) (c) (d)
b c d a 3 2 6
43

31. The equation of a plane whose perpendicular 40. The angle between the normals to the planes
distance from the origin is 2 and 1, −2, 2 are the d.r’s 2x − y + z = 13 and x + y + 2z = 9 is
of this perpendicular, is π π π
(a) (b) π (c) (d)
(a) x − 2y + 2z = 0 (b) x − 2y + 2z = 2 2 3 4
(c) x − 2y + 2z = 3 (d) x − 2y + 2z = 6 41. The bisector of the acute angle between the planes
32. The equation of the yz-plane at a distance a from the 2x − y + 2z + 3 = 0 and 3x − 2y + 6z + 8 = 0 is
origin is (a) x − 13y + 32z = 0
(a) y = a (b) z = a (b) 23x − 13y + 32z + 45 = 0
(c) x = a (d) y + z = a (c) 23x − y + 32z = 0
33. The equation of the plane which contains the line of (d) none of these
intersection of the planes 3x − y + 2z + 1 = 0 and 42. The area of the triangle whose vertices are (1, 0, 0),
− x + y + 2z + 2 = 0 and which is perpendicular to (0, 1, 0) and (0, 0, 1) is
the plane x + y + z = 0 is 3 3
(a) 3 3 (b) (c) 3 (d)
(a) 7 x − 4y − 3z + 4 = 0 4 2
(b) 11x − 3y − 4z − 4 = 0 43. The area of the triangle included between the plane
(c) 6x + y − 7 z + 1 = 0 2x − 3y + 4z = 12 and the co-ordinate planes is
(d) 2x + 4z + 3 = 0 (a) 2 29 (b) 5 29
34. The perpendicular distance between the planes (c) 2 29 (d) 4 29
x + y + z = 0, 3x + 3y − 3z + 2 = 0 is 44. The angle between the planes formed by
3 3 3 2 2
(a) (b) (c) (d) x 2 + 4y 2 − z 2 + 4xy = 0 is
2 2 3 3 3
5 5 
35. The equation of the plane through the line of (a) tan −1   (b) tan −1  
 2  2
intersection of the planes x + 1 = 0, y + 3 = 0 and
 5  5
passing through the origin is (c) tan −1   (d) tan −1  
 2   2
(a) x + y = 2 (b) x − y + z = 1
(c) x + y − z = 2 (d) x − y = 2 45. The volume of the tetrahedron whose vertices are
the points (0, 0, 0), (1, 1, 1), (1, 2, 3), (2, 5, 6) is
36. The equation of the plane parallel to y-axis does not
1 1 1
contain the variable (a) (b) (c) (d) 0
2 3 6
(a) x (b) y
46. The planes represented by
(c) z (d) both x and z
ax 2 + by 2 + cz 2 + 2 fyz + 2gzx + 2hxy = 0 will be
37. All normals to a plane are always at right angles if
(a) perpendicular (b) intersect (a) a + b + c = 1 (b) abc = 0
(c) parallel (d) none of these 1 1 1
(c) + + = 0 (d) a + b + c = 0
a b c
38. In 3-dimension geometry z = a represents a
47. The equation of the plane perpendicular to z-axis is
(a) plane (b) line (a) x = a (b) y = a
(c) point (d) sphere (c) z = a (d) x + y = a
39. The plane ax + cz + d = 0 is parallel to 48. The area of a triangle whose vertices are given by
the co-ordinates (0, 0, 0), (0, 1, 0) and (0, 0, 1) is
(a) x-axis (b) y-axis
1 1
(a) 1 (b) (c) (d) 2
(c) z-axis (d) yz-plane 2 2 2
44

49. In 3-dimension geometry y = a represents a plane 58. The angle between the planes
parallel to x 2 + 4xy + 4y 2 − z 2 = 0 is formed by
(a) xy-plane (b) yz-plane 3 2
(a) cos −1   (b) cos −1  
(c) xz-plane (d) none of these  2  3
4 3
50. The number of arbitrary constants in the intercept (c) cos −1   (d) cos −1  
form of the plane is  3  4

(a) 4 (b) 3 59. The angle between the planes 2x − y + z = 7 and


(c) 2 (d) 1 x + y + 2z = 9 is
π π
51. The planes x − y + z = 7 and 3x + 2y − z + 9 = 0 (a) (b)
3 2
are
π π
(a) parallel (b) intercept each other (c) (d)
4 6
(c) perpendicular (d) none of these 60. The distance between two planes
52. If the area of projections of ∆ ABC on the 2x − 2y + z + 1 = 0 and 4x − 2y + 2z + 3 = 0 is
co-ordinate planes yz, zx and xy are respectively Ax, 1 5
(a) (b)
Ay and Az then the area of ∆ ABC is 3 6
(a) Ax + Ay + Az (b) Ax 2 + Ay 2 + Az 2 7 1
(c) (d)
6 6
(c) Ax 2 + Ay 2 + Az 2 (d) Ax + Ay + Az 61. If the plane x + y + z = 5 intersect the co-ordinate
53. If the foot of the perpendicular drawn from the axes then the area of triangle formed by them is
origin to a plane is (12, − 4, 3) then the plane is 25 25 3
(a) (b)
(a) 4x − 3y + z = 103 2 2

(b) 12x − 4y + 3z = 169 15 3 5 3


(c) (d)
2 2
(c) 12x − 4y + 3z = 103
62. The equation of the plane passing through (2, 4, 6)
(d) 4x − 3y + z = 169
and makes equal intercepts on co-ordinate axes is
54. The distance between the planes
(a) 2x + 3y + 2z = 12 (b) x + 3y + 2z = 12
4x − 2y + 2z + 3 = 0 and 2x − 2y + z + 1 = 0 is
(c) x + y + z = 12 (d) 2x + 3y + z = 12
2 1 1 1
(a) (b) (c) (d) 63. The equation of the plane passing through the
3 2 3 4
points (1, 2, 1), (1, 1, 0) and (−2, 2, − 1) is
55. If the planes represented by
(a) 2x + 3y − 3z = 5 (b) 2x + 3y + 3z = 5
x 2 − 3y 2 + az 2 + 2yz + 3zx + xy = 0 will be right
(c) 2x − 3y + 3z = 5 (d) −2x + 3y + 3z = 5
angle then a is
64. The angle between the planes 3x − 4y + 5z = 0 and
(a) 6 (b) 3
2x − y − 2z = 5 is [Kanpur 2018]
(c) 2 (d) 0 π π
(a) (b)
56. The distance of the point (1, 2, 0) from the plane 3 6
4x + 3y + 12z + 16 = 0 is π π
(c) (d)
2 4
(a) 2 (b) 4
65. The angle between the planes x + y = z = 1 and
(c) 12 (d) 16
2x − y + z = 2 is [Avadh 2018]
57. The image of the point (3, 5, 7) in the plane π π
2x + y + z = 6 is (a) (b)
2 3
(a) (1, 3. 5) (b) (1, − 3, 5)  2 2
(c) cos −1   (d) cos −1  
(c) (1, − 3, − 5) (d) (−5, 1, 3)  3   3
45

66. The length of perpendicular from the point (1, 2, 3) to 69. The equation of plane passing through the point
the plane x + y − z + 5 = 0 is [Avadh 2018] (1, 2, 1) and perpendicular to the line joining the
11 6 points (1, 4, 2) and (2, 3, 5) is [Kanpur 2018]
(a) (b)
3 3 (a) x − y + 3z = 2 (b) x + y − 2z = 0
5
(c) (d) none of these (c) x + y + 2z = 0 (d) 2x + y + 3z = 0
3
70. The image of the point (1, 3, 4) in the plane
67. A plane meets the coordinate axes at A, B, C such
2x − y + z + 3 = 0 is [Agra 2017]
that the centroid of the triangle ABC is the point
(a, b, c), then the equation of the plane ABC is (a) (3, 5, 2) (b) (−3, 5, 2)

[Kanpur 2018] (c) (3, − 5, 2) (d) none of these


x y z x y z 71. The intercepts made on the axes by the plane
(a) + + =1 (b) + + =0
a b c a b c x + 2y − 2z = 9 are [Avadh 2018]
x y z 1 2 −2
(c) + + =3 (d) none of these (a) 1, 2, − 2 (b) , ,
a b c 9 9 9
68. The distance between the parallel planes 9 −9
(c) 9, , (d) none of these
2x − y + 3z = 4 and 6x − 3y + 9z + 13 = 0 is 2 2
[Kanpur 2018] 72. The equation of the plane passing through the
23 25 points (1, 1, 1) and (2, 3, 0) and parallel to the z-axis
(a) (b)
3 14 3 14 is : [Avadh 2018]
26 28
(c) (d) (a) 2x − y = 1 (b) x − y + z − 1 = 0
3 14 3 14
(c) x − 2y + 1 = 0 (d) 4x −3y + 1 = 0

ANSWERS
MULTIPLE CHOICE QUESTIONS

1. (d) 2. (c) 3. (a) 4. (c) 5. (b) 6. (d) 7. (b) 8. (c) 9. (a) 10. (d)

11. (c) 12. (a) 13. (d) 14. (d) 15. (a) 16. (a) 17. (d) 18. (a) 19. (d) 20. (d)

21. (b) 22. (a) 23. (d) 24. (a) 25. (b) 26. (a) 27. (d) 28. (a) 29. (b) 30. (c)

31. (d) 32. (c) 33. (b) 34. (d) 35. (d) 36. (a) 37. (c) 38. (a) 39. (b) 40. (c)

41. (b) 42. (d) 43. (a) 44. (c) 45. (b) 46. (d) 47. (c) 48. (c) 49. (a) 50. (b)

51. (c) 52. (c) 53. (b) 54. (a) 55. (c) 56. (a) 57. (d) 58. (b) 59. (a) 60. (d)

61. (b) 62. (c) 63. (a) 64. (c) 65. (c) 66. (c) 67. (c) 68. (b) 69. (a) 70. (b)

71. (c) 72. (a)


46

HINTS AND SOLUTIONS


3. Equation of plane passing through (2, 0, 1) is 1 2 2
Given d.r. are 1, –2, 2 so d.c.’s are , − , and
a (x − 2) + b(y − 0) + c (z − 1) = 0 i.e. 3 3 3
1 2 2
ax + by + cz − 2a − c = 0. Since this is perpendi- p = 2 so we get x− y+ z=2
cular to plane whose d.r.’s are 1, 1, 1 so 3 3 3
a = b = c = 1 i.e.required plane is x + y + z = 3. ⇒ x − 2y + 2z = 6
21
. + (−1)(1) + 1(2) 3 1
6. The direction ratio of line AB is 1, –2, 2 so equation 40. cos θ = = =
x −1 y−2 z − 3 4+ 1+ 1 1+ 1+ 4 6 2
of line AB is = = =r
1 −2 2 π
⇒θ=
so co-ordinate of B is (r + 1, − 2r + 2, 2r + 3) which 3
is lies on plane so 44. Given that x 2 + 4y 2 − z 2 + 4xy = 0
−1
1(r + 1) − 2(−2r + 2) + 2(2r + 3) = 0 i.e. r = so B Compare it with
3
 2, 8, 7  ax 2 + by 2 + cz 2 + 2 fyz + 2gzx + 2hxy = 0
is   thus AB is given by
 3 3 3 a = 1, b = 4, c = − 1, h = 2
2 2 2 The angle between two planes is
AB = 1 − 2 +  2 − 8 +  3 − 7  = 1.
       2 f 2 + g 2 + h2 − ab − bc − ac 
 3  3  3
θ = tan −1  

A (1, 2, 3) a + b + c
 
2 4 − 4 + 4 + 1 −1  5 
θ = tan −1   = tan  
 4   2 
3
90° 54. The point  0, , 0 lies on first plane so its
B  2 
perpendicular distance from the plane
x – 2y + 2z = 0
0− 3+ 0+ 1 2
2x − 2y + z + 1 = 0 is p = =
11. The point (1, 0, 0) lies on first plane so its 4+ 4+ 1 3
perpendicular distance from the plane 55. ∵ a + b + c = 0 so 1 − 3 + a = 0 ⇒ a = 2.
2x + 4y + 2z = 2 is
57. d.r.’s of OP are 2, 1, 1 so its equation is
2+ 0+ 0− 2 x − 3 y− 5 z −7
p= = 0. = = =r
4 + 16 + 4 2 1 1
14. The equation of the plane passing through the line Co-ordinates of P is (2r + 3, r + 5, r + 7)
of intersection of given planes is
O(3, 5, 7)
(x + y + z − 1) + λ (2x − y + 2z − 2) = 0 i.e.
(1 + 2λ) x + (1 − λ) y + (1 + 2λ) z − 1 − 2λ = 0 it d.c. 2, 1, 1
pass through (0, 0, 2) so 2 + 4λ − 1 − 2λ = 0 ⇒ P
−1 2x + y + z = 6
2λ = − 1, λ = so required plane y = 0
2
23. The area of a triangle ABC whose intercepts a, b and A(a, b, c)
c on the co-ordinate and is Which lies on plane so 4r + 6 + r + 5 + r + 7 = 6 ⇒
1 2 2 2 2 2 2
A= b c + c a + a b so put a = 4, b = − 3, 6r = − 12, r = − 2
2
So, co-ordinate of P is (−1, 3, 5). If (a, b, c) are the
c = 12 we get A = 3 26 a+ 3
co-ordinate of image point A then = − 1,
30. x + 0y + 0z = a, 0x + 1y + 0z = b so angle is 2
cos θ = l1l 2 + m1m2 + n1n2 = 1.0 + 1.0 + a.0 = 0 ⇒ b+ 5 c+7
π = 3, = 5 i.e. co-ordinate of image point
θ= . 2 2
2 is A(−5, 1, 3).
31. Required plane is lx + my + nz = p ❍❍❍
C HAPTER Unit-II
47

7 The Straight Line

THE EQUATIONS OF STRAIGHT LINE 3. The condition for the line


x −α
=
y−β
1. The two non-parallel planes intersect in a line. l m
z−γ
Therefore, two equations of the first degree in = to lie in a plane ax + by + cz + d = 0 is
n
x , y, z say, a 1x + b1y + c1z + d 1 = 0 and
al + bm + cn = 0 and aα + bβ + cγ + d = 0
a 2 x + b2 y + c2 z + d 2 = 0 together represent a
straight line. These are called the general 4. The equation of any plane through the line
equations of a straight line of the unsymmetric x −α y−β z − γ
= =
form of the line. l m n
2. The equation of straight line passing through a is a(x − α) + b(y − β) + c(z − γ ) = 0
given point A(α, β, γ ) and having d.c.’s l, m, n is where al + bm + cn = 0
x −α y−β z − γ 5. The equation of the plane through the line
= =
l m n x −α y−β z − γ
= = and parallel to the line
3. The equation of straight line passing through a l1 m1 n1
given point A(α, β, γ ) and having d.r.’s a, b, c is x y z
= = is
x −α y−β z − γ l 2 m2 n 2
= =
a b c x −α y−β z−γ
4. The equation of straight line through two given
l1 m1 n1 =0
points (x 1, y 1, z 1) and (x 2 , y 2 , z 2 ) is
x − x1 y − y1 z − z1 l2 m2 n2
= =
x 2 − x 1 y2 − y1 z 2 − z 1
PROJECTION OF A LINE ON PLANE
5. The direction ratios of the line given in (point 1)
1. The projection of a line on a given plane is the
are b1c1 − b2 c1, a 2 c1 − a 1c2 , a 1b2 − a 2 b1
line of intersection of the two planes normally
THE STRAIGHT LINE AND THE PLANE (i) the given plane and (ii) the plane through the
x −α y−β given line and perpendicular to the given plane.
1. The conditions that the line =
l m 2. If P be the point of intersection of the given line
z−γ with the given plane and Q be the foot of the
= may be parallel to the plane
n perpendicular from any point on the line to the
ax + by + cz + d = 0 is al + bm + cn = 0 and plane, then the line PQ is called the projection
aα + bβ + cγ + d ≠ 0 of the given line on the given plane.
2. The conditions that the line
x −α y−β z − γ
COPLANAR LINES
= = is perpendicular to the
l m n 1. The condition that two given lines
plane ax + by + cz + d = 0 is x −α y−β z − γ
= = and
a b c l m n
= =
l m n
48

x − α′ y − β′ z − γ ′ INTERSECTION OF THREE PLANES


= = be coplanar is
l′ m′ n′ 1. Let the equation of three planes be given by
α′ − α β′ − β γ′ − γ
a 1x + b1y + c1z + d 1 = 0
l m n =0
a 2 x + b2 y + c2 z + d 2 = 0
l′ m′ n′
And a 3 x + b3 y + c3 z + d 3 = 0
2. The equation of plane containing the lines b1 c1 d1
given in (point 1) is
And ∆ 1 = b2 c2 d2
x −α y−β z−γ
b3 c3 d3
l m n =0

l′ m′ n′ a1 c1 d1

∆ 2 = a2 c2 d2
3. The condition that two given lines
a3 c3 d3
a 1x + b1y + c1z + d 1 = 0
= a 2 x + b2 y + c2 z + d 2 a1 b1 d1
and a 3 x + b3 y + c3 z + d 3 = 0 ∆ 3 = a2 b2 d2
= a 4 x + b4 y + c4 z + d 4 a3 b3 d3
is
a1 b1 c1
a1 b1 c1 d1
∆ 4 = a2 b2 c2
a2 b2 c2 d2
=0 a3 b3 c3
a3 b3 c3 d3

a4 b4 c4 d4 Case I: If ∆ 4 ≠ 0 then the three planes intersect


in a point.
4. The condition that the lines
Case II: If ∆ 4 = 0 and ∆ 3 ≠ 0 (or ∆ 1 ≠ 0, or
x −α y−β z − γ
= =
l m n ∆ 2 ≠ 0) then the three planes form a triangular
prism.
and a 1x + b1y + c1z + d 1 = 0
Case III: If ∆ 4 = 0 = ∆ 1 = ∆ 2 = ∆ 3 then the
= a 2 x + b2 y + c2 z + d 2 may intersect is
a 1α + b1β + c1γ + d 1 a 2α + b2β + c2 γ + d 2 three planes intersect in a common line.
=
a 1l + b1m + c1n a 2l + b2m + c2n SHORTEST DISTANCE
and the equation of plane containing them is 1. The lines which do not lie in a plane are called
a 1x + b1y + c1z + d 1 a 2 x + b2 y + c2 z + d 2 skew lines. The length of the line intercepted
= .
a 1l + b1m + c1n a 2l + b2m + c2n between two lines which is perpendicular to
5. If the equation of the two lines are u 1 = 0 = v 1 both is the shortest distance between them. The
and u 2 = 0 = v 2 then the equation of the line straight line which is perpendicular to each of
intersecting both the given lines are the two skew lines is called the line of shortest
distance (S.D.).
u 1 + λ 1v 1 = 0 and u 2 + λ 2v 2 = 0.
49

2. If two lines are coplanar then the shortest 3. By a proper choice of axes the equation of two
distance between them is zero. skew lines can be given by the equations
y = x tan α, z = c and y = − x tan α, z = − c.

EXERCISE
MULTIPLE CHOICE QUESTIONS 7. The distance of the point (1, 1, 0) from the plane
x + y + z = 0 measured parallel to the line
1. The equation of straight line through the points
− x = y = z is
(1, 2, 3) and (3, 5, 7) is
x − 1 y − 2 z −7 (a) 3 2 (b) 2 3
(a) = =
2 3 4 (c) 2 (d) 3
x−3 y−5 z−2 8. The co-ordinates of the point where the line joining
(b) = =
2 3 4 the points (2, − 3, 1) and (3, − 4, − 5) meets the plane
x −1 y − 2 z − 7 2x + y + z = 7 is
(c) = =
2 3 2
(a) (2, 1, 1) (b) (1, − 2, 7)
x −1 y− 2 z − 3
(d) = = (c) (1, − 1, 2) (d) (1, 7, 2)
2 3 4
9. The image of the point (1, 3, 4) in the plane z − 3 = 0
2. The direction ratio of the line
is
3x + 2y − z − 4 = 0 = 4x + y − 2z + 3 is
(a) (2, − 4, 7) (b) (3, − 6, 4)
(a) 3, − 2, 5 (b) 3, 2, 5
(c) (1, 3, 2) (d) (1, − 3, 7)
(c) 3, 2, − 5 (d) −3, 2, 5
10. Two non-parallel planes always intersect
3. The lines x = ay + b, z = cy + d and x = a′ y + b′,
(a) at a point (b) in a plane
z = c ′ y + d ′ are perpendicular if aa′ + cc ′ is equal
(c) in a line (d) none of these
to :
(a) 0 (b) 1 11. The direction ratios of the line

(c) –1 (d) 4 1
2x − 1 = y − = 3z − 2 are
2
4. The equations ax + by + cz + d = 0 and
(a) 3, 1, 2 (b) 2, 3, 6
a′ x + b′ y + c ′ z + d ′ = 0 together represents
(c) 3, 2, 1 (d) 3, 6, 2
(a) a plane (b) two planes
(c) a straight line (d) two straight lines 12. The equation of the plane through the point
(1, 2, − 1) and perpendicular to the line
5. The co-ordinates of the point of intersection of the
3x − y + z + 1 = 0 = 5x + y + 3z is
line 6x = 3y = 2z with the plane x + y + z = 6 is
(a) x − y + 2z = 5 (b) x + y + z = 5
(a) (2, 4, 6) (b) (2, 1, 3)
(c) x + y − 2z = 5 (d) x + y − 2z = 0
(c) (3, 1, 2) (d) (1, 2, 3)
x −α y−β z − γ
13. If the line = = is perpendicular to
6. The distance of the point (4, 6, 7) from the point l m n
x−3 y−4 z−5
where the line = = meets the the plane ax + by + cz + d = 0 then
1 2 2
(a) al + bm + cn = 0 (b) al + bm + cn = 1
plane x + y + z = 22 is
a b c
(a) 1 (b) 2 (c) = = (d) al 2 + bm 2 + cn2 = 1
l m n
(c) 3 (d) 2 3
50

14. The image of the point (0, 0, 0) with respect to the 22. The line 2x − 4 = 3y + 6 = 6z − 30 meets the plane
plane 2x + y = 5 is y + z = 0 at the point
(a) (2, 2, 2) (b) (4, 2, 0) (a) (1, 4, − 4) (b) (−1, 4, − 4)
(c) (1, 2, 5) (d) (2, 1, 5)
(c) (−1, − 4, 4) (d) (−1, − 4, − 4)
x−2 y+1 z−2
15. The point in which the line = = 23. The direction ratios of the line 4x + 4y − 5z = 12,
3 4 12
8x + 12y + 3z = 32 are
meets the plane x − 2y + z = 20 is
(a) 3, 4, 2 (b) 2, 3, 4
(a) (8, 7, 25) (b) (8, 7, 24)
(c) 4, 3, 2 (d) 2, 4, 3
(c) (8, 7, 26) (d) (8, 7, 27)
x−3 y−4 z−5
16. The distance of the point (1, 3, 4) from the plane 24. The angle between the line = =
2 3 4
2x − y + z = 3 measured parallel to the line
and the plane 4x + 4y − 5z = 0 is
x y z
= = is π
−2 1 1 [Kanpur 2018] (a) 0 (b)
2
(a) 0 (b) 1 π π
1 (c) (d)
(c) (d) 2 4 3
2
25. The equation to the plane through the line y + z = 1,
x y z
17. The angle between the line = = and the plane x = 0 and parallel to the line x − z = 1, y = 0 is
2 3 4
(a) x + y + z = 1 (b) x − y + z = 1
3x + 2y − 3z = 4 is
π π (c) x + y − z = 1 (d) x − y − z = − 1
(a) (b)
2 3 26. The image of the point (0, 0, 0) in the plane x + y = 2
π is
(c) (d) 0
4 (a) (2, 2, 2) (b) (1, 1, 0)
18. The equation to the plane through the line (c) (1, 2, 1) (d) (2, 2, 0)
x−2 y−3 z−4
= = parallel to the x-axis is 27. A straight line passing through a given point
2 4 5
A(α, β, γ) and having d.c.’s l, m, n. If P(x, y, z) be any
(a) 5y − 4z = 1 (b) 5z − 4y = 1
point on it such that AP = r then
(c) 4z − 5y = 1 (d) 4y − 5z = 1
(a) x + α = lr (b) x − α = lx
19. The equation of the plane containing the lines
(c) x − r = αl (d) x + r = αl
x −1 y− 2 z − 3 x−2 y−3 z−4
= = and = = is
2 3 4 3 4 5 28. The equation of straight line passing through
(1, − 1, 3) and (0, 2, 1) is
(a) x − 2y + z = 1 (b) x − 2y + z = 0
x −1 y+ 1 z − 3
(c) x + 2y − z = 0 (d) x + 2y − z = 1 (a) = =
1 −3 2
20. The plane through the line 3x − 4y + 5z = 10, x y−2 z −1
(b) = =
2x + 2y − 3z = 4 and parallel to the line x = 2y = 3z −1 2 3
is x + 1 y−1 z + 3
(c) = =
(a) x + 20y + 27 z = 14 (b) x − 20y + 27 z = 14 1 3 2

(c) x − 20y − 27 z = 14 (d) none of these x y− 2 z −1


(d) = =
1 3 2
21. The sum of the d.c.’s of the line
3x − 3 = 3y − 1 = 6z − 9 is 29. The sum of the d.c.’s of the line 3x + 2y − z − 4 = 0,
1 4x + y − 2z + 3 = 0 is
(a) (b) 1
2 (a) – 3 (b) 2
(c) 0 (d) 3 (c) – 5 (d) – 6
51

30. The lines x = ay + 2, z = y + 3 and x = y + 1, 38. The angle between the straight line
z = y − 1 are perpendicular if x−3 y+ 4 z
= = and the plane y − z + 2 = 0 is
(a) a = 0 (b) a = − 1 1 −1 0
π
(c) a = − 2 (d) a = − 3 (a) 0 (b)
2
x −α y−β
31. One of the condition that the line = = π π
l m (c) (d)
3 6
z−γ
lie in the plane ax + by + cz + d = 0 is
n 39. The point of intersection of the lines 4x − 3y + 1 = 0
x −1 y− 2 z − 3
aα + bβ + cγ + d = 0 then the other is = 5x − 3y + 2 and = = is
2 3 4
(a) l + m + n = 0
(a) (1, 1, 1) (b) (1, − 1, − 1)
(b) al + bm + cn = 0
(c) (−1, − 1, − 1) (d) (−1, − 1, 1)
(c) a + b + c = 0
x −α y−β z − γ
(d) al + bm + cn ≠ 0 40. The condition that the line = = is
l m n
32. The equation of the plane containing two parallel parallel to the plane ax + by + cz + d = 0 is
x y z x −1 y z −1
lines = = and = = is (a) al + bm + cn = 0
1 0 −1 1 0 −1
(b) aα + bβ + cγ + d ≠ 0
(a) x = y (b) x = z
(c) either (a) or (b)
(c) y = z (d) y = 0
(d) (a) and (b) both
33. The equation of the plane through the line
41. The equation of straight line passing through (1, 0, 2)
x−2 y−3 z−4
= = and parallel to z-axis is and whose d.c.’s are proportional to 2, 1, 3 is
2 3 5
x −1 y z − 2 x −1 y z − 2
(a) 3x = 2y (b) 2x = 3y (a) = = (b) = =
2 1 3 3 2 6
(c) x = 2y (d) 2x = y x −1 y z − 2
(c) = = (d) none of these
34. The foot of the perpendicular from the point (1, 6, 3) 2 3 1
to the line 6x = 3y − 3 = 2z − 4 is 42. The co-ordinate of the foot of the perpendicular
(a) (6, 3, 3) (b) (2, 4, 6) from the point (3, − 1, 11) to the line
(c) (1, 3, 5) (d) (0, 1, 2) x y−2 z−3
= = is
2 3 4
35. The shortest distance between the lines
x −1 y− 2 z − 3 (a) (3, 2, 0) (b) (0, 2, 3)
= = and x − 2 = y − 3 = z − 6 is
1 0 0 (c) (2, 3, 4) (d) (2, 5, 7)
x y z x y z x y z
(a) 2 2 (b) 6 43. The lines = = , = = and = =
aα bβ cγ α β γ α β γ
(c) 2 (d) 3 a b c
36. The surface generated by the lines which intersect are coplanar if
the lines y = x, z = 1, y = − x, z = − 1 and x-axis is (a) a = b and b = c
(b) a = b and a = c
(a) y = xz (b) x = yz
(c) b = c and c = a
(c) z = xy (d) xyz = 0
(d) a = b or b = c or c = a
37. The direction cosines of any straight line perpendi
44. The point of intersection of the lines
-cular to x-axis are x −1 y− 3 z − 3
= = and 4x − 3y + 1 = 0
1 1 1 1 2 3 4
(a) , ,0 (b) , 0,
2 2 2 2 = 5x − 3z + 2 is
1 1 1 1 1 (a) (−1, − 2, 1) (b) (−1, − 2, − 2)
(c) 0, , (d) , ,
2 2 2 2 2 (c) (−1, − 1, − 1) (d) (1, 1, 1)
52

45. The perpendicular distance between the point 51. x y z


The line = = is perpendicular to the plane
x−2 y−3 z−4 a b c
(1, 2, 3) and the straight line = = is
3 4 5 lx + my + nz + d = 0 only when
3 3 l m n
(a) (b) (a) al + bm + cn = 0 (b) = =
5 5 a b c
5 5
(c) (d) (c) al + bm + cn = 1 (d) al + bm + cn ≠ 0
3 3
52. The image of the point (−1, 2, 3) in the yz-plane is
46. The shortest distance between the lines
x −1 y− 2 z − 3 x−2 y−4 z−5 (a) (1, 2, 3) (b) (−1, 2, − 3)
= = and = =
2 3 4 3 4 5 (c) (1, 2, − 3) (d) (1, − 2, 3)
is :
53. The point of intersection of the lines
2 1 x −1 y−1 z −1
(a) (b) = = and x − 2 = y − 2 = z − 2 is
6 6 2 2 2
1 2 (a) (1, 2, 3) (b) (2, 2, 2)
(c) (d)
3 3 (c) (1, 1, 1) (d) (3, 3, 3)
47. The trace planes x − y + z = 4, 2x − y − z = − 4 and 54. The distance of the point (0, 0, 0) from the line
x + y − 5z = − 14 intersect at through (1, 1, 1) whose d.c.’s are proportional to 1, 0,
(a) a point 1 is

(b) common line of intersection (a) 2 2 (b) 3

(c) triangular prism (c) 6 (d) 1

(d) plane 55. The shortest distance between the lines x = y = z


and x = 1, y = 0, is
48. The distance of the point (4, 1, 1) from the line of
(a) 1 (b) 2 2
intersection of x + y + z − 4 = 0 = x − 2y − z − 4
1
is : (c) 2 (d)
2
5 42 3 42
(a) (b) 56. The d.c.’s of the straight line y = 0 = z are
14 14
(a) 0, 0, 1 (b) 1, 0, 0
42 42
(c) (d)
14 7 (c) 0, 1, 0 (d) 1, 0, 1

49. The distance of the point A(1, 0, 0) from the line 57. The equation of any plane through the line
6x = 3y = 2z measured parallel to the plane x −α y−β z − γ
= = is only when al + bm + cn is
x − y + z = 0 is l m n

11 9 (a) equal to zero


(a) (b)
2 2 (b) equal to any number
13 (c) not equal to zero
(c) (d) 3
2
(d) may or may not equal to zero
50. The shortest distance between the lines
58. The length of the shortest distance between the lines
2x = y = 2z − 2 and x + y + z = 1 = x − y − z is
x + y + 2z − 3 = 0 = 2x + 3y + 4z − 4 and z-axis
4 3
(a) (b) is :
11 11
(a) 3 (b) 4
2 1
(c) (d)
11 11 (c) 2 (d) 1
53

59. The shortest distance between the z-axis and the 67. Which one of the following represents a straight line
line x − 3 = 0, y + z − 1 = 0 is in three dimension geometry?
(a) 2 (b) 3 (a) x = 0
(c) 3 (d) 2 (b) x + 2y + 3z = 1
60. One of the condition that the line (c) x + 3y + 4z = 1, z = 3
x −α y−β z − γ (d) x 2 + y 2 + z 2 = 2
= = is parallel to the plane
l m n
x −1 y− 3 z − 4
ax + by + cz + d = 0 is al + bm + cn = 0, then the 68. If the line = = lies on the plane
2 1 3
other is
x + 2y − pz − 3 = 0 then p is
(a) α + β + γ + d ≠ 0
(a) 1 (b) – 1
(b) a + b + c + d ≠ 0
(c) d ≠ 0 (c) 2 (d) 3
(d) aα + bβ + cγ + d ≠ 0 69. The shortest distance between any edges of a cube
and diagonal which does not intersect it is
61. The equation of the plane which pass through the
line of intersection of the planes a
(a) a (b)
u1 ≡ a1x + b1y + c1z + d1 = 0 and 2

u 2 ≡ a2x + b2y + c 2z + d2 = 0 (c) a 2 (d) 2a


and is parallel to x-axis is 70. The condition that the planes x = cy + bz,
(a) a2u1 = a1u 2 (b) b2u1 = b1u 2 y = az + cx and z = bx + ay pass through a line is
(c) c 2u1 = c1u 2 (d) d2u1 = d1u 2 (a) a 2 + b2 + c 2 + 2abc = 1
62. The equation of the plane through the points
(b) a 2 + b2 + c 2 = 1
(0, 0, 0),(1, 1, 1) and parallel to the line 3x = 2y = z is
(a) 4x − 3y + z = 3 (b) 3x − 4y + z = 0 (c) 2abc = 1
(c) x + 3y − 4z = 0 (d) x − 3y + 4z = 0 (d) a 2 + b2 + c 2 = 2abc
63. The shortest distance between the lines
71. The equation of the plane through the point
x −1 y z x−2 y z
= = and = = is (2, − 1, 1) and the line 4x − 3y + 5 = 0; y − 2z − 5 = 0
0 1 0 0 0 1
(a) 0 (b) 1 is [Kanpur 2018]

(c) 2 (d) 3 (a) 2x − y − 3z = 5 (b) 2x + 3y − z = 5


64. If the straight line x = 2y = az is parallel to the plane (c) 3x − y + 2z = 4 (d) 4x − y − 4z = 5
x + 2y + 3z + 4 = 0 then a is
72. The equation of the line through the point (1, 2, 3)
3 1 1 3
(a) − (b) (c) − (d) and parallel to the line x − y + 2z − 5 = 0;
2 2 2 2
3x + y + z − 6 = 0 [Kanpur 2018]
65. The image of the point (−1, 2, 3) in the xz-plane is
x+1 y−2 z+ 3
(a) (−1, − 2, 3) (b) (1, 2, 3) (a) = =
3 −5 4
(c) (1, − 2, 3) (d) (−1, − 2, − 3)
x −1 y− 2 z − 3
(b) = =
66. The shortest distance between the lines −3 5 4
x−2 y z x −1 y z x −1 y+ 2 z + 3
= = and = = is (c) = =
0 0 1 0 1 0 3 −5 4
(a) 0 (b) 1 x −1 y+ 2 z − 3
(d) = =
(c) 2 (d) 2 3 5 4
54

73. The co-ordinates of the point of intersection of the x −1 y− 6 z − 3


(a) = =
x+1 y+ 3 z−2 0 2 −3
line = = with the plane
1 3 2 x −1 y− 6 z − 3
(b) = =
3x + 4y + 5z = 20 is [Kanpur 2018] 2 −3 6
(a) (0, 0, 3) (b) (0, 0, 4) x −1 y− 6 z − 3
(c) = =
0 −3 2
(c) (0, 1, 4) (d) (1, 0 4)
x −1 y− 6 z − 3
(d) = =
74. The shortest distance between the lines 3 0 2
x−3 y−8 z−3 x+ 3 y+7 z−6
= = ; = = is 80. The angle between the plane 2x + 3y − z = 7 and
3 −1 1 −3 2 4
x y−1 z − 2
[Kanpur 2018] line = = is
1 2 3 [Avadh 2018]
(a) 3 30 (b) 4 30
5 5
(c) 5 30 (d) none of these (a) cos −1   (b) sin −1  
 14   14 
75. The equation of shortest distance between 1 1
(c) cos −1   (d) sin −1  
3x − 9y + 5z = 0 =x+ y−z=0 and  14   14 
6x + 8y + 3z − 13 = 0 = x + 2y + z − 3 is
81. The equation of the projection of the line
[Kanpur 2018] x −1 y+ 1 z − 3
= = on the plane x + 2y + z = 6
(a) 10x + 29y + 16z = 0 = 0, 2 −1 4
13x + 82y + 55z + 109 = 0 is
(b) 10x − 29y + 16z = 0 = 0, [Kanpur 2018]
13x + 82y + 55z − 109 = 0 (a) 9x − 2y − 5z = 4
(c) 10x + 29y − 16z = 0, (b) 9x − 2y − 5z + 4 = 0
13x − 82y − 54z − 107 = 0 (c) 9x − 2y + 5z − 5 = 0
(d) none of these (d) 9x − 2y − 5z + 5 = 0
76. The equation of the perpendicular from the point 82. The shortest distance between the line
x y−1 2− z x + a = 2y = − 12z and x = y + 2a = 6z − 6a is
(1, 6, 3) to the line = = is
1 2 3
[Kanpur 2018] [Kanpur 2018]
(a) 0 (b) a (c) 2a (d) 3a
(a) x − 1 = 0, 2y + 3z = 21
83. The equation of plane through (2, 1, 4) and
(b) x − 2 = 0, 3y + 2z = 21
perpendicular to the line of intersection of the plane
(c) x − 9 = 0, 2y + 3z = 23
3x + 4y + 7 z + 4 = 0 and x − y + 2z + 3 = 0 is
(d) x + 1 = 0, 2y + 3z = 21 [Kanpur 2018]
77. The shortest distance between the lines y = mx, (a) 15x + y − 7 z − 3 = 0
z = c; and y = − mx, z = − c is [Avadh 2018] (b) 15x − y − 7 z − 3 = 0
(a) c (b) c / 2 (c) 2c (d) 0 (c) 15x + y + 7 z − 3 = 0
x−2 y−3 z−y x −1 y− 4 (d) 15x + y + 7 z + 3 = 0
78. The lines = = and =
1 1 −k k 2 84. The image of the point (1, 3, 4) in the plane
z−5
= are coplanar if 2x − y + z + 3 = 0 is [Agra 2017]
1 [Avadh 2018]
(a) (3, 5, 2) (b) (−3, 5, 2)
(a) k = 1 (b) k = 2
(c) (3, − 5, 2) (d) none of these
(c) k = 3 (d) k = − 5
85. Two lines, which do not lie in the same plane are
79. The equation of perpendicular from the point
x y−1 z − 2 called [Avadh 2018]
(1, 6, 3) to the line = = is
1 2 3 (a) parallel (b) intersecting
[Avadh 2018] (c) co-incident (d) skew
55

ANSWERS
MULTIPLE CHOICE QUESTIONS

1. (d) 2. (a) 3. (c) 4. (c) 5. (d) 6. (c) 7. (b) 8. (b) 9. (c) 10. (c)

11. (d) 12. (c) 13. (c) 14. (b) 15. (c) 16. (a) 17. (d) 18. (c) 19. (b) 20. (b)

21. (a) 22. (c) 23. (b) 24. (a) 25. (d) 26. (d) 27. (b) 28. (a) 29. (d) 30. (c)

31. (b) 32. (d) 33. (a) 34. (c) 35. (c) 36. (a) 37. (c) 38. (d) 39. (c) 40. (d)

41. (a) 42. (d) 43. (d) 44. (c) 45. (a) 46. (b) 47. (c) 48. (b) 49. (a) 50. (a)

51. (b) 52. (a) 53. (d) 54. (d) 55. (d) 56. (b) 57. (a) 58. (c) 59. (b) 60. (d)

61. (a) 62. (b) 63. (b) 64. (a) 65. (d) 66. (b) 67. (c) 68. (a) 69. (c) 70. (a)

71. (d) 72. (b) 73. (b) 74. (a) 75. (b) 76. (a) 77. (d) 78. (c) 79. (c) 80. (b)

81. (b) 82. (c) 83. (a) 84. (b) 85. (d)

HINTS AND SOLUTIONS


x−b y z−d
3. Given lines are = = and O(1, 3, 4)
a 1 c
x − b′ y z − d ′
= = so these are perpendicular
a′ 1 c′ P
when aa′ + 1.1 + cc ′ = 0 i.e. aa′ + cc ′ = − 1.
5. Let 6x = 3y = 2z = r so intersection point is A(a, b, c)
 r , r , r  which lies on x + y + z = 6 i.e.
 
 6 3 2 The co-ordinate of P is given by (1, 3, r + 4) it lies on
r r r given plane so r + 4 − 3 = 0 ⇒ r = − 1
+ + = 6 i.e. r = 6 so required co-ordinate is
6 3 2 So, P is (1, 3, 3) thus A is (1, 3, 2).
(1, 2, 3). x y z
16. Since given line is = =
6. Let the point of intersection is given by −2 1 1
x−3 y−4 z−5 A(1, 3, 4)
= =` = r so P (r + 3, 2r + 4,
1 2 2
–2, 1, 1 dr
2r + 5) it lies on given plane x + y + z = 22 so
r + 3 + 2r + 4 + 2r + 5 = 22 i.e. r = 2 so P(5, 8, 9). 2x – y + z = 3 P
Thus its distance from A(4, 6, 7) is
AP = 1 + 4 + 4 = 3
So equation of line AP is
9. The d.r’s of OP are 2, –1, 1 so its equation
x −1 y− 3 z − 4
x −1 y− 3 z − 4 = = =r
= = =r −2 1 1
0 0 1
56

Co-ordinate of P is (−2r + 1, r + 3, r + 4) which lies −1 3


= + = 2
on given plane so 2 2

−4r + 2 − r − 3 + r + 4 = 3 ⇒ r = 0 a1 b1 c1 1 −1 1

So co-ordinate of P is (1, 3, 4) thus AP = 0 47. ∆ 4 = a2 b2 c2 = 2 −1 −1 = 0


18. Equation of plane containing the line is a3 b3 c3 1 1 −5
a(x − 2) + b(y − 3) + c(z − 4) = 0 a1 b1 d1 1 −1 4
Since it is parallel to x-axis so
Now ∆ 3 = a2 b2 d2 = 2 −1 −4 ≠ 0
a ⋅ 1 + b⋅ 0 + c ⋅ 0 = 0 …(1)
a3 b3 d3 1 1 −14
Also 2a + 4b + 5c = 0 …(2)
So it is a triangular prism.
After solving (1) and (2) we get b = 4, c = − 4
54. P is (r + 1, 1, r + 1)
Put it in plane the required plane is 5y − 4z + 1 = 0
d.r. of OP are r + 1, 1, r + 1 which is perpendicular
32. Equation of plane containing first line is
to 1, 0, 1 (d.r. of line) so r + 1 + r + 1 = 0 ⇒ r = − 1
ax + by + cz = 0 since it containing line (2) so plane
so P(0, 1, 0)
pair through (1, 0, 1) i.e.
O(0, 0, 0)
a + 0b + c = 0 

Also a + 0b − c = 0
P
After solving we get a = 0, b = − 2, c = 0
x–1 y–1 z–1
Thus, the plane is y = 0 = = =r
1 0 1
35. Let a, b, c be the d.r. of shortest distance then
Thus OP = 1
a + 0b + 0c = 0
x = 2y = az
with a+ b+ c = 0
C (1, 2, 3) x + 2y + 3z + 4 = 0

x y z x = 2y = az
64. Line is = +
S.D. 1 1 / 2 1 / a x + 2y + 3z + 4 = 0
1 1
Its d.r. are 1, , which are perpendicular to the d.r.
D 2 a
(2, 3, 6)
of normal to the plane i.e. 1, 2, 3 so
We get d.r.’s are 0, − 1, 1 1 3
1⋅ 1 + 2 + =0
0 −1 1 2 a
So d.c. of S.D. are , ,
2 2 2 −3
⇒ a=
0 −1 1 2
Thus CD = (2 − 1) (3 − 2) + (6 − 3) ❍❍❍
2 2 2
C HAPTER Unit-III
G-57

8 The Sphere

EQUATIONS OF A SPHERE 8. If C(−u, − v, − w) be the centre and


2 2 2
1. A sphere is a locus of a point in space which r = u + v + w − d be the radius of the
moves so that its distance from a fixed point sphere then :
(called centre) is constant. The constant Case I: If u 2 + v 2 + w 2 > d, the radius r is real
distance is called the radius of sphere.
and so the sphere is real.
2. The equation of the sphere when centre is
Case II: If u 2 + v 2 + w 2 = d, then r = 0 and so
(x 1, y 1, z 1) and radius is r is
the sphere reduce to a point, called a point
(x − x 1)2 + (y − y 1)2 + (z − z 1)2 = r 2
sphere.
3. If the centre is origin i.e. (0, 0, 0) in (point 2) Case III: If u 2 + v 2 + w 2 < d, then r is
then the equation of the sphere is
imaginary and then the sphere is called a virtual
x 2 + y2 + z 2 = r 2 sphere.
4. The general equation of the sphere is PLANE SECTION OF A SPHERE
x 2 + y 2 + z 2 + 2ux + 2vy + 2wz + d = 0
1. The plane section of a sphere is a circle.
where centre of the sphere is C (−u, − v, − w)
2. Let R and r be the radius of the sphere S and if
and radius is r = u 2 + v 2 + w 2 − d
plane section i.e. circle C respectively then
5. The diameter of a sphere always subtends a Case I: If R < r , the circle C is called a small
right angle at any point on the surface of the
circle.
sphere.
Case II: If R = r , the circle C is called a great
6. The equation of the sphere when the end points
circle and centre of circle coincide with the
of its diameter are (x 1, y 1, z 1) and (x 2 , y 2 , z 2 ) is
centre of the sphere.
(x − x 1)(x − x 2 ) + (y − y 1)(y − y 2 )
Case III: If R > r , then circle is a imaginary
+ (z − z 1)(z − z 2 ) = 0
circle i.e. the sphere and the circle do not
7. The equation of the sphere passing through intersect.
four given points (x 1, y 1, z 1),
3. The general equation of a circle is
(x 2 , y 2 , z 2 ), (x 3 , y 3 , z 3 ) and (x 4 , y 4 , z 4 ) is
x 2 + y 2 + z 2 + 2ux + 2vy + 2wz + d = 0 with
x 2 + y2 + z 2 x y z 1 lx + my + nz = p
4. The curve of intersection of two spheres is a
x 12 + y 12 + z 12 x1 y1 z1 1
circle. If sphere are S1 = 0 and S2 = 0 then the
x 22 + y 22 + z 22 x2 y2 z2 =0 circle is
1
S1 = 0, S1 − S2 = 0 or S2 = 0, S1 − S2 = 0
x 32 + y 32 + z 32 x3 y3 z3 1 5. The equation of the sphere through the circle
x 42 + y 42 + z 42 S = 0, P = 0 is S + λP = 0, where λ is arbitrary
x4 y4 z4 1
constant.
58

TANGENT PLANE AND TANGENT LINE 1 1 2


+ = , then the locus of the point R
1. A line which meets a sphere in two coincident AP AQ AR
points is called the tangent line to the sphere. is a plane called the polar plane of the point A
2. The locus of the tangent line to a sphere with respect to the given sphere and the fixed
through a point lying on sphere is called its point A is called the pole of the polar plane.
tangent plane at that point. 2. The equation of the polar plane of a point
3. The equation of the tangent plane at the point A(x 1, y 1, z 1) with respect to a sphere
(x 1, y 1, z 1) of the sphere x 2 + y 2 + z 2 + 2ux x 2 + y 2 + z 2 + 2ux + 2vy + 2wz + d = 0 is
+ 2vy + 2wz + d = 0 is xx 1 + yy 1 + zz 1 + u(x + x 1) + v(y + y 1)
xx 1 + yy 1 + zz 1 + u(x + x 1) + v(y + y 1)
+ w(z + z 1) + d = 0
+ w(z + z 1) + d = 0
3. The pole of the plane lx + my + nz = p with
4. The equation of the tangent plane at (x 1, y 1, z 1)
respect to the sphere x 2 + y 2 + z 2 = r 2 is
to the sphere x 2 + y 2 + z 2 = r 2 is
 l 2 m 2 n 2
 r , r , r .
xx 1 + yy 1 + zz 1 = r 2 p p p 
5. The tangent line and the tangent plane both are 4. The distance of two points from the centre of a
perpendicular to the radius through the point of sphere and proportional to the distance of each
contact.
from the polar plane of the other.
6. The condition that the plane lx + my + nz = p is
5. The polar plane of a point P with respect to a
a tangent plane to the sphere
sphere is perpendicular to the line joining the
x 2 + y 2 + z 2 + 2ux + 2vy + 2wz + d = 0 is
point P to the centre of the sphere.
(lu + mv + nw − p)2
6. If the line joining the centre O of the sphere and
= (l 2 + m 2 + n 2 ) (u 2 + v 2 + w 2 − d). a point P meets the polar plane of P in the point
7. The condition that the plane lx + my + nz = p, Q, then OP ⋅ OQ = (radius)2
touches the sphere x 2 + y 2 + z 2 = r 2 is
7. The two planes such that the pole of each lies
p2 = r 2 (l 2 + m 2 + n 2 )
on the other, are called conjugate planes.
8. If straight line in any direction and through a
given point A meets the sphere in two points P
THE ANGLE OF INTERSECTION OF TWO
SPHERES
and Q, then AP. AQ is constant and is called the
power of the point A. 1. The angle of intersection of any two spheres at
9. The plane of contact is the locus of the point of a common point in the angle between the
contact of the tangent planes which pass tangent planes at that point. Two spheres are
through a given point (not on the sphere). Its said to cut orthogonally if their angle of
equation is same as tangent plane. intersection is a right angle.

POLE AND POLAR PLANE 2. Let P be the point of intersection of the two
spheres whose centres are C 1 and C 2 with radii
1. Consider a line through a fixed point A to
r1 and r2 . If θ be the angle of intersection then
intersect a given sphere in the points P and Q.
Take a point R on this line such that
59

P 2. The equation of the radical plane of two


r1 r2 spheres S1 = 0 and S2 = 0 is S1 − S2 = 0.

C1 C2
3. The radical plane of two spheres is
d perpendicular to the line joining their centre.
4. The radical plane of two spheres passes
through their points of intersection.
 r 2 + r22 − d 2  5. The radical planes of three spheres taken two at
θ = cos−1  1  a time pass through a common line which is
 2r1r2 
said to be the radical line of three spheres and
3. Two spheres x 2 + y 2 + z 2 + 2u 1x + 2v 1y the radical lines of the four spheres taken three
+ 2w 1z + d 1 = 0 and x 2 + y 2 + z 2 + 2u 2 x at a time meet in one point called radical centre
+ 2v 2 y + 2w 2 z + d 2 = 0 cut orthogonally when of the four spheres.
2u 1u 2 + 2v 1v 2 + 2w 1w 2 = d 1 + d 2 COAXIAL SYSTEM OF SPHERES
4. If the distance between the centres of the two
spheres is equal to the difference of their radii, 1. A system of sphere is called a co-axial system of
then the two spheres touch internally. If the sphere if for all spheres any two of them have
distance between the centres of the two spheres same radical plane.
is equal to the sum of their radii, then the two 2. The general equation of a co-axial system of
spheres touch externally. sphere S1 = 0 and S2 = 0 is S1 + λS2 = 0, and
5. The length of the tangent from the point S1 + µ(S1 − S2 ) = 0 represent a co-axial system
(x 1, y 1, z 1) to the sphere of sphere. Similarly if S = 0 be a sphere and
x 2 + y 2 + z 2 + 2ux + 2vy + 2wz + d = 0 is P = 0 be a plane then S + λP = 0 represents a
co-axial system of sphere.
x 12 + y 12 + z 12 + 2ux 1 + 2vy 1 + 2wz 1 + d.
3. The centres of the sphere of co-axial system
THE RADICAL PLANES which have zero radius are called the limiting
points of co-axial system. The limiting point of
1. The radical plane of two spheres is the locus of
x 2 + y 2 + z 2 + 2ux + d = 0 are ( d , 0, 0) and
points whose power with respect to two spheres
are equal. (− d , 0, 0).

EXERCISE
MULTIPLE CHOICE QUESTIONS 3. The diameter of the sphere (x − 1)(x − 2) + (y − 2)
(y − 3) + (z − 3)(z − 4) = 0 is
1. The co-ordinates of the centre of the sphere
(x − 1)2 + (y + 2)2 + (z − 3)2 = 4 are (a) 2 (b) 3 (c) 2 (d) 3
4. If a sphere of constant radius 2k passes through the
(a) (1, 2, 3) (b) (1, − 2, − 3)
origin and meets the axes in A, B, C then the
(c) (−1, − 2, − 3) (d) (1, − 2, 3) centroid of the tetrahedron OABC lies on
2. The minimum independent condition that deter- (a) x 2 + y 2 + z 2 = 4 k 2
mined the sphere completely and uniquely are :
(b) x 2 + y 2 + z 2 = k 2
(a) 5 (b) 4 (c) 3 (d) 2
(c) 9 (x 2 + y 2 + z 2) = k 2

(d) 9 (x 2 + y 2 + z 2) = 4 k 2
60

5. The radius of the sphere 12. The angle between the spheres

x 2 + y 2 + z 2 + 2 ux + 2 vy + 2 wz + d = 0 is x 2 + y 2 + z 2 + 6y − u = 0 and
[Agra 2017] x 2 + y 2 + z 2 − 2x − 5z + 4 = 0 is
(a) u+v+w−d (b) u 2 + v 2 + w 2 − d π π
(a) (b)
2 2 2 2 4
(c) u + v + w − d (d) u +v +w −d
π π
(c) (d)
6. The angle at any point on the surface of sphere that 3 6
the diameter subtends is 13. If the plane 2x + y + 3z = a touches the sphere
π x 2 + y 2 + z 2 = 4 then a is
(a) (b) π
2
(a) 14 (b) 2 14
π π
(c) (d)
4 3 (c) 3 14 (d) 5 14
7. The equation of the sphere when the end points of 14. If any tangent plane to the sphere
its diameter are (0, 0, 0) and (−1, 2, 1) is 2 2 2 2
x + y + z = R makes intercepts a, b and c on
(a) x 2 + y 2 + z 2 + 2x − y + z = 0 the co-ordinate axes then
[Avadh 2018]
(b) x 2 + y 2 + z 2 + x − 2y − z = 0
1 1 1 1
(a) + + = (b) a + b + c = R
(c) x 2 + y 2 + z 2 − x − 2y − 2z = 0 a b c R
1 1 1 1
(d) x 2 + y 2 + z 2 + x + 2y + z = 0 (c) a 2 + b2 + c 2 = R 2 (d) + + =
a2 b2 c2 R2
8. Which is same in the sphere and its great circle?
15. The number of spheres touching the three
(a) only centre
co-ordinate axes of fixed radius r is
(b) only radius
(a) 8 (b) 6
(c) centre and radius both
(c) 4 (d) 2
(d) neither centre nor radius
16. The pole of the plane lx + my + nz = p with respect
9. If the sphere x 2 + y 2 + z 2 = 9 touches the plane
to the sphere x 2 + y 2 + z 2 = a 2 is [Avadh 2018]
x + y + z = a 3, then the value of a is
 al am an 
(a) 3 (b) 3 3 (a)  , , 
p p p
(c) 9 (d) 3  pl pm pn 
(b)  , , 
10. The radius of the sphere for which the circle  a 2 b2 c 2 
x 2 + y 2 + z 2 + 7 y − 2z + 2 = 0, 2x + 3y + 4z = 8  a 2l a 2m a n 
(c)  , , 
is a great circle is  p p p 

(a) 4 (b) 2  al 2 am 2 an2 
(d)  , , 
(c) 1 (d) 2 2  p p p 

11. If a point moves so that the sum of the squares of its 17. If the line joining the centre of the sphere of radius r
distance from the six faces of a cube is constant then and a point P meets the polar plane of P in the point
its locus is Q, then OP ⋅ OQ is
(a) cylinder (b) circle (a) r (b) 2r
2
(c) sphere (d) cone (c) 2 r (d) r 2
61

18. If two spheres of radii 3 and 4 cut orthogonally then 26. If R and r be the radius of the sphere S and its plane
the radius of the common circle is section C respectively and R = r, then the circle C is
3 7
(a) (b) (a) small circle (b) imaginary circle
5 5
11 12 (c) large circle (d) great circle
(c) (d)
5 5 27. The polar plane of (1, − 1, 1) with respect to the
19. The angle between the radius through any point sphere x 2 + y 2 + z 2 = 4 is
and the tangent plane at that point in a sphere is
(a) x − y + z = 0 (b) − x + y − z = 2
(a) 0 (b) π
π π (c) − x + y − z = − 4 (d) − x + y − z = 4
(c) (d)
2 4 28. The pole of the plane x + 2y + 3z = 3 with respect
20. The locus of a point which moves so that its distance to the sphere x 2 + y 2 + z 2 = 9 is
from a fixed point is constant in space is called
(a) (3, 6, 9) (b) (1, 2, 3)
(a) circle (b) plane
(c) sphere (d) cone (c) (1, 1, 1) (d) (3, 3, 3)

21. The centre of the sphere 29. If the tangent plane to the sphere x 2 + y 2 + z 2 = r 2

2x 2 + 2y 2 + 2z 2 − 4x − 3y + 4z + 2 = 0 is makes intercept a, b, c on the axes then


3 3 (a) a + b + c = r
(a)  1, , − 1 (b)  1, , 1
 4   4  1 1 1 1
(b) + + =
3
(c)  2, , − 2
3
(d)  2, , 2 a2 b2 c2 r2
 2   2 
1 1 1
(c) + + =r
22. The radius of the sphere a b c
x 2 + y 2 + z 2 + 6x − 8y − 10z + 1 = 0 is (d) a 2 + b2 + c 2 = r 2
(a) 7 (b) 7 30. If two spheres x 2 + y 2 + z 2 + 2x + 1 = 0 and
(c) 17 (d) 17
x 2 + y 2 + z 2 + ax + 2 = 0 cut orthogonally then
23. The curve of intersection of two spheres is
the value of a is
(a) line (b) plane
(a) 1 (b) 2
(c) circle (d) sphere
(c) 3 (d) 4
24. If a plane passes through a fixed point (a, b, c) and
31. The number of spheres that can be made to pass
cuts the axes in A, B, C then the locus of the centre
of the sphere OABC is through the three points (a, 0, 0), (0, b, 0) and (0, 0, c)

x y z a b c are
(a) + + =1 (b) + + =1
a b c x y z (a) 3 (b) 2
a b c a b c (c) 1 (d) infinite
(c) + + =2 (d) + + = −1
x y z x y z
32. The equation
25. The radius of the circle x 2 + y 2 + z 2 = 25,
ax 2 + by 2 + cz 2 + 2 ux + 2 vy + 2 wz + d = 0
3
x+ y − 2z = 0 is represents a sphere of
2
(a) u = v = w (b) a = b = d
(a) 3 (b) 2
(c) a = b = c (d) a = c = d
(c) 5 (d) 25
62

33. The tangent plane of the sphere x 2 + y 2 + z 2 = 25 40. If two spheres of radius r1 and r2 cut orthogonally
then radius of the common circle is
at the point (2, − 3, 4) is
[Avadh 2018]
(a) 2x − 3y + 4z = 25 (b) 2x − 3y + 4z = 1
(c) 2x − 3y + 4z = 5 (d) none of these (a) r1r2 (b) r1r2 r12 + r22
r1r2
34. If the plane 2x − 2y + z + 12 = 0 touches the sphere (c) (d) r12 + r22
2 2 2
x + y + z − 2x − 4y + 2z − 3 = 0 then the point r1 + r22
2

of contact is 41. The radius of the circle in which the sphere


(a) (1, 4, 2) (b) (1, 4, − 2) x 2 + y 2 + z 2 − 2x − 4y − 6z = 0 is cut by the plane
x + y + z = 0 is
(c) (−1, 4 − 2) (d) (−1, − 4, − 2)
(a) 5 (b) 2 2
35. If the distance between the centres of two spheres is
(c) 2 (d) 2
equal to the sum of their radii then the two spheres
touches 42. The equation of the sphere which passes through
the points (1, 0, 0),(0, 1, 0), (0, 0, 1) and radius as small
(a) internally
or possible is
(b) externally
(a) x 2 + y 2 + z 2 − 2x − 2y − 2z = 1
(c) may be internally or externally
(b) 3 (x 2 + y 2 + z 2) − 3x − 3y − 3z = 1
(d) none of these
(c) 3 (x 2 + y 2 + z 2) − x − y − z = 1
36. The angle between two spheres
(d) 3 (x 2 + y 2 + z 2) − 2x − 2y − 2z = 1
x 2 + y 2 + z 2 + 6y + 2z + 8 = 0 and
43. The members of the co-axial system
x 2 + y 2 + z 2 + 6x + 8y + 4z + 20 = 0 is 2 2 2
x + y + z + 2ux + d = 0 touch one another
π π only when
(a) (b)
3 4 (a) d > 0 (b) d < 0
π π (c) d = 0 (d) disimaginary
(c) (d)
6 2
44. If a sphere of radius 3 passes through the origin and
37. The angle between the radical plane and the line meets the axes in A, B, C then the centroid of the
joining their centres is triangle ABC is
π
(a) 0 (b) (a) a (x 2 + y 2 + z 2) = 4
2
(b) x 2 + y 2 + z 2 = 9
π π
(c) (d)
3 6 (c) x 2 + y 2 + z 2 = 4
38. The limiting point of the co-axial system of sphere (d) x 2 + y 2 + z 2 = 3
2 2 2
x + y + z + 24x + d = 0 is
45. The equation of the sphere with centre at (0, 0, 0)
(a) (d, 0, 0) (b) (0, d, 0) and touching the plane x + y + z = 1 is
(c) (0, d , 0) (d) ( d , 0, 0) (a) x 2 + y 2 + z 2 = 1
39. The locus of a point whose power with respect to the
(b) x 2 + y 2 + z 2 = 3
two given spheres are equal is called
(a) conjugate planes (b) polar planes (c) 3 (x 2 + y 2 + z 2) = 2

(c) radical planes (d) parallel planes (d) 3 (x 2 + y 2 + z 2) = 1


63

46. If a point in 3-dimensional geometry moves so that 54. The radius of the circle
the ratio of its distance from two fixed points is 3x 2 + 3y 2 + 3z 2 + x − 5y − z = 0, x + y = 2 is
constant then its locus is 1
(a) 2 (b) (c) 2 2 (d) 2
(a) circle (b) sphere 2
(c) cylinder (d) cone 55. If 2x 2 + 2y 2 + cz 2 + 2x − 3y + 4z − 1 = 0
47. The equation of the sphere circumscribing the represent a sphere then c is
tetrahedron whose faces are x = 0, y = 0, z = 0, (a) 1 (b) 2 (c) 3 (d) not exist
x y z
+ + = 1 is 56. If the plane 2x − 2y + z − 9 = 0 touches the sphere
a b c
x 2 + y 2 + z 2 = a 2 then a is [Kanpur 2018]
(a) x 2 + y 2 + z 2 + ax + by + cz = 1
(a) 1 (b) 2 (c) 9 (d) 3
(b) x 2 + y 2 + z 2 − x − y − z = a 2 + b2 + c 2
57. The equation of sphere with centre at (2, 3, − 4) and
(c) x 2 + y 2 + z 2 − ax − by − cz = 0 touching the plane 2x + 6y − 3z + 15 = 0 is
[Kanpur 2018]
(d) x 2 + y 2 + z 2 = a 2 + b2 + c 2
2 2 2
(a) x + y + z + 4x − 6y + 8z + 20 = 0
48. The section of the sphere by a plane is
(b) x 2 + y 2 + z 2 + 4x + 6y + 8z + 20 = 0
(a) line (b) cone
(c) cylinder (d) circle (c) x 2 + y 2 + z 2 + 4x − 6y − 8z + 20 = 0

49. If the distance between the centres of the two (d) x 2 + y 2 + z 2 − 4x − 6y + 8z − 20 = 0


spheres is equal to the difference of their radius, 58. The equation of sphere having its centre on the line
then the two spheres touches 2x − 3y = 0 = 5y + 2z and passing through the
(a) internally (b) externally points (0, − 2, − 4) and (2, − 1, − 1) is [Kanpur 2018]
(c) intersect (d) none of these (a) x 2 + y 2 + z 2 − 6x − 4y − 10z + 12 = 0
50. The radical planes of four spheres taken in pairs
(b) x 2 + y 2 + z 2 − 6x − 4y + 10z + 12 = 0
always meet in a
(c) x 2 + y 2 + z 2 + 6x + 4y − 10z + 12 = 0
(a) sphere (b) circle
(c) plane (d) point (d) x 2 + y 2 + z 2 + 6x − 4y − 10z − 12 = 0
51. The power of the point (2, − 1, 3) with respect to the 59. The angle of intersection of the spheres
sphere x 2 + y 2 + z 2 − 3x + y − z + 10 = 0 is x 2 + y 2 + z 2 − 2x − 4y − 6z + 10 = 0 and
(a) 10 (b) 14 x 2 + y 2 + z 2 − 6x − 2y + 2z + 2 = 0 is
(c) 8 (d) 6 [Kanpur 2018]
2 2
52. The radical axes for the spheres (a) sinh −1   (b) cosh −1  
 3  3
(x − 3)2 + y 2 + z 2 = 11, x 2 + (y − 2)2 + z 2 = 6 and
2 2
(c) sin −1   (d) cos −1  
(x + 1)2 + y 2 + (z − 2)2 = 7 is  3  3
x y z x y z 60. Equation of the sphere through the circle
(a) = = (b) = =
3 2 2 3 3 2 x 2 + y 2 + z 2 = 9, x + y − 2z + 4 = 0 and the origin
x y z x y z
(c) = = (d) = = is [Avadh 2018, Kanpur 2018]
2 3 4 3 3 4
(a) 4 (x 2 + y 2 + z 2) − 9x + 9y − 18z = 0
53. The pole of the plane x − y + 5z = 3 with respect to
the sphere x 2 + y 2 + z 2 = 9 is (b) 4 (x 2 + y 2 + z 2) + 9x − 9y − 18z = 0
(c) 4 (x 2 + y 2 + z 2) + 9x + 9y − 18z = 0
(a) (3, − 3, 15) (b) (−3, 3, 15)
(c) (3, 3, 15) (d) (3, − 3, − 15) (d) 4 (x 2 + y 2 + z 2) − 9x − 9y + 18z = 0
64

61. The equation of sphere which touches the plane (c) x 2 + y 2 + z 2 + ax + by + cz = 0


3x + 2y − z + 2 = 0 at the point (1, − 2, 1) and also
(d) x 2 + y 2 + z 2 − ax − by − cz = 0
cuts orthogonally the sphere
x 2 + y 2 + z 2 − 4x + 6y + 4 = 0 is 64. The radius of the sphere
[Kanpur 2018] 2x 2 + 2y 2 + 2z 2 + x + y + z = 0 is [Avadh 2018]

(a) x 2 + y 2 + z 2 − 7 x + 10y − 5z + 24 = 0 3 1
(a) (b)
4 4
(b) x 2 + y 2 + z 2 + 7 x + 10y − 5z + 12 = 0
3
(c) (d) none of these
(c) x 2 + y 2 + z 2 + 7 x − 10y − 5z + 12 = 0 4
(d) x 2 + y 2 + z 2 + 7 x − 10y + 5z − 12 = 0 65. The equation of the sphere whose centre (−3, 4, 5)
and radius 7 is [Avadh 2018]
62. The equation of sphere joining (2, − 3, 4) and
(−5, 6, − 7) as diameter is [Avadh 2018] (a) x 2 + y 2 + z 2 − 6x − 8y − 10z + 1 = 0
(a) x 2 + y 2 + z 2 + x + y + z = 0 (b) x 2 + y 2 + z 2 − 6x + 8y + 10z + 1 = 0
(b) x 2 + y 2 + z 2 + x − y + z = 1
(c) x 2 + y 2 + z 2 + 6x − 8y − 10z + 1 = 0
2 2 2
(c) x + y + z + 3x − 3y + 3z − 56 = 0
(d) none of these
(d) none of these
66. The centre of the sphere
63. The equation of the sphere passing through the
point (0, 0, 0), (a, 0, 0), (0, b, 0) and (0, 0, c) is (x + 1) (x + 3) + (y − 2) (y − 4) + (z + 1) (z + 3) = 0
[Avadh 2018] is [Avadh 2018]
2 2 2 2 (a) (−1, 1, − 1) (b) (1, − 1, 1)
(a) x + y + z = a
2 2 2 2 2 2 (c) (2, − 3, 2) (d) (−2, 3, − 2)
(b) x + y + z = a + b + c

ANSWERS
MULTIPLE CHOICE QUESTIONS

1. (d) 2. (b) 3. (d) 4. (b) 5. (d) 6. (a) 7. (b) 8. (c) 9. (d) 10. (b)

11. (c) 12. (a) 13. (b) 14. (d) 15. (a) 16. (c) 17. (d) 18. (d) 19. (c) 20. (c)

21. (a) 22. (a) 23. (c) 24. (c) 25. (c) 26. (d) 27. (c) 28. (a) 29. (b) 30. (c)

31. (d) 32. (c) 33. (c) 34. (c) 35. (b) 36. (d) 37. (b) 38. (d) 39. (c) 40. (c)

41. (d) 42. (d) 43. (c) 44. (c) 45. (d) 46. (b) 47. (c) 48. (d) 49. (a) 50. (d)

51. (b) 52. (c) 53. (a) 54. (b) 55. (b) 56. (d) 57. (d) 58. (b) 59. (d) 60. (c)

61. (b) 62. (c) 63. (d) 64. (c) 65. (c) 66. (d)
65

HINTS AND SOLUTIONS


4. The equation of sphere OABC is x 2 + y 2 + z 2 − px − qy − rz = 0 its centre is
2 2 2
x + y + z − ax − by − cz = 0 its radius is  p , q , r  . Let the centre is (x, y, z) then p = 2x,
 
 2 2 2
a 2 + b2 + c 2 = 2k ⇒ a 2 + b2 + c 2 = 16 k 2
q = 2y and r = 2z their locus of centre is
a b c a b c
If x, y, z be the centroid then x = ,y = ,z = + + =2
4 4 4 x y z
So locus is 16(x 2 + y 2 + z 2) = 16 k 2 30. Two sphere cut orthogonally then
2 2 2 2 2u1u 2 + 2v1v2 + 2w1w 2 = d1 + d2
⇒ x + y + z =k
a
10. The equation of sphere is so 2 (1)   + 2 ⋅ 0 + 2 ⋅ 0 ⋅ 0 = 1 + 2 ⇒ a = 3
 2
x 2 + y 2 + z 2 + 2λx + (7 + 3λ)y
34. The centre of the sphere is (1, 2, − 1) and d.r. of the
− 2 (1 − 2λ)z + 2 − 8λ = 0 line CA are 2, –2, 1 so equation of line CA is
3λ + 7 x −1 y− 2 z + 1
and its centre is  − λ, − , 1 − 2λ  which lies = = =r
 2  2 −2 1
on given plane so λ = − 1 thus sphere is

x 2 + y 2 + z 2 − 2x + 4y − 6z + 10 = 0 and its
C A
radius = 2
13. The centre of the sphere is (0, 0, 0) and radius is 2 so
length of perpendicular from centre on tangent is
equal to sphere i.e. A (2r + 1, − 2r + 2, r − 1)
a a
i.e. 2 = = ⇒ a = 2 14 A lies on tangent plane so
4+ 1+ 9 14
4r + 2 + 4r − 4 + r − 1 + 12 = 0 ⇒ r = − 1
14. Let the tangent plane in intercept from is
Thus co-ordinate of point of contact A is (−1, 4, − 2).
x y z
+ + = 1.
a b c 43. The equation of sphere is

Since, it is tangent on given sphere s. x 2 + y 2 + z 2 + 2ux + 2vy + 2wz + d = 0 …(1)


1
= radius = r It pass through (1, 0, 0), so 1 + 2u + d = 0
1 1 1
+ + 1 + d
a2 b2 c2 ⇒ u = −  
 2 
1 1 1 1
⇒ + + = 1 + d
a2 b2 c2 r2 Similarly it pass through (0, 1, 0) so v = −  
 2 
r1r2 3 × 4 12
18. r= = = 1 + d
r12 + r22 5 5 and pass through (0, 0, 1) so w = −  
 2 
x y z
24. Let the plane be + + = 1 it pass through Thus radius of the sphere is R = u 2 + v 2 + w 2 − d
p q r
a b c 1 + d
2
3d 2 + 2d + 3
(a, b, c) so + + = 1 equation of sphere is
p q r or R 2 = 3   −d =
 2  4
66

For minima D (R 2) = 0 ⇒ 6d + 2 = 0 i.e. or 3 (x 2 + y 2 + z 2) − 2x − 2y − 2z = 1


1
d=− 48. In this case sphere pass through (0, 0, 0) (a, 0, 0)
3
(0, b, 0) and (0, 0, c) so its equation is
1
∴ u =v=w = −
3 x 2 + y 2 − ax − by − cz = 0
❍❍❍
So by equation (1) required sphere is
2x 2y 2z 1
x 2 + y2 + z 2 − − − − =0
3 3 3 3
67

C HAPTER Unit-III

9 The Cone

CONE ax 2 + 2hxy + by 2 + 2gx + 2 fy + c = 0, z = 0 is


a (αz − γx )2 + 2h (αz − γx )(βz − γy)
1. A cone is a surface generated by a moving
straight line which passes through a fixed point + b(βz − γy)2 + 2g (αz − γx )(z − γ )
and intersects a given curve or touches a given
surface. The fixed point is called the vertex and + 2 f (βz − γy)(z − γ ) + c (z − γ )2 = 0
the given curve is called the guiding curve of the 7. The condition that the general equation of
cone. The moving straight line is called a second degree ax 2 + by 2 + cz 2 + 2 fyz + 2gzx
generator of the cone.
+ 2hxy + 2ux + 2vy + 2wz + d = 0 represents a
2. A cone which is cut by a straight line other than cone is
the generators only at two points is called a
a h g u
quadratic cone.
h b f v
3. The equation of a quadratic cone with its vertex =0
at the origin is a homogeneous equation of g f c w
second degree in x, y, z i.e. u v w d
ax 2 + by 2 + cz 2 + 2 fyz + 2gzx + 2hxy = 0.
Thus every homogeneous equation of second
TANGENT PLANE
degree in x, y and z represent a cone whose 1. The tangent plane at a point to the cone is the
vertex is at the origin. locus of the tangent lines through that point
drawn in all directions.
x y z
4. The lines = = is a generator of the cone 2. The equation of the tangent plane at the point
l m n
(x 1, y 1, z 1) of the cone ax 2 + by 2 + cz 2 + 2 fyz
whose vertex at the origin i.e.
f (x , y, z ) ≡ ax 2 + by 2 + cz 2 + 2 fyz + 2gzx + 2hxy = 0 is x (ax 1 + hy 1 + gz 1)
+ y(hx 1 + by 1 + fz 1) + z (gx 1 + fy 1 + cz 1) = 0
+ 2gzx + 2hxy = 0 iff its direction ratios l, m, n The tangent plane at any point of a cone passes
satisfy the equation of cone i.e. f (l , m, n) = 0 through its vertex.

5. The general equation of a cone of second 3. The condition that the plane lx + my + nz = 0
degree which passes through the co-ordinate may touch the cone ax 2 + by 2 + cz 2 + 2 fyz
axes, the axes being rectangular is + 2gzx + 2hxy = 0 is Al 2 + Bm 2 + Cn 2
fyz + gzx + hxy = 0. + 2Fmn + 2Gnl + 2Hlm = 0 where

6. The equation of a cone whose vertex is the A = bc − f 2 , B = ca − g 2 , C = ab − h 2 ,


point (α, β, γ ) and base the conic F = gh − af , G = hf − bg , H = fg − ch.
68

RECIPROCAL CONE it makes a constant angle with a fixed line


1. The reciprocal cone of a given cone is the locus (called axis) through the fixed point. The
of the normal through the vertex of the cone to constant angle is called the semi-vertical angle.
the tangent planes of the given cone. 2. The equation of the right circular cone with
2. The equation of the cone reciprocal to the cone vertex at (α, β, γ ) and l, m, n as direction ratios of
ax 2 + by 2 + cz 2 + 2 fyz + 2gzx + 2hxy = 0 is the axis is [l (x − α) + m (y − β) + n (z − r )]
= cos2 θ (l 2 + m 2 + n 2 ) x
Ax 2 + By 2 + Cz 2 + 2Fyz + 2Gzx + 2Hxy = 0
where A, B, C, F, G, H are defined in point 3. [(x − α)2 + (y − β)2 + (z − γ )2 ]

3. Two cones which are such that each is the locus 3. The equation of the right circular cone with
of the normals through the vertex to the tangent vertex at origin, axis the z-axis and
planes to the other are called reciprocal cones. semi-vertical angle θ is x 2 + y 2 = z 2 tan 2 θ.

4. The condition of perpendicularity between the 4. If the vertex at the origin, axis the y-axis
lines in which the plane ux + vy + wz = 0 cuts (respectively x-axis) and semi-vertical angle is θ
the cone f (x , y, z ) ≡ ax 2 + by 2 + cz 2 is
+ 2 fyz + 2gzx + 2hxy = 0 is (a + b + c) x 2 + z 2 = y 2 tan 2 θ
2 2 2
(u + v + w ) = f (u, v, w).
(respectively y 2 + z 2 = x 2 tan 2 θ)
5. The condition of the tangency of a plane and a
cone given in point 4 is THE ENVELOPING CONE
2 2 2
Au + Bv + Cw + 2Fvw + 2Gwu 1. The locus of the tangents from a given point to
a given surface is a cone called the enveloping
+ 2Huv = 0
cone.
THREE MUTUALLY PERPENDICULAR
GENERATORS 2. The equation of the (tangent cone or)
enveloping cone from the point (x 1, y 1, z 1) to
1. The condition that the cone
the sphere x 2 + y 2 + z 2 = a 2 is SS1 = T 2
ax 2 + by 2 + cz 2 + 2 fyz + 2gzx + 2hxy = 0 where S = x 2 + y 2 + z 2 − a 2,
may have three mutually perpendicular S1 = x 12 + y 12 + z 12 − a 2
generators is a + b + c = 0.
and T = xx 1 + yy 1 + zz 1 − a 2
2. The condition that the cone
3. The equation of the enveloping cone for the
ax 2 + by 2 + cz 2 + 2 fyz + 2gzx + 2hxy = 0
conicoid ax 2 + by 2 + cz 2 = 1 from the point
may have three mutually perpendicular tangent
(x 1, y 1, z 1) is (ax 2 + by 2 + cz 2 − 1)
planes is f 2 + g 2 + h 2 = ab + bc + ac.
(ax 12 + by 12 + cz 12 − 1)
RIGHT CIRCULAR CONE
= (axx 1 + byy 1 + czz 1 − 1)2
1. A right circular cone is a surface generated by a
straight line which moves in such a way that it
passes through a fixed point (called vertex) and
69

EXERCISE
MULTIPLE CHOICE QUESTIONS 6. The reciprocal cone of the given cone
2 2 2
1. Every homogeneous equation of second degree in ax + by + cz = 0 is

x, y and z always represent a x 2 y2 z 2


(a) + + =0
a b c
(a) cone (b) cylinder
x 2 y2 z 2
(c) circle (d) sphere (b) + + =1
a b c
2. The equation of the cone whose vertex is the origin (c) ax 2 + by 2 + cz 2 = 0
and base the curve z = k, f (x, y) = 0 is
x2 y2 z2
[Avadh 2018] (d) + + =0
2 2
a b c2
xk yk 
(a) f  ,  =0
 z z  7. If the cone x 2 + ay 2 − 3z 2 − hz + zx − 2xy = 0

 zk zk  have three mutually perpendicular generators then


(b) f  ,  = 0
 x y the value of a is

xz yz (a) −2 (b) −1 (c) 1 (d) 2


(c) f  ,  = 0
k k  8. The equation

k k  ax 2 + by 2 + cz 2 + 2 fyz + 2gzx + 2hxy + d = 0


(d) f  ,  = 0
 xz yz 
represents a cone only when
3. The vertex of the cone (a) a = b = c = d (b) f = g = h = d
2 2 2
4x − y + 2z + 2xy − 3yz + 12x (c) d = 0 (d) d = 1
9. The general equation of second degree of a cone
− 11y + 6z + 4 = 0 is [Kanpur 2018]
which pass through the co-ordinate axes is
(a) (1, 2, 3) (b) (1, − 2, 3) [Agra 2018]
(c) (1, − 2, − 3) (d) (−1, − 2, − 3) (a) x 2 + y 2 + z 2 + fyz + gzx + hxy = 0
4. One of the equation of the line in which plane (b) fyz + gzx + hxy = 0
2x + y − z = 0 cuts the cone 4x 2 − y 2 + 3z 2 = 0 is (c) y 2 + fyz + gzx = 0

(a)
x y z
= = (b)
x y z
= = (d) x 2 + fyz + gzx + hxy = 0
−1 2 0 −1 2 1
10. The condition that the equation
x y z x y z
(c) = = (d) = =
2 −1 0 −1 0 2 ax 2 + by 2 + cz 2 + 2ux + 2vy + 2wz + d = 0

5. The equation to reciprocal cone of the given cone represents a cone is


u v w
fx ± gy ± hz = 0 is (a) + + =d
a b c
(a) fyz + gzx + hxy = 0
a 2 b2 c 2
(b) + + =d
(b) y fx + z gy + x hz = 0 u v w
(c) fx + gy + hz = 0 a b c
(c) + + =d
u v w
(d) fyz + gzx + hxy = 0
u 2 v2 w 2
(d) + + =d
a b c
70

11. The plane ax + by + cz = 0 cuts the cone 17. If a right circular cone has three mutually
yz + zx + xy = 0 in perpendicular lines only when perpendicular tangent planes then its semi-vertical
[Kanpur 2018, Avadh 2018] angle is
−1
(a) a + b + c = 0 (b) (a + b + c) = abc (a) tan −1 ( 2) (b) cot −1 ( 2)
1 1 1
(c) + + =0 (d) abc = 0 (c) sin −1 ( 2) (d) cos −1 ( 2)
a b c
12. The angle between the lines of section of the plane 18. The surface generated by among straight line which
3x + y + 5z = 0 and the cone 6yz − 2zx + 5xy = 0 passes through a fixed point and intersects a given
is curve is called

1  2  (a) cylinder (b) conicoid


(a) cos −1   (b) cos −1  
 3 2  3 (c) sphere (d) cone

1  1 19. The equation of the cone with vertex at the origin


(c) cos −1   (d) cos −1  
 6  6 and direction cosines of its generators satisfy the

13. The equation of cone whose vertex is (1, 1, 0) and the relation 2 l 2 + 3 m 2 − n2 = 0 is

guiding curve is x 2 + z 2 = 4, y = 0 is (a) 3x 2 + 2y 2 + z 2 = 0

(a) x 2 + 3y 2 + z 2 + 2yz + 6x = 4 (b) 2x 2 + 3y 2 = 0

(c) 2x 2 + 3y 2 − z 2 = 0
(b) x 2 − 3y 2 + z 2 − 2xy + 8y = 4
(d) 3y 2 − z 2 = 0
(c) x 2 + 3y 2 − z 2 − 2yz + 6x = 4
20. The equation
(d) none of these
x 2 + y 2 + z 2 + 2ux + 2vy + 2wz + d = 0
14. The equation of the cone with vertex at the origin,
semi-vertical angle is θ and axis as x-axis is represents a cone if
[Avadh 2018] (a) u + v + w = d
2
(a) y + z = x 2 2 1 1 1
(b) + + =d
u v w
(b) y 2 − z 2 = x 2
(c) u 2 + v 2 + w 2 = d
2 2 2 2
(c) y + z = x tan θ
(d) u + v + w = d 2
2 2 2
(d) y + z = tan θ
21. The equation of the tangent plane at the point
15. If the cone px 2 + y 2 + 2z 2 − 2yz + 4zx + 2xy have (1, 0, 0) of the cone
three mutually perpendicular tangent planes then x 2 + y 2 + z 2 + 2yz + 2zx + 2xy = 0 is
the value of p is
(a) x + y + z = 0
−4 4
(a) (b) (b) x + 2y + 3z = 0
3 3
3 −3 (c) 2x + y + 3z = 0
(c) (d)
4 4 (d) x − y − z = 0
16. The section of a right circular cone by any plane 22. The tangent plane at any point of a cone always
perpendicular to its axis is passes through its
(a) circle (b) ellipse (a) guiding curve (b) generator
(c) triangle (d) square (c) guiding surface (d) vertex
71

23. If the angle between the lines given by 29. The locus of the perpendiculars drawn from the
π origin to the tangent planes to the cone
x + y + z = 0, ayz + bzx + cxy = 0 is then
2 ax 2 + by 2 + cz 2 = 0 is
(a) a + b + c = 0 (a) cone (b) circle
1 1 1
(b) + + = 0 (c) sphere (d) cylinder
a b c
30. The number of sets of three mutually perpendicular
(c) a 2 + b2 + c 2 = 0
generators of the cone are
1 1 1
(d) + + =0 (a) unique (b) finite
a2 b2 c2
(c) infinite (d) only 1
24. The equation of the cone whose vertex is (1. 2, − 2)
31. If the cone 2x − 5y + az 2 + 3yz + 4zx + 5xy = 0
2 2
and guiding curve is the parabola y 2 = 4ax, z = 0 is
have three mutually perpendicular generators then
(a) (y + 2)2 = a (z − 2x)(z − 2)
a is equal to
2
(b) (y + 2) = a (z + 2x)(z + 2) (a) 4 (b) 3
2 (c) 5 (d) 2
(c) (y + 2) = 4a (z − 2x)(z − 2)

(d) (y + 2)2 = 4a (z + 2x)(z + 2) 32. The equation of right circular cone where vertex is
at the origin, axis the x-axis and semi-vertical angle
25. If the plane 2x − y + pz = 0 cuts the cone is 30° is
yz + zx + xy = 0 in perpendicular lines then the
(a) 3(y 2 + z 2) = x 2
value of p is
(a) 1 (b) 2 (b) y 2 + z 2 = x 2

(c) 3 (d) 4 (c) 3(x 2 + y 2) = z 2


26. If the plane 3x − y + az = 0 cuts the cone (d) 3(x 2 + z 2) = y 2
yz + zx + xy = 0 in perpendicular lines then the
33. The locus of the tangent from a given point to a
value of a is
given surface is called
(a) 2 (b) 3
(a) reciprocal cone
3 2
(c) (d)
2 3 (b) cylinder

27. The vertex of the cone through which all the tangent (c) right circular cone
planes pass is called (d) enveloping cone
(a) nodal point 34. The equation of the cone reciprocal to the cone
(b) conjugate point x 2 + y 2 + z 2 = 0 is
(c) limit point
(a) x 2 + y 2 + z 2 + xy + yz + zx = 0
(d) singular point
(b) x 2 + y 2 + z 2 = 0
28. The equation fx + gy + hz = 0 represents a
(a) plane (b) cone (c) xy + yz + zx = 0
(c) cylinder (d) conicoid
(d) x 2 + y 2 + z 2 + 2xy + 2yz + 2zx = 0
72

35. The equation of the cone formed by rotating the line 41. If the plane x + y + z = 1 meets the co-ordinate
2x + 3y = 6, z = 0 about the y-axis is axes in A, B, C then the equation of the cone
2 2
(a) x + 9y + 4z − 36y − 36 = 0 2 generated by the lines drawn from O to meet the
circle ABC is
(b) 4x 2 + 9y 2 + 2z 2 + 36y − 36 = 0
(a) x 2 + y 2 + z 2 + xy + yz + zx = 0
2 2 2
(c) x + 9y + 4z − y − 4 = 0
(b) xy + yz + zx = 0
2 2 2
(d) 4x − 9y + 4z + 36y − 36 = 0
(c) x 2 + y 2 + z 2 = xy
36. The condition that the cone
(d) none of these
2 2 2
ax + by + cz + 2 fyz + 2gzx + 2hxy = 0 42. If a plane x+ y+ z=0 cuts the cone
have three mutually perpendicular generators is ax 2 + by 2 + cz 2 = 0 in perpendicular generators
1 1 1 then
(a) a + b + c = 0 (b) + + =0
a b c
(a) a + b + c = 0
1 1 1
(c) ab + bc + ca = 0 (d) + + =0 (b) a + b + c = 1
a2 b2 c2
1 1 1
37. If a right circular cone has three mutually (c) + + =0
a b c
perpendicular generators then its semi-vertical 1 1 1
(d) + + =1
angle is a b c
(a) cot −1( 2) (b) sin −1( 2)
43. If the plane ax − by + cz = 0 cut the cone
(c) tan −1( 2) (d) cos −1( 2) x 2 + y 2 + z 2 = 0 in perpendicular generator then
38. If the generator of the cone (a) a + b + c = 0
2 2 2 1 1 1
ax + by + cz + 2 fyz + 2gzx + 2hxy = 0 (b) + + = 0
a b c
is y-axis then the value of b is (c) a 2 + b2 + c 2 = 0
(a) 0 (b) 1 1 1 1
(d) + + =0
(c) −1 (d) any number a2 b2 c2
39. The equation of the tangent plane at the point 44. If the cone 2x 2 + y 2 + az 2 + 2yz + 6zx + 4xy = 0
(1, 1, 1) of the cone
have three mutually perpendicular tangent planes
x 2 + y 2 + z 2 + 4yz + 4zx + 4xy = 0 is only when
(a) 4x + 2y + 3z = 0 (b) x + y + z = 0 (a) a = 2 (b) a = 3
(c) x + 4y + 2z = 0 (d) 2x + 3y + 4z = 0 (c) a = 4 (d) a = 6
40. The equation 45. If the angle between the lines in which the plane
2 2 2 ux + vy + wz = 0 cuts the cone
ax + by + cz + 2x + 2y + 2z + 1 = 0
π
x 2 + y 2 + z 2 + 2yz + 2zx + 2xy = 0 is then
represents a cone if 2
(a) a + b + c = 1 (a) f (u, v, w) = uvw
2 2 2
(b) a + b + c = 1 (b) f (u, v, w) = u + v + w

(c) a + b + c = 0 (c) f (u, v, w) = u 2 + v 2 + w 2


1 1 1
(d) + + =1 (d) f (u, v, w) = 0
a b c
73

46. The vertex of the cone 53. The number of concurrent lines that must be
2 2 2
2x + 2y + 7 z − 10yz − 10zx + 2x required to determine a cone are

+ 2y + 26z − 17 = 0 is (a) 2 (b) 3

(a) (2, 1, 2) (b) (1, 2, 2) (c) 4 (d) 5


2 2 2
(c) (2, 2, 1) (d) (2, 2, 2) 54. If ax + by + cz + d = 0 represents a cone then

47. The vertex of the cone xy + 2yz + 3zx = 0 is (a) a = b = c (b) a = b = d

(a) (0, 0, 0) (b) (1, 2, 3) (c) b = c = d (d) d = 0

(c) (1, 0, 0) (d) (0, 2, 0) 55. The equation of the cone with vertex at origin
π
48. If a cone have three mutually perpendicular tangent semi-vertical angle is and axis as z-axis is
4
planes then its reciprocal cone has three mutually
(a) x 2 + y 2 − z 2 = 0
(a) parallel generators
(b) perpendicular generators (b) y 2 + z 2 − x 2 = 0

(c) perpendicular tangent planes (c) z 2 + x 2 − y 2 = 0


(d) parallel tangent planes (d) x 2 + y 2 + z 2 = 0
x y z
49. The straight lines = = is a generator of the 56. The section of a right circular by any plane is
l m n
cone 3x 2 − 2y 2 + 4z 2 = 0 if l, m, n satisfy the (a) circle (b) ellipse

condition (c) hyperbola (d) (a) or (b)

(a) l 2 + m 2 + n2 = 1 57. The equation x + y+ z = 0 represents a


(a) plane (b) line
(b) l 2 + m 2 + n2 = 0
(c) cylinder (d) cone
(c) 3 l 2 − 2 m 2 + 4n2 = 0
58. The coordinates of vertex of the cone
(d) none of these
4x 2 − y 2 + 2z 2 + 2xy − 3yz + 12x − 11y + 6z + 4 = 0
50. A straight line lying on the surface of a cone is is
called : [Kanpur 2018]
(a) tangent (b) generator (a) (−1, 2, 3) (b) (−1, − 2, 3)
(c) normal (d) axis (c) (−1, − 2, − 3) (d) (1, − 2, − 3)
51. The tangent plane at any point of a cone always 59. Enveloping cone of the sphere
pass through its
x 2 + y 2 + z 2 + 2x − 2y = 2 with its vertex at (1, 1, 1)
(a) guiding curve (b) tangent plane
is
(c) normal plane (d) vertex [Kanpur 2018]
52. The number of arbitrary constants in the 2 2
(a) 3x − y + 4zx + 10x + 2y − 4z + 6 = 0
homogeneous equation of second degree in x, y, z of
the cone are (b) 3x 2 − y 2 + 4zx + 10x − 2y − 4z − 6 = 0
(a) 4 (b) 5 (c) 3x 2 − y 2 + 4zx + 10x − 2y + 4z − 6 = 0
(c) 6 (d) 3
(d) 3x 2 − y 2 + 4zx − 10x + 2y − 4z + 6 = 0
74

60. What is the vertex of the cone 63. The angle between the lines of section of the plane
6x − y − 2z = 0 and the cone 108x 2 − 7 y 2
ax 2 + by 2 + cz 2 + 2 fyz + 2gzx + 2hxy = 0 ?
− 20z 2 = 0 is [Kanpur 2018]
[Avadh 2018]
20
(a) ( f , g, h) (b) (a, b, c) (a) cos −1  
 21 
(c) (1, 1, 1) (d) (0, 0, 0)
20
61. Equation of right circular cone whose axis is z-axis (b) cos −1  
 23 
and semi-vertical angle α is [Avadh 2018]
19
(c) cos −1  
(a) x 2 + y 2 + z 2 tan 2 α = 0  20 

(b) x 2 + y 2 = z 2 tan 2 α (d) none of these


64. The enveloping cone of the sphere
(c) x 2 + z 2 = y 2 tan 2 α
x 2 + y 2 + z 2 − 2x + 4z = 1 with the vertex (1, 1, 1)
(d) y 2 + z 2 = x 2 tan 2 α
is : [Avadh 2018]
62. The cone generated by normals at vertex to a given
2 2 2
(a) 4x + 3y − 5z − 6yz − 8x + 16z − 4 = 0
cone is called as
[Avadh 2018] (b) x 2 + y 2 + z 2 + 2xy = 0
(a) original cone (b) enveloping cone (c) x 2 + y 2 − z 2 + xy + 2yz = 0
(c) reciprocal cone (d) none of these
(d) none of these

ANSWERS
MULTIPLE CHOICE QUESTIONS

1. (a) 2. (a) 3. (d) 4. (a) 5. (d) 6. (a) 7. (d) 8. (c) 9. (b) 10. (d)

11. (c) 12. (d) 13. (b) 14. (c) 15. (b) 16. (a) 17. (b) 18. (d) 19. (c) 20. (c)

21. (a) 22. (d) 23. (a) 24. (b) 25. (b) 26. (c) 27. (d) 28. (b) 29. (a) 30. (c)

31. (b) 32. (a) 33. (d) 34. (b) 35. (d) 36. (a) 37. (c) 38. (a) 39. (b) 40. (d)

41. (b) 42. (a) 43. (c) 44. (c) 45. (c) 46. (c) 47. (a) 48. (b) 49. (c) 50. (b)

51. (d) 52. (b) 53. (d) 54. (d) 55. (a) 56. (d) 57. (d) 58. (c) 59. (d) 60. (d)

61. (b) 62. (c) 63. (a) 64. (a)


75

HINTS AND SOLUTIONS


3. Let f = 4x 2 − y 2 + 2z 2 + 2xy − 3yz through y-axis and z-axis then b = 0 and c = 0. Thus
the required cone is fyz + gzx + hxy = 0
+ 12xt − 11yt + 6zt + 4t 2
11. Let the equation of a line of section of given cone by
∂f x y z
= 8x + 2y + 12t = 0 the plane is = = then mn + nl + lm = 0 and
∂x l m n
∂f al + bm + cn = 0. By last two equations we get
= − 2y + 2x − 3z − 11t = 0
∂y 2
l l l l b
a   + (a + b − c)   + b = 0 i.e. 1 ⋅ 2 =
m m m1 m2 a
∂f
= 4z − 3y + 6t = 0
∂z l1l 2 m1m2 n1n2
Thus = = . So the lines of section will
1 1 1
∂f
= 12x − 11y + 6z + 8t = 0 a b c
∂t
be perpendicular if l1l 2 + m1m2 + n1n2 = 0 i.e.
After solving these equations we get x = − 1, 1 1 1
+ + =0
a b c
y = − 2, z = − 3
x −1 y−1 z
13. Equation of line through (1, 1, 0) is = =
5. Given fx ± gy ± hz = 0 l m n
⇒ fx ± gy = ∓ hz x − 1 −1 z
it meets at y = 0 at the point = = i.e.
l m n
Squaring both sides we get  1 − l , 0, − n  . This point lie on x 2 + z 2 = 4. So
 
fx + gy − hz = ∓ 2 fgxy again squaring we get  m m
f 2x 2 + g 2y 2 + h2z 2 − 2 fgxy − 2 fhzx − 2ghyz = 0 2 2
 1 − l  + n = 4, by the line l = x − 1 and
 
 m m2 m y−1
it represents a cone with vertex at origin.
n z
= . Put in above equation we get
Let its reciprocal cone is m y−1
2 2 2
Ax + By + Cz + 2Fyz + 2Gzx + 2Hxy = 0 x 2 − 3y 2 + z 2 − 2xy + 8y = 4
2
where A = bc − f = 0, B = C = 0,
15. The cone ax 2 + by 2 + cz 2 + 2 fyz + 2gzx
F = gh − af = 2 f gh, G = 2g hf , H = 2h2 fg
2 2 2

2hxy = 0 has three mutually perpendicular tangent


So reciprocal cone is fyz + gzx + hxy = 0.
plane when ab + bc + ac = f 2 + g 2 + h2 so by
7. If the cone given cone p + 2 + 2p = 1 + 4 + 1 ⇒ 3p = 4 ⇒
2 2 2 4
ax + by + cz + 2 fyz + 2gzx + 2hxy = 0 has p=
3
three mutually perpendicular generator then
a + b + c = 0 so by given cone 1 + a − 3 = 0 ⇒ a = 2 26. Required condition is (a + b + c)(u 2 + v 2 + w 2)
3
9. Let the cone is = f (u, v, w). So we get 0 = − 3 − a + 3a ⇒ a =
2
ax 2 + by 2 + cz 2 + 2 fyz + 2gzx + 2hxy = 0 28. After solving it we get a homogeneous equation of
If x-axis is its generator then d.c. of x-axis 1, 0, 0 degree two so it represents a cone.
satisfy it i.e. a = 0. Similarly if generator pass 31. 2 − 5 + a = 0⇒a = 3
76

32. The direction ratios of OP and OX are x, y, z and 1, ⇒ 3(y 2 + z 2) = x 2


0, 0 respectively
39. Tangent plane at (x1, y1, z1) of the cone
y, z)
P(x,
2 2 2
ax + by + cz + 2 fyz + 2gzx + 2hxy = 0 is
z
x, y,
d.r. x (ax1 + hy1 + gz1) + y (hx1 + by1 + fz1)
(0, 0, 0) O 30° x + z (gx1 + fg1 + cz1) = 0.
d.c. 1, 0, 0
So required plane is
x (1 + 2 + 2) + y (2 + 1 + 2) + z (2 + 2 + 1) = 0
x ⋅1 + y⋅ 0 + z ⋅ 0 3 ⇒ x+ y+ z=0
So cos 30° = =
1+ 0+ 0 x + y + z 2 2 2 2 ❍❍❍
77

C HAPTER Unit-III

10 The Cylinder

THE CYLINDER
1. A cylinder is a surface generated by a straight RIGHT CIRCULAR CYLINDER
line which is parallel to a fixed line, and satisfies 1. A cylinder whose guiding curve is a fixed circle
one more condition is it may intersect a fixed and whose generator is normal to the plane of
curve or touch a given surface. Any line on the the circle is called right circular cylinder. The
surface of the cylinder is called its generator. radius of the circle is called the radius of the
The fixed line is called the axis of the cylinder. right circular cylinder.
If all the generators of a cylinder intersect a
curve, then curve is called the guiding curve. 2. The equation of the right circular cylinder
whose radius is r and axis the line
2. The equation of a cylinder whose generators x −α y−β z − γ
x y z = = is
are parallel to the line = = and intersect l m n
l m n
2 2 [(y − β) n − (z − γ ) m]2 + [(z − γ ) l − (x − α) n]2 +
the conic ax + by + 2hxy
[(x − α) m − (y − β) l ]2 = r 2 (l 2 + m 2 + n 2 )
+ 2gx + 2 fy + c = 0, z = 0 is a(nx − lz )2 + 2h
3. The equation of the right circular cylinder
(nx − lz ) (ny − mz ) + b(ny − mz )2 + 2gn whose axis is the z-axis, and radius r is

(nx − lz ) + 2 fn(ny − mz ) + cn 2 = 0 x 2 + y2 = r 2

3. If z-axis be the axis of cylinder then by point 2 ENVELOPING CYLINDER


the equation of cylinder becomes 1. The enveloping cylinder is the locus of the
2 2
a + 2hxy + by + 2gx + 2 fy + c = 0 tangents to a surface (sphere or conicoid) which
is parallel to a given line.
4. The equation of the cylinder whose axis is
2. The equation of the enveloping cylinder of the
z-axis and whose generators intersect the circle
sphere x 2 + y 2 + z 2 = a 2 , whose generator are
x 2 + y 2 = a 2 , z = 0 is x 2 + y 2 = a 2
x y z
parallel to the line = = is
5. The equation f (x , y) = 0, f (y, z ) = 0 and a m n
f (z , x ) = 0 respectively represents a cylinder
(x 2 + y 2 + z 2 − a 2 )(l 2 + m 2 + n 2 )
with its generators parallel to z-axis, x-axis and
y-axis respectively. = (lx + my + nz )2
78

EXERCISE
MULTIPLE CHOICE QUESTIONS 7. The equation of the right circular cylinder of
radius 2 whose axis is the y-axis is
1. The equation of the cylinder in three-dimensions
whose generators are parallel to the z-axis is (a) x 2 + y 2 = 4

(a) F (x, y) = 0 (b) F (y, z) = 0 (b) y 2 + z 2 = 4


(c) F (z, x) = 0 (d) F (x, y, z) = 0 (c) x 2 + y 2 + z 2 = 4
2 2
2. In three-dimension geometry x + y = a always (d) x 2 + z 2 = 4
represents a
8. The surface generated by a variable line which
(a) circle (b) cone
moves parallel to a fixed line and intersect a given
(c) sphere (d) cylinder
cone is called
3. The equation of the cylinder whose axis is z-axis (a) plane (b) sphere
and whose generator intersect the circle (c) cone (d) cylinder
x 2 + y 2 = a 2, z = 0 is 9. The equation of the right circular cylinder described
(a) x 2 + y 2 + z 2 = a 2 on the circle through the points (1, 0, 0), (0, 1, 0),
(0, 0, 1) as the guiding curve
(b) x 2 + z 2 = a 2
(a) x 2 + y 2 + z 2 + xy + yz + zx + 1 = 0
(c) x 2 + y 2 = a 2
(b) x 2 + y 2 + z 2 − xy − yz − zx − 1 = 0
(d) y 2 + z 2 = a 2
(c) x 2 + y 2 + z 2 + xy − yz − zx − 1 = 0
4. The generators of the cylinder y 2 + z 2 = a 2 are
(d) x 2 + y 2 + z 2 − xy − yz + zx + 1 = 0
straight lines parallel to
(a) y-axis (b) z-axis 10. In three-dimension geometry F (y, z) = 0 represents

(c) yz-axis (d) x-axis (a) sphere (b) cylinder


(c) hyperboloid (d) ellipse
5. The equation of a right circular cylinder whose axis
is y-axis is [Avadh 2018] 11. The envelope of the sphere x 2 + y 2 + z 2 = a 2
x y z
(a) x 2 + y 2 = r 2 whose generators are parallel to the line = =
l m n
(b) y 2 + z 2 = r 2 represents
2 2 2
(c) x + z = r (a) cylinder (b) sphere
2 2
(d) x + y + z = r 2 2 (c) cone (d) circle
12. The equation of the cylinder with generator parallel
6. The equation of the cylinder with generator parallel
to z-axis and passing through two curves
to z-axis and passing through the curve
2 2
x + y = z, x + y + z = 1 is
x 2 + y 2 = 2z, x + y + z = 1 is
(a) x 2 + y 2 + z 2 + x + y = 2
(a) x 2 + y 2 + z 2 + x + y = 2
(b) x 2 + y 2 + x + y = 1
(b) x 2 + y 2 + 2x + 2y = 2
(c) x 2 + y 2 + z 2 + 2x + 2y = 2
(c) x 2 + y 2 + z 2 + 2x + 2y = 2
(d) x 2 + y 2 − 2x − 2y = 2
(d) x 2 + y 2 − 2x − 2y = 2
79

13. All the generators of a cylinder are straight line 19. If the equation of enveloping cylinder of conicoid
which are x 2 + y 2 + z 2 = 1 whose generators are parallel to
(a) perpendicular the line x = y = z is given by
(b) parallel x 2 + y 2 + z 2 − xy − yz − zx = p then p is
(c) intersect at a point 3
(a) 1 (b)
(d) intersect at infinite point 2
2
14. The surface generated by the tangent’s lines to the (c) (d) 2
3
sphere x 2 + y 2 + z 2 = a 2 which are parallel to the
20. The equation of the right circular cylinder whose
x y z
line = = is a axis is x = z + 2, y = 0 and which pass through the
l m n
point (3, 0, 0) is
(a) sphere (b) cone
(a) x 2 + y 2 + z 2 − 2zx + 4x + 4z + 3 = 0
(c) cylinder (d) conicoid
15. The equation of the cylinder which intersect two (b) x 2 + y 2 + 3zx − 4x + y + 3 = 0

curves and whose generators are parallel to z-axis (c) x 2 + 2y 2 + z 2 − 2zx − 4x + 4z + 3 = 0


is : (d) x 2 + y 2 + z 2 − 4xy + y + 3 = 0
(a) f (y, z) = 0 (b) f (x, z) = 0
21. The surface generated by a variable line which
(c) f (x, y) = 0 (d) f (x, y, z) = 0
moves parallel to a fixed line and intersects a given
16. The equation of the cylinder with generators parallel curve is
to y-axis and intersect the curve x 2 + y 2 + 2z 2 = 12, (a) x 2 + y 2 + z 2 = 0
x − y + z = 1 is
(b) x 2 + y 2 + 2hxy = 0
(a) x 2 + y 2 + 2z 2 = 12
(c) x 2 + y 2 − z 2 = a 2
(b) x 2 + (x + z + 1)2 + z 2 = 12
(d) x 2 + y 2 = r 2
2 2 2
(c) x + (x + z − 1) + 2z = 12
22. The equation of the cylinder whose generators are
(d) x 2 + (x + z − 1)2 − z 2 = 12 x y z
parallel to the line = = and whose guiding
0 1 −1
17. In three-dimension geometry
curve is x 2 + 2z 2 = 1, y = 0, is
ax 2 + 2hxy + by 2 + 2gx + 2 fy + c = 0 represents
(a) x 2 + 2(y − z)2 = 1
a:
(b) y 2 + 2(z − y)2 = 1
(a) cone (b) cylinder
(c) x 2 + 2 (y + z)2 = 1
(c) sphere (d) conic
(d) x 2 + 2(y + z)2 = 2
18. The equation of the cylinder whose generators are
parallel to the line x = − y = z and whose guiding 23. The equation of the right circular cylinder of radius
curve is the ellipse x 2 + y 2 = 1, z = 0 is x y z
1 whose axis is the line = = is
1 0 0
(a) x 2 + y 2 + 2z 2 + 2yz − 2zx = 1 (a) y 2 + z 2 + 2y + 2z − 2 = 0
(b) x 2 + y 2 + z 2 + 2yz + 2zx + 2xy = 1 (b) y 2 + z 2 + x + 2y − 2 = 0

(c) x 2 + y 2 + 2z 2 + 2yz + 2zx + 2xy = 0 (c) y 2 + z 2 + 2x + 2y − 2 = 0

(d) x 2 + y 2 − z 2 + 2yz + 2zx − 2xy = 1 (d) y 2 + z 2 + 2x − 2 = 0


80

24. The equation of the cylinder with generator parallel 31. The locus of the lines parallel to a given line r or to
to z-axis and passing through the curve touch a given surface is called
2 2
x + y = 3z, x + y + z = 1 is (a) circle
2 2
(a) x + y + 3x + 3y + 2 = 0 (b) enveloping cylinder
(c) cylinder
(b) x 2 + y 2 + 3y + 3z = 3
(d) sphere
(c) x 2 + y 2 + 3x + 3y = 3
32. The normal to the plane of guiding circle through its
(d) x 2 + y 2 + 3x − y = 3 centre for a cylinder is called
25. The equation of the cylinder whose axis is x-axis (a) generator
and whose generators intersect the circle (b) guiding curve
y 2 + z 2 = a 2 and x = 0 is (c) axis

(a) x 2 + y 2 = a 2 (d) envelope


33. The equation of the cylinder whose generators are
(b) y 2 + z 2 = a 2
parallel to z-axis and which pass through the circle
(c) z 2 + x 2 = a 2
whose centre is (a, 0, 0) and radius a lying in the
(d) x 2 + y 2 + z 2 = 0 plane z = 0 is

26. The equation of the cylinder which intersect two (a) x 2 + y 2 + z 2 + ax = 0


curves and whose generators are parallel to x-axis (b) x 2 + y 2 + ax = 0
is :
(c) x 2 + y 2 = a 2
(a) f (x, y) = 0 (b) f (y, z) = 0
(d) x 2 + y 2 − ax = 0
(c) f (z, x) = 0 (d) f (x, y. z) = 0
27. F (y, z) = 0 represents a cylinder in three-dimension 34. The equation of the cylinder whose generators are
x y z
geometry, whose generators are parallel to the parallel to the line = = and whose guiding
1 −2 3
(a) x-axis (b) y-axis
curve is ellipse x 2 + 2y 2 = 1, z = 3 is
(c) z-axis (d) any axis
[Kanpur 2018]
28. The equation of a right circular cylinder whose axis 2 2 2
(a) 3x + 6y + 3z + 8yz + 2zx
is z-axis is
+ 6x − 24y − 18z + 24 = 0
(a) x 2 + y 2 + z 2 = a 2
2 2 2
(b) 3x + 6y + 3z + 8yz − 2zx
(b) x 2 + y 2 = a 2
+ 6x − 24y − 18z + 24 = 0
(c) y 2 + z 2 = a 2 2 2 2
(c) 3x + 6y + 3z + 8yz − 2zx
(d) z 2 + x 2 = a 2
− 6x − 24y + 18z − 24 = 0
29. The envelope of any sphere whose generators are (d) 3x 2 + 6y 2 + 3z 2 + 8yz − 2zx
parallel to any line represents a
− 6x − 24y + 18z − 24 = 0
(a) cone (b) sphere
35. Guiding curve of a right circular cylinder is
(c) cylinder (d) circle
[Avadh 2018]
30. The section of a right circular cylinder by any plane (a) Circle
perpendicular to its axis is
(b) Ellipse
(a) parabola (b) hyperbola
(c) Any closed curve
(c) ellipse (d) circle
(d) none of these
81

ANSWERS
MULTIPLE CHOICE QUESTIONS

1. (a) 2. (d) 3. (c) 4. (d) 5. (c) 6. (b) 7. (d) 8. (d) 9. (b) 10. (b)

11. (a) 12. (b) 13. (b) 14. (c) 15. (c) 16. (c) 17. (b) 18. (a) 19. (b) 20. (b)

21. (d) 22. (c) 23. (d) 24. (c) 25. (b) 26. (b) 27. (a) 28. (b) 29. (c) 30. (d)

31. (b) 32. (c) 33. (d) 34. (b) 35. (a)

HINTS AND SOLUTIONS


6. Since the generator of the cylinder are parallel to Eliminating r between (3) and (4) we get
z-axis so it does not contain the terms of z so x12 + y12 + z12 − y1z1 − z1x1 − x1y1 = 1
eliminate z between two curves i.e.
so required locus is
x 2 + y2
=1− x − y
2 x 2 + y 2 + z 2 − xy − yz − zx = 1

⇒ x 2 + y 2 + 2x + 2y = 2 12. Since generator is parallel to z-axis so eliminate z


between two equations we get
9. Equation of sphere OABC is
x 2 + y2 = 1 − x − y
x 2 + y2 + z 2 − x − y − z = 0 …(1)
⇒ x 2 + y2 + x + y = 1
and plane ABC is x + y + z = 1 …(2)
Let (x1, y1, z1) lies on the cylinder then its generator 16. Since the axis of cylinder is y-axis so eliminate y

is between two equation, the required equation of


cylinder is
x − x1 y − y1 z − z1
= = =r
1 1 1 x 2 + (x + z − 1)2 + 2z 2 = 12
i.e. a point on generator is (r + x1, r + y1, r + z1) it 18. Let (x1, y1, z1) be the point lies on cylinder then
lies (1) and (2) x − x1 y − y1 z − z1
equation of generator is = = it
1 −1 1
So (r + x1)2 + (r + y1)2 + (r + z1)2
meets z = 0
− (r + x1) − (r + y1) − (r + z1) = 0
x − x1 y − y1
So, = = − z1
and (r + x1) + (r + y1) + (r + z1) = 1 1 −1
⇒ 3r 2 + r (2x1 + 2y1 + 2z1 − 3) i.e. (x1 − z1, y1 + z1, 0) lies on x 2 + y 2 = 1 i.e.

+ (x12 + y12 + z12 − x1 − y1 − z1) = 0 …(3) (x1 − z1)2 + (y1 + z1)2 = 1

1 ⇒ so, locus is (x − z)2 + (y + z)2 = 1


and r= (1 − x1 − y1 − z1) …(4)
3
⇒ x 2 + y 2 + 2z 2 − 2xz + 2yz = 1
82

2
1 
(x − 2)2 + y 2 + z 2 =  
x−2 y z  2
20. Axis of cylinder is = =
1 0 1 2
1 1
A (3, 0, 0) +  (x − 2) + (2)
 2 2 
B After solving we get
(2, 0, 0) C
x 2 + y 2 + 3zx − 4x + y + 3 = 0

d.c.’s 1 , 0, 1 22. If P(x1, y1, z1) lies on cylinder then equation of


Ö2 Ö2 generator is
It pass through B(2, 0, 0) so AB = 1, BC = projection x − x1 y − y1 z − z1
= = , it lies on y = 0 so
1 1 0 1 −1
of AB on BC i.e. BC = + 0+ 0=
2 2 x − x1 − y1 z − z1
= = i.e. P is (x1, 0, z1 + y1) it also
0 1 −1
1
∴ Radius AC = AB 2 − BC2 =
2 lies on x 2 + 2z 2 = 1 so x12 + 2 (z1 + y1)2 = 1, i.e.

Consider a point P(x, y. z) on surface of cylinder locus is x 2 + 2 (z + y)2 = 1]


then by projector rule we get ❍❍❍
C HAPTER Unit-IV
83

11 The Central Conicoids

EQUATION OF CENTRAL CONICOIDS 2. The condition that the plane lx + my + nz = p


may touch the central conicoid
1. The surface ax 2 + by 2 + cz 2 = 1 is called a
2 2 2 l 2 m2 n 2
conicoid because the sections of this surface by ax + by + cz = 1 is + + = p2
a b c
planes parallel to co-ordinate planes are conics.
Then the tangent plane becomes
The conicoid is called a central conicoid
because every surface has a centre. l 2 m2 n 2
lx + my + nz = ± + +
2. The origin is the centre of the central conicoid a b c
ax 2 + by 2 + cz 2 = 1 and the co-ordinate 3. The condition that the plane lx + my + nz = p
planes bisect all chords perpendicular to them. x2 y2 z2
may touch the ellipsoid + + = 1 is
3. The central conicoid is symmetric with respect a2 b2 c2
to all the three co-ordinate planes which are
a 2l 2 + b2m 2 + c 2n 2 = p2
called the principal planes. The three principal
planes taken in pairs intersect in three lines 4. The line perpendicular to the tangent plane to a
called the principal axes of the central conicoid. conicoid ax 2 + by 2 + cz 2 = 1 at any point P on
4. In the central conicoid ax 2 + by 2 + cz 2 = 1, it and passing through P called the normal to
there are four cases with respect to the signs of the conicoid at P.
a, b, c :
The equation of the normal to the central
(i) If all are positive then surface is an
conicoid ax 2 + by 2 + cz 2 = 1 at the point
ellipsoid.
(x 1, y 1, z 1) is
(ii) If only one is negative, the surface is
x − x 1 y − y1 z − z 1
hyperboloid of one sheet. = =
ax 1 by 1 cz 1
(iii) Two are negative, the surface is
hyperboloid of two sheets. In terms of d.c's its equation is
(iv) All are negative, the surface is virtual x − x 1 y − y1 z − z 1
= =
quadric. ax 1 p by 1 p cz 1 p

THE TANGENT AND NORMAL PLANES 5. The equation of the normal to the ellipsoid
x2 y2 z2
1. The equation of the tangent plane to the central + + = 1 at (x 1, y 1, z 1) is
a2 b2 c2
conicoid ax 2 + by 2 + cz 2 = d at the point
(x 1, y 1, z 1) is axx 1 + byy 1 + czz 1 = d x − x 1 y − y1 z − z 1
= =
x 1p y1p z 1p
So the equation of the tangent plane to the a2 b2 c2
2 2 2
x y z
ellipsoid + + at the point (x 1, y 1, z 1) 6. The number of normals that can be drawn to
2 2
a b c2
the conicoid which pass through a given point
xx yy zz
is 1 + 1 + 1 = 1 are six.
2 2
a b c2
84

7. There are S1x feet of normals that can be drawn P and Q satisfying this property are called the
to a conicoid (respectively ellipsoid) from a conjugate points.
given point are the points of intersection of a
4. Two points P(x 1, y 1, z 1) and Q(x 2 , y 2 , z 2 ) are
certain cubic curve with the conicoid
the conjugate points with respect to
(respectively ellipsoid).
ax 2 + by 2 + cz 2 = 1 if and only if
THE DIRECTOR SPHERE ax 1x 2 + by 1y 2 + cz 1z 2 = 1
1. The director sphere of a central conicoid is the 5. The polar line or polar plane of a given line AB
locus of the points of intersection of three is another line PQ such that the polar planes of
mutually perpendicular tangent planes to that all points on AB pass through PQ.
central conicoid.
6. Two lines AB and PQ are called the conjugate
2. The equation of the director sphere of the lines if when PQ intersects the polar line of the
central conicoid ax 2 + by 2 + cz 2 = 1 is line AB then the line AB also intersects the polar
1 1 1 line of PQ.
x 2 + y2 + z 2 = + +
a b c
DIAMETRAL PLANE
Similarly director sphere of the ellipsoid is
1. A diametral plane for a conicoid is the locus of
x 2 + y 2 + z 2 = a 2 + b2 + c 2
the middle points of a system of parallel chords.
THE POLAR PLANE AND CONJUGATE PLANES 2. If the equation of central conicoid is
2 2 2
1. Let ax 2 + by 2 + cz 2 = 1 be the conicoid and ax + by + cz = 1 and equations of a system
A(x 1, y 1, z 1) be any point. Draw line APQ to x −α y−β z − γ
of parallel chords are = =
meet the conicoid in the points P and Q, then l m n
the locus of the point R such that AR is the then the diametral plane is
harmonic mean of AP and AQ is called the alx + bmy + cnz = 0. In case of ellipsoid,
lx my nz
polar plane of the point A with respect to the diametral plane is + + =0
conicoid. The equation of the polar plane is a 2 b2 c 2

axx 1 + byy 1 + czz 1 = 1 3. Every plane through the centre is a diametral


l m n plane of the central conicoid corresponding to
The pole of the polar plane is , , .
ap bp cp some direction.

2. Let u 1 = 0 and u 2 = 0 be the equations of two 4. If the diametral plane of a line OP passes
planes. If the pole of the plane u 1 = 0 with through the point Q then the diametral plane of
respect to the conicoid ax 2 + by 2 + cz 2 = 1 lies the line OQ will also pass through the point P.

on the plane u 2 = 0 then the pole of the plane 5. The three semi-diameters OP, OQ and OR of
u 2 = 0 will lie on the plane u 1 = 0. Then two an ellipsoid which are such that the plane
such planes u 1 = 0 and u 2 = 0 are called containing any two is the diametral plane of the
conjugate planes. third, are called the conjugate semi-diameters.

3. If the polar plane of the point P with respect to a 6. The three planes POQ, QOR and ROP which
central conicoid pass through a point Q, then are such that each is the diametral plane of the
the polar plane of Q pass through P. The points line of intersection of the other two, are called
85

the conjugate planes or conjugate diametral 8. The sum of the squares of the projection of
planes. three conjugate semi-diameters of an ellipsoid
on a line is constant.
7. The sum of the squares of any three conjugate
semi-diameters of an ellipsoid is constant i.e. 9. The sum of the squares of projections of three
semi-diameters of an ellipsoid on any plane is
OP 2 + OQ2 + OR 2 = a 2 + b2 + c 2
constant.

EXERCISE
MULTIPLE CHOICE QUESTIONS 6. If the plane x + y + z = 2 touches the central

1. The surface represented by the equation conicoid px 2 + y 2 + z 2 = 1 the value of p is

x2 y2 z2 (a) 2 (b)
1
+ − = 1 is
a 2
b 2
c 2
[Avadh 2018] 2

(a) ellipsoid 1
(c) 3 (d)
3
(b) sphere
(c) hyperboloid of one sheet x2 y2 z2
7. If a tangent plane to the ellipsoid + + =1
2 2
(d) hyperboloid of two sheets a b c2
2. The centre of the central conicoid meets the co-ordinate axes in A, B and C then the
2
ax + by + cz = 1 is 2 2 locus of the centroid of the tetrahedron OABC is

(a) (0, 0, 0) (b) (a, 0, 0) a2 b2 c2 a2 b2 c2


(a) + + = 16 (b) + + =4
(c) (0, b, 0) (d) (a, b, c) x2 y2 z2 x2 y2 z2
3. The plane lx + my + nz = p touches the conicoid a2 b2 c2 a2 b2 c2
2 2 2 (c) + + =1 (d) + + =2
ax + by + cz = 1 if [Avadh 2018] x2 y2 z2 x2 y2 z2
(a) al 2 + bm 2 + cn2 = 1 8. The equation of tangent plane to the surface
2
l m n 2 2
x 2 y2 z 2
(b) + + = p2 + − = 1 at (2, 3, 5) is
a b c 4 a 25
l 2 m 2 n2 (a) 15x + 10y − 6z = 10
(c) + + =p
a b c
(b) 15x + 10y − 6z = 20
(d) al 2 + bm 2 + cn2 = p 2
(c) 15x + 10y − 6z = 30
4. The centre of the director sphere of the central
(d) 15x + 10y − 6z = 40
conicoid 3x 2 + 4y 2 − 5z 2 = 1 is
9. The equation of the plane which cuts
(a) (3, 4, − 5) (b) (3, 4, 5)
3x 2 + 2y 2 − 15z 2 = 4 in a conic whose centre is
(c) (−3, 4, 5) (d) (0, 0, 0)
(−2, − 3, − 1) is given by
5. The equation of the director sphere of the central
1 (a) 2x + 2y + z + 2 = 0
conicoid x 2 + y 2 + z 2 = is
3 (b) 2x − 2y + 5z + 2 = 0
2 2 2 2 2 2
(a) x + y + z = 3 (b) x + y + z = 1 (c) 2x + 2y − 5z + 2 = 0
1
2 2 2
(c) x + y + z = (d) none of these (d) 2x − 2y − 5z + 2 = 0
3
86

10. If OP, OQ and OR are the conjugate semi-diameters 16. The equation of the tangent plane at (1, 1, − 1) to the
2 2 2
x y z conicoid 2x 2 + 3y 2 − z 2 = 4 is
of the ellipsoid + + =1 then
2 2 2
a b c (a) 2x + 3y − z = 4 (b) 2x − 3y + z = 4
OP 2 + OQ2 + OR 2 is (c) −2x + 3y + z = 4 (d) 2x + 3y + z = 4
1
(a) a + b + c (b) 17. The equation of the polar plane of the point (1. 2, 3)
a+ b+ c
with respect to the conicoid x 2 + 2y 2 + z 2 = 1 is
2 2 2 1
(c) a + b + c (d) (a) x + 4y + 3z = 1 (b) x + 2y + 3z = 1
a 2 + b2 + c 2
(c) x + 2y + z = 2 (d) x + 2y + 3z = 2
11. The number of normals that can be drawn to the
18. The pole of the plane 2x − 3y + 2z = 1 with respect
x2 y2 z2
ellipsoid + + = 1 from a given point is to the conicoid x 2 − y 2 + z 2 = 1 is
a2 b2 c2
[Kanpur 2018] (a) 2, –3, 2 (b) 2, 3, 2

(a) 1 (b) 2 (c) 1, –1, 1 (d) 2, –3, 1

(c) 3 (d) 6 19. The radius of the director sphere of the ellipsoid

12. If the normal at p(α, β, γ) of a central conicoid x 2 y2 z 2


+ + = 1 is
6 4 15
ax 2 + by 2 + cz 2 = 1 meets the three principal
(a) 5 (b) 6 (c) 4 (d) 15
planes at A, B, C then PA : PB : PC is
1 1 1 20. If the plane lx + my + nz = p touches the ellipsoid
(a) a : b : c (b) : :
a b c x2 y2 z2
+ + = 1 then the value of (lx + my + nz)2
2 2
2 2
(c) a : b : c 2
(d)
1 1 1
: : a b c2
a 2 b2 c 2 is
13. The equation of the polar plane of the point (α, β, γ) a 2 b2 c 2
(a) al + bm + cn (b) + +
with respect to the conicoid ax 2 + by 2 + cz 2 = 1 is l m n

(a) ax + by + cz = 1 (c) a 2l 2 + b2m 2 + c 2n2 (d) al 2 + bm 2 + cn2

(b) αx + βy + γz = 1 21. The equation of the director sphere of the central


(c) aαx + bβy + cγz = 1 x2 y2 z2
conicoid + + = 1 is
2 2
(d) none of these a b c2
14. The pole of the plane lx + my + nz = p with respect (a) x 2 + y 2 + z 2 = a + b + c
to the conicoid ax 2 + by 2 + cz 2 = 1 is (b) x 2 + y 2 + z 2 =
1 1 1
+ +
a b c
a b c p p p
(a) , , (b) , ,
lp mp np al bm cn (c) x 2 + y 2 + z 2 = a 2 + b2 + c 2
1 1 1
(c)
l m n
, , (d)
l m n
, , (d) x 2 + y 2 + z 2 = + +
a b c ap bp cp a2 b2 c2

15. If the planes l1x + m1y + n1z = p1 and 22. If P (x1, y1, z1), Q(x 2, y2, z 2) and R (x 3, y3, z 3) be the
extremities of three conjugate semi-diameters of the
l 2x + m2y + n2z = p 2 are the conjugate planes with
respect to the conicoid ax 2 + by 2 + cz 2 = 1 then x2 y2 z2
ellipsoid + + = 1 then
2 2
l1l 2 m1m2 n1n2 a b c2
+ + is equal to
a b c x1z1 + x 2z 2 + x 3z 3 is equal to
(a) p1p 2 (b) p1 (a) a 2 (b) b2
(c) p 2 (d) p1 + p 2 (c) c 2 (d) 0
87

23. The diametral plane of the central conicoid 30. The equation of the director sphere of the central
2 2 2
ax + by + cz = 1, while l,m, n are d.c.’s of parallel conicoid x 2 + 2y 2 + 3z 2 = 1 is
chords is (a) x 2 + y 2 + z 2 = 0
(a) lx + my + nz = 0
(b) x 2 + y 2 + z 2 = 1
(b) alx + bmy + cnz = 0
11
(c) alx + bmy + cnz = 1 (c) x 2 + y 2 + z 2 =
6
(d) none of these 11
(d) x 2 + 2y 2 + 3z 2 =
24. The volume of the parallel piped formed by three 6
conjugate semi-diameters as coterminous edges in 31. The direction cosines of the normals to the conicoid
x2 y2 z2 ax 2 + by 2 + cz 2 = d at the point (α, β, γ) are
an ellipsoid + + = 1 is
2 2 2
a b c (a) a, b, c (b) aα, bβ, γ
(a) abc (b) a 2b2c 2 (c) aαp, bβp, cγp (d) αp, βp, γp
(c) a + b + c (d) a 2 + b2 + c 2 32. The equation of the normal to the ellipsoid
25. The pole of the plane 4x + 8y − 3z = 1 with respect 3x 2 + 4y 2 + 5z 2 = 64 at the point (1, 2, 3) on it is
to the conicoid 2x 2 + 4y 2 − z 2 = 1 is (a)
x −1
=
y−2
=
z−3
3 4 5
(a) (2, 4, − 1) (b) (4, 8, − 3)
x −1 y−2 z−3
(c) (2, 2, 3) (d) (2, 3, 4) (b) = =
3 8 15
26. The point A(x1, y1, z1) and B (x 2, y2, z 2) are the x −1 y−2 z−3
(c) = =
conjugate points with respect to the conicoid 3 16 5
ax 2 + by 2 + cz 2 = 1 if ax1x 2 + by1y2 + cz1z 2 is (d) none of these
(a) 0 (b) 1 (c) 2 (d) 3 33. The length of the perpendicular drawn from the
27. The equation of the normal to the central conicoid centre of the conicoid x 2 − y 2 + z 2 = 1 to the
x 2 + 2y 2 + 3z 2 = 1 at the point (1, 0, 1) is tangent plane to it at the point (1, 1, 1) is
x −1 y− 0 z −1 1
(a) = = (a) 1 (b) 3 (c) 3 (d)
1 0 3 3
x −1 y− 0 z −1 34. The tangent planes at the extremities of any
(b) = =
1 2 3 diameter of an ellipsoid are always
x −1 y− 0 z −1
(c) = = (a) perpendicular (b) parallel
1 0 1
(d) none of these (c) intersect (d) touches

28. If P (x1, y1, z1), Q(x 2, y2, z 2), and R (x 3, y3, z 3) be the 35. The equation of tangent plane to the conicoid
extremities of three conjugate semi-diameters of the x 2 + 2y 2 + z 2 = 3 at the point (1, 2, 3) is
x2 y2 z2 (a) x + 4y + 3z = 1
ellipsoid + + = 1 then z12 + z 22 + z 32 is
2 2
a b c2 (b) x + 4y + 3z = 0
equal to (c) x + 4y + 3z = 3
(a) a 2 (b) b2 (d) x + 2y + z = 3
2 2 2
(c) c (d) b + c 36. If the plane x + y + z = p touches the ellipsoid
29. In the conicoid the number of diameters are 6x 2 + 3y 2 + 2z 2 = 6 then p is
(a) 1 (b) 3 (a) ± 2 (b) ± 6
(c) finite (d) infinite (c) ± 13 (d) ± 14
88

37. The equation of the director sphere of the ellipsoid 44. If P (x1, y1, z1), Q(x 2, y2, z 2) and R (x 3, y3, z 3) be the
2 2 2 extremities of three conjugate semi-diameters of the
x y z
+ + = 1 is
2 3 4 ellipsoid 2x 2 + y 2 + 3z 2 = 6 then z12 + z 22 + z 32 is
2 2 2 2 2 2
(a) x + y + z = 1 (b) x + y + z = 0 equal to
2 2 2 (a) 2 (b) 1 (c) 3 (d) 6
x y z
(c) + + =9 (d) x 2 + y 2 + z 2 = 9
2 3 4 45. The point of contact of the tangent plane
38. If the tangent plane to the conicoid 2x + 3y + 10z = 25 with respect to ellipsoid
2 2 2 2 2 2
x + y + z = 1 meets the co-ordinate axes in P, Q 2x + 3y + 5z = 25 is
and R then the centroid of the triangle PQR lies on (a) (1, 2, 2) (b) (1, 2, 1)
1 1 1
(a) x 2 + y 2 + z 2 = 9 (b) + + =1 (c) (1, 1, 2) (d) (2, 2, 1)
x 2 y2 z 2
46. A cubic curve meets a plane at
1 1 1 2 2 2
(c) + + =9 (d) x + y + z = 1 (a) three points (b) four points
x2 y2 z2
(c) finite points (d) infinite points
39. The direction ratios of the normal to the ellipsoid
47. The equation of the plane that cuts
x2 y2 z2 2 2 2
+ + = 1 at the point (α, β, γ) are 2x − 3y + 5z = 1 in a conic whose centre is
a2 c2b2
(2, 1, 3) is given by
α β γ α β γ
(a) , , (b) , , (a) 4x + 3y + 15z = 50 (b) 4x + 3y − 15z = 50
a 2 b2 c 2 a b c
(c) −4x + 3y + 15z = 50 (d) 4x − 3y + 15z = 50
a b c a 2 b2 c 2
(c) , , (d) , ,
α β γ α β γ 48. If OP, OQ and OR be the conjugate semi-diameters
x 2 y2 z 2
40. Two points P (1, 1, 2) and Q(2, 2, 1) are the conjugate of the ellipsoid + + =1 then
4 9 16
points with respect to the conicoid
OP 2 + OQ2 + OR 2 is
ax 2 + by 2 + cz 2 = 1 if
(a) 4 (b) 9 (c) 16 (d) 29
(a) a + b + 2c = 1 (b) 2a + 2b + c = 1
1 49. The radius of the director sphere of the conicoid
(c) a + b + c = (d) a + b + c = 1
2 x 2 y2 z 2
+ − = 1 is
4 15 10
41. If (1, 1, 1) is one conjugate point with respect to the
(a) 3 (b) 2 (c) 4 (d) 5
conicoid x 2 + y 2 + z 2 = 1 then the other one is
50. The radius of the director sphere of
(a) (0, 1, 1) (b) (1, 1, 0)
x 2 + 2y 2 + 3z 2 = 1 is
(c) (1, 0, 0) (d) (1, 0, 1)
49
42. The equation of the director sphere of the central (a) 1 (b)
36
conicoid 36x 2 + 9y 2 + 4z 2 = 36 is 11
(c) 14 (d)
(a) x 2 + y 2 + z 2 = 49 (b) x 2 + y 2 + z 2 = 14 6

(c) x 2 + y 2 + z 2 = 36 (d) x 2 + y 2 + z 2 = 1 51. The diametral plane of the central conicoid


x 2 − 2y 2 + 3z 2 = 1 whose d.c. of parallel chords are
43. If the plane 2x + y + 3z = 3 touches the conicoid
1, 1, 1 is
x 2 + y 2 + pz 2 = 1 then the value of p is
(a) x − 2y + 3z = 1
3 8
(a) (b) (b) x − 2y + 3z = 0
4 3
9 2 (c) x − 2y + 3z = 3
(c) (d)
4 3 (d) none of these
89

52. If (x1, x 2, x 3), (y1, y2, y3) and (z1, z 2, z 3) be the (a) Sphere
extremitier of the three conjugate semi-diameters of the (b) Ellipsoid
x 2 y2 z 2 (c) Hyperbola of one sheet
ellipsoid + + = 1 then x12 + x 22 + x 32 is :
2 4 6 (d) Hyperbolic paraboloid
(a) 2 (b) 4 (c) 6 (d) 1
57. The surface represented by the equation
53. The equation of tangent plane to the central 4 (x 2 + y 2) = − 9z is a [Kanpur 2018]
conicoid 3x 2 − 5y 2 + z 2 + 2 = 0 at the point (1, 1, 0)
(a) Parabolic cylinder
is [Kanpur 2018]
(b) Hyperbolic cylinder
(a) −3x − 3y + 2 = 0 (b) 3x + 5y + 2 = 0
(c) Elliptic paraboloid
(c) 3x − 5y + 2 = 0 (d) none of these
(d) Paraboloid of revolution
54. The pole of 4x + 8y − 3z = 15 with respect to
58. The centre of the director sphere of the central
3x 2 + 7 y 2 + 2z 2 = 12 is [Kanpur 2018] conicoid 3x 2 + 4y 2 − 5z 2 = 1 is [Avadh 2018]
−16 32 −6  16 −32 6 
(a)  , ,  (b)  , ,  (a) (0, 0, 0) (b) (1, 1, 1)
 15 35 5   15 35 5 
(c) (2, 2, 2) (d) None of these
16 −32 −6  16 32 −6 
(c)  , ,  (d)  , , 
 15 35 5   15 35 5  x2 y2 2z
59. The equation + = represents
2
a b2 c [Avadh 2018]
55. The equation of the plane which cuts
(a) Ellipsoid
3x 2 + 2y 2 − 15z 2 = 4 in a conic whose centre is at
(b) Elliptic paraboloid
the point (−2, 3, − 1) is [Kanpur 2018]
(c) Hyperbolic paraboloid
(a) 2x + 2y − 5z + 2 = 9
(d) Hyperboloid of one sheet
(b) 2x − 2y − 5z + 2 = 0
60. The equation of an ellipsoid is [Agra 2017]
(c) 2x + 2y + 2z + 3 = 0
2 2 2 2 2
x y z x y z2
(d) x − y + z − 1 = 0 (a) − + =1 (b) + − =1
a2 b2 c2 a2 b2 c2
56. The surface represented by the equation
x2 y2 z2
2 2
5x + 6y − 7 z = 1 is 2 (c) + + =1 (d) None of these
[Kanpur 2018] a2 b2 c2

ANSWERS
MULTIPLE CHOICE QUESTIONS
1. (c) 2. (a) 3. (b) 4. (d) 5. (b) 6. (b) 7. (a) 8. (c) 9. (d) 10. (c)

11. (d) 12. (b) 13. (c) 14. (d) 15. (a) 16. (d) 17. (a) 18. (b) 19. (a) 20. (c)

21. (c) 22. (d) 23. (b) 24. (a) 25. (c) 26. (b) 27. (a) 28. (c) 29. (d) 30. (c)

31. (c) 32. (b) 33. (d) 34. (a) 35. (c) 36. (b) 37. (d) 38. (b) 39. (a) 40. (c)

41. (c) 42. (b) 43. (c) 44. (a) 45. (c) 46. (a) 47. (d) 48. (d) 49. (a) 50. (d)

51. (b) 52. (a) 53. (c) 54. (d) 55. (b) 56. (c) 57. (d) 58. (a) 59. (b) 60. (c)
90

HINTS AND SOLUTIONS


5. Central conicoid is 3x 2 + 3y 2 + 3z 2 = 1 so its perpendicular distance from centre (0, 0, 0) is
1 1
director sphere is =
1+ 1+ 1 3
1 1 1 3
x 2 + y2 + z 2 = + + = =1
3 3 3 3 36. The condition that the plane lx + my + nz = p
1 1 1 1 1 x2 y2 z2
6. + + = 4⇒ = 2⇒p = touches the ellipsoid + + = 1 is
p 1 1 p 2 a2 b2 c2

8. Tangent of ax 2 + by 2 + cz 2 = 1 at (x1, y1, z1) is a 2l 2 + b2m 2 + c 2n2 = p 2


2x 1 1
axx1 + byy1 + czz1 = 1 i.e. + 3y − 5z = 1 so p 2 = 1.1 + 2.1 + 3.1 = 6 so p = ± 6
4 9 25
x y z 40. Two points (x1, y1, z1) and (x 2, y2, z 2) are the
i.e. + − =1
2 3 5 conjugate points with respect to
⇒ 15x + 10y − 6z = 30 ax 2 + by 2 + cz 2 = 1
17. The equation of the polar plane of (α, β, γ) with iff ax1x 2 + by1y2 + cz1z 2 = 1
respect to ax 2 + by 2 + cz 2 = 1 is
so we get a.1.2 + b.1.2 + c.1.2 = 1
aαx + bβy + cγz = 1 so required polar plane is ⇒ 2a + 2b + 2c = 1
1x + 4y + 3z = 1
43. The plane lx + my + nz = p touches the conicoid
25. The pole of the plane lx + my + nz = p with respect
l 2 m 2 n2
 l m n ax 2 + by 2 + cz 2 = 1 if p 2 =
+ + so we
to conicoid ax 2 + by 2 + cz 2 = 1 is  , ,  so a b c
 ap bp cp  4 1 9 9 9
get + + = 9 ⇒ = 4 ⇒ p =
4 8 −3
required pole is  , ,  i.e.(2, 2, 3). 1 1 p p 4
 2 4 −1 
44. Since z12 + z 22 + z 32 = C2 so z12 + z 22 + z 32 = 2
2 2 2
27. The equation of normal of ax + by + cz = 1 at
48. Since OP, OQ and OR are conjugate semi-diameters
(x1, y1, z1) is
so OP 2 + OQ2 + OR 2 = a 2 + b2 + c 2 i.e.
x − x1 y − y1 z − z1
= = so required normal is 2 2 2
OP + OQ + OR = 4 + 9 + 16 = 29
ax1 by1 cz1
x −1 y− 0 z −1 50. The director sphere of x 2 + 2y 2 + 3z 2 = 1 is
= =
1 2.0 3.1 1 1 11
x 2 + y2 + z 2 = 1 + + = so radius of the
x −1 y− 0 z −1 2 3 6
⇒ = =
1 0 3 11
director sphere is .
6
33. The equation of tangent plane of x 2 − y 2 + z 2 = 1 ❍❍❍
at the point (1, 1, 1) is x − y + z = 1 and its
C HAPTER Unit-IV
91

12 The Paraboloids

THE ELLIPTIC PARABOLOID Also the point of contact is given by


1. The equation of the elliptic paraboloid is  lc −mc − p
− , , .
x2 y2 2z  an bn n 
+ = . The planes x = 0 and y = 0 are
2 2 c
a b 6. The polar plane of the point (x 1, y 1, z 1) with
called the principal planes. The sections of the respect to the paraboloid ax 2 + by 2 = 2cz is
elliptic paraboloid by the planes parallel to the
axx 1 + byy 1 = c (z + z 1).
zx-plane (y = 0) are parabolas.
7. The locus of the middle points of a system of
2. The equation of the hyperbolic paraboloid is
parallel chords is called a diametral plane.
x2 y2 2z
− = . The planes x = 0 and y = 0 are 8. Two diametral planes are called conjugate if
2 2 c
a b
each bisects the chords parallel to the other.
called the principal planes.
9. The equation of the normal at any point
3. The general equation ax 2 + by 2 = 2cz . If a and
(x 1, y 1, z 1) of the paraboloid ax 2 + by 2 = 2cz
b are both of the same sign then the general
x − x 1 y − y1 z − z 1
equation ax 2 + by 2 = 2cz represents an elliptic is = =
ax 1 by 1 −c
parabola, if a and b are of opposite signs then it
represents a hyperbolic paraboloid. 10. There are five normals can be drawn from a
given point to a paraboloid.
4. The equation of the tangent plane at the point
(x 1, y 1, z 1) on the paraboloid ax 2 + by 2 = 2cz 11. The five feet of normals from any point to the
paraboloid lie on a cubic curve.
is axx 1 + byy 1 = c (z + z 1).

5. If the plane lx + my + nz = p touches the


paraboloid ax 2 + by 2 = 2cz , then the
2 2
l m 2np
condition for tangency is + + = 0.
a b c

EXERCISE
MULTIPLE CHOICE QUESTIONS 2. The principal planes of the elliptic paraboloid
x2 y2 2z
1. The equation ax 2 + by 2 = 2cz represents an elliptic + = are
2
a b2 c
paraboloid if a and b are [Avadh 2018]
(a) x = 0, y = 0 (b) z = 0
(a) +ve sign only (b) −ve sign only
(c) x = 0, z = 0 (d) y = 0, z = 0
(c) same sign (d) none of these
92

3. The equation of the tangent plane at (x1, y1, z1) on 11. The equation ax 2 + by 2 = 2cz represents a
the paraboloid ax 2 + by 2 = 2cz is hyperbolic paraboloid if a and b are
(a) ax + by = c (z + z1) (a) same sign (b) opposite sign
(b) xx1 + yy1 = c (z − z1) (c) positive sign only (d) negative sign only
(c) axx1 + byy1 = c (z + z1) 12. If x + 3y = 3 be the one conjugate diametral plane
(d) axx1 + byy1 = cz1 + z of the paraboloid 2x 2 + 3y 2 = 4z then the other one
4. The point of contact of the tangent plane is
x + y + z = 1 to the paraboloid x 2 + y 2 = 2z is (a) 2x + y = 1 (b) x − 2y = 1
(a) (1, 1, 1) (b) (0, 0, 0) (c) x + 2y = 1 (d) 2x − y = 1
(c) (−1, − 1, − 1) (d) (−1, 0, − 1) 13. The condition that the plane lx + my + nz = p
5. The polar plane of the point (1, 1, 1) with respect to touches the paraboloid ax 2 + by 2 = 2cz is
2 2
the paraboloid 2x + y = 2z is
a 2 b2 2np a 2 b2 2np
(a) + = (b) + + =0
(a) 2x + y + z = 1 (b) 2x + y − z = 1 l m c l m c
(c) x + 2y + 2z = 1 (d) x + 2y − 2z = 1 l 2 m 2 2np l 2 m 2 2np
2 2 (c) + = (d) + + =0
6. The centre of the conic x + y = 2z, x + y + z = 1 a b c a b c
is 14. If the plane 2x + 3y − z = p touches the paraboloid
(a) (1, 1, 3) (b) (1, 1, 1) x 2 + y 2 = 2z then the value of p is
(c) (−1, − 1, 3) (d) (1, 1, 2) 11 13
(a) (b) 2 (c) 7 (d)
7. The vertex of the paraboloid 2 2
3x 2 + 2y 2 − 12z − 6x + 8y = 13 is 15. The locus of the middle points of a system of parallel
chords for the paraboloid is called
(a) (0, 0, 0) (b) (−2, − 2, 1)
(a) conjugate plane (b) semi-conjugate plane
(c) (1, 2, 2) (d) (1, − 2, − 2)
(c) diametral plane (d) none of these
8. If the plane lx + my + nz = p be the tangent plane
2 2 16. If the plane 8x − 6y − z = 5 touches the paraboloid
of the paraboloid ax + by = 2z at the point
3x 2 − 2y 2 = 6z then the co-ordinates of the point of
(α, β, γ) then the point of contact is
l m p al bm p  contact is
(a)  , ,  (b)  , , 
 an bn n   n n n (a) (9, 8, 5) (b) (8, 9, 5)
− al − bm − p  −l −m − p  (c) (5, 8, 9) (d) (3, 2, 6)
(c)  , ,  (d)  . , 
 n n n  an bn n  17. The principal planes of the hyperbolic paraboloid
9. 2x 2 − 3y 2 = 4z represents x2 y2 2z
− = are
2
(a) ellipse (b) hyperbola a b2 c

(c) elliptic paraboloid (a) x = c, y = c (b) x = a, y = b


(d) hyperbolic paraboloid (c) x = 0, y = 0 (d) x = c, y = a
10. The equation of the normal at any point (α, β, γ) of 18. The section of the elliptic paraboloid by the planes
the paraboloid is parallel to the yz-plane is
x −α y−β z − γ (a) circles (b) ellipses
(a) = =
aα bβ cγ (c) parabolas (d) hyperbolas
x −α y−β z − γ
(b) = = 19. The equation of the polar plane of the point (1, 1, 1)
αa bβ −c
with respect to the paraboloid x 2 + y 2 = 2z is
x −α y−β z − γ
(c) = =
−a bβ cγ (a) x + y + z = 0 (b) x + y + z = 1
(d) none of these (c) x + y − z = 1 (d) x + y − z = 0
93

x2 y2 (c) (3, 3, 2) (d) (3, 2, 3)


20. The section of the surface − = z by the planes
2 2
a b 29. The locus of the mid points of a system of parallel
x = a is chords of ax 2 + by 2 = 2cz is called
(a) parabolas (b) hyperbolas
(a) parallel plane (b) conjugate plane
(c) ellipses (d) cones
(c) diametral plane
21. Two diametral planes alx + bmy − cn = 0 and (d) conjugate semi-diameter
al ′x + bm ′y − cn′ = 0 are conjugate diametric
30. The locus of the normals from the point (x1, y1, z1)
planes only when
on the paraboloid x 2 + y 2 = 2az is
(a) all ′ + bmm ′ = cnn′ (b) ll ′ + mm ′ = nn′
(c) all ′ + bmm ′ = 0 (d) ll ′ + mm ′ = 0 (a) cone (b) cylinder
(c) sphere (d) circle
22. The equation of the tangent plane to the paraboloid
x 2 y2 31. The locus of the point of intersection of three
− = z at the point (8, 9, 5) is
2 3 mutually perpendicular tangent planes of the
paraboloids
(a) 8x − 6y + z = 1 (b) 8x − 6y − z = 5
1 1
(c) 8x − 6y + z = 5 (d) 8x − 6y − z = 1 (a) 2z + + = 0 (b) 2z + a + b = 0
a b
x2 y2 2z 1 1
23. The sections of the ellipsoid + = by the (c) z + a + b = 0 (d) z + − = 0
2 2 c
a b a b
planes z = k is 32. The surface represented by x 2 + 2y 2 = 1 − z is
(a) parabolas (b) hyperbolas
(a) hyperboloid of one sheets
(c) circles (d) ellipses
(b) hyperboloid of two sheets
24. The number of normals that can be drawn to the
paraboloid ax 2 + by 2 = 2cz from a given point is (c) elliptic paraboloid (d) ellipsoid

(a) 5 (b) 6 (c) 1 (d) 4 33. The section of the paraboloid x 2 − 2z 2 = 3y by the
25. The locus of the point of intersection of three XOZ plane is
mutually perpendicular tangent plane to the (a) parabola (b) hyperbola
paraboloid x 2 + y 2 + 2z = 1 is
(c) circle (d) pair of plane
(a) z = − 1 (b) z = 1
34. The equation of tangent plane to the paraboloid
(c) y + z = 1 (d) y + z + 1 = 0 x 2 y2
− = 2 at the point (8, 9, 5) on it is
26. The direction ratios of the normal at any point 2 3
(α, β, γ) of the paraboloid ax 2 + by 2 = 2cz are [Kanpur 2018]
(a) a, b, c (b) aα, bβ, c (a) 8x − 6y − z = 5 (b) 8x + 6y − z = 5
(c) α, β, γ (d) aα, bβ, −c (c) 8x + 6y + z = 5 (d) 8x − 6y + z = 5
27. The sections of the hyperbolic paraboloid
35. The locus of the perpendiculars from the vertex of
x2 y2 2z
− = by the varying plane z = a is x2 y2
a2 b2 c the paraboloid − = 2z to the generators of
a2 b2
(a) hyperbolas (b) parabolas
one system is [Kanpur 2018]
(c) ellipses (d) circles
(a 2 + b2)
28. The point of contact of the plane x + 2y − 2z = 4 (a) x 2 + y 2 + 3z 2 ± xy = 0
ab
touches the paraboloid 3x 2 + 4y 2 = 24z is
(a 2 + b2)
(a) (2, 3, 2) (b) (2, 2, 3) (b) x 2 + y 2 − 3z 2 ± xy = 0
ab
94

(a 2 + b2)  x 2 y2   x2
2
y2
(c) x 2 + y 2 + 2z 2 ± xy = 0 (b) a 2b2  + − 2  +

− 1 = x 4
ab
 a2 b2
  a 4
b4

(a 2 + b2)
(d) x 2 + y 2 − 2z 2 ± xy = 0 2
ab  x 2 y2   x2 y2 
(c) a 2b2  + + 2  + + 1 = x 4
2 2 4 4
36. The locus of the centre of sections of the paraboloid a b  a b 
x2 y2
+ = 2z, which are of constant area πx 2 is (d) None of these
2
a b2
37. The plane x + 2y − 2z = 4 touches the paraboloid
[Kanpur 2018]
3x 2 + 4y 2 = 24z at the following point
2
 x 2 y2   x2 y2  [Agra 2017]
(a) a 2b2  + − 2  + + 1 = x 4
2 2 4
a b  a b4  (a) (1, 2, 3) (b) (2, 3,2)
(c) (3, 2, 2) (d) None of these

ANSWERS
MULTIPLE CHOICE QUESTIONS

1. (c) 2. (a) 3. (c) 4. (c) 5. (b) 6. (c) 7. (d) 8. (d) 9. (c) 10. (b)

11. (b) 12. (d) 13. (d) 14. (d) 15. (c) 16. (b) 17. (c) 18. (c) 19. (c) 20. (a)

21. (c) 22. (b) 23. (d) 24. (a) 25. (a) 26. (d) 27. (a) 28. (a) 29. (c) 30. (c)

31. (a) 32. (c) 33. (d) 34. (a) 35. (c) 36. (a) 37. (b)

HINTS AND SOLUTIONS


4. The point of contact of the tangent plane 13
⇒ 2p = 13 ⇒ p= .
lx + my + nz = p with respect to the paraboloid 2
−lc −mc − p 
ax 2 + by 2 = 2cz is  , ,  so point of 16. The point of contact of lx + my + nz = p with
 an bn n 
respect to paraboloid ax 2 + by 2 = 2cz is
−11. −11 . −1 
contact is  , ,  i.e., (−1, − 1, − 1).  −lc , −mc , − p  so required point of contact is
 11
. 11
. 1  an bn n 
5. The polar plane of (x1, y1, z1) with respect to  +8.3 , 6.3 , + 5  i.e., (8, 9, 5)
ax 2 + by 2 = cz 2 is axx1 + byy1 = c (z + z1) so  3 2 1
required polar plane is 2x + y = 1(z + 1) 19. The polar plane of ax 2 + by 2 = 2cz with respect to
⇒ 2x + y − z = 1 the point (x1, y1, z1) is axx1 + byy1 = c (z + z1) i.e.
14. The plane lx + my + nz = p touches the paraboloid 11
. x + 11
. y = 1(z + 1) ⇒ x + y − z = 1
ax 2 + by 2 = 2cz then 28. Point of contact is 
−lc −mc − p 
, ,  so (2, 3, 2).
 an bn n 
l 2 m 2 2np 4 9 2p
+ + = 0 so + − =0 ❍❍❍
a b c 1 1 1
C HAPTER Unit-IV
95

13 Generating Lines

GENERATING LINES OF HYPERBOLOID OF ONE conicoid ax 2 + by 2 + cz 2 = 1 are


SHEET
2 2 2
aα + bβ + cγ = 1, alα + bmβ + cnγ = 0,
1. A ruled surface is defined as a surface which is
generated by the motion of one parameter al 2 + bm 2 + cn 2 = 0.
family of straight lines and the straight line itself
is called its generating line, generator or ruling.
THE GENERATING LINES OF A HYPERBOLIC
PARABOLOID
2. The generating lines of a hyperboloid of one
1. The generating lines of a hyperbolic paraboloid
x 2 y2 z 2 x z  y
sheet + − = 1 are − = λ 1 −  , x2 y2 x y
x y 2
a 2 b2 c 2 a c  b − = 2z are − = λ2,
+ =
2
x z 1 y a b2 a b
a b λ
+ = 1 +  called λ-system family of x y 2
a c λ  b called λ-system of generators and − = ,
a b µ
x z  y
straight lines and − = µ 1 +  , x y
a c  b + = µz called µ-system of generators.
x z 1 y a b
+ = 1 −  called µ-system family of
a c µ b 2. The locus of the point of intersection of two
straight lines. perpendicular generators of a hyperbolic
x2 y2
3. One generator each of λ- and µ- systems passes paraboloid − = 2z is a 2 − b2 + 2z = 0
2 2
through every point of the hyperboloid. a b
together with hyperbolic paraboloid.
4. No two generators of the same systems
intersect. 3. The generators of the λ- and µ- systems of the
x2 y2 2z
5. Any two generators of the different systems hyperbolic paraboloid − = are
2
a b2 c
intersect.
x y
parallel to the planes ± = 0
6. The tangent plane at a point of a hyperboloid of a b
one sheet meets the hyperboloid in two
4. One generator each of λ- and µ- systems passes
generators through that point.
through every point of hyperbolic paraboloid.
7. The locus of the point of intersection of two
5. No two generator of the hyperbolic paraboloid
perpendicular generators of hyperboloid of one
of the same system intersect.
sheet is x 2 + y 2 + z 2 = a 2 + b2 − c 2
6. In the hyperbolic paraboloid any generator of
8. If three points of a straight line lie on a conicoid the λ-system intersects any generator of the
then the straight line lies wholly on the conicoid. µ-system.
9. The conditions that the straight line 7. The tangent plane at any point of a hyperbolic
x −α y−β z − γ paraboloid meets it in two generators through
= = is a generator of the
l m n that point.
96

EXERCISE
MULTIPLE CHOICE QUESTIONS 6. One of the generating line of the hyperboloid
x 2 y2 z 2
1. In the hyperboloid of one sheet, one generator each + − = 1 passing through the point (2, 3, 4)
4 9 16
of λ- and µ-systems pass through
is
(a) only one point (b) no point x−2 y−3 z−4
(a) = =
(c) only two points (d) every point 1 3 4
2. The locus of the point of intersection of two x−2 y−3 z−4
(b) = =
perpendicular generators of hyperboloid of one 1 0 4
x−2 y−3 z−4
sheet is [Kanpur 2018] (c) = =
1 0 0
2 2 2 2 2 2
(a) x + y + z = a + b − c x−2 y−3 z−4
(d) = =
0 3 4
(b) x 2 + y 2 + z 2 = a 2 + b2 + c 2
7. In the hyperboloid of one sheet two generators of
(c) x 2 + y 2 + z 2 = a 2 − b2 − c 2 the same system intersect only
(a) one point (b) two points
(d) x 2 + y 2 + z 2 = − a 2 − b2 − c 2
(c) infinite points (d) no point
3. If a straight line lies wholly on the conicoid then x y 2
8. If + = be a λ-system generating line of the
minimum number of points of straight line that lies a b λ
on conicoid are x2 y2
hyperbolic paraboloid − = 2z then other one
(a) 1 (b) 2 a2 b2
is
(c) 3 (d) infinite
x y
4. The locus of the point of intersection of two (a) + = λz
a b
perpendicular generators of a hyperbolic
x y
x 2
y 2 (b) − = λz
paraboloid − = 2z is a b
a2 b2
x y 2
2 2
(c) − =
(a) a + b = 2z a b λ

(b) a 2 − b2 + 2z = 0 (d) none of these


9. The locus of the point of intersection of two
(c) a 2 + b2 − 2z = 0
x 2 y2 z 2
perpendicular generators of + − = 1 is
2 2 4 25 9
(d) a + b + 2z = 0

5. The generators of the λ- and µ-systems of the (a) x 2 + y 2 + z 2 = 4 (b) x 2 + y 2 + z 2 = 25


y2 z2 2x
hyperbolic paraboloid − = are parallel to (c) x 2 + y 2 + z 2 = 9 (d) x 2 + y 2 + z 2 = 20
2
b c2 a
the plane 10. The locus of the point of intersection of two
x 2 y2
x y y z perpendicular generators of − = 2z is
(a) ± =0 (b) ± =a 16 9
a b b c
y z x y (a) 2z + 7 = 0 (b) 2z + 25 = 0
(c) ± =0 (d) ± =c
b c a b (c) 2z + 1 = 0 (d) 2z − 7 = 0
97

11. The generators of the λ- and µ-systems of the 18. The locus of the point of intersection of two
y 2 − z 2 = 2x are parallel to the planes perpendicular generators of hyperboloid of one

(a) y = ± z (b) y = ± 2z sheet x 2 + y 2 − z 2 = 2 is

(c) z = ± 2y (d) x = 0, y = 0 (a) x 2 + y 2 + z 2 = 1


12. If two generators intersect then systems are (b) x 2 + y 2 + z 2 = 0
(a) same (c) x 2 + y 2 + z 2 = 2
(b) different
(d) x 2 + y 2 + z 2 = 4
(c) never intersect
x −α y−β z − γ
(d) may be same or different 19. The condition that = = is a
l m n
13. The generators of the λ- and µ-systems of the
generator of ax 2 + by 2 + cz 2 = 1 is
hyperbolic paraboloid x 2 − y 2 = 2z are parallel to
the planes (a) aα 2 + bβ 2 + cγ 2 = 1
(a) x ± y = 0 (b) alα + bmβ + cnγ = 0 only
(b) x + y = 0 only
(c) al 2 + bm 2 + cn2 = 0 only
(c) x − y = 0 only
(d) x ± y = 2 (d) All the above

14. The locus of the points of intersection of two 20. The locus of the point of intersection of two
perpendicular generators of the hyperboloid y2
perpendicular generators of x 2 − = z is
2 2 2 4
2x + 3y − 4z = 1 is

(a) x 2 + y 2 + z 2 = 0 (a) 4z − 3 = 0 (b) 2z − 3 = 0


(c) 4z − 1 = 0 (d) 4z − 5 = 0
(b) 2x 2 + y 2 + z 2 = 1
21. The equation of the generators of hyperboloid
(c) 2x 2 + 3y 2 − 4z 2 = 1
x2 y2 z2
7 + − = 1 which passes through the point
(d) x 2 + y 2 + z 2 = a2 b2 c2
12
(a cos α, b sin α, 0) is [Kanpur 2018]
15. The tangent plane at a point of a hyperboloid of one
x − cos α y − b sin α z
sheet meets the hyperboloid then the number of (a) = =
a sin α b cos α ± c
generators through that point are
(a) 1 (b) 2 x − a cos α y − b sin α z
(b) = =
a sin α − b cos α ± c
(c) 3 (d) nil
16. The locus of the point of intersection of two x + a cos α y − b sin α z
(c) = =
perpendicular generators of a hyperbolic a sin α b cos α c

paraboloid x 2 − y 2 = 2z is (d) None of these

(a) 2z + 1 = 0 (b) 2z − 1 = 0 22. The generators of the λ-and µ-systems of the

(c) z = 0 (d) z − 1 = 0 x2 y2
hyperbolic paraboloid − = 2z are parallel to
2
a b2
17. One of the generating line of the hyperboloid
yz + 2zx + 3xy + 6 = 0 which pass through the planes [Kanpur 2018]
(−1, 0, 3) is x y x y
(a) ± =0 (b) ± =1
a b a b
(a) x = − 1, z = 3 (b) x = 1, z = 3
x y
(c) x = − 1, z = − 3 (d) x = + 1, z = − 3 (c) ± =2 (d) None of these
a b
98

23. The locus of the point of intersection of two 24. Number of system of generating liners in a
perpendicular generators of a hyperbolic hyperbolic paraboloid is [Avadh 2018]
paraboloid is [Avadh 2018] (a) 1 (b) 2
(c) 3 (d) 4
(a) a 2 + b2 + 2z = 0
25. Two generators of the same system of hyperboloid
(b) a 2 − b2 + 2z = 0 of one sheet [Avadh 2018]
(c) a 2 − b2 − 2z = 0 (a) Intersect at a point
(b) Never intersect
(d) a 2 + b2 − 2z = 0
(c) Intersect on the hyperboloid
(d) None of these

ANSWERS
MULTIPLE CHOICE QUESTIONS

1. (d) 2. (a) 3. (c) 4. (b) 5. (c) 6. (d) 7. (d) 8. (b) 9. (d) 10. (a)

11. (a) 12. (b) 13. (a) 14. (d) 15. (b) 16. (c) 17. (a) 18. (c) 19. (d) 20. (a)

21. (b) 22. (a) 23. (b) 24. (b) 25. (b)

HINTS AND SOLUTIONS


9. The locus of the point of intersection of two 13. The generators of the λ- and µ- systems of
x2 y2 z2 x2 y2
2z x y
perpendicular generators of + − = 1 is − are parallel to the planes + = 0.
=
2 2 2 2 2c a b
a b c a b
2 2 2 2 2 2 x y
x + y + z = a + b − c so required locus is So, required plane is ± = 0 ⇒ x ± y = 0
1 1
x 2 + y 2 + z 2 = 4 + 25 − 9 = 20. 14. The locus of the points of intersection of
10. The locus of the point of intersection of two perpendicular generators of 2x 2 + 3y 2 − 4z 2 = 1 is
x2 y2 1 1 1 7
perpendicular generators of − = 2z is x 2 + y 2 + z 2 = a 2 + b2 − c 2 = + − =
2
a b2 2 3 4 12
❍❍❍
2z = b2 − a 2 so required locus is

2z = 9 − 16 = − 7 i.e. 2z + 7 = 0
99

C HAPTER Unit-IV

14 The Plane Sections of Conicoids

THE PLANE SECTIONS OF CONICOIDS 3. The condition for the section to be a rectangular
1. The nature of a plane section (projection) of the hyperbola in point 2 is
central conicoid ax 2 + by 2 + cz 2 = 1 by the (a + b) n 2 + am 2 + bl 2 = 0.
plane lx + my + nz = p is a hyperbola, a 4. The section of the conicoid by the planes are
parabola or an ellipse according as circles since every plane must meet a sphere in
bcl 2 + acm 2 + abn 2 <, =, or > 0. a circle.

2. If the central plane section is an ellipse in point 1 5. Any two circular section of an ellipsoid which
1 are not parallel always lie on a sphere.
 l 2 + m2 + n 2 2
then its area is π  .
2 2 2 6. The hyperbolic paraboloid has no real circular
 bcl + acm + abn 
sections.
If the conicoid be the ellipsoid
2 2 2
UNBILICS
x y z
+ + = 1 then the area of the central 1. Let P (α, β, γ ) be a point on any conicoid. If the
a2 b2 c2
1 planes parallel to the tangent plane at P
 l 2 + m2 + n 2 2 determine circular sections of the conicoid, then
plane section is π abc  .
2 2 2 2 2 2 the tangent plane at the point P itself will be a
 a l + b m + c n 
circular section of radius zero. Such a point P is
3. The section of the conicoid called an unbilic of the conicoid.
ax 2 + by 2 + cz 2 = 1 by any tangent plane to
2. The number of real unbilics of the ellipsoid
x2 y2 z2
the cone + + = 0 is a x2 y2 z2
(b + c) (c + a) (a + b) + + = 1 are four.
a2 b2 c2
rectangular hyperbola.
3. The unbilics of the ellipsoid lie on a sphere.
PLANE SECTIONS OF A PARABOLOID
4. The length p of the perpendicular distance from
1. The nature of the plane section (projection) of the centre to the tangent plane at an unbilic of
the paraboloid ax 2 + by 2 = 2cz by the plane ac
the ellipsoid is .
lx + my + nz = p is a hyperbola, a parabola or b
an ellipse according as abn 2l 2 <, =, or > 0. 5. The number of real unbilics of the paraboloid

2. If the plane section is an ellipse then its area is ax 2 + by 2 = 2cz are two.

πc  cl 2 cm 2  l 2 + m2 + n 2
 + + 2np .
3
η  a b  ab
100

EXERCISE
MULTIPLE CHOICE QUESTIONS 8. The plane x + 2y + 3z = 1, cuts the paraboloid

1. The plane sections of a surface represented by an x 2 + y 2 = 2z in a


equation of second degree are called (a) parabola (b) plane
(a) conics (b) conicoids (c) ellipse (d) hyperbola
(c) surfaces (d) central conicoids 9. The number of real circular sections of the
2. All parallel plane sections of a conicoids are hyperbolic paraboloid are
(a) similar (a) 1 (b) 2
(b) different (c) 3 (d) not exist
(c) may be similar or may be different 10. The unbilics of the ellipsoid lie on a
(d) none of these (a) circle (b) sphere
3. The section of the ellipsoid 9x + 6y + 14z = 3 2 2 2 (c) ellipsoid (d) cylinder

by the plane x + y + z = 0 always represents 11. The plane lx + my + nz = 0 cuts the conicoid
(a) a circle (b) an ellipse ax 2 + by 2 + cz 2 = 1 in a parabola only when
(c) a hyperbola (d) a plane (a) bcl 2 + cam 2 + abn2 = 0
4. The plane lx + my + nz = p, n ≠ 0, cuts the (b) bcl 2 + acm 2 + abn2 > 0
2 2
paraboloid ax + by = 2z in an ellipse if
(c) bcl 2 + cam 2 + abn2 < 0
2 2
(a) abn = 0 (b) abn > 0
(d) none of these
2
(c) abn < 0 (d) none of these 12. The number of real circular sections of the
5. The plane lx + my + nz = 0 cuts the conicoid hyperboloid 4x 2 − 13y 2 = 2z are
ax 2 + by 2 + cz 2 = 1 in a hyperbola if (a) 1 (b) 2
bcl 2 + cam 2 + abn2 is (c) 3 (d) does not exist
(a) > 0 (b) = 0 13. The real unbilics of the ellipsoid are
(c) < 0 (d) none of these (a) 1 (b) 2
6. If A1, A2 and A3 are the areas of three mutually (c) 4 (d) 6
perpendicular central sections of an ellipsoid then 14. The condition that any two lines x = y = z and
(a) A1 + A2 + A3 = constant 2x = 2y = − z be the axes of the section of the
(b) A12 + A22 + A32 = constant conicoid ax 2 + by 2 + cz 2 = 1 by a plane through
1 1 1 them is
(c) + + = constant
A1 A2 A3 (a) a + b − c = 0 (b) 2a + b − c = 0
1 1 1
(d) + + = constant (c) a + b − 2c = 0 (d) a + b + c = 0
A12 A22 A32
15. The plane lx + my + nz = 0 cuts the paraboloid
7. The plane x + y + z = 0, cuts the conicoid ax 2 + by 2 = cz in a parabola if abn2
2 2 2
x + 2y + 3z = 1 in a
(a) > 0 (b) < 0
(a) ellipse (b) parabola
(c) equal to 0 (d) none of these
(c) hyperbola (d) circle
101

16. If the section of the surface xy + yz + zx = a 2 by the 25. If r1 and r2 be the semi axis of a rectangular
plane lx + my + nz = p is a parabola then hyperbola then

(a) l + m + n = 0 (b) l 2 + m 2 + n2 = 0 (a) r1 = r2 = 0 (b) r1 + r2 = 0


(c) r1r2 = 0 (d) r12 + r22 = 0
(c) l + m+ n=0 (d) none of these
17. The plane x − y + z = 2 cuts the paraboloid 26. The area of the central plane section of the conicoid
2 2
x − 2y = 2z in a ax 2 + by 2 + cz 2 = 1 by the plane lx + my + nz = 1
is
(a) parabola (b) ellipse
π π
(c) hyperbola (d) circle (a) (b)
p abc abc
18. The equation 2x 2 + 3y 2 = 4z represents a
(c) π abc (d) p π abc
(a) parabola (b) ellipse 27. The real central circular sections of the ellipsoid
(c) paraboloid (d) elliptic paraboloid x 2 y 2 3z 2
+ + = 1 are
8 4 4
19. The plane lx + my = p cuts the paraboloid
2 2
ax + by = cz in a (a) z ± 2x = 0 (b) x ± 2z = 0
(c) y ± 2z = 0 (d) z ± 2y = 0
(a) parabola (b) hyperbola
28. The circular sections of the paraboloid
(c) ellipse (d) circle 13 2
y + 2z 2 = x through the (0, 0, 0) are
20. The plane x + y + z = 2, cuts the paraboloid 2
2x 2 + 3y 2 = 2z in a (a) 3x ± 2y = 0 (b) 3x ± 2z = 0
(c) 2x ± 3z = 0 (d) 2x ± 3y = 0
(a) parabola (b) ellipse
29. The area of the plane section (central) of the
(c) hyperbola (d) hyperboloid
x2 y2 z2
ellipsoid + + =1 by the plane
21. The hyperbolic paraboloid has a 2
b 2
c2
(a) one real circular section lx + my + nz = p is
abc p abc
(b) many real circular sections (a) π (b)
p π
(c) no real circular section π p
(c) abc (d) abc
(d) none of these p π
22. The section by the plane 3x + 2y − 4z = 1 on the 30. The distance from the centre of the tangent plane at
paraboloid x 2 + y 2 = 2z is x2 y2 z2
an unbilic of the ellipsoid + + = 1 is
(a) hyperbola (b) parabola a2 b2 c2
bc ab
(c) ellipse (d) circle (a) (b)
a c
23. If r1 and r2 be the length of semi axis and plane ac
(c) abc (d)
section be an ellipse then area of plane section is b

(a) r1r2 (b) r1 + r2 31. The real unbilics of the paraboloid ax 2 + by 2 = 2cz

(c) π r1r2 (d) π(r1 + r2) are

24. The section by the plane x − y + z = 0 on the (a) 1 (b) 2


2 2 2
conicoid 2x − y + 3z = 1 represents a (c) 3 (d) 4
32. The real unbilics of hyperboloid of one sheet are
(a) ellipse (b) hyperbola
(a) 1 (b) 2
(c) parabola (d) conicoid
(c) 3 (d) 4
102

33. The real central circular sections of the ellipsoid (a) 2 (x + 2)2 − 10 (x + 2) − 9y 2 = 0
x 2 y2 z 2
+ + = 1 are (b) 2 (x + 2)2 + 10 (x + 2) + 9y1 = 0
8 4 2
(a) x ± z = 0 (b) x ± z 2 = 0 (c) 2(x + 2)2 + 10(x + 2) − 9y 2 = 0

(c) x ± 2z = 0 (d) x ± 2 2z = 0 (d) 2(x + 2)2 − 10(x + 2) + 9y 2 = 0


34. The circular section of the paraboloid 39. If the section of conicoid ax 2 + by 2 + cz 2 = 1 by the
2 2
4x + y − 4z = 0 are
plane lx + my + nz = 0 is hyperbola, then it section
(a) z ± 3x = 0 (b) x ± 3z = 0 by the plane ln + my + nz = p is [Kanpur 2018]
(c) x ± 3z = 0 (d) z ± 3x = 0 (a) parabola (b) circle
35. The circular section of the hyperbolic paraboloid (c) ellipse (d) hyperbola
x 2 − y 2 = 2z are 40. The two confocal paraboloids always cut at the
(a) x ± y = 0 (b) x ± 2y = 0 angle
(c) y ± 2x = 0 (d) not exist π π
(a) (b)
4 2
36. The area of the central plane section of the conicoid
(c) π (d) 0
3x 2 + 3y 2 + 4z 2 = 1 by the plane x + 2y + z = 2 is
41. The locus of the point of intersection of
π
(a) π (b) x 2
y 2
6 perpendicular generators of − = 2z is
π π a2 b2
(c) (d)
6 12 (a) b2 − a 2 = 2z (b) b2 − a 2 = − 2z
37. The two circular sections of an ellipsoid which are (c) b2 − a 2 = 2y (d) b2 − a 2 = − 2y
not parallel lies on a
42. If A1, A2, A3 are the areas of three mutually
(a) circle (b) sphere
perpendicular central reactions of
(c) ellipse (d) ellipsoid
x2 y2 z2 1 1 1
+ + = 1 then + + is equal to
38. The equation of the real central circular sections of a2 b2 c2 A12 A22 A32
the conicoid 6x 2 + 13y 2 + 6z 2 − 10yz + 4zx
(a) constant (b) variable
− 10xy − 1 = 0 are [Kanpur 2018]
(c) abc (d) 0

ANSWERS
MULTIPLE CHOICE QUESTIONS

1. (a) 2. (a) 3. (b) 4. (b) 5. (c) 6. (d) 7. (a) 8. (c) 9. (d) 10. (b)

11. (a) 12. (d) 13. (c) 14. (c) 15. (c) 16. (c) 17. (c) 18. (d) 19. (a) 20. (b)

21. (c) 22. (c) 23. (c) 24. (a) 25. (d) 26. (a) 27. (b) 28. (d) 29. (a) 30. (d)

31. (b) 32. (d) 33. (b) 34. (a) 35. (d) 36. (d) 37. (b) 38. (d) 39. (d) 40. (b)

41. (a) 42. (a)


103

HINTS AND SOLUTIONS


π abc (l12 + m12 + n12)1/ 2 + (3)(2)(−1)2 + (2)(−1)(1)2
6. Here A1 =
(a 2l12 + b2m12 + c 2n12)1/ 2 = − 3 + 6 − 2 = + 1 > 0 so it represents an ellipse.
1 a 2l12 + b2m12 + c 2n12 27. The real pair of central circular sections of the
So = , in a similar way A2
A12 2 2 2 2
π a b c x2 y2 z2
ellipsoid + + = 1 are
2 2
1 1 1 1 a b c2
and A3 exists so + + =
A12 A22 A32 π 2a 2b2c 2 x 2 z 2
a − b2 ± b − c2 = 0
a c
[a 2 (l12 + l 22 + l 32) + b2 (m12 + m22 + m32)
x z 3 4
so we get 2± 4− = 0
2 (a 2 + b2 + c 2) 2 2 2 3
+ c (n12 + n22 + n32)] = = constant
π 2a 2 b2 c 2 x z 3 2 2
⇒ ± ⋅ = 0 i.e. x ± 2z = 0.
2 2 2 2 2 3
7. Here, bcl + cam + abn = 6 + 3 + 2 = 11 > 0 so
it represents an ellipse. 28. Real circular section through the origin are given by
13
8. Here, abn2l 2 = 11 . = 9 > 0 so it represents an
. .91 y a − b ± x b = 0⇒y −2± x 2=0
2
ellipse.
⇒ 2x ± 3y = 0.
17. Here abm 2l 2 = (1)(−2)(−1)2 (1)2 = − 2 < 0 so it
represents a hyperbola.
24. bcl 2 + cam 2 + abn2 = (−1)(3)(1)2
C HAPTER Unit-IV
105

15 Reduction of General
Equation of Second Degree

GENERAL EQUATION OF SECOND DEGREE 4. By rotation of axes, f (x , y, z ) = ax 2 + by 2


1. The most general equation of second degree is + cz 2 + 2 fyz + 2gzx + 2hxy
F (x , y, z ) ≡ ax 2 + by 2 + cz 2 + 2 fyz
transforms to λ 1x 2 + λ 2 y 2 + λ 3 z 2 where λ 1,
+ 2gzx + 2hxy + 2ux + 2vy λ 2 , λ 3 are the roots of the equation
+ 2wz + d = 0 a−λ h g
By the transformation of axes it can be reduced
h b− λ f =0
to
(i) λ 1x 2 + λ 2 y 2 + λ 3 z 2 = µ (First form) g f c−λ

(ii) λ 1x 2 + λ 2 y 2 = 2µ z (Second form) It is called the discriminating cubic.


5. The roots of the discriminating cubic are all real.
2. First form have the following surfaces
6. If the given equation of surface is homogeneous
(i) Ax 2 + By 2 + Cz 2 = 1 (Ellipsoid)
then the centre of the given surface is origin.
(ii) A (x 2 + y 2 + z 2 ) = 1 (Sphere) 7. If the given equation of surface F (x , y, z ) = 0 is
2 2
(iii) Ax + By − Cz = 1 2 not homogeneous then centre of the surface are
∂F ∂F
(Hyperboloid of one sheet) found by solving the equations = 0, = 0,
∂x ∂y
(iv) Ax 2 − By 2 − Cz 2 = 1 ∂F
= 0.
(Hyperboloid of two sheet) ∂z
(v) Ax 2 + By 2 + Cz 2 = 0 (Cone) PROCESS OF REDUCING A GENERAL EQUATION
2 2
(vi) Ax + By = 1 (Elliptic cylinder)
TO THE STANDARD FORM
1. Form the discriminating cubic and solve it.
(vii) Ax 2 − By 2 = 1 (Hyperbolic cylinder)
2. If all the characteristics roots are different from
(viii) Ax 2 − By 2 = 0 (Pair of intersecting planes)
zero, then find the co-ordinates (α, β, γ ) of the
(ix) Ax 2 = 1 or By 2 = 1 or Cz 2 = 1 ∂F ∂F
centre by solving the equations = 0, = 0,
(Pair of parallel planes) ∂x ∂y
3. Second form have the following surfaces ∂F
= 0.
∂z
(i) Ax 2 + By 2 = 2Cz (Elliptic paraboloid)
3. Shifting the origin to the centre (α, β, γ ) and
(ii) Ax 2 − By 2 = 2Cz
then rotating the axes, the reduced equation
(Hyperbolic paraboloid)
becomes λ 1x 2 + λ 2 y 2 + λ 3 z 2 = µ where
2 2
(iii) A(x + y ) = 2Cz
µ = − (uα + vβ + wγ + d). Thus in this case the
(Paraboloid of revolution) surface can be determined.
106

4. If one root say λ 3 = 0 then corresponding to 6. If k = 0, there is a line of centres, given by any
this value find the principal direction l 3 , m3 , n 3 ∂F ∂F ∂F
two of the equations = 0, = 0, = 0.
from any two of the following equations. ∂x ∂y ∂z
al 3 + hm3 + gn 3 = 0 Let any point (α, β, γ ) on the line of centres be
chosen or centre; then the reduced equation is
hl 3 + bm3 + fn 3 = 0
gl 3 + fm3 + cn 3 = 0 λ 1x 2 + λ 2 y 2 = µ

and evaluate k = ul 3 + vm3 + wn 3 . where µ = − (uα + vβ + wγ + d).


5. If k ≠ 0, the reduced equation is
λ 1x 2 + λ 2 y 2 + 2kz = 0 and represents an
elliptic or hyperbolic paraboloid.

EXERCISE
MULTIPLE CHOICE QUESTIONS 5. The surface represented by the equation

1. 2 2
In 3-dimension geometry Ax + By = 1 represents ax 2 + by 2 − cz 2 = 1 is
(a) ellipsoid
(a) ellipse (b) cone
(b) a hyperboloid of one sheet
(c) hyperboloid (d) elliptic cylinder
(c) sphere
2. The surface of the equation (d) a hyperboloid of two sheet
2
(3x − 4y + z) + 9x − 12y + 3z − 10 = 0 represents 6. The latus rectum of the normal section of the

(a) cone parabolic cylinder (3x − 4y + z)2 = 26(x + y + z) is

(b) ellipsoid (a) 3 (b) 2


(c) 3 (d) 2
(c) pair of parallel planes
7. The centre of the conicoid
(d) hyperboloid
3x 2 − y 2 − z 2 + 6yz − 6x + 6y − 2z − 2 = 0 is
3. The discriminating cubic of the conicoid
(a) (1, 1, 1) (b) (1, 0, 1)
x 2 + y 2 + z 2 + yz + zx + xy = 1 is
(c) (1, 0, − 1) (d) (−1, 0, 1)
3 2
(a) 4λ + 2λ + 9λ − 2 = 0 8. The discriminating cubic of the conicoid

(b) 4λ3 − 12λ2 + 7 λ + 1 = 0 2x 2 + 2y 2 + z 2 − 4xy + 2yz − 2zx + x + y = 0 is

(a) λ3 + λ2 + 2λ = 0 (b) λ3 − 2λ2 + 5λ = 0


(c) 4λ3 + 12λ2 − 9λ + 1 = 0
(c) λ3 − λ2 + 5λ = 0 (d) λ3 − 5λ2 + 2λ = 0
(d) 4λ3 + 12λ2 + 7 λ + 1 = 0
9. The normal form of the equation
4. The centre of the conicoid
3x 2 + 5y 2 + 3z 2 + 2yz + 2zx + 2xy
2 2 2
3x − y + z + 6yz − 6x + 6y − 2z − 2 = 0 is
− 4x − 8z + 5 = 0 is
(a) (1, 1, 1) (b) (1, 0, 1) (a) 2x 2 + 3y 2 + 6z 2 = 1 (b) 2x 2 + 3y 2 − 6z 2 = 1
(c) (1, 0, − 1) (d) (−1, 0, 1)
(c) 2x 2 − 3y 2 − 6z 2 = 1 (d) −2x 2 − 3y 2 + 6z 2 = 1
107

10. Ax 2 = ± 2cz always represents a 19. The equation y 2 + z 2 − x 2 = 1 represents a


(a) elliptic paraboloid (b) parabolic cylinder (a) hyperboloid of one sheet
(c) hyperbolic cylinder (d) elliptic cylinder (b) hyperbolic cylinder
11. The centre of the conicoid (c) hyperboloid of revolution

x 2 + 2yz − 4x + 6y + 2z = 0 is (d) hyperboloid of two sheets


20. The centre of the surface
(a) (2, − 1, − 3) (b) (2, 1, 3)
(c) (2, − 1, 3) (d) (2, 1, − 3) 2x 2 + 5y 2 + 10z 2 + 12yz + 6zx + 4xy − 1 = 0 is

12. The surface x + y+ z = 0 always represents (a) (1, 0, 2) (b) (0, 0, 0)


(a) paraboloid (b) right circular cylinder (c) (0, 1, 2) (d) (1, 2, 0)
(c) hyperboloid (d) none of these 21. The roots of the discriminating cubic of the conicoid
13. The surface represented by the equation x 2 + 2y 2 − z 2 = 1 are

Ax 2 + By 2 = Cz is (a) 1, 2, 2 (b) 1, 2, 1
(c) 1, 2, − 1 (d) 1, − 2, 1
(a) sphere (b) elliptic paraboloid
(c) ellipse (d) hyperboloid 22. The surface Ax = 1 or By 2 = 1 or Cz 2 = 1 always
2

14. The roots of the discriminating cubic of the conicoid represents a


2 2 2 (a) cone (b) pair of parallel planes
2x + 5y + 10z + 12yz + 6zx + 4xy − 1 = 0 are
(c) one plane
(a) 1, 1, 5 (b) 1, 5, 5
(d) pair of perpendicular planes
(c) 1, 1, 15 (d) 1, 5, 1
23. The characteristics roots of the equation
15. The line of centre of the hyperbolic cylinder
x 2 − y 2 + 4yz + 4zx − 6x − 2y − 8z + 5 = 0 passes x + y+ z = 0 are

through the point (a) 1, 2, 2 (b) 1, − 2, 2

(a) (0, 0, 0) (b) (1, 0, 1) (c) −1, 2, 2 (d) −1, − 2, 2

(c) (0, 1, 0) (d) (1, 1, 1) 24. The equation A(x + y + z 2) = 1 represents a


2 2

16. The centre of the conicoid (a) cone (b) cylinder


2 2 2 (c) ellipsoid (d) sphere
x + y + z + yz + zx + xy
25. The roots of the discriminating cubic of the conicoid
+ 3x + y + 4z + 4 = 0 is
(a) (1. 1, 2) (b) (−1, 1, − 2) 2x 2 − 3y 2 + 4z 2 = 5 are
(c) (1, 1, − 2) (d) (−1, 1, 2) (a) 2, 3, 4 (b) −2, 3, − 4
17. The roots of the discriminating cubic of the surface (c) 2, − 3, − 4 (d) 2, − 3, 4
2 2 2
2y + 4zx − 6x − 8y + 2z + 5 = 0 are 26. The equation x − y + 2yz − 2zx − x − y + z = 0

(a) 2, 2, 2 (b) 2, 2, − 2 always represents a

(c) 2, − 2, − 2 (d) −2, − 2, − 2 (a) ellipsoid (b) hyperboloid

18. The roots of discriminating cubic are (c) hyperbolic paraboloid

(a) purely real (d) elliptic paraboloid

(b) real and imaginary both 27. The surface equation 3x 2 + 4y 2 = 1 represents a

(c) purely imaginary (a) circle (b) ellipse


(d) none of these (c) ellipsoid (d) elliptic cylinder
108

28. In 3-dimension Ax 2 − By 2 = 0 represents (c) 3, 6, 18 (d) 3, 4, 12


(a) cone (b) cylinder 2 2 2
30. 3x − 4y − 12z = 1 represents a
(c) pair of planes (d) pair of lines (a) hyperboloid of one sheet
29. The roots of discriminating cubic for 11x 2 + 10y 2 (b) hyperbolic paraboloid
+ 6z 2 − 8yz + 4zx − 12xy + 72x − 72y + 362 (c) hyperboloid of two sheets
+ 150 = 0 are (d) elliptic paraboloid
(a) 3, 4, 6 (b) 3, 6, 12

ANSWERS
MULTIPLE CHOICE QUESTIONS
1. (d) 2. (c) 3. (a) 4. (c) 5. (b) 6. (c) 7. (c) 8. (d) 9. (a) 10. (b)

11. (a) 12. (b) 13. (b) 14. (c) 15. (d) 16. (b) 17. (b) 18. (a) 19. (c) 20. (b)

21. (c) 22. (b) 23. (c) 24. (d) 25. (d) 26. (c) 27. (d) 28. (c) 29. (c) 30. (c)

HINTS AND SOLUTIONS


2. (3x − 4y + z)2 + 3(3x − 4y + z) − 10 = 0 ∂f
= 2x − 4 = 0 ⇒ x = 2
−3 ±
9 + 40 ∂x
so 3x − 4y + z = = 2, − 5 i.e. we get ∂f
2 = 2y + 2 = 0 ⇒ y = − 1
∂y
3x − 4y + z − 2 = 0 and 3x − 4y + z + 5 = 0 a pair
∂f
of parallel planes. = 2z + 6 = 0 ⇒ z = − 3
∂z
1 1
1− λ So its centre is (2, − 1, − 3).
2 2
1 1 23. x + y=− z
3. 1− λ = 0 After solving we get
2 2 x + y + 2 xy = z⇒ x + y − z = − 2 xy
1 1
1− λ x 2 + y 2 + z 2 + 2xy − 2yz − 2xz = 4xy
2 2
i.e. x 2 + y 2 + z 2 − 2xy − 2yz − 2zx = 0
4λ3 − 12λ2 + 9λ − 2 = 0.
So roots of discriminating cubic are
4. Let f = 3x 2 − y 2 + z 2 + 6yz − 6x + 6y − 2z − 2 1− λ −1 −1
∂f ∂f
= 6x − 6 = 0, = − 2y + 6z + 6 = 0, −1 1− λ −1 = 0
∂x ∂y
∂f −1 −1 1− λ
= 2z + 6y − 2 = 0
∂z i.e. λ3 − 3λ2 + 4 = 0 we get λ = − 1, 2, 2
Thus x = 1, y = 0, z = − 1 so centre is (1, 0, − 1).
25. The roots of the discriminating cubic are given by
8. The discriminating cubic is
2− λ 0 0
2− λ −2 −1
0 −3 − λ 0 =0
−2 2− λ 1 = 0 i.e. λ3 − 5λ2 + 2λ = 0
0 0 4− λ
−1 1 1− λ
i.e. (2 − λ) (−3 − λ) (4 − λ) = 0
11. Let f = x 2 + 2yz − 4x + 6y + 2z = 0
i.e. λ = 2, − 3, 4.
❍❍❍
C HAPTER Unit-V
109

16 Vector Calculus
DIFFERENTIATION OF VECTORS → →
d → → da db
1. If to each point P(x , y, z ) of a region R in space (i) (a + b) = +
dt dt dt
there corresponds a unique scalar f (P), then f is
→ →
called a scalar point function. Similarly if to d → → db da →
(ii) (a ⋅ b) = a ⋅ + ⋅b
each point P(x , y, z ) of a region R in space there dt dt dt
→ → → →
corresponds a unique vector f (P), then f is d → → → db da →
(iii) (a × b) = a × + ×b
called a vector point function. dt dt dt
→ → →
2. Let r = f (t) be a vector function of the scalar d → d a dφ →
(iv) (φ a ) = φ + a
→ → → dt dt dt
variable t. We define r + δ r = f (t + δt) so
→ →  →   → 
δ r = f (t + δt) − f (t), consider the vector d → → →  d a → → → d b →
(v) [ a b c] =  b c +  a c
→ → →
dt  dt   dt 
δr f (t + δt) − f (t)    
= .
δt δt → → → 
d c
→ + a b
δr f (t + δt) − f (t)  dt 
If lim = lim exists, then  
δt → 0 δt δt → 0 δt
→ →
dr d → → → da → → →
the value of this limit, denoted by is called (vi) { a × (b × c )} = × (b × c ) + a
dt dt dt
the derivative of the vector function r with  →   →
respect to the scalar t.  d b → →  → d c 
× × c + a ×  b ×
 dt   dt 
→ → → →    
dr (r + δ r) − r
So = lim
dt δt → 0 δt → →
dr d r ds
→ → (vii) = (chain rule)
f (t + δt) − f (t) dt ds dt
= lim .
δt → 0 δt →
dc →
→ (viii) = 0 if c is constant vector
dr → dt
If exists, r is said to be differentiable. If we
dt 4. Some important results are :
→ 2→
dr d r (i) The necessary and sufficient condition for
differentiate again, we get which is →
dt dt 2 the vector function a (t) to be constant is
called the second derivative and so on. →
da
→ → → that =0
3. If a , b and c are differentiable vector function dt
of a scalar t and Q is a differentiable scalar
function of the same variable t then
110

→ →  → →  →
→ → da ∂2 f ∂  ∂ f  ∂2 f ∂ ∂ f 
(ii) If a has constant length, then a and = , = ,
dt ∂x 2 ∂x  ∂x  ∂x∂y ∂x  ∂y 

   
da
are perpendicular provided ≠0 →  →
dt ∂2 f ∂ ∂ f 
= etc.
∂y ∂x ∂y  ∂x 
 
(iii) The necessary and sufficient condition for
→ 2. The vector differential operator ∇ (del or n abla)
the vector a (t) to have constant magnitude
is defined as

→ da → ∂ $ ∂ $ ∂ $ $ ∂ ∂ ∂
is a ⋅ =0 ∇≡ i+ j+ k≡i + $j + k$ .
dt ∂x ∂y ∂z ∂x ∂y ∂z
(iv) The necessary and sufficient condition for 3. Let f (x , y, z ) be defined and differentiable at
→ each point (x , y, z ) in certain region of
the vector a (t) to have constant direction is
space.Then the gradient of f (∇f or grad f ) is

→ da defined as
a× =0
dt ∂f $ ∂f $ ∂f $
∇f = i+ j + k
→ ∂x ∂y ∂z
5. If r be the position vector of a particle and the
→ The gradient of a scalar field defines a vector
dr →
field. Some important results are
scalar variable t be the time, then V = is the
dt
(i) ∇( f + g ) = ∇f + ∇g
velocity vector at instant t and is along the
tangent to the path of the particle. The second (ii) ∇( fg ) = f∇g + g∇f
→  f  g ∇f − f ∇g
→ d2 r (iii) ∇   =
derivative a = is the acceleration vector g g2
dt 2
at instant t. (iv) The necessary and sufficient condition for
a scalar point function to be constant is that
GRADIENT, DIVERGENCE AND CURL ∇f = 0
→ 4. The directional derivative of a scalar field f at a
1. Let f be a vector function depends on more
point P(x , y, z ) in the direction of a unit vector a$
that one scalar variable and let
df
→ → is given by = ∇f ⋅ a$
f = f (x , y. z , ...). Suppose the variable x dS
increases from x to x + δx while the other 5. The gradient f is a vector in the direction of
→ which the maximum value of directional
remains constant. Let δ f be the corresponding
df
→ → derivative of f is occurs.
∂f→ δf dS
increment in f then = lim
∂x δx→ 0 δx →
6. Let v be any given differentiable vector point
→ →
f (x + δx , y, z ) − f (x , y, z ) →
= lim is called the function then the divergence of v is
δx→ 0 δx
→ → →
→ → → → ∂v $ ∂v ∂v
partial derivative of f w.r.t. x. The higher div v = ∇ ⋅ v = $i ⋅ + j⋅ + k$ ⋅
→ ∂x ∂y ∂z
partial derivative of f can be defined as :
111

→ → → →
It should be noted that div v is a scalar quantity f (t) w.r.t. t and symbolically ∫ f (t) dt = F (t) + C.
→ → →
A vector v is said to be solenoidal if ∇ ⋅ v = 0 →
The function f (t) to be integrated is called the
→ integrand.
7. Let f be any given differentiable vector point
→ 2. Some important results are given by
function. Then the curl of f is defined as curl
→ → →
 → →
→ → → ∂f $ ∂f ∂f d r → → d s → →
f = ∇ × f = $i × + j× + k$ × curl (i) ∫  dt ⋅ s + r ⋅ dt = r ⋅ s + C
∂x ∂y ∂z dt 
 

f is a vector quantity.  →
 → d r →
→ → → → (ii) ∫  2 r ⋅  dt = r 2 + C
A vector f is said to be irrotational if ∇ × f = 0 dt 
 
8. Some properties of divergence and curl are
 → →
defined as → d 2 r  → dr
(iii) ∫  r ×  dt = r × dt + C
→ → → → → → → dt 2 
(i) ∇⋅ (A + B) = ∇⋅ A + ∇⋅ B  
→ → → → → → →  →
(ii) ∇ × (A + B) = ∇ × A + ∇ × B → d r  → →
(iv) ∫  a × dt = a × r + C
→ → → → → → dt 
(iii) ∇⋅ (φ A) = (∇ φ) ⋅ A + φ (∇⋅ A)  

→ → → → → → 3. Any integral which is to be evaluated along a


(iv) ∇ × (φ A) = (∇ φ) × A + φ (∇ × A) curve is called a line integral and denoted by
→ → → → → → → → → → →
(v) ∇⋅ (A × B) = B⋅ (∇ × A) − A ⋅ (∇ × B) ∫c F ⋅ d r.
→ → → → → → → → → →
(vi) ∇ × (A × B) = (B ⋅ ∇ ) A − B (∇⋅ A) 4. The total work done W by the force F to
→ → → → → → →
− (A⋅ ∇ ) B + A (∇ ⋅ B) displace the particle d r along the curve C is
→ →
→ →
(vii) ∇ ⋅ (∇ φ) = ∇ φ 2 W= ∫c F ⋅ d r .
→ → → 5. Any integral which is to be evaluated over a
(viii) ∇ × (∇ φ) = O surfaces is called the surface integral and is
→ → → →
(ix) ∇ ⋅ (∇ × A) = O denoted by ∫ ∫s F ⋅ n$ ds

→ → → → → → 2→ dxdy dydz F n
(x) ∇ × (∇ × A) = ∇ (∇⋅ A) − ∇ A where ds = =
| n$ ⋅ k$ | | n$ ⋅ $i|
INTEGRATION OF VECTORS dxdz ds P
= .
1. Integration is the reverse process of | n$ ⋅ $j|
→ →
differentiation. Let f (t) and F (t) be two vector 6. If f (x , y, z ) is a scalar point function, then its
d→ → volume integral over a region v is defined as
function of the scalar t such that F (t) = f (t).
dt ∫ ∫ ∫ fdv = ∫ ∫ ∫ f (x , y, z ) dxdydz .
v v
→ →
Then F (t) is called the indefinite integral of f (t)
112

GREEN’S, GAUSS’S AND STOKE’S THEOREMS continuous and has continuous first partial
1. Let C be a closed regular curve in the xy-plane derivative in v then
bounding a region R and M (x , y) and N (x , y) → → →
$
∫ ∫ ∫v ∇ ⋅ F dv = ∫ ∫s F ⋅ nds
be continuously differentiable functions inside
and on C then where n$ is the outwards drawn unit normal
 ∂N ∂M  vector to s. This is called Gauss’s theorem.
∫c (Mdx + Ndy) = ∫ ∫s  ∂x −  dxdy
∂y  →
3. Let F be a continuous differentiable vector
The area bounded by a simple closed curve C is point function and S be an open, two sided
1 surface bounded by a simple closed curve C
given by A = ∫ (xdy − ydx )
2 c then
The Green’s theorem in plane is → → →
$
∫c F ⋅ d r = ∫ ∫s curl F ⋅ nds
→ → → →
∫ ∫R (∇ × F ) ⋅ k$ dR = ∫ c F ⋅ d r Where C is traversed in the positive direction
2. Let v is the volume bounded by a closed and n$ is the unit normal vector at any point of S.

piecewise smooth surface S. Suppose F (x , y, z ) This is called Stoke’s theorem.
is a vector function of position which is

EXERCISE
MULTIPLE CHOICE QUESTIONS → d2 r

4. If r = sin t $i + cos t $j + tk$ then is
Differentiation of Vectors dt 2

1. If the vector function a(t) is constant then (a) − sin t $i − cos t $j (b) 2
→ →
→ da → da (c) –1 (d) 1
(a) a ⋅ =0 (b) a × =0
dt dt

→ 5. If r = (cos n t) $i + (sin n t) $j, where n is a constant
da →
(c) =0 (d) a = 0 →
dt → dr
and t varies then r × is
→ dt
→ da
2. If a has constant length with ≠ 0 then
dt (a) k$ (b) sin nt k$

→ → (c) cos n t k$ (d) nk$


da
→ →da
(a) a ⋅ =0 (b) a ⋅ =0
dt dt → → → → →
6. If r = e nt a + e − nt b , where a and b are constant

→ da → →
(c) a =0 (d) a = 0 d2 r
dt vector then is equal to
dt 2
3. The locus of a point whose co-ordinates are
→ →
function of a single parameter is called (a) 0 (b) r
(a) surface (b) curve → →
(c) n r (d) n2 x
(c) point (d) none of these
113

→ → → → → →
7. If a vector a(t) have constant magnitude then 14. If r = cos ωt a + sin ωt b , where a, b are constant
→ → →
→ da da d2 r →
(a) a × =0 (b) =0 vectors and ω is constant then + ω2 r is equal
dt dt dt 2
→ to
→ → da
(c) a = 0 (d) a ⋅ =0 → → →
dt (a) a + b (b) ω a
 → → →
d → d r d2 r (c) ω b (d) 0
8. The value of r, ,  is
dt  dt dt 2  →
  nt → − nt → → →
15. If r = e a+ e b where a and b are constant
 d→ → → → d 2 → →
r d2 r d2 r  r d3 r 

(a)  , ,  (b)  r , ,  d2 r
vectors then is equal to
 dt dt 2 dt 2   dt 2 dt 2  dt 2
   
→ →
→ d → → → d → → (a) r (b) n2 r
r d2 r  r d2 r 
(c)  r , ,  (d)  r , ,  → →
 dt dt 3   dt dt 2  (c) n r (d) − n2 r
   
→ →
→ 16. If A = a $i + $j − 2k$ and B = 2$i − $j + bk$ and
9. If a = t 2$i − t$j + (2t + 1) k$ and
→ →
A × B = 0 then the value of a and b are
→ d → →
b = (2t − 3) $i + $j − tk$ then ( a ⋅ b ) at t = 1 is
dt (a) 2 and –1 (b) 1 and –2
(a) 6 (b) – 6 (c) 2 and –2 (d) 2 and 3
(c) 5 (d) – 5 17. The necessary and sufficient condition for r t to


10. If a = a cos t $i + a sin t $j + at tan α k$ then have constant magnitude is
→ →
• •• dr → dr
| r × r | is (a) =0 (b) r × =0
[Kanpur 2018; Avadh 2018] dt dt
(a) a sec α (b) a tan α →
→ dr →
2 2 (c) r ⋅ =0 (d) r = 0
(c) a sec α (d) a tan α dt
11. A particle moves along the curve 18. If a particle moves along the curve x = t 3 + 1, y = t 2,
$
−t t $ $
r = e i + e sin t j + k then the magnitude of its z = 2t + 5 where t is time, then the components of
velocity at t = 0 is its velocity at t = 1 in the direction $i + $j + 3k$ is
(a) 2 (b) 2 (a) 7 (b) 6
(c) 2 2 (d) 0 (c) 11 (d) 3
12. The necessary and sufficient condition for the →
→ 19. If r = a cos t $i + a sin t $j + a t tan α k$ then [&r &&r &&&]
r
vector a t to have constant direction is
is [Kanpur 2018]
→ 3
→ da (a) a tan α (b) a tan α
(a) a = 0 (b) =0
dt
(c) a sec α (d) a 3 sec α
→ →
→ da → da
(c) a × =0 (d) a ⋅ =0 →
→ → d2 r
dt dt 20. If r = (cos ωt) $i + (sin ωt)$j then r × is

dt 2
→ dr
13. If r = sin t $i + cos t $j + tk$ , then is equal to [Avadh 2018]
dt
(a) 1 (b) sin ωt
(a) 1 (b) 0 (c) 2 (d) 2 (c) cos ωt (d) 0
114

GRADIENT, DIVERGENCE AND CURL  1


29. grad   is
21. If F is a scalar point function, then gradient of f (∇f ) → 
r
is a → →
r r
(a) scalar point function (a) − (b) −
r r2
(b) vector point function

(c) either scalar or vector point function r 1
(c) − (d) −
(d) none of these r3 r2
22. The necessary and sufficient condition for a scalar 30. →
grad f (r) × r is
point function to be constant is that

→ (a) r (b) f (r)
(a) ∇⋅ F = 0 (b) ∇ × f = 0

→ (c) r f (r) (d) 0
(c) ∇f = 0 (d) none of these
31. The unit vector normal to the level surface
23. If f (x, y, z) = 3x 2y − y 3z 2 then grad f at the point x 2 + y − z = 1 at the point (1, 0, 0) is
(1, 1, 1) is $i + $j + k$ 2$i + $j − k$
(a) 6$i + 2$j − 2k$ (b) 6$i − $j + 4k$ (a) (b)
6 6
(c) 6$i − 2k$ (d) none of these i$ + $j − 2k$ i$ + $j
(c) (d)
→ → 6 6
24. If r = | r |where r = x $i + y$j + zk$ then the value of
32. The directional derivative of f = x 2yz + 4xz 2 at
∇r is
r (1, − 2, − 1) in the direction 2$i − $j − 2k$ is
(a) 0 (b)
→ [Kanpur 2018]
r
→ → (a) 3 (b) 37
r r 3 37
(c) − (d) + (c) (d)
r r 37 3
→ →
25. The value of ∇( r ⋅ a) is 33. The greatest value of the direction derivative of the
→ → function 2x 2 − y − z 4 at the point (2, − 1, 1) is
(a) a (b) r

(a) 7 (b) 6
(c) 0 (d) ∇ × a (c) 8 (d) 9
26. For the surface f (x, y, z) = c where c is a constant 34. The equation of the tangent plane to the surface
the vector ∇f is x 2 + 2y 2 + 3z 2 − 12 = 0 at the point (1, 2, − 1) is
(a) parallel vector (b) normal vector (a) x + 4y − 3z = 12 (b) x + 3y − 4z = 12
(c) intersect the surface (d) none of these (c) 3x + y − 4z = 12 (d) 3x + 4y − z = 12
27. The vector in the direction of which the maximum 35. The directional derivative of f = xyz at the point
value of the directional derivative of f occurs is (1, 1, 1) in the direction $i is
(a) curl F (b) div F (a) 3 (b) 3
(c) grad f (d) none of these (c) 1 (d) 2

28. The normal to be surface x 4 − 3xyz + z 2 + 1 = 0 at 36. The div f is a

the point (1, 1, 1) is (a) scalar


(b) vector
(a) $i + 3$j + k$ (b) $i + 3$j − k$
(c) may be scalar or vector both
(c) i$ − 3$j + k$ (d) i$ − 3$j − 3k$ (d) none of these
115

→ →
37. The vector f is called solenoidal if [Avadh 2018] 46. If the vectro A = (ax + 2y) $i + (2y − 3) $j − 3zk$ is
→ → solenoidal; then the constant a is
(a) ∇ f = 0 (b) ∇ ⋅ f = 0
(a) 0 (b) 1
→ →
(c) ∇ × f = 0 (d) f = 0 (c) 2 (d) 3
→ →
38. The vector f is called irrotational if 47. If the vector A = ay $i − 3z$j + 2bk$ is irrotational then
→ → the constant a is
(a) ∇ f = 0 (b) ∇ ⋅ f = 0
→ →
(a) 1 (b) 3
(c) ∇ × f = 0 (d) f = 0
(c) 2 (d) 0

39. div r is [Avadh 2018] 48. The greatest rate of increase of xyz 2 at the point
(a) 1 (b) 2 (1, 0, 3) is
(c) 3 (d) 0 (a) 0 (b) 3
→ (c) 1 (d) 9
40. Curl r is
49. The maximum value of the directional derivative of
(a) 0 (b) 1
x 2yz at the point (1, 4, 1) is
(c) 2 (d) 3
(a) 9 (b) 4
→ →
41. If f = xy 2$i + 2x 2yz$j − 3yz 2k$ then div f at the (c) 1 (d) none of these
point (1, − 1, 1) is
50. If f = x − y + 4z then the value of ∇2 f is
2 2
(a) 0 (b) 3
(a) 4 (b) 1
(c) 6 (d) 9
(c) −1 (d) 0

42. If f is a constant vector then →
51. The vector r n r is solenoidal only when

(a) div f = 0 only (a) n = 0 (b) n + 1 = 0
→ (c) n + 2 = 0 (d) n + 3 = 0
(b) curl f = 0 only
→ →
→ →
(c) div f = 0 but curl f ≠ 0 52. If F = x 2y + 2xy + z 2 then curl grad F is

→ → (a) x $i + y$j + 2zk$ (b) 2x $i + 2y$j + 2zk$


(d) div f = 0 and curl f = 0
→ → → (c) 2x $i + y$j (d) 0
43. If a is a constant vector then div ( r × a) is
Integration of Vectors
→ →
(a) r (b) a → 1
→ → 53. If f (t) = (t 2 − 1) $i + t 2$j − k$ then ∫ f (t) dt is
0
(c) r ⋅ a (d) 0
1 1
44. ∇ × (∇f ) is (a) (2i$ − $j + 3k$ ) (b) (i$ + $j − 3k$ )
3 3

(a) f (b) 0 1 1 $ $
(c) − (2$i − $j + 3k$ ) (d) (− i − j + 3k$ )
(c) 0 (d) 1 3 3
→ →
45.
→ → →
If A and B are irrotational then A × B is

54. If a = t $i − t 2$j + (t − 1)k$ and b = 2t 2$i + 6tk$ then
2→ →
(a) solenoidal only
∫0 a ⋅ b dt is
(b) irrotational only
(a) 10 (b) 12
(c) either solenoidal or irrotational
(d) none of these (c) −12 (d) −10
116

→ →
d2 r → → 62. The circulation of F round the curve C, where
55. The solution of the equation = a, where a is a
2
dt →
F = (x − y) $i + (x + y) $j and C is the circle
→ →
constant vector and it is given that r = 0 and x 2 + y 2 = 16 , z = 0 is

dr → (a) 2π (b) 8π
= 0 when t = 0 is
dt (c) 16π (d) 32π
1→ 1 → →
(a) a (b) t a 63. If F = cos y $i − x sin y$j and curve C is the circle
2 2
1 2→ 1 → x 2 + y 2 = 1 in the xy-plane from (1, 0) to (0, 1) then
(c) t a (d) t a+1
2 2 → →
∫c F ⋅ d r is
→ 2$i − $j + 2k$ when t = 2
56. If r (t) =  (a) 0 (b) 1
4$i − 2$j + 3k$ when t = 3
(c) −1 (d) π

3 → dr 
→ →
then ∫  r ⋅ dt is equal to 64. The circulation of F round the curve C where
2 dt  →
  F = y $i + z$j + xk$ and C is the circle x 2 + y 2 = 1,
(a) 10 (b) 6 z = 0 is
(c) −7 (d) 0 (a) − π (b) π (c) 2π (d) 0
57. Any integral which is evaluated along curve is called 2
65. The value of the integral ∫ ∫ ∫v 45 x y, where v is the
(a) volume integral (b) surface integral
closed region bounded by the planes
(c) line integral (d) none of these
4x + 2y + z = 8, x = 0, y = 0, z = 0 is

58. If F = x $i + $j and curve C is the line y = x from (0, 1) (a) 26 (b) 64 (c) 128 (d) 0
→ → →
to (1, 2) then ∫ F ⋅ d r is
c
66. If F = 4x $i − 2y 2$j + z 2k$ and surfaces is the cylinder
2 x 2 + y 2 = 4 and the circular discs z = 0 and z = 3
(a) (b) 1
3 →

(c) 0 (d)
3
then ∫ ∫s F ⋅ n$ ds over the plane face z = 0 of the

2 cylinder is

59.

The value of ∫ F ⋅ dr, where F = x 2$i − xy$j, from the (a) 36π (b) x 2 + y 2
c
(c) 2π (d) 0
point (0, 0) to (1, 1) along the curve y 2 = x is

1 1 67. For a closed surface, ∫ ∫ r ⋅ n$ ds is
(a) 12 (b) −12 (c) (d) − s [Avadh 2018]
12 2
(a) 3 v (b) 2 v (c) v (d) 0
60. The value of the integral ∫ (xdy − ydx) around the
68. If S is the surface of unique sphere and
circle x 2 + y 2 = 1 is →
F = ax $i + by$j + czk$ where a, b, c are constants then
(a) π (b) 2π (c) 0 (d) 1 →
→ ∫ ∫s F ⋅ n$ ds is
61. If F = (x + y )$i − 2xy$j and curve C is the line y = 0
2 2

→ →
from (0, 0) to (a, 0) then the value of ∫ F ⋅ d r is π 4
c (a) (a + b + c) (b) π(a + b + c)
3 3
a a2 a3 −a
(a) (b) (c) (d) 2π
3 2 3 3 (c) (a + b + c) (d) 0
3
117

If a = t $i + tk$ , b = $i − $j, c = − $i + t$j + 2k$ then the Green’s, Gauss’s and Stoke’s Theorems
→ → →
69.
1→ → →

value of ∫ ( a ⋅ b × c) dt is 76. According to Gauss divergence theorem ∫ F ⋅ n$ ds is
0 s

6 −7 equivalent to
(a) (b)
7 6 → →
(a) ∫v grad F dv (b) ∫v div F dv
−6
(c) (d) 0
7 → →

(c) ∫v curl F dv (d) ∫v curl F ⋅ n$ dv
70. If F = 2x $i − y$j and C is the curve in the xy-plane,

→ → 77. If v is the volume of the surface S then ∫s r ⋅ n$ ds is
y = x 2, from (0, 0) to (2, 4) then the value of ∫ F ⋅ d r
c
equal to
is
(a) −2 (b) −6 (a) v (b) 2v
(c) −4 (d) 0 (c) 3v (d) 0
→ →
71. If F = (2x + y)$i + (3y − x)$j and C is the straight line 78. The value of ∫ ∫ r × n$ ds is
s
→ →
from (0, 0) to (2, 0) then ∫ F ⋅ d r is (a) 1 (b) 2
c

(a) 1 (b) 2 (c) −1 (d) 0


(c) 4 (d) 8 →
79. For any closed surface S, ∫ ∫ curl F ⋅ n$ ds is
→ s
72. If F = (2y + 3)$i + xz$j + (yz − x)k$ and C is the
(a) 1 (b) $i
straight line from (0, 0, 0) to (0, 0, 1) then the value of
→ → →
is (c) 2 F (d) 0
∫c F ⋅ d r

80. If v is the region bounded by S and vector N is
(a) 0 (b) 4

(c) −5 (d) 10 always normal to S then ∫ curl N dv is
v

73. If F = x $i + y$j and curve C is the arc of the curve (a) 0 (b) 2v
→ → (c) 3v (d) v
y = x 2 from (0, 0) to (1, 1) then ∫ F ⋅ d r is
c
→ →
81. If F = x $i + y$j + zk$ then the value of ∫ div F dv over
(a) 0 (b) 1 v
the rectangular parallelopiped 0 ≤ x ≤ 9, 0 ≤ y ≤ b,
(c) 2 (d) none of these
0 ≤ z ≤ c is
74. The work done in moving a particle in a force field
→ (a) a + b + c (b) 3abc
F = 3x 2$i + (2xz − y) $j + 3k$ along the line joining
(c) ab + bc + ac (d) 0
(0, 0, 0) to (2, 1, 3) is
→ →
(a) 0 (b) 24 82. The value of ∫ F ⋅ n$ ds, where F = 4xz $i − y 2$j + yzk$
s

(c) 10 (d) 16 and S is the surface of the cube bounded by x = 0,


→ → x = 1, y = 0, y = 1, z = 0, z = 1 is
75. If F = x $i + y$j + zk$ then ∫ ∫s F ⋅ n$ ds, where S is the 2 4
(a) (b)
surface of unit sphere is 3 3
(a) π (b) 2π 3 3
(c) (d)
(c) 4π (d) 0 4 2
118

→ → →
83. If F = yz $i + z$j + 2xyk$ over the tetrahedron 90. According to Stoke’s theorem ∫ F ⋅ d r is
C
bounded by the planes x = 0, y = 0, z = 0 and →→ → → → →
→ (a) ∫S( ∇ F) ⋅ d S (b) ∫S( ∇ ⋅ F) ⋅ d S
x + y + z = 1 then the value of ∫ F ⋅ n$ ds is
s
→ →
1 1 (c) ∫S ( ∇ × F) ⋅ dS (d) none of these
(a) (b)
2 4
→ →
(c) 2 (d) 0 91. ∫r ⋅ d r is
→ →
84. If F = x $i + y$j + zk$ then the value of ∫ F ⋅ n$ ds over (a) 1 (b) 2
s
(c) 3 (d) 0
the rectangular parallelopiped 0 ≤ x ≤ a, 0 ≤ y ≤ b,

0 ≤ z ≤ c is 92. If F = (2x − y) $i − yz 2$j − y 2zk$ , where S is the upper
(a) abc (b) 2abc half surface of the sphere x 2 + y 2 + z 2 = 1 and C is
(c) 0 (d) 3abc → →
its boundary then ∫ F ⋅ d r is
→ C
85. If F = x $i + xz$j + y k over the region bounded by a
2$
(a) – 3 (b) π

cube of unit length then ∫ F ⋅ n$ ds is (c) 0 (d) 1
s

(a) 6 (b) 2 93. If F = 2$i + 3$j + 9k$ and S is the upper half surface
(c) 1 (d) 0 of the sphere x 2 + y 2 + z 2 = 9 and C be its

86. If F = (2x − z) $i + (3y − z) $j − zk$ taken over the boundary then ∫ F ⋅ d r is
→ →

region bounded by x = 0, x = a, y = 0, y = a, z = 0,
→ (a) 9π (b) 6π
z = a then ∫ div F dv is
v (c) −12 (d) 0
(a) 4a 3 (b) a 3 94.

If F = x i + xy$j and S is the rectangle in xy-plane
2$

(c) 4a (d) 0
whose sides are x = 0, y = 0, x = 1, y = 1 then
87. If S is the surface of the unit cube then → →
∫F⋅dr is
2 2
∫S x dydz + y dzdx + 2z (xy − x − y) dxdy, is
1
(a) 2 (b)
(a) 1 (b) 4 2
1 (c) 3 (d) 0
(c) 2 (d)
2 x
95. The value of ∫C (e dx + y dy − 3 dz) over the cure

88. If F = (2x + 3z) $i − (xz + y) $j + (y 2 + 2z) k$ and S is
x 2 + y 2 = 1, z = 0 is

the surface of the sphere of radius 3 then ∫S F ⋅ n$ ds (a) π (b) 8
is (c) –1 (d) 0

(a) 18π (b) 108π (c) π (d) 0 96. If F = 4xy $i + (2x 2 + y)$j and C is the boundary of

89. If F = x $i − y$j + 2zk$ over the sphere the triangle with vertices at (0, 0) (2, 0) (2, 1) then the
→ →
→ value of ∫ F ⋅ d r is
x 2 + (y − 1)2 + (z − 1)2 = 1 then ∫ F ⋅ n$ ds is C
S
(a) 0 (b) 2π
2π 4π
(a) (b)
3 3 (c) –1 (d) none of these

(c) (d) π
3
119

97. If C be the boundary of the rectangle 0 ≤ x ≤ 1, 104. If C is the circle x 2 + y 2 = 1 then the value of
0 ≤ y < 1, z=3 then the value of
(sin z dx − cos x dy + sin y dz ) is
∫c [(cos x sin y − xy) dx + sin x cos y dy] is
∫C π
(a) 0 (b)
(a) 1 (b) 2 2
(c) π (d) − π (c) π (d) 1

98. If F = x 2$i + xy$j and S is the rectangle in the plane 105. If A be the area bounded by a simple closed curve C
then ∫ (x dy − y dx) is
z = 0, whose sides are x = 0, y = 0, x = 1, z = 1 then c
→ (a) A (b) 2 A
∫S curl F ⋅ n$ dS is
(c) 3 A (d) 4 A
1
(a) (b) 2 →
2 106. If a vector field F is not irrotational then
1 → →
(c) (d) 0 (a) F = ∇φ (b) ∇ × F = 0
3
→ → →
99. If C is the curve x 2 + y 2 = a, z = y 2 then the value of (c) ∫C F ⋅ d r =0 (d) ∇⋅ F = 0

∫c (yz dx + zx dy + xy dz) is → →
107. If F = x $i + y$j + zk$ such that F = ∇φ then the value
(a) a (b) 2a
of φ is
(c) 8a (d) 0
(a) (xy + yz + zx) + c (b) (x 2 + y 2 + z 2) + c
→ →
100. If ∫c F ⋅ d r = 0 for every closed curve C lying in a 1 2
(c) (x + y + z) + c (d) (x + y 2 + z 2) + c
simple connected region R then 2
→ → 108. If the vector
(a) grad F = 0 (b) div F = 0

→ → F = (3x − 3y + az) $i + (bx + 2y − 4z)$j
(c) curl F = 0 (d) grad div F = 0
+ (2x + cy + z)k$ is irrotational then (a, b, c) is
101. If C is the boundary of the region enclosed by x = y 2
(a) (2, − 3, − 4) (b) (−3, 2, − 4)
and y = x 2 describe in the positive sense then the
(c) (2, 3, 4) (d) (2, − 3, 5)
value of ∫ ((2xy − x 2) dx + (x 2 + y 2) dy) is
MISCELLANEOUS
(a) 2π (b) − π
→ →
(c) 8 (d) 0 109. If a = $i − $j + k$ , b = 3$i + p$j + 5k$ and
102. The value of ∫ ((x 2 − y cos x) dx + (y + sin x) dy), →
c c = $i + 2$j − k$ are coplanar then p is given by
π
where C is the rectangle with vertices (0, 0),  , 0 , (a) 6 (b) 4
2 
(c) – 4 (d) – 6
 π , 1 , (0, 1) is
2  →
110. If the vector A = (ax + ay) $i + 4y$j + 3k$ is
(a) 0 (b) π solenoidal then a is
π
(c) (d) 2π (a) – 4 (b) – 2
2
(c) 0 (d) 2
2
103. The value of ∫c ((x − y 2) dx + 2xy dy) around the
→ → →
111. If F = x $i + 2y$j + 3zk$ then evaluate ∫ ∫ F ⋅ n$ d S is
rectangular boundary x = 0, x = a, y = 0, y = b is S

(a) 11ab (b) 2ab (a) 3 S (b) 6 V


(c) 2ab 2
(d) 2a b 2 (c) 6 S (d) 3 V
120

112. If C is the curve given by x 2 + y 2 = 1, y 2 = z then (c) divergence theorem

∫C yz dx + zx dy + xy dz is (d) none of these


121. The maximum value of directional derivative of
(a) 0 (b) 1

(c) – 1 (d) 2 A = x 2 − 2y 2 + 4z 2 at (1, 1, − 1) is
→ ∂ 2r 7 7
113. If r = xyz $i + xz 2$j − y 3k$ then at (0, 0, 1) is (a) 2 (b)
∂x ∂y 3 3
3 7
(a) $i + $j (b) $i − $j (c) 2 (d) 6
7 3
(c) $i (d) $j
→ →
→ d a → → db → → d → →
114. The vector A = e x sin y λ + e x cos y$j is 122. If = r × a, = r × b then ( a × b ) is
dt dt dt
(a) solenoidal → → →
(a) r ( a × b )
(b) rotational
→ → →
(c) irrotational (b) r ⋅ ( a × b )
(d) (a) and (c) both → → →
(c) r × ( a × b )
2 2
115. If C is the curve x + y = 4, z = 0 then → → →
(d) r × ( a ⋅ b )
x
∫c (e dx + 2y dy − dz) is
123. If the surface ax 2 − yz = (a + z) x is orthogonal to
(a) 1 (b) 2
the surface 4x 2y + z 3 = 4 at (1, − 1, 2) then a is
(c) 4 (d) 0
2 5
(a) (b)
116. ∇ ⋅ (∇r n) is [Avadh 2018] 5 2
(a) n(n + 1) r n− 2 (b) n(n + 1) r n−1 3
(c) (d) 5
2
n− 2 n
(c) n(n + 3) r (d) n(n + 1) r
124. If (x + 3y) $i + (y − 2z) $j + (x + az) k$ is solenoidal
117. The projection of i$ − 2$j + k$ on 4i$ − 4$j + 7 k$ is then a is equal to
17 19 (a) 2 (b) 3
(a) (b)
9 9
(c) −2 (d) 0
19 17
(c) (d) ( 2, 01
,)
3 3 125. The value of ∫ (ze x dx + 3yz dy + e x + y 2) dz is
2
( 0, 21
,)
118. ∇e r is given by
(a) e − 1 (b) e 2 + 1
→ r2 2
(a) 2 r e (b) 2e r (c) e 2 − 5 (d) e + 1
→ r2 → r
(c) r e (d) r e 126. If n$ is the unit outward drawn normal to any closed
→ → $
surface s then ∫ div ndv is
119. ∇ f( r ) × r is given by v
→ (a) S (b) v
(a) 0 (b) r f ′(r)

(c) S + v (d) S − v
f ′( r ) → →
(11
,)
(c) (d) r f( r )
→ 127. The value of ∫ [(x + y 2) i$ + (x 2 − y 2) $j] ⋅ dr over
2
r ( 0, 0)

120. Green’s theorem is a special case of y 2 = x is


[Avadh 2018] (a) 7 (b) 10
(a) Gauss theorem 7 10
(c) (d)
(b) Stoke’s theorem 10 7
121

128. A particle moves along the curve x = 4 cos t, 133. →


If r = x ^

i + y^
j + 2^
k, then the value of ∇ log| r | is
y = 4sin t, z = 6 t, then the acceleration at time t is
[Kanpur 2018]
(Kanpur 2018)
→ →
r r
(a) 4 cos t^
i + 4 sin t^
j+^
k (a) (b)
r r2
^ ^ ^
(b) 4 sin t i − 4 cos t j + k →
r
(c) (d) None of these
^
(c) 4 cos t i − 4 sin t j − k^ ^ r3
134. The value of grad φ, where φ is given by
(d) −4 cos t^
i − 4 sin t^
j
φ = x 3 − y 3 + xz 2, at the point (1, − 1, 2) is
129. The position vector of a moving particle at time t is
[Kanpur 2018]

given by r = (3t − 4)^
i + (t 2 − 2)^
j + 4t 3 ^
k , then ^ ^ ^
(a) 3 i − 7 j + 4 k
the acceleration at time t = 2 is [Kanpur 2018]
(b) −7^
i + 3^
j + 4^
k
(a) 2^
i + 2^
j + 48^
k
(c) −7^
i − 3^
j + 4^
k
(b) 2^
j + 48^
k
(d) 7^
i − 3^
j + 4^
k
(c) 2^
i +^
j + 24^
k
135. A unit normal vector to the level surface
(d) 3^
j + 4^
k
x 2y + 2xz = 4 at the point (2, − 2, 3) is

130. For r = 2t 2^
i + t^
j − 3t 3 ^
k, the value of [Kanpur 2018]
 →
2 → d2 r  (a) 1 / 3^
i + 2 / 3^
j + 2 / 3^
k
∫1  r × dt is
dt 
  [Kanpur 2018]
(b) 1 / 3^
i − 2 / 3^
j + 2 / 3^
k
(a) −42^
i + 90^
j − 6^
k
(c) −1 / 3^
i + 2 / 3^
j + 2 / 3^
k
^ ^ ^
(b) 42 i − 90 j − 6 k
(d) 1 / 3^
i + 2 / 3^
j − 2 / 3^
k
(c) 42^
i + 90^
j + 6^
k
→ →
(d) −42^
i − 90^
j + 6^
k 136. If F = x 2y^
i − 2x 2^
j + 2yz ^
k, then curl F is
131. A particle moves along the curve [Kanpur 2018]

r = e cos t i + e sin t $j + e − t ^
−t ^ −t
k. The magnitude ^ 2
(a) (2x + 2) i − (x + 2z) k ^

of its velocity at time t = 0 is [Kanpur 2018]


(b) (2x + 2z)^
i − (x 2 + 2z)^
k
(a) 2 3 (b) 3 / 2
(c) 3 (d) 2 / 3 (c) (2x + z)^
i + (x 2 + 2z)^
k

132. If F (t) = t^i + (t 2 − 2t)^
j + (3t 2 + 3t 3)^
k then the (d) (x + 2z)^
i − (x 2 + 2z)^
k
1 r
value of ∫ F (t)dt is → →
0 [Kanpur 2018] 137. If F = 2x 2 2^
i − xy 2z^
j + 3y 2x ^
k then div. F is

(a) ^
i / 2 + 2 / 3^
j + 7 / 4^
k [Kanpur 2018]
(a) 2xz(2 − y)
(b) ^
i / 2 − 2 / 3^
j + 7 / 4^
k
(b) xz(1 − y)
(c) −1 / 2^
i + 2 / 3^
j + 7 / 4^
k (c) xy(1 − z)
(d) xy(2 − z)
(d) −1 / 2^
i − 2 / 3^
j + 7 / 4^
k
122

138. The value of div. (r 4 r ) is


→ 145. By Green’s theorem the value of
[Kanpur 2018] 2
∫ C (x − cos h y)dx + ( y + sin x)dy, where C is the
(a) (n + 2)r 4 + 1
rectangle with vertices (0, 0), ( π, 0), ( π, 1), (0, 1)
(b) (n + 3)r 4 + 2
(a) cos 1 − 1 (b) sin 1 − 1
(c) (4 + 3)r 4 (c) cos h 1 − 1 (d) sin h 1 − 1
(d) (n + 2)r 4 146. By Green’s theorem the value of
139. The value of div. [r grad r ] is −3
[Kanpur 2018] ∫ [(cos x sin y − xy)dx + sin x cos y dy], where ρ is
(a) 2r −3
(b) 3r 4 the circle x 2 + y 2 = 1

(c) 2r 3 (d) 3r −4 (a) 0 (b) 1

140. The value of the line integral ∫ (xdy − ydx) around (c) 2 (d) 3
→ →
the circle x 2 + y 2 = 1 is 147. If a = ^
i + 2^
j + 3^
k, then curl a is equal to

[Kanpur 2018, Avadh 2018] [Avadh 2018]



(a) π (b) 2π (c) 3π (d) 4π (a) 0 (b) 0
→ → →
141. The value of ∫ F ⋅ d r , where (c) 14 (d) a
c
→ d → →
F = (x 2 + y 2)^
i − 2xy^
j and the curve c is the 148. The value of ( a × b ) is
dt (Avadh 2018)
rectangle in the xy-plane bonded by y = 0, x = a,
→ →
y = b , x = 0 is [Kanpur 2018] → db da
(a) a × + b×
2 2 dt dt
(a) 2ab (b) 2a b
→ →
2
(c) −2a b (d) −2ab2 (b)
d a db
×
dt dt
142. The workdone in moving a particle in a force field

→ → db
F = 3x 2^
i + (2xz − y)^
j + 3^
k along the line joining (c) a ×
dt
(0, 0, 0) to (2, 1, 3) is [Kanpur 2018]
→ →
(a) 12 (b) 16 d a → → db
(d) ×b+ a×
dt dt
(c) 0 (d) 20
→→
149. If φ is a constant function, then the value of gred φ is
143. The value of ∫ ∫ F n ds, where
S (a) | gred φ|
→ ^ ^ ^ (b) zero vector
F = yz i + 2x j + xy k and s is the part of surface of
(c) zero scalar
the sphere x 2 + y 2 + z 2 = 1 which lies in the first
(d) None of these
octant is [Kanpur 2018]
→ → →
(a) 3 / 8 (b) 3 / 10 150. The vector ^
i × (a ×^
i) + ^
j × (a ×^
j) + ^
k × (a × ^
k)
(c) 3 / 11 (d) 3 / 13 is equal to
→ → →
144. If F = (2x 2 − 32)^
i − 2xy^
j − 4x ^
k, then the value of (a) 0 (b) a
→ → →
∇⋅ F dv, where V is the closed region (c) 2 a (d) None of these
∫ ∫ ∫V
151. The Green’s theorem establishes a relation between
bounded by the planes x = 0 , y = 0 , z = 0 and
[Avadh 2018]
2x + 2y + z = 4 is [Kanpur 2018]
(a) Line integral and surface integral in a plane
(a) 2/3 (b) 4/3
(b) Line integral and volume integral
(c) 7/3 (d) 8/3
123

(c) Surface integral and volume integral (c) Green’s theorem


(d) None of these (d) None of these

152.
→ → → → →
The value of [ a − b , b − c, c − a] is
→ 158. A necessary and sufficient condition that the line
→ →
[Avadh 2018] integral ∫ A⋅ d V = 0 for any closed curve C is that
C
(a) 0 (b) 1 [Avadh 2018]
(c) 2 (d) 3 → →
(a) div. A = 0 (b) curl A = 0
2  1
153. The value of ∇   is → →
 r [Avadh 2018] (c) div. A ≠ 0 (d) curl A ≠ 0

(a) 2 / r 3 (b) −2 / r 3 159.



If F = y^
i + (x − 2xz)^
j − xy ^
k, then the value of
(c) 0 (d) None of these → →
∫ ∫S ( ∇ × F ) ⋅^
n ds is where S is the surface of sphere
154. The directional derivative of φ = xy + yz + zx in the
x 2 + y 2 + z 2 = a 2 above the xy-plane
direction of the vector ^
i + 2^
j + 2^
k at (1, 2, 0)
[Kanpur 2018]
(Avadth 2018)
(a) 0 (b) 1
(a) 5/3 (b) 8/3 (c) 2 (d) 3
(c) 10/3 (d) 7/3 160. The value of (x + y 2)ds is where S is the
2
∫ ∫S
→ →
155. If r × d r = 0 then unit vector ^
r is [Avadh 2018] surface of the cone z 2 = 3 (x 2 + y 2) bonded by

(a) constant (b) zero z = 0 and z = 3. [Kanpur 2018]


(a) 3π
(c) one (d) None of these
(b) 6π
156. The unit normal vector to the surface z = x 2 + y 2 at
(c) 9π
the point (1, 1, 2) is [Avadth 2018]
(d) 11π
^ ^ ^ ^ ^ ^
(a) 2 i + 2 j − k (b) 2 i + 2 j + k 161. If S denotes the surface of the cube bonded by the
planer x = 0, x = a, y = 0, y = a, z = 0, z = a, then by
^
i +^
j + 2^
k 2^
i + 2^j −^
k the application of Gauss divergence theorem the
(c) (d)
6 3
value of ∫ ∫ (x ^
i + y^
j + z^
k) ⋅ ^
n ds is
→ → → ^ S
157. The formula ∫ ∫ ∫V ∇⋅ F dv = ∫ ∫S F ⋅ n ds is
[Kanpur 2018]
governed by [Avadh 2018] (a) a 3 (b) 2a 3
(a) Stoke’s theorem (c) 3a 3 (d) 0
(b) Gauss’s theorem
124

ANSWERS

MULTIPLE CHOICE QUESTIONS

1. (c) 2. (a) 3. (b) 4. (d) 5. (d) 6. (d) 7. (d) 8. (d) 9. (b) 10. (c)

11. (b) 12. (c) 13. (d) 14. (d) 15. (b) 16. (c) 17. (c) 18. (c) 19. (b) 20. (d)

21. (b) 22. (c) 23. (c) 24. (d) 25. (a) 26. (b) 27. (c) 28. (d) 29. (c) 30. (d)

31. (b) 32. (d) 33. (d) 34. (a) 35. (c) 36. (a) 37. (b) 38. (c) 39. (c) 40. (a)

41. (d) 42. (d) 43. (d) 44. (b) 45. (a) 46. (b) 47. (d) 48. (d) 49. (b) 50. (d)

51. (b) 52. (d) 53. (c) 54. (b) 55. (c) 56. (a) 57. (c) 58. (d) 59. (c) 60. (b)

61. (c) 62. (d) 63. (c) 64. (a) 65. (c) 66. (d) 67. (a) 68. (b) 69. (b) 70. (c)

71. (c) 72. (a) 73. (b) 74. (d) 75. (c) 76. (b) 77. (c) 78. (d) 79. (d) 80. (a)

81. (b) 82. (d) 83. (d) 84. (d) 85. (c) 86. (a) 87. (d) 88. (b) 89. (c) 90. (c)

91. (a) 92. (b) 93. (c) 94. (b) 95. (d) 96. (a) 97. (b) 98. (a) 99. (d) 100. (c)

101. (d) 102. (a) 103. (c) 104. (a) 105. (b) 106. (d) 107. (d) 108. (a) 109. (d) 110. (a)

111. (b) 112. (a) 113. (c) 114. (d) 115. (d) 116. (a) 117. (b) 118. (a) 119. (a) 120. (b)

121. (d) 122. (c) 123. (b) 124. (c) 125. (c) 126. (a) 127. (c) 128. (d) 129. (b) 130. (a)

131. (c) 132. (b) 133. (b) 134. (d) 135. (c) 136. (b) 137. (a) 138. (c) 139. (d) 140. (b)

141. (d) 142. (b) 143. (a) 144. (d) 145. (c) 146. (a) 147. (b) 148. (d) 149. (b) 150. (c)

151. (a) 152. (a) 153. (c) 154. (c) 155. (a) 156. (d) 157. (b) 158. (b) 159. (a) 160. (c)

161. (c)
125

HINTS AND SOLUTIONS


→ − a sin t a cos t a tan α
4. r = sin t $i + cos t $j + t k$
19. [ r& &&r &&&]
r = − a cos t − a sin t 0

dr a sin t − a cos t 0
= cos t $i − sin t $j + k$
dt
= a tan α (a 2) = a 3 tan α

d2 r
and = − sin t $i − cos t $j
dt 2 23. Here f = 3x 2y − y 3z 2

→ ∂f ∂f ∂f
d2 r ∵ grad f = i + j +k
so = sin 2 t + cos 2 t = 1. ∂x ∂y ∂z
dt 2
∴ ∇f = i (6xy) + j (3x 2 − 3y 2z 2) + k (−2y 3z)

5. If r = cos nt $i + sin nt $j then
At (1, 1, 1), ∇f = 6$i − 2k$
i j k
→ dr

31. Let f = x 2 + y − z − 1, normal vector is ∇f
r × = cos nt sin nt 0
dt
∂f ∂f ∂f
− n sin nt n cos nt 0 so ∇f = i + j +k = 2x $i + $j − k$
∂x ∂y ∂z
= k$ |n cos 2 nt + n sin 2 nt| = nk$ $ $ $
$ unit vector is 2i + j − k
At (1, 0, 0), ∇f = 2$i + $j − k,
→ 6
11. Here r = e − t $i + e t sin t $j + k$ then velocity
32. $
The directional derivative is ∇f ⋅ a

→ dr
v = = − e − t $i + (e t cos t + e t sin t) $j ∂f ∂f ∂f
dt Here grad f = ∇f = i$ + $j + k$
∂x ∂y ∂z
→ →
at t = 0, v = − $i + $j so v = |v | = 1 + 1 = 2 ∇f = (2xyz + 4z 2) $i + (x 2z) $j + (x 2y + 8xz) k$

16.
→ →
Here A = a $i + $j − 2k$ & B = 2$i − $j + bk$ At (1, − 2, − 1) ∇f = 8$i − $j − 10k$

$ $ $

Given A × B = 0

and $ = 2i − j − 2k
a
3
i j k
So directional derivative
i.e. a 1 −2 = 0
 2$i − $j − 2 k$  37
2 −1 b = (8$i − $j − 10 k$ ) ⋅   =
 3  3

⇒ i (b − 2) + j (−4 − ab) + k (− a − 2) = 0
34. f = x 2 + 2y 2 + 3z 2 − 12
i.e. b = 2, ab = − 4
grad f = ∇f = $i (2x) + $j (4y) + k$ (6z)
and a = − 2 so (a, b) = (−2, 2)
At (1, 2, − 1), ∇f = 2$i + 8$j − 6k$
126

→ 2
equation of tangent is (R r ) ⋅ grad f = 0  t4 
= + 2t 3 − 3t 2 
 2 0
$ →
where R = x $i + y$j + zk, r = $i + 2$j − k$
= (8 + 16 − 12) = 12
So we get
→ →
58. ∫c F ⋅ d r = ∫ (xdx + dy)
[(x − 1) $i + (y − 2) $j + (z + 1) k$ ] ⋅ (2 $i + 8 $j − 6 k$ ) = 0

∵ y = x so dy = dx
i.e. 2(x − 1) + 8(y − 2) − 6(z + 1) = 0
1 1
i.e. 2x + 8y − 6z = 24 ⇒ x + 4y − 3z = 12 = ∫0 xdx + dx = ∫0(1 + x) dx
∂f ∂f ∂f
44. ∇f = $i + $j + k$ 1
∂x ∂y ∂z  x2 1 3
= x +  =1+ 2 = 2
 2 0
$i $j k$ 60. For circle x 2 + y 2 = 1, x = cos t, y = sin t
2
 ∂ f 2
∂ ∂ ∂ ∂ f 
Curl (∇f ) = = $i  − 
∂x ∂y ∂z  ∂y∂z ∂z ∂y  dx = − sin t dt, dy = cos t dt so

∂f ∂f ∂f 2π
∂x ∂y ∂z ∫ xdy − ydx = ∫0 (cos t ⋅ cos t dt + sin t ⋅ sin t dt)

 ∂2 f ∂2 f   ∂2 f ∂2 f  = ∫0 dt = 2 π
+ $j  −  + k$  −  =0
 ∂ z ∂ x ∂ x∂ z   ∂ x∂ y ∂ y ∂ x 
→ →
64. The circulation is ∫ F ⋅ d r = ∫ ydx + zdy + xdz
→ →
46. A is solenoidal then div. A = 0
where curve is x 2 + y 2 = 1, z = 0
∂ ∂ ∂
Thus (ax + 2y) + (2y − 3) + (−3z) = 0
∂x ∂y ∂z so x = cos t, y = sin t, z = 0

i.e. a + 2 − 3 = 0 ⇒ a = 1 i.e. dx = − sin tdt, dy = cos tdt, dz = 0

→ → → → 2π
47. A is irrotational then curl A = 0 So ∫ F ⋅ d r = ∫ (− sin 2 tdt + 0 + 0)

i$ $j k$ 2π
= −∫ sin 2 t dt = − π
0
∂ ∂ ∂
Thus curl A = =0
∂x ∂y ∂z 66. Over a plane face z = 0, normal vector is −k so
ay −3z 2y
→ $ 2$
∫ ∫ F⋅ n$ ds = ∫ ∫ (4x i − 2y j) ⋅ (− k$ ) ds = ∫ ∫ ods = 0
i.e. $i (2 + 3) + $j (0 − 0) + k$ (0 − a) = 0
70. ∵ y = x 2 so dy = 2x dx
→ →
54. a⋅ b = 2t 3 + (6t 2 − 6t)
→ → $ $ $ $ $
2→ → 2 3
∫c ( F⋅ d r) = ∫c (2x i − y j) ⋅ (dx i + dy j + dzk)
∴ ∫0 a ⋅ b = ∫0(2t + 6t 2 − 6t) dt
= ∫ (2x dx − y dy)
127

= ∫ 2x dx − x 2 2xdx → → $ $
So ∫cF⋅ d r = ∫ ∫s k ⋅ k ds = ∫ ∫ ds = π
2 3
= ∫0 (2x − 2x ) dx = − 4
→ → →
→ →
94. By Stoke’s theorem ∫ F ⋅ d r =
c ∫ ∫s curl F⋅ n$ ds
74. The work done w = ∫ F ⋅ d r
$i $j k$
= ∫ [3x 2dx + (2xz − y)dy + 3dz]
→ ∂ ∂ ∂
curl F = = yk$
(2, 1, 3) ∂x ∂y ∂z

x2 xy 0
x y z
Line is = = (0, 0, 0)
2 1 3 → → $ $ 1 1 1
So ∫ F ⋅ d r = ∫ ∫s y k ⋅ k ds = ∫0 ∫0 y dx dy = 2
c
x 3
∴ y= ,z = x $i $j
2 2 k$

1 3 → ∂ ∂ ∂
i.e. dy = dx, dz = dx 99. Here curl F = =0
2 2 ∂x ∂y ∂z
yz zx xy
→ → 2  3x 2dx +  3x 2 − x  dx + 9 dx  = 16
∴ ∫ F⋅ d r = ∫0   2 2 2  → → →
So by Stoke’s theorem ∫ F ⋅ d r =
c ∫ ∫s curl F⋅ n$ ds = 0
→ a b c
81. ∫ ∫ ∫ div. F dv = ∫0 ∫0 ∫0 3 dx dy dz = 3 abc  ∂N ∂M 
104. ∵ ∫ (Mdx + Ndy) = ∫ ∫R  ∂x −  dx dy
∂y 
→ →
83. ∫ ∫sF⋅ n$ ds = ∫ ∫ ∫ ∇⋅ F dv M = (cos x sin y − y), N = sin x cos y$j
(by Gauss divergence theorem)
∂M ∂N
= cos x cos y − x, = cos x cos y
 ∂ ∂ ∂  ∂y ∂x
= ∫ ∫ ∫  (yz) + ( z) + (2xy) = 0
 ∂x ∂y ∂z 

Hence ∫ ∫ dx dy = ∫0 ∫ r cosθ rdθ dr = 0
→ → R
85. ∫ ∫ F ⋅ n$ ds = ∫ ∫ ∫ div. F dv
→ →
(By Gauss divergence theorem) 108. Since F is irrotational curl F = 0 curl

→ $i $j k$
so ∫ F ⋅ n$ ds = ∫ ∫ ∫ (1 + 0 + 0) dx dy dz
s
→ ∂ ∂ ∂
1 1 1
curl F =
∂x ∂y ∂z
= ∫0 ∫0 ∫0 dx dy dz = 1
3x − 3y + az bx + 2y − 4 z 2x + cy + z
→ → →
92. By Stoke’s theorem ∫ F ⋅ d r = $
c ∫ ∫s curl F⋅ nds
=0
$i $j k$ $i (c + 4) + $j (a − 2) + k$ (b + 3) = 0
→ ∂ ∂ ∂
where curl F = = k$
∂x ∂y ∂z ⇒ a = 2, b = − 3, c = − 4
2x − y − yz 2 − y 2z
128


So A is both solenoidal and irrotational.
111. By Gauss divergence theorem → → →
115. By Stoke’s theorem ∫ F ⋅ d r = ∫ ∫S curl F⋅ nds
C
→ →
∫ ∫S F ⋅ n$ ds = ∫ ∫ ∫ div. F dv $i $j k$
→ → ∂ ∂ ∂
div. F = 1 + 2 + 3 = 6 curl F =
∂x ∂y ∂z
→ 0
So ∫ ∫ F ⋅ n$ ds = ∫ ∫ ∫ 6 dv = 6 v ex 2y
S

→ = $i (0) + $j (0 − 0) + k$ (0 − 0) = 0
114. ∵ div A = e x sin y − e x sin y = 0
→ →
$i $j k$ So ∫ F ⋅ d r = ∫ ∫S O ds = 0.
C

→ ∂ ∂ ∂ ❍❍❍
and curl A = =0
∂x ∂y ∂z

e x sin y e x cos y 0
1

BRA University, Agra, Examination-2019


Paper-III Geometry and Vector Calculus
[Time : 2 Hours] [Maximum Marks : 70]

Note : Attempt all questions. Each question carries equal marks.


1. The greater rate of increase of φ = xyz 2 at (1, 0, 3) 7. If φ = x 2 y + y2 z + z 2 x, then Curl grad φ is
is : ^ ^ ^
(a) 2(z i + x j + y k)
(a) 0 (b) 1 (c) 3 (d) 9
^ ^ ^
2. The angle between the surface x + y2 + z 2 = 9 2 (b) y i + (4 xz − 3 z 3 ) j + 2z k

and x 2 + y2 + z − 3 = 0 at the point (2, − 1, 2) is : (c) 2x ^


i + 3y^
j + 2z ^
k
 4   8 
(a) cos −1   (b) cos −1   (d) 0
 3 21   5 21 
8. For what value of a, b, c such that
 2   8 
(c) cos −1   (d) cos −1   → 3 ^
f = (6 xy + az ) i + (bx − z) j 2 ^ 2
+ (3 xz + cy) k
^
 3 21   3 21 
→ →
is irrotational?
^ ^ ^
3. If r = x i + y j + z k then div r is equal to : (a) a = 1, b = − 1, c = 3
(a) 0 (b) 3 (b) a = 1, b = + 3, c = − 1
(c) Can’t find (d) None of these (c) a = − 1, b = 3, c = − 1
→ ^ ^ ^ (d) a = − 1, b = − 3, c = − 1
4. If R = x i + y j + z k and
→ ^ ^ ^ →
→ 9. If F = x 2 y i + y2 z j + z 2 x k, then Curl Curl F is
r = | r | = x 2 + y2 + z 2 then the value of
^ ^ ^ ^ ^ ^
1 (a) 2(z i + x j + y k) (b) z i + x j + y k
divgrad   is :
r ^ ^ ^
(c) 3(z i + x j + y k) (d) None of these
(a) 0 (b) 1
→ ^ ^ → →
(c)
2
(d) None of these 10. If F = x 2 i + xy j , then the value of ∫c F ⋅ d r
r
from (0, 0) to (1, 1) along the straight line joining
5. For what value of ‘a’ so that the vector
these points :
→ ^ ^
f = (ax + 2y + 4 z) i + (2x − 3 y + 3 z) j 1 2 4
(a) 1 (b) (c) (d)
^
3 3 3
+ (3 x + 2y − 2 z) is solenoidal :

11. The value of the integral ∫ ∫s F ⋅ nds on the
(a) 3 (b) 4 (c) 5 (d) 6
→ ^ ^ ^ → yz-plane is :
6. If f = (x + y + 1) i + j + (− x − y) k, then Curl f
→ ^ dydz → ^ dydz
is : (a) ∫ ∫S F⋅ n (b) ∫ ∫S F⋅ n
^^ ^^
[n ⋅ j ] [n ⋅ i ]
^ ^ ^ ^ ^ ^
(a) i + j + k (b) − i − j + k → ^ dydz
(c) ∫ ∫S F⋅ n (d) None of these
^ ^ ^ ^ ^ ^ ^^
(c) − i + j − k (d) − i − j − k [n ⋅ k]
2

→ → 19. PSP′ is the focal chord of the conic


12. By Stroke’s theorem, ∫ F ⋅ dr is equal to :
C l 1 1
= 1 + e cos θ, then the value of + is :
→ ^ → → ^ r SP SP ′
(a) ∫c ( ∇ × F ) ⋅ n d r (b) ∫ ∫S ( ∇ × F ) ⋅ n ds (a) 1 (b) 0
r → ^ 1 2
(c) ( ∇ × F ) ⋅ n ds (d) None of these (c) (d)
∫ ∫S l l
13. By Gauss divergence theorem, the value of 20. The equation of tangent at a point ‘α’ on the
^ ^ ^ ^ l
conic = 1 + e cos θ is :
∫S (ax i + by j + cz k) n ds is : r
2π 4π l
(a) (a + b + c) (b) (a + b) (a) = e sin θ + sin (θ − α)
3 3 r
4π 4π l
(c) (b + c) (d) (a + b + c) (b) = e cos θ + cos (θ − α)
3 3 r
14. By Green’s theorem, the value of l
(c) = cos θ + e cos (θ − α)
where is r
∫C (2x − y) dx + (x + 3y) dy C
(d) None of the above
x 2 y2
+ =1: 21. If S is a pole, SS′ is the initial line, SS′ = 2ae and a
4 1
point P moves in such a way that SP + S′ P = 2a,
(a) π (b) 2π (c) 4 π (d) 6π
where 0 < e < 1, then the locus of P is :
15. The work done in moving a particle in a force
a (1 − e2 )
→ 2^ ^ ^ (a) = 1 − e cos θ
field F = 3 x i + (2xz − y) j + 3 k along the line r2
joining (0, 0, 0) to (2, 1, 3) is a (1 − e2 )
(b) = 1 + e cos θ
(a) 0 (b) 8 (c) 16 (d) 20 r2
l
16. The equation = 1 + e cos θ and a (1 − e2 )
r (c) = (1 + e cos θ)2
r
l
= − 1 + e cos θ represent : a (1 − e2 )
r (d) = (1 − e cos θ)
r
(a) Different conics
(b) Two straight lines 22. If the normals at the points whose vectorial angles
l
(c) Tangents of each other are α , β and γ on the parabola = 1 + cos θ meet
r
(d) The same conic
in a point (ρ, φ) then :
l
17. The line = A cos θ + B sin θ touches the conic (a) φ = α + β + γ (b) 2φ = α + β + γ
r
l (c) 3φ = α + β + γ (d) 4φ = α + β + γ
= 1 + e cos θ, if :
r 23. The curve represented by the equation
(a) ( A − e)2 + B 2 = 1 (b) ( A + e)2 + B 2 = 1 r = 2a cos (θ − α ) is :
(c) ( A + e)2 − B 2 = 1 (d) None of these (a) A circle (b) A parabola
4 (c) An ellipse (d) A hyperbola
18. The polar equation = 3 + 4 cos θ + 3 sin θ
r 24. The latus rectum of the conic (cosec α + cos θ) = 1
represents : is :
(a) An ellipse (b) An parabola (a) 2 cos α (b) 2 cosec α
(c) A hyperbola (d) A circle
(c) 2 sin α (d) sin α
3

25. The equation ax + by = 1 represents : 1 1 1


(b)  , , 
(a) Ellipse  3 3 3

(b) Pair of straight line 1 1 1 


(c)  ± ,± ,± 
(c) Parabola  2 2 2

(d) Pair of perpendicular straight line (d) (1, 1, 1)


26. The centre of the conic 33. The angle between two lines whose direction
14 x 2 − 4 xy + 11 y2 − 44 x − 58 y + 71 = 0 is : ratios are : (1, 1, 2) and ( 3 − 1, − 3 − 1, 4 ) is :
(a) (2, 2) (b) (3, 2) (a) 30° (b) 45°
(c) (3, 3) (d) (2, 3) (c) 90° (d) None of these
27. The following equation represents 34. If a line makes angles α , β, γ with the coordinates
3 x 2 + 12xy + 12y2 + 435 x − 9 y + 72 = 0 is: axes, then sin 2 α + sin 2 β + sin 2 γ is equal to :
(a) Ellipse (b) Parabola (a) 0 (b) 1
(c) Hyperbola (d) Circle (c) 2 (d) 3
x2 y2 35. The angle between the lines whose direction
28. The confocal conics of the ellipse + = 1 is :
a2 b2 cosines satisfy the equations l + m + n = 0 and
x2 y2 x2 y2 2 lm + 2 nl − mn = 0 is :
(a) + =1 (b) + =1
a2 b2 aλ bλ (a) 60°
2 2 (b) 90°
x y
(c) + =1 (d) None of these
a 2λ 2 b2λ 2 (c) 120°
29. Confocals cut at : (d) 150°
(a) Right angle (b) Acute angle 36. The shortest distance between the lines
(c) Obtuse angle (d) None of these x − 1/2 y − 1/3 z − 1/4
= = and
1 2 3
30. The length of the major axis of the conic
3 y − 1 4 z − 1 2x − 1
22x 2 − 12xy + 17 y2 − 112x + 92y + 178 = 0 is: = = is :
3 4 2
(a) 2 (b) 2 2 4
(a) 0 (b)
5
(c) 2 (d) 3 2
4
31. If the coordinates of the points A, B, C and D are (c) (d) None of these
3
(3, 4, 5), (4, 6, 3), (−1, 0, 4 ) and (1, 2, 5)
respectively the projection of CD on AB is : 37. The shortest distance between the z-axis and the
3 4 line x + y + 2z − 3 = 0, 2x + 3 y + 4 z − 4 = 0 is :
(a) (b)
4 3 (a) 1 (b) 2
3 5 (c) 3 (d) 4
(c) (d)
5 3
38. The lines x = py + q, z = ry + s and
32. Direction cosines of the lines equally inclined to x = p′ y + q′ , z = r ′ y + s′ are perpendicular if the
the coordinates axes, are : value of pp′ + rr ′ is equal to :
1 1 1 
(a)  ± ,± ,±  (a) 0 (b) 1
 3 3 3
(c) −1 (d) −2
4

39. A plane cuts the axes in A, B, C and the centroid of (b) a1a2 + b1b2 + c 1c 2 = 0
the triangle ABC is (a, b, c). The equation of plane is : (c) a1a2 + b1b2 + c 1c 2 + d1d2 = 0
x y z x y z
(a) + + =1 (b) + + =2 (d) None of the above
a b c a b c
47. The angle between the planes 3x + 4y − 5z = 9 and
x y z x y z
(c) + + =3 (d) + + =0 2x + 6y + 6z = 7 is :
a b c a b c
π π
40. The shortest distance between the lines (a) (b)
6 3
x + 3 y−6 z x + 2 y z −7
= = and = = is : π
−4 3 2 −4 1 1 (c) (d) None of these
2
(a) 81 (b) 9
(c) 27 (d) 1 48. If the cone x 2 + y 2 − z 2 tan 2 α = 0 has three
mutually perpendicular generators, then α equals :
41. The straight line x = 2y = λ z is parallel to the
plane 3 x − 4 y + 5 z − 10 = 0 if λ is equal to : π π
(a) (b)
2 3
(a) 0 (b) −3
π
(c) −4 (d) −5 (c) (d) tan −1 2
4
42. The equation of the plane passing through the 49. If the equation
point (1, − 2, 3) and perpendicular line
αx 2 + βy 2 + 2z 2 − 2αx − 4βy − 40 = 0 represents a
x −2 y−4 z−6
= = is : sphere, its radius is :
3 5 7
(a) 3 x − 5 y + 7 z = 0 (b) 3 x − 5 y + 7 z = 8 (a) 2 30 (b) 2 10

(c) 3 x − 5 y + 7 z = 14 (d) None of these (c) 3 5 (d) 5

43. The distance of the origin from the plane 50. If the plane lx + my + nz = p touches the sphere
2x + 6 y − 3 z + 7 = 0 is : x 2 + y 2 + z 2 = a 2 then l 2 + m 2 + n2 is equal to :
(a) 0 (b) 1 p2
(a) (b) p 2
(c) 2 (d) 3 a2
x − 4 y− 2 z −k
44. If the lines = = lies on the plane a2
1 1 2 (c) a 2 (d)
p2
2x − 4 y + z = 7, then the value of K is :
51. The angle between the radical plane of two spheres
(a) 7 (b) −7
and the line joining their centres is :
(c) 0 (d) None of these
(a) 0° (b) 30°
45. The acute angle between two diagonals of a cube
is : (c) 90° (d) None of these

 1   1  52. The equation ax + by + cz 2 + 2ux + 2vy + 2wz


2 2
(a) cos −1   (b) cos −1  − 
 3  3 + d = 0 represents a cone if :
−1 1 (a) d = au 2 + bv 2 + cw 2
(c) cos −1   (d) cos −1  
 3  3
u 2 v2 w 2
46. If the planes a1x + b1y + c 1z + d1 = 0 and (b) d = + +
a b c
a2 x + b2 y + c 2 z + d2 = 0 are perpendicular to
u 2 v2 w 2
each other then : (c) + + + d=0
a b c
a b c
(a) 1 = 1 = 1
a2 b2 c 2 (d) au 2 + bv 2 + cw 2 + d = 0
5

a b c
53. The equation + + = 0 represents (a) 6 (b) 12
y−z z−x x−y
(c) 18 (d) 24
a:
61. The vertex of the cone xy + 2yz + 3zx = 0 is the
(a) Cylinder (b) Cone
point :
(c) Pair of planes (d) Sphere
(a) (−1, 0, 0) (b) (0, 0, 0)
54. If a tangent plane to the sphere x 2 + y 2 + z 2 = r 2
(c) (−1, − 2, 0) (d) (−1, 2, 2)
makes intercepts a, b, c on the coordinate axes, then
62. The equation of the right circular cylinder whose
1 1 1 1 1 1 1 1
(a) + + = (b) + + = axis is z-axis and radius ‘a’ is:
a b c r a2 b2 c2 r2
1 1 1 (a) y 2 + z 2 = a 2 (b) z 2 + x 2 = a 2
(c) + + =r (d) None of these
a b c (c) x 2 + y 2 = a 2 (d) x 2 + y 2 + z 2 = a 2
55. The vertex of the cone
63. The equation of the cylinder with generator parallel to
(x − 1)2 + (y − 3z) (2y + z) − (x − 1) (4y + 6z) = 0 is :
x-axis and passing through the circle
(a) (−1, 0, 0) (b) (1, 0, 0) x 2 + y 2 + z 2 = 9, 2x = y + z is :
(c) (−1, 2, 3) (d) (1, 2, − 3) (a) 5y 2 + 2yz + 4z 2 = 36
56. The plane αx + βy + γz = 0 cuts the cone
(b) 5y 2 + 2yz + 5z 2 = 36
yx + zx + xy = 0 in perpendicular generators if :
α β γ (c) x 2 + z 2 + 2yz = 81
(a) + + =0 (b) α + β + γ = 0
β γ α
(d) 3y 2 + z 2 + 2yz = 81
(c) βγ + γα + αβ = 0 (d) None of these
64. Which one of the following equations represents a
57. The radius of the smallest spheres passing through cone not having three mutually perpendicular
the points (1, 0, 0), (0, 1, 0) and (0, 0, 1) is : generators :
2 2 (a) xy + 2yz + 3zx = 0 (b) xy + 2yz − 3zx = 0
(a) (b)
3 3
(c) x 2 + 2y 2 − 3z 2 = 0 (d) x 2 − 2y 2 + 3z 2 = 0
3
(c) (d) 1
2 65. Two spheres x 2 + y 2 + z 2 = 25 and
2 2 2
58. Two spheres of radii 3 and 4 cut orthogonally. The x + y + z − 24x − 40y − 18z + 225 = 0 :
radius of common circle is : (a) Touch internally
12 (b) Touch externally
(a) 12 (b)
5
(c) Intersect in two points
12
(c) 12 (d) (d) Do not intersect
5
59. The pole of the plane 3x + 4y + 5z = 6 with respect 66. The number of normals that can be drawn to the

to the spherex 2 + y 2 + z 2 = 12 is : x2 y2 z2
Ellipsoid + + = 1 from any given point
2 2
a b c2
(a) (1, 1, 0) (b) (0, 0, 0) (α, β, γ) is :
(c) (6, 8, 10) (d) (3, 4, 5) (a) 2 (b) 4
60. The number of radical planes and radical centres of (c) 6 (d) 8
n spheres are equal if n is equal to :
6

67. The equation of the normal to the conicoid 72. The surface represented by the equation
2 2 2 2 2
ax + by + cz = 1 at (α, β, γ) is : 5(x + y ) = − 6z is a :
x −α y−β z − γ (a) Parabolic cylinder
(a) = =
aα bβ cγ (b) Elliplic paraboloid
(b) aαx + bβy + cγz = 1 (c) Right circular cylinder of revolution
x −α y−β z−γ (d) Paraboloid of revolution
(c) = =
α/a 2 β/b2 γ /c 2
73. If the section of conicoid ax 2 + by 2 + cz 2 = 1 by the
(d) None of the above
plane lx + my + nz = 0 is hyperbola, then its section
68. The equation of tangent plane to the central by the plane lx + my + nz = p is :
conicoid 5x 2 − 3y 2 + z 2 + 2 = 0 at the point (1, 1, 0) (a) A hyperbola (b) A circle
is :
(c) A ellipse (d) Pair of straight line
(a) 5x − 3y + 2z + 2 = 0
74. The surface represented by the equation
(b) 5x − 3y + 2 = 0 2 2 2
2x + 5y − 9z = 1 is :
(c) 5x + 2z + 2 = 0
(a) An ellipsoid
(d) −3y + 2z + 2 = 0
(b) A hyperboloid of one sheet
69. The centre of the central conicoid
ax 2 + by 2 + cz 2 = 1 is : (c) Elliptic paraboloid

(a) (0, 0, 0) (b) (0, 1, 0) (d) Elliptic cylinder



(c) (0, 0, 1) (d) (1, 0, 0) 75. If r = 3 t 2^
i + (t 2 − 2 t) ^
j + t3^
k then the value of
70. The equation of director sphere of the ellipsoid →
d2 r
at t = 1 is :
x2 y2 z2
+ + = 1 is : dt 2
a2 b2 c2
(a) 6^
i + 2^
j − 6^
k (b) 6^
i − 2^
j + 6^
k
(a) x 2 + y 2 + z 2 = a 2 + b2 + c 2
1 1 1 (c) 6^
i + 2^
j + 6^
(b) x 2 + y 2 + z 2 = + + k (d) None of these
2 2 2
a b c r
1 1 1 76. If r = a cos λt + b sin λt, where a and b are constant
(c) x 2 + y 2 + z 2 = + + →
a b c d2 r →
vectors, then + λ 2 r is equal to :
2
(d) None of these dt
71. The condition that the plane → →
(a) −λ r (b) λ 3 r
x cos α + y cos β + z cos γ = p touches the Ellipsoid

x2 y2 z2 (c) λ 4 r (d) 0
+ + = 1 is :
2 2 2
a b c → →
77. If r = t^
i − t 2^
j + (t − 1) ^
k and S = 2t 2^
i + 6t^
j, then
2
(a) a cos α + b cos β + c cos γ = p 2→ →
the value of ∫ ( r ⋅ s )dt is :
(b) a 2 cos 2 α + b2 cos 2 β + c 2 cos 2 γ = p 2 0

(c) a cos α + b cos β + c cos γ = p (a) 0 (b) 4


(d) None of these (c) 8 (d) 12
7


78. If r = 5t 2^
i + t^
j − t3^
k, then the value of →
r

r
(a) (b) −
 →
2 → d 2 r 
r r3
∫1  r x dt is : r
dt 2  r
 (c) (d) None of these
r2
(a) 14^
i + 75^
j + 15^
k (b) 14^
i − 75^
j + 15^
k 80.
1
Find the directional derivative of in the direction of
r
(c) −14^
i + 75^
j − 15^
k (d) −14^
i − 75^
j − 15^
k → →
r where r = x ^
i + y^
j + z^
k is :

79. If r = x ^
i + y^
j + z^
k and (a) −
1
(b) −
1
→ r2 r3
r = | r | = (x 2 + y 2 + z 2)1/ 2
1
1 (c) − (d) None of these
then the value of grad is : r4
r

ANSWERS

1. (c) 2. (b) 3. (d) 4. (c) 5. (c) 6. (c) 7. (d) 8. (a) 9. (a) 10. (d)

11. (b) 12. (c) 13. (d) 14. (c) 15. (c) 16. (d) 17. (a) 18. (a) 19. (d) 20. (b)

21. (d) 22. (b) 23. (a) 24. (a) 25. (d) 26. (b) 27. (b) 28. (c) 29. (a) 30. (b)

31. (b) 32. (b) 33. (d) 34. (c) 35. (b) 36. (c) 37. (d) 38. (c) 39. (c) 40. (b)

41. (a) 42. (d) 43. (b) 44. (b) 45. (d) 46. (c) 47. (b) 48. (d) 49. (b) 50. (a)

51. (c) 52. (b) 53. (c) 54. (b) 55. (d) 56. (c) 57. (c) 58. (d) 59. (d) 60. (a)

61. (d) 62. (c) 63. (a) 64. (c) 65. (a) 66. (c) 67. (a) 68. (c) 69. (a) 70. (b)

71. (b) 72. (c) 73. (d) 74. (b) 75. (c) 76. (d) 77. (c) 78. (c) 79. (b) 80. (a)

MULTIPLE CHOICE QUESTIONS

HINTS AND SOLUTIONS


( 3 − 1) + 1 ( 3 − 1) + 2 × 4 6 1
3. cos a = = = =0
2 2 6 24 2
1 + 1 + 4 ( 3 − 1) + (− 3 − 1) + 16
1 π
−1+ 3− 3 −1+ 8 θ = cos −1   =
=−  2 3
6 3 + 1 − 2 3 + 3 + 1 + 2 3 + 16
8

→ → l
5. Vector F will be solinoidal then ∇⋅ F = 0 or i. e., = 1 + e cosθ1
r1
 ∂ ∂ ∂ 
 i+ j+ k
 ∂x ∂y ∂z  Thus every point p on the curve (1) also lies on the
curve (2). Hence the equation (1) and (2) represent
⋅ {(dx + 2y + 4z) i + (2x − 3y + 3z) j
the same conic
+ (3x + 2y − 2z) k} = 0
67. Suppose the line
⇒a − 3 − 2 = 0⇒a = 5
l
= A cos θ + B sin θ …(1)
^
i ^
j ^
k r

9. Curl F = ∇ × F = x 2y y 2z z 2x is a tangent to the conic
∂ ∂ ∂
l
∂x ∂y ∂z = 1 + e cosθ …(2)
r
= − y 2^
i − z 2^
j − x 2^
k at the point whose vectorial angle is α. The equation
^ ^ ^
of the tangent to (2) at the point ‘α’ is
i j k
∂ ∂ ∂ l
Curl Curl F = ∇ × (∇ × F ) = = cos (θ − α) + e cos θ
∂x ∂y ∂z r
− y2 −z 2 −x 2 l
= (e + cos α) cos θ + sin θ ⋅ sin α
r
=^ j {0 + 2x} + k$ {0 + 2y}
i {0 + (2z)} + ^ the equation (1) and (3) should represent the same
1
^ ^ ^
= 2 ( 3 i + x j + y k) line so comparing the coefficients of , cosθ and
r
l e + cos α sin α
66. = 1 + e cosθ …(1) sinθ we have 1= =
r A B
l cosα = A − e and sinα = B
= − 1 + e cosθ …(2)
r
Squaring and adding, we have ( A − e)2 + B 2 = 1
Let p(r1, θ1) for any point on the curve (1) so that
l 86. When you will draw an ellipse and a hyperbola in
= 1 + e cosθ1
r1 which the focus and incident point has to be same,
the angle between the tangents at the incident point
l
= − 1 + e cos (θ1 + π) will be 90° which is right angle.
− r1
❍❍❍
9
Allahabad State University, Examination-2019
Paper-III Analytical Geometry
[Time : 2 Hours] [Maximum Marks : 50]

Note : All questions are compulsory. This question paper consists of 80 objective questions.
1. The distance between the points (2 , 90° ) and 8. The radius of the circle r = 6 cos α is :
(3, 30° ) is : (a) 2 (b) 4 (c) 6 (d) 3
(a) 7 (b) 7 9. The equation of a directrix of the conic
(c) 13 (d) 13 l
= 1 − e cos θ is :
2. The perpendicular distance from the pole to the r
10 l l
line = 3 cos θ + 4 sin θ is : (a) = e cos θ (b) = − e cos θ
r r r
(a) 1 (b) 2 (c) 3 (d) 5 l l
(c) = e cos 2 θ (d) = − e cos 2 θ
r r
3. The cartesian equation of polar equation
7
r 2 = a2 cos 2 θ is : 10. The conic = 5 + 4 cos θ represents :
r
(a) x 2 − y2 = a2 (x 2 + y2 )
(a) A circle (b) A parabola
(b) x 2 + y2 = a2 (x 2 − y2 ) (c) A hyperbola (d) An ellipse
(c) (x 2 − y2 )2 = a2 (x 2 + y2 ) 6
11. The latus rectum of the conic = 3 + 4 cos θ is :
r
(d) (x 2 + y2 )2 = a2 (x 2 − y2 )
(a) 2 (b) 3
4. If (x, y) and (r , θ) be the cartesian and polar
(c) 4 (d) 6
coordinates of a point, then :
1
(a) x = r sin θ, y = r cos θ 12. The line = A cos θ + B sin θ will touch the conic
r
(b) x = r cos θ, y = r sin θ 1
= 1 + e cos θ if :
(c) r = x sin θ, r = y cos θ r
(d) r = x cos θ, r = y sin θ (a) ( A − e)2 + B 2 = 1 (b) ( A − e)2 + B 2 = 0
5. The polar co-ordinates of the point whose (c) ( A + e)2 + B 2 = 1 (d) A2 + (B − e)2 = 0.3
cartesian co-ordinates are (1, 3) are :
13. The equation of tangent to the conic
(a) (2, π /3) (b) (2, π /6)
1
(c) (1, π /3) (d) (1, π /6) = 1 − e cos θ at point ‘α’ is :
r
5 1
6. The polar equation = 3 cos θ + 5 sin θ (a) = e cos θ + cos (θ − α )
r r
represents :
1
(a) A straight line (b) A circle (b) = − e cos θ + cos (θ − α )
r
(c) A parabola (d) A hyperbola 1
(c) = e cos (θ − α )
7. The centre of the circle r = 2a cos (θ − α ) is: r
(a) (a, a) (b) (a, α ) 1
(d) = e cos θ + cos 2 (θ − α )
(c) (a, 0) (d) (0, α ) r
10

14. Which one of the following conics have 21. The direction ratios of the line joining of the
asymptote : points (2 , 3, 5) and (6, − 5, 8) are :
(a) Circle (b) Parabola (a) (4 , − 8, 3) (b) (8, − 2 , 13)
(c) Ellipse (d) Hyperbola (c) (4 , 6, 7) (d) None of these
15. The equation of normal to the conic 22. The direction cosines of the line equally inclined
1
= 1 + e cos θ at point ‘α’ is : to the positive direction of coordinate axes are :
r
1 1 1 1 1 1
le sin α (a) , , (b) , ,
(a) = e sin θ + e cos (θ − α ) 3 3 3 3 3 3
r (1 + e cos α )
1 1 1
le sin α (c) 1, 1, 1 (d) , ,
(b) = e sin θ + sin (θ − α ) 2 2 2
r (1 + e cos α )
le sin α 23. The sum of intercepts made by the plane
(c) = sin θ + e cos (θ − α ) 2x + 3 y + 4 z = 12 on axes is :
r (1 + e cos α )
(d) None of the above (a) 9 (b) 10
(c) 12 (d) 13
16. The equation of normal to the parabola
1 24. The equation of plane passing through the points
= 1 + cos θ at point π/2 is :
r (2, 3, − 4 ) and (1, − 1, 3) and parallel to z-axis is :
1 1 (a) 4 x + y = 5 (b) 4 x − y = 5
(a) = cos θ + sin θ (b) = sin θ − cos θ
r r
(c) 2x + 3 z = 7 (d) None of these
1 1
(c) = cos θ (d) = 2 cos θ 25. A plane meets the co-ordinate axes at points
r r
A, B, C such that the centroid of ∆ABC is the
17. The equation of the director circle of the conic
1 point (a, b, c). Then the equation of plane is :
= 1 + e cos θ is :
r (a) x/a + y/b + z/c = 2 (b) x/a + y/b + z/c = 3
2 2 2 (c) ax + by + cz = 2 (d) ax + by + cz = 3
(a) r (1 − e ) + 2 ler cos θ + 2 l = 0
2 2 2 26. The length of perpendicular drawn from origin to
(b) r (1 + e ) − 2 ler cos θ − 2 l = 0
the plane 3 x + 4 y − 5 z = 50 is:
(c) r 2 (1 − e2 ) + 2 ler cos θ − 2 l 2 = 0
(a) 12 (b) 25
(d) None of the above
1 (c) 5 2 (d) None of these
18. For the conic = 1 + e cos θ, the sum of the
r 27. Planes a1x − b1y + c 1z + d1 = 0 and
reciprocals of the segment of any focal chord is a2 x + b2 y + c 2 z + d2 = 0 will be perpendicular if :
equal to : (a) a1a2 + b1b2 + c 1c 2 = 0
(a) l (b) 2l a b c
(b) 1 + 1 + 1 = 0
(c) 1 / l (d) 2 / l a2 b2 c 2
19. If PSP′ is a focal chord of a conic, then the (c) a1a2 + b1b2 + c 1c 2 = 1
tangents P at and P′ intersect on : a b c
(d) 1 + 1 + 1
(a) Initial line (b) Vertex a2 b2 c 2
(c) Directrix (d) None of these
28. Planes 2x − 3 y − 5 z = 6 and
4
20. The point on the conic = 1 + 2 cos θ is: −8 x + 12y + 20 z = 13 are :
r
(a) Parallel (b) Perpendicular
(a) (2, π/3) (b) (4, π / 6)
(c) Equal (d) None of these
(c) (1, π/2) (d) None of these
11

29. The angle between the planes 2x − y + z = 5 and 36. The equation of plane containing the line
x + y + 2z = 7 is : x −1 y− 2 z − 3
= = and point (0, 0, 5) is :
0 3 4
(a) π/6 (b) π/4
(a) 14x − 4y + 3z = 15 (b) x + 2y + 5z = 6
(c) π/3 (d) π/2
(c) 3x + 4y + 5z = 9 (d) None of these
x −1 y−1 z − 2
30. The line = = meets the plane
2 3 3 37. The equation of line passing through the point
2x − 3 y + z + 5 = 0 at point : x −1 y− 2 z + 3
(1, − 1, 2) and parallel to line = =
2 3 4
(a) (2, 3, 3) (b) (1, 2, 3)
is :
(c) (2, − 3, 1) (d) (7, 10, 11) x −1 y−1 z − 2
(a) = =
31. The equation of straight line passing through the 2 3 4
points(2, 3, 4 ) and (3, 4 , 5) is : x −1 y+ 1 z − 2
(b) = =
x −2 y−3 z−4 2 3 4
(a) = =
1 1 1 x −1 y− 2 z + 3
(c) = =
x −3 y−4 z−5 1 −1 2
(b) = =
1 −1 1 (d) None of the above
x + 2 y+ 3 z+ 4 38. The equation of plane passing through the line of
(c) = =
1 −1 −1 intersection of planes 2x − 2y + z = 1 and
(d) None of the above x − 2y + 3 z = 2 and point (1, − 1, 1) is:
32. The lines x = a1y + b1, z = c 1y + d1 and (a) x + 2z = 1 (b) y + 2z = 1
x = a2 y + b2 , z = c 2 y + d2 will be perpendicular (c) y − 2z = 1 (d) x − 2z + 1 = 0
if :
39. The shortest distance between the lines
(a) a1a2 + b1b2 = 1 (b) b1b2 + c1c 2 = 1 x − 2 y z −1 x−4 y−3 z
= = and = = is :
(c) a1a2 + c1c 2 + 1 = 0 (d) None of these 1 2 2 2 2 3
(a) 1 (b) 2
33. The image of the point (3, 5, 7) in the plane
(c) 3 (d) 4
2x + y + z = 6 is :
40. The shortest distance between the lines
(a) (−1, 3, 5) (b) (1, 3, 5)
x −1 y z x y−2 z
= = and = = is :
(c) (3, 5, 7) (d) None of these 0 0 3 2 0 0
x −1 y− 3 z − 4 (a) 1 (b) 2
34. The angle between the line = =
2 2 −1 (c) 3 (d) 5
x + 1 y+ 5 z −7
and = = is : 41. The radius of the sphere 3 x 2 + 3 y2 + 3 z 2 + 12x
1 2 −2
+ 18 y − 24 z − 12 = 0 is :
(a) π/3 (b) cos −1(1/3)
(a) 2 (b) 3
(c) π/2 (d) None of these (c) 4 (d) 5
35. The co-ordinates of foot of perpendicular drawn 42. The equation of sphere passing through origin
from the point (1, 3, 7) on the line and points (a, 0, 0), (0, b, 0) and (0, 0, c) is:
x −1 y− 2 z − 3
= = is : (a) x 2 + y2 + z 2 + ax + by + cz = 0
1 2 3
(b) x 2 + y2 + z 2 − ax + by − cz = 0
(a) (1, 2, 3) (b) (2, 4, 6)
(c) x 2 + y2 + z 2 − ax − by − cz = 0
(c) (3, 6, 9) (d) (1, 0, 4)
(d) x 2 + y2 + z 2 = a + b + c
12

43. If two spheres of radii 3 and 4 cut orthogonally. 50. The equation ax 2 + by 2 + cz 2 + 2ux + 2vy
Then the radius of common circle is : + 2wz + d = 0 represents a cone if :
(a) 12/5 (b) 5/12
(a) au 2 + bv 2 + cw 2 = d
(c) 5 (d) 12
u 2 v2 w 2
44. If the co-ordinates of ends of diameters of a (b) + + =d
a b c
sphere are (1, 2 , 3) and (2, − 3, 4 ), then the
a 2 b2 c 2
equation of sphere is : (c) + + =d
u v w
(a) x 2 + y2 + z 2 − 3 x + y − 7 z + 8 = 0
(d) None of these
(b) x 2 + y2 + z 2 + 3 x − y + 7 z + 8 = 0
51. The equation of right circular cone whose vertex is
(c) x 2 + y2 + z 2 − 3 x + y − 7 z = 0 at origin, x-axis and semivertical angle ‘α’ is :
(d) None of the above (a) y 2 + z 2 = x 2 tan 2 α
45. The pole of the plane lx + my + nz = p w.r.t the (b) x 2 + y 2 = z 2 tan 2 α
2 2 2 2
sphere x + y + z = r is :
(c) x 2 + z 2 = y 2 tan 2 α
 lr mr nr   lr 2 mr 2 nr 2 
(a)  2 , 2 , 2  (b)  , , 
p p p   p p p  (d) x 2 + y 2 + z 2 = tan 2 α

 l 2r m 2r n 2r  52. The vertex of the cone x 2 + 2y 2 + 3z 2 − 2xy


(c)  , ,  (d) None of these
 p p p  + 3yz + 2xy = 0 is :

46. The point of contact of plane 2 x + y − 2 z = 9 (a) (0, 0, 0) (b) (1, 1, 1)

and sphere x 2 + y2 + z 2 = 9 is : (c) (1, 2, 3) (d) (1, − 2, − 3)

(a) (2 , 1, 2) (b) (−2 , 1, 2) 53. The cone, reciprocal to the cone


2 2 2
(c) (1, 1, 2) (d) (2 , 1, − 2) ax + by + cz = 0 is:
2 2 2
47. The radius of the circle x + y + z − 2y (a) ax 2 + by 2 + cz 2 = 0
− 4z − 11 = 0, x + 2y + 2z = 15 is : x 2 y2 z 2
(b) + + =0
(a) 7 (b) 4 a b c
(c) 7 (d) None of these x2 y2 z2
(c) + + =0
2 2
48. The equation of cone which has vertex at origin and a b c2
passing through the curve ax 2 + by 2 = z, (d) None of these
lx + my + nz = p is : 54. The equation of right circular cylinder whose axis is
2 2 x-axis and radius ‘r’ is :
(a) p(ax + by ) = z (lx + my + nz)

(b) p 2(ax 2 + by 2) = z (lx + my + nz) (a) x 2 + y 2 = r 2 (b) y 2 + z 2 = r 2

(c) p(ax 2 + by 2) = z (lx + my + nz)2 (c) x 2 + z 2 = r 2 (d) None of these

(d) None of the above 55. The equation of cylinder with generator parallel to
x-axis and passing through the circle
49. The cone has three mutually perpendicular
generators is : x 2 + y 2 + z 2 = 4, x − y − z = 0 is :

(a) x 2 − y 2 + 2z 2 = 0 (b) x 2 + y 2 + 2z 2 = 0 (a) y 2 + z 2 + yz = 2 (b) y 2 + z 2 − 2yz = 4

(c) x 2 + 2y 2 − 3z 2 = 0 (d) x 2 + y 2 + z 2 = 0 (c) y 2 + z 2 + 4yz = 0 (d) None of these


13

56. The equation of axis of right circular cylinder 62. The equation of tangent plane to the conicoid
2 2
x + z = 4 is : 2x 2 + 6y 2 − 3z 2 = 5 at point (−1, 0, 1) is :

x y z x y z (a) 2x + 3z + 5 = 0 (b) 2x − 3y = 4
(a) = = (b) = =
1 0 0 0 1 0 (c) 2x + 3y − 5z = 7 (d) 2x + 5y = 3z

x y z 63. The condition that the plane lx + my + nz = p


(c) = = (d) x = y = z
0 0 1 x2 y2 z2
touches the ellipsoid + + = 1 is :
2 2
57. The equation of cylinder whose generators are a b c2
x y z a2 b2 c2
parallel to line = = and passes through the (a) + + = p2
1 2 1 l 2
m 2
n2
curve x 2 + y 2 = 3, z = 0 is :
l2 m2 n2
(b) + + = p2
(a) x 2 + y 2 + 5z 2 − 2zx − 4yz = 3 a2 b2 c2
(c) a 2l + b2m + c 2n = p 2
(b) x 2 − y 2 − 5z 2 + 2zx + 4yz = 6
(d) a 2l 2 + b2m 2 + c 2n2 = p 2
(c) x 2 + y 2 − 5z 2 + 2zx + 4yz = 3
64. The equation of normal to the conicoid
(d) None of these ax 2 + by 2 + cz 2 = 1 at point (α, β, γ) is :

58. The surface generated by a straight line which is x −α y−β z − γ


(a) = =
a b c
always parallel to a fixed line and intersect a given
x−a y−b z−c
curve or touches a given surface is called a : (b) = =
α β γ
(a) Sphere (b) Cone
x −α y−β z − γ
(c) = =
(c) Cylinder (d) Central conicoid aα bβ cγ
59. The equation of cylinder whose generators are (d) None of these
x y z
parallel to the line = = and passing through 65. The feet of six normals that can be drawn from a
1 −2 3
point on a conicoid lie on :
the curve x 2 + 2y 2 = 1, z = 3 is :
(a) Sphere (b) Cone
(a) 3x 2 + 6y 2 + 3z 2 − 2xz + 8yz − 3 = 0 (c) Cylinder (d) Conicoid
2 2 2
(b) 3x − 6y − 3z + 2xz + 8yz = 3 66. The sum of squares of any three conjugate

(c) 3x 2 + 6y 2 − 3z 2 − 2xz + 8yz − 3 = 0 x2 y2 z2


semidiameters of the ellipsoid + + = 1 is
2 2
a b c2
(d) None of the above
equal to :
60. The equation of cone reciprocal to the cone (a) a + b + c (b) 2abc
x 2 − y2 + z 2 = 0 :
(c) ab + bc + ca (d) a 2 + b2 + c 2
2 2 2 2 2 2
(a) x + y + z = 0 (b) x − y − z = 0
x2 y2 z2
67. The equation + − = 1 represents :
(c) x 2 − y 2 + z 2 = 0 (d) None of these a2 b2 c2

61. The locus of point of intersection of three mutually (a) Ellipsoid


perpendicular tangent planes to a conicoid is a : (b) Hyperboloid of one sheet

(a) Circle (b) Sphere (c) Hyperboloid of two sheet


(c) Conicoid (d) Cone (d) Elliptic paraboloid
14

68. The equation of director sphere of the conicoid 74. If the plane x + 2y − 2z = 4, touches the paraboloid
2 2 2
2x + 3y + 4z = 1 is : 3x 2 + 4y 2 = 24z, then the point of contact is :

(a) x 2 + y 2 + z 2 = 9 (b) x 2 + y 2 + z 2 = 16 (a) (2, 3, 2) (b) (3, 3, 2)

(c) x 2 + y 2 + z 2 = 29 (d) x 2 + y 2 + z 2 = 1 (c) (−2, 3, − 2) (d) (2, − 3, − 3)

69. The pole of the plane lx + my + nz = p w.r.t the 75. The equation of tangent plane to the paraboloid
2 2
conicoid ax + by + cz = 1 is : 2 2x 2 − 5y 2 = 10z at (5, − 2, 3) is :

 al bm cn  a b c  (a) x + y − z = 3 (b) x + 2y − 3z = 4
(a)  , ,  (b)  , , 
p p p  lp mp np  (c) x + y − z = 4 (d) 2x + 2y − z = 3
 l m n 76. How many normals can be drawn from any point to
(c)  , ,  (d) None of these
 ap bp cp 
a paraboloid?
70. The volume of the parallelopiped formed with three (a) 2 (b) 4
conjugate semi-diameter as coterminous edges of
(c) 5 (d) 6
x2 y2 z2
+ + = 1 is equal to : 77. The equation of normal to the paraboloid
a2 b2 c2
x 2 − y 2 = 2z at point (2, 3, 4) is :
(a) ab + bc + ca (b) abc
(c) a 2b2c 2 (d) (a + b + c)2 x−2 y−3 z−4
(a) = =
2 −3 −1
71. The condition that the plane lx + my + nz = p, may
x−2 y−3 z−4
(b) = =
touch the paraboloid ax 2 + by 2 = 2cz is : 2 −3 4
 a2 b2  x−2 y−3 z−4
(a) c  + + 2pn = 0 (c) = =
 l2 m 2  2 3 1

 a 2 b2  (d) None of these
(b) c  +  + 2pn = 0
 l m 
 78. The locus of chord of the paraboloid x 2 − y 2 = 2z
 a2 m 2  bisected at point (2, − 3, 4) is :
(c) c  +  + 2pn = 0
 l b 
 (a) 2x + 3y − z + 9 = 0 (b) 2x − 3y + z = 9
(d) None of these (c) 2x − 3y + 2z = 10 (d) None of these
72. The locus of mid point of a system of parallel chords 79. The polar of point (1, − 2, 3) w.r.t the paraboloid
of a paraboloid is called : 3x 2 + 2z 2 = 4y is :
(a) Conjugate planes
(a) 3x + 2y + 6z = 4 (b) 3x − 2y + 6z + 4 = 0
(b) Diametral planes
(c) Parallel planes (c) 2x + 3y + 6z = 4 (d) None of these
(d) None of these x2 y2 2z
80. The equation − = represents a :
2
73. The feet of the normals from any point on the a b2 c

paraboloid x 2 + y 2 = 2az lie on : (a) Elliptic paraboloid

(a) A sphere (b) A cone (b) Parabolic paraboloid

(c) A cylinder (d) A cube (c) Hyperbolic paraboloid


(d) None of these
15

ANSWERS
MULTILE CHOICE QUESTIONS

1. (b) 2. (d) 3. (d) 4. (b) 5. (a) 6. (a) 7. (c) 8. (d) 9. (b) 10. (d)

11. (c) 12. (a) 13. (b) 14. (d) 15. (b) 16. (b) 17. (c) 18. (d) 19. (c) 20. (a)

21. (a) 22. (b) 23. (d) 24. (b) 25. (b) 26. (c) 27. (a) 28. (a) 29. (c) 30. (d)

31. (a) 32. (c) 33. (a) 34. (d) 35. (b) 36. (a) 37. (b) 38. (d) 39. (c) 40. (b)

41. (d) 42. (c) 43. (a) 44. (a) 45. (b) 46. (d) 47. (c) 48. (d) 49. (c) 50. (d)

51. (a) 52. (d) 53. (c) 54. (b) 55. (a) 56. (b) 57. (b) 58. (c) 59. (a) 60. (c)

61. (b) 62. (a) 63. (d) 64. (c) 65. (b) 66. (d) 67. (b) 68. (c) 69. (c) 70. (b)

71. (d) 72. (b) 73. (a) 74. (a) 75. (d) 76. (c) 77. (b) 78. (c) 79. (c) 80. (c)

HINTS AND SOLUTIONS


1. The distance between the two points in polar form 2 4
10. = 1 + cosθ
r 3
= r12 + r22 − 2r1r2 cos (θ1 − θ2)
latus rectum = 2l = 2 × 2 = 4
Here r1 = 2, θ1 = 90, r2 = 3, θ2 = 30 24. The equation of plane passing (2, 3, − 4) is
= 4 + 9 − 2 × 2 × 3 cos 60° = 7 a(x − 2) + b(y − 3) + c (z + 4) = 0 passed (1, − 1, 3)
3. We have x = r cos θ, y = r sin θ − a − 4b + 7 c = 0, parallel to z-axis with dc. (0, 0, 1)
2 2 2
x + y =r ⇒ c = 0 a = − 4b

∴ r 2 = a 2 cos 2θ −4b (x − 2) + b (y − 3) = 0
−4x + 8 + y − 3 = 0
r 2 = a 2(cos 2 θ − sin 2 θ)
4x − y = 5
⇒ r 4 = a 2(r 2 cos 2 θ − r 2 sin 2 θ)
50 50 50 50
27. = = × = 50 = 5 2
⇒ (x 2 + y 2)2 = a 2(x 2 − y 2) 16 + 9 + 25 50 50 50

6. l = r cos θ, 3 = r sin θ 2−1+ 2 3 1


29. cosθ = = =
2 4+ 1+ 1 1+ 1+ 4 6 2
r = 1 + 3 = 4, r = 2, tanθ = 3
π
θ=
θ = tan −1( 3) 3
π x − a1 y − b1 z − c1
= 30. = =
3 a2 − a1 b2 − b1 c 2 − c1
16

2+ 4+ 2 8 55. x 2 + y2 + z 2 = 4 …(1)
34. cosθ = =
9 9 9
x−y−z=0 …(2)
38. (2x − 2y + z − 1) + λ (x − 2y + 3z − 2) = 0
make homogenous equation of (1) with the help of
⇒ (2 + 2 + 1 − 1) + λ (1 + 2 + 3 − 2) = 0
(2) from (2) x = y + z
4 + 4λ = 0, λ = − 1
⇒ ( y + z) 2 + y 2 + z 2 = 4
⇒ x − 2z + 1 = 0
2y 2 + 2z 2 + 2yz = 4
39. Shortest distance between two lines is
x 2 − x1 y2 − y1 z 2 − z1 y 2 + z 2 + yz = 2
a1 b1 c1 62. The equation of tangent plane to the conic
a2 b2 c2
2x 2 + 6y 2 − 3z 2 = 5 at (−1, 0, 1) is
2 2 2 2x (−1) + 6 (y) (0) − 3(z) (1) = 5
(b1c 2 − b2c1) + (c1a2 − a1c 2) + (a1b2 − a2b1)

4 − 2 3 − 0 −1 2 3 −1 −2x − 3z = 5
⇒ 1 2 2 = 1 2 2 ⇒ 2x + 3y + 5 = 0
2 2 3 2 2 3
68. The equation of director circle of conicoid
1 1 1
(6 − 4)2 + (4 − 3)2 + (2 − 4)2 = ax 2 + by 2 + cz 2 = 1 is x 2 + y 2 + z 2 = + +
a b c
2
2 (6 − 4) + 3 (4 − 3) −1 (2 − 4) = 4 + 3 + 2 = =3 1 1 1 6 + 4 + 3 13
3 = + + = =
2 3 4 12 12
41. x 2 + y 2 + z 2 + 4x + 6y − 8z − 4 = 0
75. The equation of tangent plane to 2x 2 − 5y 2 = 10 z
radius = u 2 + v 2 + w 2 − d = 4 + 9 + 16 + 4 at (5, − 2, 3) is
43. Two spheres of radius r1 and r2 then radius of 2(5x) − 5 (−2y) = 5 (z + 3)
common circle is ⇒ 2x + 2y − z = 3
r1r2 12
= 77. The equation of normal at (α, β, γ) of paraboloid
r12 + r22 5 x −α y−β z − γ
ax 2 + by 2 = 2cz is = =
aα bβ cγ
Here r1 = 3, r2 = 4
Here a = 1, b = − 1, c = 1
44. (x − 1) (x − 2) + (y − 2) (y + 3) + (z − 3) (z − 4) = 0
α = 2, β = 3, γ = 4
⇒ x 2 + y 2 + z 2 − 3x + y − 7 z + 8 = 0
x−2 y−3 z−4
= =
2 −3 4
❍❍❍
17
CSJM University, Kanpur, Examination-2019
Paper-III Geometry and Vector Calculus
[Time : 2 Hours] [Maximum Marks : 70]

Note : All questions are compulsory. This question paper consists of 80 objective type questions.
→ → → → 7. The equation of the asymptotes of the conic
1. If a is any vector, then the value of ∇.( ∇ × a ) is
equal to : x 2 − 3 xy + y2 + 10 x − 10 y + 21 = 0 is

(a) −∇ 2 a

(b) 0 (a) x 2 − 3 xy + y2 + 10 x − 10 y + 20 = 0
→ →
(c) 2 a (d) −2 a (b) x 2 + 3 xy + y2 + 10 x + 10 y + 20 = 0

2.
→ →
If curl a = 0 and curl b = 0, then div ( a × b ) is
→ → (c) x 2 − 3 xy + y2 + 10 x − 10 y − 20 = 0

equal to : (d) x 2 − 3 xy + y2 + 10 x + 10 y − 20 = 0
→ → → →
(a) grad ( a ⋅ b ) (b) a × b 1 1
8. The equation +
→ → →
x + y−a x −y+ a
(c) 0 (d) | a × b| div a 1
+ = 0 represents a :
→ → → ^ ^ ^
y−x + a
3. Find ∫c F ⋅ d r where F = y i + z j + x k and C
(a) Circle (b) Pair of straight lines
is the circle x 2 + y2 = 1, z = 0 :
(c) Parabola (d) Ellipse
π
(a) 0 (b) 9. The principal axes of the conic
2
2 2
(c) π (d) −π ax + 2hxy + by + 2gx + 2 fy + c = 0 are
→ ^ parallel to the lines ;
4. For any closed surface S the value ∫∫ curl F ⋅ n dS
S (a) (x 2 − y2 )h = (a + b)xy
is : (b) (x 2 + y2 ) = + h (a − b)xy

(a) 0 (b) div F
(c) (x 2 − y2 ) = h (a + b)xy
→ →
(c) curl F (d) − F
(d) None of these
5. Evaluate ∫∫ (ax + by + dz 2 )dS over the sphere
2 2
10. There pass two confocals through any point in
S
the plane of an ellipse of which :
x 2 + y2 + z 2 = 1 :
(a) One is ellipse and other is parabola
(a) 0 (b) abc
4 4 (b) One is ellipse and other is also ellipse
(c) π(a2 + b 2 + c 2 ) (d) π(a + b + c)
3 3 (c) One is ellipse and other is a hyperbola
2 2
6. The conic 9 x + 25 y − 90 x − 150 y + 225 = 0 (d) One is parabola and other is a hyperbola
represent a :
11. Passing through a given point two confocals cut
(a) Pair of straight lines (b) Parabola at angle :
(c) Ellipse (d) Hyperbola
(a) 45° (b) 30° (c) 60° (d) 90°
18

12. The point of intersection of the two perpendicular 1 1 1


(a) 1, 1, 1 (b) , ,
tangents one to each of two given confocals lie on 2 2 2
a: 1 1 1 1 1 1
(c) , , (d) ± ,± ,±
(a) Circle (b) Parabola 3 3 3 3 3 3
(c) Ellipse (d) Hyperbola 19. The length of a segment of a line whose
13. The equation to the directrix of the conic projections on the axes are 2, 3 and 6 is :
l (a) 2 (b) 3
= 1 + e ⋅ cos θ is :
r (c) 6 (d) 7
l l
(a) = e (b) = e cos θ 20. The angle between any two diagonals of a cube is :
r r
(a) 90° (b) 30°
l l
(c) = 1 − e cos θ (d) = e tan θ  1
r r (c) 60° (d) cos −1  
 3
14. The equation of the asymptotes of the conic
l 21. The co-ordinates of a point A are (2, 3, –5). If O is
= 1 + e cos θ is :
r the origin, then the equation of the plane through
le A and perpendicular to OA is ;
(a) = (e2 − 1) cos θ ± e2 − 1 sin θ
r (a) 2x + 3 y + 5 z = 38 (b) 2x + 3 y − 5 z = 38
le (c) x + y + z = 18 (d) 2x + 3 y − 5 z = 20
(b) = (1 − e2 ) cos θ ± 1 + e2 sin θ
r
22. The distance between the parallel planes
l
(c) = (e2 − 1) cos θ ± e2 − 1 sin θ 2x − y + 3 z − 4 = 0 and 6 x − 3 y + 9 z + 13 = 0
r
is :
l
(d) = (1 − e2 ) cos θ ± 1 − e2 sin θ 1 10
r (a) (b)
126 126
l
15. If the line = A cos θ + B sin θ is tangent to the 25
r (c) (d) None of these
126
l
conic = 1 + e cos θ then :
r 23. The equation of the plane through the point (0, 7,
2 2 2 2 x +1 y−3 z+ 2
(a) ( A − e) + B = 1 (b) A + (B − e) = 1 –7) and containing the line = =
−3 2 1
2 2 2
(c) A + B = e (d) None of these is :
16. The perpendicular distance of the point (x, y, z) (a) x + y + z = 1 (b) x − y + z = 0
from the z-axis is : (c) x + y + z = 0 (d) x + y + z = 7
2 2 x +1 y−3 z+ 2
(a) Z (b) x +y 24. The lines = = and
−3 2 1
2 2 2 2
(c) x +z (d) y +z x y−7 z + 7
= = interesect at the point :
17. The distance between the points (1, –1, 3) and 1 −3 2
(2, 3, –5) is : (a) (0, 0, 0) (b) (1, –1, 1)
(a) 2 (b) 5 (c) (–1, 3, –2) (d) (2, 1, –3)
(c) 12 (d) 9 25. If the distance of a point from the x-axis is twice of
18. If a straight line is equally inclined to the three its distance from the yz plane, then the locus of
co-ordinate axes, then the direction cosines of the the point is :
line are : (a) 4 x 2 − y2 − z 2 = 0 (b) x 2 − 4 y2 − z 2 = 0
(c) x 2 − y2 − 4 z 2 = 0 (d) x 2 + y2 + z 2 = 4
19

26. The length of the shortest distance between the 32. The equation of the plane which is horizontal and
z-axis and the line x + y + 2z − 3 = 0 passes through the point (1, − 2, − 5) is :
= 2x + 3 y + 4 z − 4 is : (a) x − 2y − 5 z = 0 (b) 2y + 5 z = 0
(a) 0 (b) 1 (c) z + 5 = 0 (d) x − 2y = 0
1
(c) 2 (d) 33. A plane meets the co-ordinate axes in A, B, C
2
such that the centroid of the triangle ABC is the
27. The co-ordinates of the foot of the perpendicular point ( p, q, r ), then the equation of the plane is :
from the point (2, 3, 7) to the plane 3 x − y − z = 7 (a) px + qy + rz = 1 (b) px + qy + rz = 3
are : p q r x y z
(c) + + =3 (d) + + =3
(a) (−1, 1, 1) (b) (0, 1, 1) x y z p q r
(c) (5, 2, 6) (d) (1, 1, 0) 34. If the lines x = ay + b, z = cy + d and
28. The equation of the straight lines which bisect the x = a ′ y + b ′ , z = c ′ y + d ′ are perpendicular
x y z then the condition is :
angles between the lines = = ,
l 1 m 1 n1 a b c
(a) = =
x y z a′ b ′ c ′
= = is :
l 2 m 2 n2
(b) aa ′ + bb ′ + cc ′ = d + d ′
l 1x m 1y n1z
(a) = = (c) aa ′ + cc ′ + 1 = 0
l 1 ± l 2 m 1 ± m 2 n1 ± n2
(d) aa ′ + bb ′ + cc ′ + dd ′ = 0
x y z
(b) = = 35. The equation of the plane through the point
l 1 ± l 2 m 1 ± m 2 n1 ± n2
(2,−1, 1) and the line 4 x − 3 y + 5 = 0 = y − 2z − 5
x y z
(c) = = is :
l 12 ± l 22 m 12 ± m 22 n12 ± n22
(a) 4 x − y − 4 z = 5 (b) x − 4 y + 4 z = 1
(d) None of the above (c) 4 x − 4 y + z = 5 (d) 4 x + y + 4 z = 5
29. The co-ordinates of the point of intersection of
36. The number of spheres that can be made to pass
x +1 y+ 3 z−2
the line = = with the plane through the three given points (1, 0, 0), (0, 1, 0)
1 3 2
and (0, 0, 1) is :
3 x + 4 y + 5 z = 20 is :
(a) 1 (b) 2
(a) (1, 0, 0) (b) (0, 1, 0)
(c) 3 (d) Infinite
(c) (0, 0, 1) (d) (0, 0, 4)
37. A sphere is described on the line joining
30. The angle between the planes 3 x − 4 y + 5 z = 0
A(2, − 1, 4 ) and B(−2 , 2 , − 2 ) as diameter. Then
and 2x − y − 2z = 5 is :
the co-ordinates of the centre of the sphere are :
π π π π
(a) (b) (c) (d)  1 
3 2 6 4 (a) (0, 0, 0) (b)  0, , 1
 2 
31. The normal form of the plane x + 2y − 2z − 9 = 0
 1 
is : (c) (0, 1, 2) (d)  0, − , − 1
 2 
1 2 2
(a) x + y − z = 3
3 3 3 38. The condition that the plane lx + my + nz = p
1 2 2 touches the sphere x 2 + y2 + z 2 = a2 is :
(b) x + y − z = 1
3 3 3
(a) p 2 = a2 (l 2 + m 2 + n 2 )
1 2 2
(c) x + y + z = 3 (b) p = a lmn
3 3 3
1 (c) p 2 = a2 − (l 2 + m 2 + n 2 )
(d) x + y + z =
3 (d) p 2 = a2 + (l 2 + m 2 + n 2 )
20

39. If the sphere 44. The equation 2x 2 + 2y2 + 2z 2 − x − y − z = 0


2 2 2
x + y + z − 2x − 4 y + 2z − 3 = 0 is touched represents a :
by the plane 2x − 2y + z + 12 = 0 then the point (a) Shpere (b) Pair of planes
of contact is :
(c) Cone (d) End points
(a) (−1, 4 , − 2) (b) (1, − 4 , 2)
45. The equation of the cone whose vertex is origin
(c) (1, 2, 1) (d) (−1, 2 , − 1)
and which passes through the curve given by
40. The equation of the sphere through the circle
x 2 + y2 = 2 z, x + y + z = 1 :
x 2 + y2 + z 2 = 9, x + y − 2z + 4 = 0 and the
origin is : (a) x 2 + y2 − 2 z + (x + y + z)2 = 0

(a) x 2 + y2 + z 2 + x + y − 2z = 0 (b) x 2 + y2 = 2 z
(b) x 2 + y2 + z 2 − x − y + 2z = 0 (c) x 2 + y2 + z 2 = 1
(c) 4 (x 2 + y2 + z 2 ) − 9 x − 9 y + 18 z = 0 (d) x 2 + y2 = 2 z(x + y + z)
(d) 4 (x 2 + y2 + z 2 ) + 9 x + 9 y − 18 z = 0 x y z
46. If the line = = is a generator of the cone
41. The angle of intersection of the sphere l m n
2 2 2
x + y + z + 6 y + 2z + 8 = 0 and 2x 2 + 3 y2 − 5 z 2 = 0 then the condition is :

x 2 + y2 + z 2 + 6 x + 8 y + 4 z + 20 = 0 is given (a) 2 l 2 + 3m 2 − 5n 2 = 0
by : l 2 m 2 n2
(b) + − =0
 2 π 2 3 5
(a) cos −1   (b)
 3 2 (c) 2 l 2 + 3m 2 − 5n 2 = 1
π π
(c) (d) l 2 m 2 n2
3 6 (d) + − =1
2 3 5
42. The pole of the plane lx + my + nz = p with
respect to the sphere x 2 + y2 + z 2 = a2 is : 47. The reciprocal cone of ax 2 + by2 + cz 2 = 0 is

 a2 given by :
a2 a2 
(a)  − ,− , −  (a) ax 2 + by2 + cz 2 = 1
 p p p

 la2 x 2 y2 z 2
ma2 na2  (b) + + =1
(b)  − ,− ,−  a b c
 p p p 
(c) a2 x 2 + b 2 y2 + c 2 z 2 = 0
 la2 ma2 na2 
(c)  , ,  x 2 y2 z 2
 p p p  (d) + + =0
a b c
 l 2 m 2 n2 
(d)  , ,  48. If the cone
 ap ap ap
ax 2 + by2 + cz 2 + 2 fyz + 2gxz + 2hxy = 0 have
43. The centres of the spheres of a coaxial system three mutually perpendicular generators, then
which have zero radius are called the :
condition is :
(a) Point of contact 1 1 1
(a) a + b + c = 0 (b) + + =0
(b) Limiting points a b c
(c) Boundary points f g h
(c) fa + bg + ch = 0 (d) + + = 0
(d) End points a b c
21

49. If the vertex of the right circular cone is origin, (a) 3 (b) 4
axis is z-axis and semivertical angle is θ, then the (c) 5 (d) 6
equation of this cone is : 56. The sum of the squares of any three conjugate
(a) x 2 + z 2 = y2 tan 2 θ x2 y2 z2
2 2 2 2 semidiameters of an ellipsoid + + =1
(b) x + y = z tan θ a2 b2 c2
(c) y2 + z 2 = x 2 tan 2 θ is :
2 2 2 2 1 1 1
(d) x + y + z = cot θ (a) (b) a2 + b 2 + c 2
+ +
50. The equation of the enveloping cone of the a2 b2 c2
sphere x 2 + y2 + z 2 = 1 with the vertex at the (c) abc (d) a2 b 2 c 2
point (α , β, γ) is : 57. The surface represented by the equation
(a) α 2 x 2 + β 2 y2 + γ 2 z 2 = 1 x2 y2 z2
2 2 2 2 2
+ 2
− = 1 is :
(b) (αx + βy + γz) = α + β + γ a b c2
(c) (x 2 + y2 + z 2 − 1)(α 2 + β 2 + γ 2 − 1) (a) Sphere
2 (b) Ellipsoid
= (αx + βy + γz − 1)
(d) None of the above (c) Parabolid
(d) Hyperboloid of one sheet
51. The equation of the right circular cylinder whose
axis is the z-axis and the radius r is : 58. The equations of the generators of the
2 2
(a) x + y = r 2 2 2
(b) x + y + z = r 2 2 x2 y2 z2
hyperboloid + − =1 which pass
2 2
(c) y + z = r 2
(d) x 2 + z 2 = r 2 a2 b2 c2
through the point (a cos α , b sin α , 0) are :
52. Any line on the surface of the cylinder is called : x + a cos α y + b sin α z
(a) Axis (b) Generator (a) = =
a cos α b sin α ±c
(c) Guiding curve (d) None of these x − a cos α y − b sin α z
(b) = =
53. The equation of the director sphere of the central a cos α − b sin α ±c
conicoid ax 2 + by2 + cz 2 = 1 is : x − a cos α y − b sin α z
(c) = =
(a) x 2 + y2 + z 2 = 1 a sin α − b cos α ±c

(b) x 2 + y2 + z 2 = a + b + c (d) None of the above


1 1 1 59. Any two generators of the different systems :
(c) x 2 + y2 + z 2 = + +
a b c (a) Intersect (b) Do not intersect
(d) ax 2 + by2 + cz 2 = (ax + by + cz)2 − 1
(c) Are parallel (d) None of these
54. The equation of the tangent plane to the central 60. The plane section of the central conicoid
conicoid 3 x 2 − 5 y2 + z 2 + 2 = 0 at the point ax 2 + by2 + cz 2 = 1 by the plane
(1, 1, 0) is :
lx + my + nz = p is an ellipse, if :
(a) 3 x − 5 y + 2 = 0 (b) 3 x − 5 y + z + 2 = 0
(a) bcl 2 + cam 2 + abn 2 < 0
(c) 3 x + 5 y + z + 2 = 0 (d) 3 x − 5 y − z + 2 = 0
(b) bcl 2 + cam 2 + abn 2 = 0
55. In general from a given point the number of
normals which can be drawn to the conicoid (c) bcl 2 + cam 2 + abn 2 > 0
ax 2 + by2 + cz 2 = 1 are : (d) al 2 + bm 2 + cn 2 > 0
22

61. The area of plane section of the ellipsoid 66. The necessary and sufficient condition for a
x2 y2 z2 →
vector a (t) to have a constant direction is :
2
+ 2
+ 2
=1 by the central plane
a b c → →
→ da → da
lx + my + nz = 0 is given by : (a) a × =0 (b) a ⋅ =0
1 dt dt
 a2l 2 + b 2m 2 + c 2n 2  2 2
 →
(a) πabc   → d a
 l 2 + m 2 + n2  (c) a = constant (d)  =0
 dt 
1  
 l 2 + m 2 + n2 2 → ^ ^ ^
(b) πabc  2 2 2 2 2 2 67. If r = sin t i + cos t j + t k, then the value of
a l + b m + c n 
 2 →
1 d r
 
22
π a l + b m + 2 2
c 2n 2  2  dt 2 is :
(c)    
abc  l 2 + m 2 + n2 
1
(a) 2 (b) 1
π  2
l +m +n 2 2 2 (c) 0 (d) 2
(d)  
abc  a2l 2 + b 2m 2 + c 2n 2  →
du → → d v → →

d → →
68. If = w × u, = w × v then ( u × v ) is :
62. If the section of the conicoid ax 2 + by2 + cz 2 = 1 dt dt dt
→→→
by the plane lx + my + nz = 0 is a hyperbola then (a) 0 (b) [ u v w]
the section by the plane lx + my + nz = p is : → → →
(c) w × ( u × v ) (d) 1
(a) A parabola (b) A circle
69. A particle moves along the curve x = t 3 + 1,
(c) An ellipse (d) A hyperbola
y = t 2 , z = 2t + 5 where t is time. Find the
63. The equation (3 x − 4 y + 5 z − 1)2 + 4
component of acceleration in the direction
(3 x − 4 y + 5 z − 1) + 1 = 0 represents :
^ ^ ^
(a) A pair of parallel planes i + j + 3 k at t = 1 :
8
(b) Hyperboloid (a) 11 (b)
11
(c) Paraboloid
(c) 8 (d) 6
(d) Ellipsoid
 ^ ^ ^
64. The surface x + y+ z = 0 represents : → 2 i − j + 2 k, when t=2
70. Given that r (t) = 
(a) A paraboloid ^ ^ ^
4 i − 2 j + 3 k when t=3
(b) An ellipsoid  →
(c) A hyperboloid
3 → d r 
then the value of ∫  r ⋅ dt is :
2 dt 
(d) A right circular cone with vertex (0, 0, 0)  
65. The general equation to the cone of second (a) 0 (b) 2
degree which passes through the three (c) 5 (d) 10
co-ordinate axes is : → → ^ ^ ^
71. If r = | r|, where r = x i + y j + z k, then
(a) fy2 + gx 2 + hx 2 = 0

(b) fyz + gzx + hxy = 0 ∇f (r ) × r is equal to :

(c) kxyz = constant


→ 1 →
(a) 0 (b) − 3
r
(d) None of the above r
1 → →
(c) 3
r (d) r
r
23

72. A unit normal vector to the surface x 2 y + 2xz = 4 77. The centre of the curve
at the point (2 , − 2 , 3) is : x 2 − xy − 2y2 − x − 4 y − 2 = 0 is :
1 ^ 2^ 2 ^ 1 ^ 2^ 2 ^ (a) (0, − 1) (b) (−1, 0)
(a) − i + j + k (b) i − j− k
3 3 3 3 3 3 (c) (−1, 1) (d) (1, 0)
^ ^ ^ 78. The general equation of the second degree
(c) Both (a) and (b) (d) −2 i + 4 j + 4 k
ax 2 + 2hxy + by2 + 2gx + 2 fy + c = 0 will
→ ^ ^ ^ →
73. If f = x 2 y i − 2xz j + 2yz k, then divergence f is represent a conic with centre at (0, 0) if :
equal to : (a) f = 0, c = 0 (b) f = 0, g = 0
(a) 0 (b) 2y (c) g = 0, c = 0 (d) Only c = 0
(c) 2xy (d) 2y (x + 1)
79. The equation ax + 2hxy + by2 + 2gx + 2 fy
2

74. If a is constant vector, then the value of curl + c = 0 represents a rectangular hyperbola, if :
→ →
( r × a ) is equal to : (a) ∆ = 0, h2 > ab
→ → (b) ∆ ≠ 0, h2 < ab
(a) 0 (b) 2 a
(c) ∆ ≠ 0, h2 > ab
→ →
(c) −2 a (d) a
(d) ∆ ≠ 0, h2 > ab and a + b = 0
2 2 2
75. If u = x − y + 4 z, then ∇ u is equal to :
80. The eccentricity of the hyperbola
(a) 0 (b) 1 9 x 2 − 16 y2 = 144 is :
(c) 2 x (d) −2y 4 5
(a) (b)
76. The equation x + y = a represent a : 5 4
(a) Straight line (b) Circle 9 2
(c) (d)
(c) Parabola (d) Ellipse 2 9

ANSWERS
MULTIPLE CHOICE QUESTIONS
1. (b) 2. (c) 3. (d) 4. (a) 5. (d) 6. (c) 7. (a) 8. (c) 9. (d) 10. (c)

11. (d) 12. (a) 13. (b) 14. (b) 15. (a) 16. (b) 17. (d) 18. (d) 19. (d) 20. (d)

21. (b) 22. (c) 23. (b) 24. (d) 25. (a) 26. (c) 27. (c) 28. (b) 29. (d) 30. (b)

31. (a) 32. (c) 33. (c) 34. (c) 35. (a) 36. (d) 37. (b) 38. (a) 39. (a) 40. (d)

41. (b) 42. (a) 43. (b) 44. (a) 45. (d) 46. (a) 47. (d) 48. (a) 49. (c) 50. (c)

51. (a) 52. (b) 53. (c) 54. (a) 55. (d) 56. (b) 57. (d) 58. (b) 59. (a) 60. (c)

61. (b) 62. (c) 63. (a) 64. (c) 65. (b) 66. (a) 67. (b) 68. (b) 69. (b) 70. (d)

71. (a) 72. (b) 73. (d) 74. (a) 75. (a) 76. (c) 77. (a) 78. (b) 79. (d) 80. (b)
24

HINTS AND SOLUTIONS


→ 15. The equation of tangent at α to conic is
1. Let a = a1i + a2 j + a3k. Then
l
= cos(θ − α ) + e cos θ
i j k r
→ → ∂ ∂ ∂ l
curl a = ∇ × a = compare with = A cos θ + B sin θ
∂x ∂y ∂z r
a1 a2 a3
then eliminate we get
 ∂a ∂a   ∂a ∂a 
=  3 − 2 i +  1 − 3 j ( A − e)2 + B 2 = 1
 ∂y ∂z   ∂z ∂x 
 ∂a ∂a  17. PQ = (1 − 2)2 + (−1 − 3)2 + (3 + 5)2
+  2 − 1 k
 ∂x ∂y  = 1 + 16 + 64 = 9
→ → 18. α = β = r,
Now div curl a = ∇ ⋅ (∇ × a )
⇒ cos 2 α + cos 2 β + cos 2 γ = 1
∂  ∂a3 ∂a2  ∂  ∂a1 ∂a3 
=  −  +  − 
∂x  ∂y ∂z  ∂y  ∂z ∂x  or l = m = n, ⇒ l 2 + m 2 + n 2 = 1
∂  ∂a2 ∂a1  1
+  −  ⇒ 3l 2 = 1 or l = ±
∂z  ∂x ∂y  3
→ → → → → →  1 1 1 
2. div. ( a × b ) = b ⋅ curl a − a curl b ∴ ± ,± ,± 
 3 3 3
=0−0=0 26. The S.D. between two plane

3. ∫c F ⋅ dr = − ∫ ydx = ∫ sinθ(− sin θ)dθ = − π ax + by + cz − d = 0, a ′ x + b ′ y + c ′ z − d ′ = 0
c 0
dc ′ − cd ′
5. 2
+ by2 + cz 2 )ds = ∫ from z-axis is
∫ ∫S (ax ∫sF ⋅ nds (ac ′ − a ′ c)2 + (bc ′ − b ′ c)2
=∫ ∫ ∫q div. (F)dr Here, a = 1, b = 1, c = 2, d = − 3
a ′ = 2 , b ′ = 3, c ′ = 4 , d = − 4
=∫ ∫ ∫r(a + b + c) dv −12 + 8 −4
4π = = = 2 (Numerically)
= (a + b + c) v = (a + b + c) 2
(4 − 4 ) + (4 − 6) 2 2
3
27. The equation of plane is 3 x − y − z − 7 = 0 …(1)
6. Compare 9 x 2 + 25 y2 − 90 x − 150 y + 225 = 0
The d.r. of normal to plane (1) are (3, −1, −1)
with ax 2 + 2hxy + hy2 + 2gx + 2 fy + c = 0 we
get a = 9, h = 0, b = 25, g = − 45, f = − 750, Hence, the equations of the line passing through
c = 225, (2, 3, 7) and perpendicular to plane (1) is
x − 2 y− 3 z −7
∆ = abc + 2 fgh − af 2 − bg 2 − ch2 = = =r
3 −1 −1
= 9 × 25 × − 750 + 0 − 9 × (750)2 ⇒ x = 3r + 2 , y = − r + 3, z = − r + 7
2
− 25(−45) ≠ 0 lies on plane (1)
h2 = 0, ab = 25 × 9 h2 2ab 3(3 r + 2) − (−r + 3) − (−4 + 7) − 7 = 0
9r + 6 + r − 3 + r − 7 − 7 = 0
⇒ Ellipse
11r − 11 = 0 r = 1
25

Then co-ordinate are (5, 2, 6) ⋅(x 2 yi − 2xzj + 2yzx)


x +1 y+ 3 z−2
29. = = =r = 2y(x + 1)
1 3 2
 ∂ ∂ ∂
⇒ P (−1 + r ), − 3 + 3r , 2 + 2 r 75. ∇ 2 x = ∇ ⋅ ∇x =  i + j +k 
 ∂x ∂y ∂z 
Put in plane 3 x + 4 y + 5 z = 20
⋅(2xi − 2yj + 4 )
⇒ r = 1, then P(0, 0, 4 )
= 2− 2= 0
37. The equation of sphere through diameter is
76. x + y= a
(x − 2)(x + 2) + (y + 1)(y − 2) + (z − 4 )(z + 2) = 0
x + y + 2 xy = 9
x 2 − 4 + y2 − y − 2 + z 2 − 2z − 8 = 0
2 xy = (a − x − y) square
x 2 + y2 + z 2 − y − 2z − 14 = 0 4 xy = a2 + x 2 + y2 − 2ax − 2ya + 2xy
 1 
centre  0, ,1 x 2 + y2 − 2xy + 2ax + 2ay = a2
 2 
(x − y)2 + 2ax + 2ay = a2
40. (x 2 + y2 + z 2 − 9) + λ (x + y − 2z + 4 ) = 0
9 ⇒ Parabola
At origin x = 0, y = 0, z = 0 ⇒ 4 λ = 9, λ =
4 ∂F ∂F
77. = 0, =0
4 (x 2 + y2 + z 2 − 9) + 9(x + y − 2z + 4 ) = 0 ∂n ∂y
⇒ 2x − y − 1 = 0
4 (x 2 + y2 + z 2 ) + 9 x + 9 y − 18 z = 0
− 4 y − 4 = 0 ⇒ y = − 1 then x = 0
42. The equation of the polar plane at (x 1, y1, z 1)
(0, − 1)
w.r.t to sphere is xx 1 + yy1 + zz 1 = a2
x2 y2
80. − =1
x y z a2 144 144
by comparing 1 = 1 = 1 =
e m n p 9 16
l m n  144 2 144
⇒  a2 , b 2 , a2  a2 = ,b =
p p p  9 16
12 12
54. 3 x ⋅ (1) − 5 y(1) + z.0 + 2 = 0 or 3 x − 5 y + 2 = 0 ⇒ a= ,b=
3 4
56. OP 2 + OQ 2 + OP 2 = (x 12 + y12 + z 12 )
a = 4, b = 3
+ (x 22 + y22 + z 22 ) + (x 32 + y32 + z 32 ) a = b 2 (1 − e2 )
2

= a2 + b 2 + c 2 16 = 9(1 − e2 )
∇f −2 i + 4 j + 4 x 16 2 16 25
72. Unit normal = = 1 − e2 = e =1 + = ,e= 5/3
| ∇f| 4 + 16 + 16 9 9 9
−1 2 2 ❍❍❍
= i+ j+ k
3 3 3
 ∂ ∂ ∂
73. div. f = ∇ ⋅ f = i + j +k 
 ∂x ∂y ∂z 
26
RML Avadh University, Examination-2019
Paper-III Geometry and Vector Calculus
[Time : 2 Hours] [Maximum Marks : 70]

Note : Attempt all questions. Each question carries equal marks.


1. The conic ax 2 + 2hxy + by2 + 2gx + 2 fy + c = 0 6. The equation of normal to the curve
represents an ellipse when : l
= 1 + e cos θ at any point ‘α’ is :
r
(a) h2 = ab, ∆ ≠ 0
l e sin α
(a) = e sin θ + sin (θ − α )
(b) h2 − ab < 0, ∆ = 0 r 1 + e cos α

(c) h2 − ab < 0, ∆ ≠ 0 l e cos α


(b) = e sin θ + sin (θ − α )
r 1 + e cos α
(d) h2 − ab > 0, ∆ ≠ 0
l e sin α
(c) = e cos θ + cos (θ − α )
2. The centre of the conic r 1 + e sin α
14 x 2 − 4 xy + 11 y2 − 44 x − 58 y + 71 = 0 is : l e sin α
(d) = e sin θ − sin (θ − α )
(a) (2, 3) (b) (3, 2) r 1 + e cos α

(c) (4 , 5) (d) (−5, 4 ) 7. The nature of conic


3. Polar equation of the circle when pole lying on 17 x 2 − 12xy + 8 y2 + 46 x − 28 y + 17 = 0 is :
the circumference of the circle is : (a) Parabola
(a) r = 2a cos (θ − α ) (b) r = 2a sin(θ − α ) (b) Pair of straight lines
(c) r = 2a cos θ (d) r = 2a sin θ (c) Hyperbola
4. In any conic the sum of reciprocals of the segment (d) Ellipse
of focal chord is : 8. The equation of y-axis is :
(a) 1/l (b) 2 /l (c) 4/l (d) l (a) z = 0 (b) y = 0, z = 0
5. The equation of tangent to the conic (c) z = 0, x = 0 (d) x = 0
l
= 1 + e cos θ at any point ‘α’ is : 9. The angle between two diagonals of cube is
r
l (a) cos −1 1/3 (b) sin −1 1/3
(a) = e cos θ + cos (θ + α )
r (c) cos 1/3 (d) tan −1 1/3
l
(b) = e cos θ + sin (θ + α ) 10. The general equation of plane passing through a
r
point (x 1, y1, z 1) is :
l
(c) = e cos θ + cos (θ − α ) (a) A(x − x 1) + B(y − y1) + C(z − z 1) + D = 0
r
l (b) Ax 1 + By1 + Cz 1 + D = 0
(d) = e sin θ + sin (θ − α )
r (c) A(x − x 1) + B(y − y1) + C(z − z 1) = 0
(d) Ax. x 1 + By. y1 + Cz. z 1 + D = 0
27

11. The equation of plane through (−1, 1, 1) and 17. The image of point (1, 3, 4 ) in the plane
(1, − 1, 1) and perpendicular to the plane 2x − y + z + 3 = 0 is :
x + 2y + 2z = 7 is : (a) (1, 3, 3) (b) (3, 5, 2 )
(a) x + y + z + 3 = 0 (c) (3, − 5, − 2 ) (d) (−3, 5, 2 )
(b) 2x + 2y − 3 z + 3 = 0 18. The equation of straight line passing through
(c) 2x + 2y − 3 z + 8 = 0 (a, 0, b) and parallel to x-axis is :
(d) x − y − z + 3 = 0 x−a y z−b x−a y z−b
(a) = = (b) = =
0 1 0 1 0 0
12. A variable plane is at a constant distance 3p from
x−a y z−b x−b y z−a
the origin and meets the axes in A, B and C, then (c) = = (d) = =
0 0 1 1 0 0
the locus of the centroid of the triangle ABC is :
19. If two spheres S1 ≡ 0 and S2 ≡ 0 cuts
(a) x 2 + y2 + z 2 = p 2
orthogonally then :
1
(b) x 2 + y2 + z 2 = (a) u1u2 + v 1v 2 + 2 w1w2 = d1 + d2
p2
(b) 2 u1u2 + v 1v 2 + w1w2 = d1 + d2
1 1 1
(c) + + =9 (c) u1u2 + v 1v 2 + w1w2 = d1 + d2
x2 y2 z2
(d) 2 u1u2 + 2 v 1v 2 + 2 w1w2 = d1 + d2
1 1 1 1
(d) + + =
x 2
y 2
z 2
p2 20. The equation of radical plane of two spheres
S1 ≡ x 2 + y2 + z 2 + 2 u1x + 2 v 1y
13. The co-ordinates of the points where the line
+ 2 w1z + d1 = 0
joining the points (2 , − 3, 1), (3, − 4 , − 5) cuts the
plane 2x + y + z = 7 is : and S2 ≡ x 2 + y2 + z 2 + 2 u2 x + 2 v 2 y
+ 2 w2 z + d2 = 0
(a) (1, 3, 6) (b) (1, − 2 , 5)
(c) (2 , − 3, 6) (d) (1, − 2 , 7) (a) x (u1 − u 2) + y (v1 − v2) +

14. The equation of sphere which passes through z(w1 + w 2) + (d1 − d2) = 0
origin and intercepts length a, b and c on the axis (b) x (u 2 − u1) + y (v2 − v1) +
respectively is : z(w 2 − w1) + (d1 − d2) = 0
(a) x 2 + y2 + z 2 − ax − by − cz + d = 0 (c) 2x(u1 − u 2) + 2y (v1 − v2) +
(b) x 2 + y2 + z 2 − ax − by + d = 0 2z(w1 − w 2) + 2 (d1 − d2) = 0
(c) x 2 + y2 + z 2 − ax − cz + d = 0 (d) 2x(u1 − u 2) + 2y (v1 − v2) +
(d) x 2 + y2 + z 2 − ax − by − cz = 0 2z(w1 − w 2) + (d1 − d2) = 0
15. A plane passes through a fixed point ( p, q, r ) and 21. If S1 ≡ 0 and S2 ≡ 0 be the equation of two
cuts the axis at A, B, C then the locus of centre of spheres, then the equation S1 + λ S2 = 0
the sphere OABC is : represents a sphere except only for :
x y z p q r (a) λ = 1 (b) λ = − 1
(a) + + =2 (b) + + =1
p q r x y z (c) λ ≠ 1 (d) λ ≠ − 1
p q r x y z
(c) + + =2 (d) + + =1 22. Every homogenous equation of second degree,
x y z p q r
ax 2 + by2 + cz 2 + 2 fyz + 2gzx + 2hxy = 0
16. The distance between the planes
where ∆ ≠ 0, represents :
2x + 2y − z + 2 = 0 and 4 x + 4 y − 2z + 5 = 0 is :
(a) 1/2 (b) 1/6 (a) A pair of planes (b) A sphere
(c) 1/4 (d) 2/7 (c) A cylinder (d) A cone
28

23. The equation of cone whose vertex is origin and 27. Polar plane of (α , β, γ) with respect to ellipsoid
passes through the curve of intersection of the x2 y2 z2
+ + = 1 is :
plane lx + my + nz = p and the surface a2 b2 c2
ax 2 + by2 + cz 2 = 1 is : x y z
(a) + + −1 = 0
αa βb γc
(a) (ax 2 + by2 + cz 2 ) = (lx + my + nz)2
(b) αax + βby + γcz − 1 = 0
(b) p 2 (ax 2 + by2 + cz 2 ) = (lx + my + nz)2 αx βy γz
(c) 2 + 2 + 2 − 1 = 0
(c) p 2 (ax 2 + by2 + cz 2 )2 = (lx + my + nz) a b c
αx 2 β y2 γz 2
(d) p 2 (ax 2 + by2 + cz 2 )2 = (lx + my + nz)2 (d) + + −1 = 0
a2 b2 c2
24. The equation
28. The equation of normal to the ellipsoid
ax 2 + by2 + cz 2 + 2 ux + 2 vy + 2 wz + d = 0
x2 y2 z2
represents a cone if : + + = 1 at (α , β, γ) is :
a2 b2 c2
u2 v 2 w2 x −α y−β z−γ
(a) + + =d (a) = =
a b c a 2
b 2
c2
2 2 2
u v w 1 x −α y−β z−γ
(b) + + = (b) = =
a b c d α/a 2
β/b 2
γ /c 2
2 2 2
u v w 1 x −α y−β z−γ
(c) + + = 2 (c) = =
a b c d αa 2 βb2 γc 2
2 2 2
u v w
(d) + + = d2 (d)
x −α
=
y−β
=
z−γ
a b c α 2a 2 β 2b2 γ 2c 2
25. The equation of right circular cone whose vertex
29. The equation of plane parallel to x-axis is
is origin 0 and axis is z axis and semi vertical
angle α : (a) ax + by + cz + d = 0 (b) ax + by + d = 0
2 2
(a) x + z = y tan α 2 2 (c) by + cz + d = 0 (d) ax + cz + d = 0
30. The equation of right circular cylinder whose axis
(b) x 2 + y2 = z 2 tan 2 α
is z-axis is :
(c) x 2 + z 2 = x 2 tan 2 α
(a) x 2 + y 2 + z 2 = a 2 (b) x 2 + y 2 = a 2
(d) None of the above
(c) y 2 + z 2 = a 2 (d) z 2 + x 2 = a 2
26. The condition that the plane lx + my + nz = p be
the tangent plane to the conicoid 31. The radius of director sphere of the conicoid
2 2
ax + by + cz = 1 is : 2 x 2 y2 z 2
+ − = 1 is :
4 15 10
l2 m2 n2
(a) p 2 = + + (a) 2 (b) 3 (c) 4 (d) 5
a2 b2 c2
x − 5 y−7 z + 3
l 2 m 2 n2 32. If lines = = and
(b) p = + + 4 4 −5
a b c
x −8 y−4 z−5
= = are coplanar then their
(c) p 2 = a2l 2 + b 2m 2 + c 2n 2 7 1 3
l 2 m 2 n2 point of intersection is :
(d) p 2 = + +
a b c (a) (2, 4, − 3) (b) (1, − 5, 3)
(c) (1, 3, − 2) (d) None of these
29

x y z x y z x y z
33. If the lines = = , = = , = = 4^
i + 3^j − 5^
k 3^
i − 6^j + 5^
k
α β γ aα bβ cγ l m n (a) (b)
18 7
are coplanar if :
(a) α(b − c) + β (c − a) + γ(a − b) = 0 3^
i + 2^j + 6^
k 3^
i − 2^
j + 6k
(c) (d)
l m n 7 7
(b) (b − c) + (c − a) + (a − b) = 0
α β γ → ^ ^ ^ d 2r
40. If r = sin t i + cos t j + t k then 2is :
(c)
α β γ
(b − c) + (c − a) + (a − b) = 0 dt 
l m n
(a) 1 (b) 2
(d) a + b + c = 0
(c) −1 (d) 0
34. The surface x 2 − y2 + 2yz − 2zx − x − y + z = 0
41. A particle moves along the curve x = 4 cos t, y = 4
represents : sin t, z = 6 t. The magnitude of acceleraton at any
(a) A hyperbolic paraboloid time t is :
(b) An elliptic paraboloid (a) 2 (b) 1

(c) A parabolic cylinder (c) 4 (d) 5

(d) None of the above → → ^ ^ ^


42. If r = | r| where r = x i + y j + z k then ∇f (r ) will
35. The anlge between the planes 2 x − y + z = 7 and be :
x + y + 2 z = 9 is :
→ →
(a) f (r)∇ r (b) f ′(r)∇ r
(a) π / 2 (b) π / 4
1 1 →
(c) π / 3 (d) π (c) ∇f (r) (d) ∇r
2 f ′(r)
^ ^ ^ ^ ^ ^
36. If the vecotrs i − 2x j + 3 y k and i + 2x j − 3 y k 43. The maximum value of directional derivative of
are perpendicular then the locus of (x, y) is : φ = x 2yz at point (1, 4, 1) is :
(a) Straight line (a) 8 (b) 9
(b) Pair of straight line (c) 6 (d) 7
(c) Rectangular hyperbola 44. The equation of normal to the surface xyz = 4 at
(d) None of the above (1, 2, 2) is :
x −1 y− 2 z − 2
^ → ^ ^→ ^ ^→ ^ (a) = =
37. i × ( a × i ) + j ( a × j ) + k( a × k) : 4 2 2
→ x −1 y− 2 z − 2
(a) 0 (b) a (b) = =
4 4 3
→ → x −1 y− 2 z
(c) 2 a (d) 4 a (c) = =
4 2 3
→ → → → → →
38. If a + b + c = 0 and | a| = 3,| b| = 5,| c| = 7 x −1 y− 2 z −1
(d) = =
→ → 4 2 2
then the angle between a and b is :
→ ^ ^ →
(a) π / 6 (b) 2 π / 3 45. If F = yz 2 i − zx 2 k then div. F :

(c) π / 3 (d) 4 π / 3 (a) −zx 2 (b) −x 2


39. The unit vector perpendicular to the vector (c) yz 2 (d) z 2
→ ^ ^ ^ → ^ ^ ^
a = 2 i − 6 j − 3 k and b = 4 i + 3 j − k is :
30

→ ^ ^ 53. Guiding curve of a right circular cylinder is :


46. Find curl F if F = ex sin y i + ex cos y j :
(a) Circle (b) Ellipse
(a) 1 (b) 2
(c) −1 (d) 0 (c) Parabola (d) Pair of straight lines
→ →
47. For any closed surface S ∫
→ ^
curl F ⋅ n ds : 54. If r = x ^
i + y^
j + z^
k and r = | r | then ∇r n :
∫s

(a) 0 (b) F (a) nr n−1 r (b) nr n− 2
(c) 2F (d) 1 → →
(c) nr n− 2 r (d) n(n − 2)r n− 2 r
→ →
48. Using Stroke’ theorem ∫ r ⋅ d r : →
c 55. If r = aewt + be −wt where a, b are constant vectors
→ →
(a) 2 r (b) 0 d2 r →
then − w 2 r will be :
→ 2
(c) r (d) 1 dt

49. The shortest distance between the lines (a) a b (b) a 2 + b2


x −1 y− 2 z − 3 x − 2 y− 3 z − 4
= = , = = is : (c) a 2b2 (d) 0
2 3 4 3 4 5
56. Confocal cut at an angles :
(a) 3 / 6 (b) 6
(c) 1 / 6 (d) 0 (a) π / 4 (b) π / 3

50. The enveloping cone of the sphere (c) π / 6 (d) None of these
2 2 2
x + y + z + 2x − 2y = 2 with its vertex at x2 y2
57. The foci of the confocal + = 1 are :
2 2
(1, 1, 1) is : a + λ b + λ

(a) 4x 2 + 3y 2 + 5z 2 + 6yz + 8x + 16z = 0 (a) (± a 2 − b2 , 0) (b) (0, ± a 2 − b2 )


2 2
(b) 4x + 3y + 6yz + 8x = 0
(c) (± a 2 − b2 − 2λ , 0) (d) None of these
2 2 2
(c) 4x + 3y − 5z = 0
58. The number of real umbilics of an ellipsoid is :
(d) None of the above (a) Two (b) Three (c) Four (d) Zero
51. The pole of the plane lx + my + nz = p with respect
59. The slope of tangent at ‘α’ to the
to sphere x 2 + y 2 + z 2 = a 2 is : 1
conic = 1 + e cosθ is :
 a2 a2 a2   a 2l a 2m a 2n  r
(a)  , , (b)  , ,
 pl pm pn   p p p 

(a) −
cos θ + e
(b)
cos α + e
  
sin α sin α
 l m n 
(c)  , ,  (d) None of these sin α sin α
(c) (d) −
 a 2p a 2p a 2p  cos α + e cos α + e
52. The equation of hyperboloid of two sheet is : 15
60. The semi-latus rectum of the conic = 5 − 4 cosθ is :
2 2 2 r
x y z
(a) + + =1
2 2
a b c2 (a) 3 (b) 15 (c) 6 (d) 5

x2 y2 z2 61. The direction-cosines of a line whose dr’s are 2, 3,


(b) − + =1
a 2
b2
c 2 −6 are :
2 3 −5
x2 y2 z2 (a) 2, 3, − 6 (b) , ,
(c) − − =1 7 7 7
a2 b2 c2
2 3 −6 1 2 3
(c) , , (d) , ,
(d) a 2x 2 + b2y 2 + c 2z 2 = 1 7 7 7 7 7 7
31

62. If l1 = l 2, m1 = m2 and n 1 = n2, where l1, m1, n 1, 72. The generators of the cylinder f (y, z) = 0 are parallel
l 2, m2, n2 are dc’s of two lines, then the lines are : to the axis :
(a) Perpendicular (b) Parallel (a) x-axis (b) y-axis

(c) Make an angle 30° (d) None of these (c) z-axis (d) None of these

63. The angle between the planes 2x − y + z = 6 and 73. The number of normals that can drawn to an
x + y + 2z = 7 is : ellipsoid from a given point are :

(a) π / 6 (b) π / 3 (a) 4 (b) 2

(c) π / 4 (d) π / 2 (c) 6 (d) 3

64. The intercept on x-axis of the plane


x y z
+ + = 1 is : 74. The equation ax + by + cz 2 = 1 represents an
2 2

a b c
ellipsoid if :
(a) b (b) a
(a) a, b are positive (b) a ≥ 0, b > 0, c > 0
(c) 1 (d) c
(c) a < 0, b ≥ 0, c ≥ 0 (d) None of these
65. The angle between the lines π / 4, then the lines are :
75. The centre of the surface x 2 + y 2 + z 2 = 1 is :
(a) Coplanar (b) Non-Coplanar
(a) (1, 1, 3) (b) (1, 1, 1)
(c) Collinear (d) None fo these
(c) (0, 0, 0) (d) (3, 1, 1)
66. The centres of all the spheres of coaxial system lie
on : 76. Ruled surfaces are generated by a :

(a) Straight line (b) Sphere (a) Moving line (b) Moving point
(c) Cone (d) None of these (c) Fixed line (d) Fixed point
67. The number of spheres that are touching the 77. Locus of the point of intersecton of perpendicular
Co-ordinate axis are : generators of the hyperboloid of one sheet is a :
(a) Infinite (b) 8 (a) Hyperboloid of two sheets
(c) 1 (d) None of these (b) Director sphere
68. The radical plane of two spheres cuts the line joining (c) Paraboloid
their centres at the angle :
(d) Plane
(a) π / 4 (b) π / 3
78. If a and b are irrotational, then div. (a × b) is equal to :
(c) π / 2 (d) π / 6
(a) a (b) b
69. Two spheres with centres C 1 and C 2 of radii r1 and
(c) 0 (d) a + b
r2 respectively touch externally if C 1C 2 equals :
(a) r1 (b) r2 79. The vector r n r is Solenoidal if n equals :

(c) r1 + r2 (d) r1 − r2 (a) 3 (b) −3 (c) 2 (d) 0

70. The equation of a cone reciprocal to the cone 80. The div. curl a is equal to :
fx + gy + hz = 0 is : (a) 0 (b) 1
(a) fyz + gzx + hxy = 0 (b) fyz − gzx − hxy = 0 (c) −1 (d) a
(c) fx + gy + hz = 0 (d) None of these
81. Gradient of a constant scalar field is equal to :
71. The vertex of the cone fyz + gzx + hxy = 0 is :
(a) 0 (b) 1
(a) (0, 0, 0) (b) (1, 1, 1)
(c) −1 (d) None of these
(c) (0, 1, 0) (d) (0, 0, 1)
32

82. If a vector V is Solenoial the div. V is equal to : (a) (3a, 3b, 3c) (b) (a / 3, b / 3, c / 3)
(a) 1 (b) −1 (c) 2 (d) 0 (c) (3 / a, 3 / b, 3 / c) (d) (1 / 3 a, 1 / 3 b, 1 / 3 c)
83. ∫sF × nds is equal to : 93. The equation of the plane through the Intersection
of two planes P = 0 and Q = 0 will be :
(a) ∫v ∇ × FdV (b) − ∫ ∇ × FdV
v (a) PQ = 0 (b) P = λQ = 0
(c) ∫v ∇ FdV (d) − ∫ ∇ FdV (c) P / λQ = 0 (d) P = Q = 0
v
94. The angle between the planes 2x − y + z = 16,
84. Value of ∫ r ⋅ dr is :
c x + y + 2z = 7 is :
(a) 0 (b) 2∫ ndr (a) cos −1(4 / 21) (b) π / 3
s

(c) Unit vector (d) Indeterminate (c) π / 4 (d) π / 2


85. A relation between surface and volume integrals is : 95. Two lines which do not lie in the same plane are
m m called :
(a) Stoke’s Th (b) Gauss Th
(c) Green’s Th m (d) Bernoullis Th m (a) Parallel (b) Intersecting

86. Curl grad φ is equal to : (c) Co-incident (d) Skew


(a) 2 (b) 3 (c) 0 (d) 4 96. Find the equation of the sphere described on the
87. div. r is equal to : join (2, − 3, 4) and (−5, 6, − 7) as diameter :

(a) 2 (b) 3 (c) 3.5 (d) 4 (a) x 2 + y 2 + z 2 = 1

88. To the surface f (x, y, z) = c, ∇f represents : (b) x 2 + y 2 + z 2 + 2x − 3y + 3z = 0


(a) Vector Normal (b) Vector Tangent
(c) x 2 + y 2 + z 2 + 3x − 3y + 3z − 56 = 0
(c) Unit Normal (d) None of these
(d) x 2 + y 2 + z 2 + 5x + 7 z − 56 = 0
89. A central conic has four foci of which :
(a) Two are real and two imaginary 97. The radius of the sphere 2x 2 + 2y 2 + 2z 2 = 50 is :

(b) All the four are real (a) 25 (b) 5 (c) 5 5 (d) 2
(c) All the four are imaginary 98. The centre of the sphere
(d) None of the above x 2 + y 2 + z 2 − ax − by − cz = 0 is :
90. The locus of the point from which two (a) (0, 0, 0) (b) (a / 2, b / 2, c / 2)
perpendicular tangents can be drawn one to each −a −b −c 
of two given confocals : (c)  , ,  (d) (a , b, c)
 2 2 2
(a) Pair of straight line (b) Circle
99. The plane section of a sphere cut by the plane
(c) Confocal conic (d) An Ellipse through the centre of the sphere is :
l
91. The line = sin θ + e cos θ is a tangent line to the (a) Circle (b) Sphere
r
l (c) Parabola (d) Great Circle
conic = r + e cos θ at the point :
r 100. The semi vertical angle of a right circular cone having
(a) π / 6 (b) π / 3 a set of three mutually perpendicular generators is :
(c) π / 4 (d) None of these (a) tan −1 2 (b) tan −1 2
92. The plane ax + by + cz = 1 meets the co-ordinates (c) tan −1
1
(d) π / 4
axes in A, B, C. The centroid of the triangle is : 2
33

ANSWERS
MULTIPLE CHOICE QUESTIONS
1. (c) 2. (a) 3. (c) 4. (b) 5. (c) 6. (a) 7. (d) 8. (c) 9. (a) 10. (c)

11. (b) 12. (d) 13. (d) 14. (d) 15. (b) 16. (b) 17. (d) 18. (b) 19. (d) 20. (d)

21. (d) 22. (d) 23. (b) 24. (a) 25. (b) 26. (a) 27. (c) 28. (b) 29. (c) 30. (b)

31. (b) 32. (d) 33. (c) 34. (a) 35. (c) 36. (d) 37. (c) 38. (c) 39. (c) 40. (a)

41. (c) 42. (b) 43. (b) 44. (a) 45. (b) 46. (d) 47. (a) 48. (b) 49. (d) 50. (d)

51. (b) 52. (c) 53. (d) 54. (c) 55. (d) 56. (d) 57. (a) 58. (c) 59. (a) 60. (b)

61. (c) 62. (b) 63. (b) 64. (b) 65. (d) 66. (a) 67. (b) 68. (c) 69. (c) 70. (a)

71. (a) 72. (a) 73. (c) 74. (d) 75. (c) 76. (a) 77. (b) 78. (c) 79. (b) 80. (a)

81. (a) 82. (d) 83. (a) 84. (a) 85. (a) 86. (c) 87. (b) 88. (a) 89. (a) 90. (c)

91. (d) 92. (b) 93. (b) 94. (b) 95. (d) 96. (d) 97. (b) 98. (b) 99. (a) 100. (b)

HINTS AND SOLUTIONS


2. Let f (x, y) = 14x 2 − 4xy + 11y 2 − 44x − 58y 7. Compare 17 x 2 − 12xy + 8y 2 + 46x − 28y + 17 = 0
+ 71 = 0 where ax 2 + 2hxy + by 2 + 2gx + 2 fy + c = 0
∂F
= 28x − 4y − 44 = 0 a = 17, h = − 6, b = 8 , g = 23, f = − 14, c = 17
∂x
∂F and h2 < ab, ∆ ≠ 0 so it is ellipse.
= − 4x + 22y − 58 = 0
∂y 9. Let θ be angle beween two diagonal OP and AC with
1 1 1   −1 1 1 
solve we get x = 2, y = 3 d. c are  , ,   , ,  then
 3 3 3  3 3 3
3. The equation of circle with pole on its circumference cosθ = l1l 2 + m1m2 + n1n2
is r = 2a cosθ. 1
⇒ θ = cos −1  
1 1 1  1  3
4. We have  + =
2  SP SP ′  l 11. The equation of plane passing through (−1, 1, 1) is
1 1 2 a(x + 1) + b(y − 1) + c(z − 1) = 0 …(1)
or + =
SP SP ′ l
It passed (1, − 1, 1)
6. The equation of normal at the point P in term of α is
⇒ 2a − 2b + oc = 0 …(2)
le sin α
= sin(θ − α) + e sin θ
r(1 + e cos α) and perpendicular to x + 2y + 2z = 0
34

then a + 2b + 2c = 0 ⇒ F (x, y, z, t) = ax 2 + by 2 + cz 2 + 2uxt


∴ 2a − 2b + oc = 0 + 2vyt + 2wzt + dt 2
a + 2b + 2c = 0
Put t = 0 in
a b c
= = ∂F ∂F ∂F ∂F
−4 −4 4 + 2 ∴ = 0, = 0, = 0, =0
∂x ∂y ∂z ∂t
a b c a b c
= = or = = we get x = − u / a, y = − v / b, z = − w / c
−4 −4 6 2 2 −3
∂F
∴ 2(x + 1) + 2(y − 1) − 3(z − 1) = 0 put these values in = 0 we get
∂t
⇒ 2x + 2 + 2y − 2 − 3z + 3 = 0
u 2 v2 w 2
+ + =d
⇒ 2x + 2y − 3z + 3 = 0 a b c
16. The distance between the given parallel plane 25. Here l = 0 = m = 0, l = 1
= length of perpendicular from P to the plane
∴Then from Largange’s identity
2 5 4 − 5 −1 1
= −  = = = x 2(x 2 + y 2 + z 2) sin 2 θ = x 2y 2 + x 2x 2
3 6  6   6  6

17. The given plane is 2x − y + z + 3 = 0 …(1) or x 2 + y 2 = z 2 tan 2 θ

d.c. of line perpendicular to the plane (1) are 26. The equation of tangent plane at (x1, y1, z1) to conic
(2, − 1, 1) let Q be image of given point. Then the ax 2 + by 2 + cz 2 = 1 is
equation of line PQ passing P(1, 3, 4) perpendicular axx1 + byy1 + czz1 = 1 …(1)
to plane (1) are compare with l x + my + nz = p
x −1 y− 3 z − 4 l m n
= = =λ …(2) we get x1 = , y1 = , z1 =
2 −1 1 ap bp cp
⇒ Q(2λ + 1, − λ + 3, λ + 4) …(3) put in ax12 + by12 + cz12 = 1
∴co-ordinate of middle point PQ is l 2 m 2 n2
we get + + = p2
 λ + 1, −1 λ + 3, λ + 4 a b c
 
 2 2 
28. The equation of normal to the given ellipsoid at
put in (3) λ = − 2 (α, β, γ) are

∴ Q(−3, 5, 2) x −α y−β z−γ


= =
α / a2 β / b2 γ / c2
23. ax 2 + by 2 + cz 2 = 1 …(1)
29. If the plane ax + by + cz + d = 0 parallel to x-axis
make homogenous equation of two degree with the
then normal to the plane will be right angle to x-axis,
help lx + my + nz = p
hence
lx + my + nz
⇒ =1 a × 1 + b × 0 + c × 0 or a = 0
p
2 then by + cz + d = 0
 lx + my + nz 
∴ ax 2 + by 2 + cz 2 =   α β γ
 p 
33. l m n =0
⇒ (ax 2 + by 2 + cz 2)p 2 = (lx + my + nz)2 aα bβ cγ

24. Let F (x, y, z) = ax 2 + by 2 + cz 2 + 24x ⇒


l m n
(b − c) + (c − a) + (a − b) = 0
α β γ
+ 2vy + 2wz + d = 0 …(1)
35

(2)(1) + (−1)(1) + (1)(2) 1 40. r = sin ti + cos tj + kt


34. cos θ = =
(2)2 + (−1)2 + (1)2 12 + 12 + 22 2 dr
= cos ti − sin tj + x
dt
θ= π/3
d 2r
→ → ^ → → → → → → = − sin t i − cos tj
37. i × ( a × i) + j × ( a × j ) + k × ( a × k) dt 2
a i a j → d 2r
= + + a k  2 = sin 2 + t cos 2 t = 1
i a 1 j a 1 k a 1 dt 

= a = a1i + a2 j + a3k 41. r = 4 cos ti + 4 sin tj + 6tk


dr
= a − i(i ⋅ a) + a − j(a ⋅ j + a − k(a ⋅ k) v= = − u sin ti + 4 cos tj + 6k
dt
= 3a − a1i − a2 j − a3k
→ d 2r
= 3a − a = 2a a= = − 4 cos ti − 4 sin tj + 0
dt
38. a+ b= −c →
| a| = 16 cos 2 + 16 sin 2 t = 4
(a + b) ⋅ (a + b) = c ⋅ c
∂d ∂φ ∂φ 2
→ → →→ 43. φ = x 2yz, = 2xyz, = x 2 z, x y
| a|2 + |b|2 + 2| a|b|⋅ cos θ = |c|2 ∂x ∂y ∂z
∂φ ∂φ ∂φ
9 + 25 + 2 × 3 × 5 cosθ = 49 at (1, 4, 1), = 8, = 1, =4
∂x ∂x ∂z
cos θ = 1 / 2, θ = 60 = π / 3
maximum value of directional derivative
39. Let c = c1i + c 2 j + c 3 k
|∇φ| = |8i + j + 4x|
⇒ |c| = c12 + c 22 + c 32 = 64 + 1 + 16 = 9

⇒ c12 + c 22 + c 32 =1 …(1) 45. F = yz i − zx 2k


2

c is perpendicular to A and B, A = 2i − 6 j − 3k  ∂ ∂ ∂ 
∇⋅ F =  i + j+ k  ⋅ (yz 2i − zx 2k)
 ∂x ∂y ∂z 
B = 4i + 3 j − k
⇒ C⋅ A = , C⋅ B = 0 = 0 − x2 = − x2

⇒ 2c1 − 6 c 2 − 3c 3 = 0 46. F = e x sin yi + e x cos yj

4c1 + 3c 2 − c 3 = 0 i j k
c1 c2 c3 ∂ ∂ ∂
= = ∇×F =
6 + 9 −12 + 2 6 + 24 ∂x ∂y ∂z
e x sin y e x cos y 0
c1 c 2 c 3
= = =k
15 10 30 = i(0 − 0) + j(0 − 0) + k(e x cos y − e x sin xy)
c1 c 2 c 3 =0
or = = =k
3 2 6
x2 y2 z2
52. + + = 1 (Ellipsaid)
⇒ c1 = 3k, c 2 = 2k, c 3 = 6k a 2
b2
c2
9x 2 + 4x 2 + 36x 2 = 1 x2 y2 z2
+ − = 1 (Hyperboloid in one sheet)
2 2
49x 2 = 1, x =
1 a b c2
7
x2 y2 z2
3i + 2 j + 6k − − = 1 (Hyperboloid in two sheets)
∵ c= a2 b2 c2
7
36

l Put in
60. = 1 + e cos θ, l = semi latus rectum.
r
Ax 2 + By 2 + Cz 2 + 2Fyz + 2Gzx + 2Hxy = 0
a1a2 + b1b2 + c1c 2 1
63. cosθ = =
a12 + b12 + c12 a22 + b22 + c 22 2 ⇒ fyz + gzx + hxy = 0
73. 6 normals can be drawn to conicoids
θ= π/6
98. The centre of sphere
71. Here a = f 2, b = g 2, c = h2, f = − gh
x 2 + y 2 + z 2 + 24 x + 2vy + 2wz + d = 0 is
g = − fh, h = − fg
 −u , −v , −w 
 
A = bc − f 2 = g 2h2 − g 2h2 = 0, B = 0, C = 0  2 2 2 
❍❍❍
F = gh − af = 2 f 2gh, G = 2g 2hf , H = 2h2 fg

You might also like